Download as pdf or txt
Download as pdf or txt
You are on page 1of 232

Contents Serial

No.
UNIT - 3
Page
No.

1. Centre of Mass 1-76


PHYSICS

2. Rigid Body Dynamics 77-176

Year Long Revision Exercises 177-226


MODULE CONTENTS
THEORY
1. Comprehensive theory covering all concepts & subtopics for excellence in both school level as
well as competitive exams.
Symbols used for categorization
SL Þ Topics required for school level preparations.

AL Þ Topics required for Advance level preparations useful for competitive exams.

2. Illustrations : Subtopic based solved questions to get comfortable in problem solving.


[Students should go through these after the sub-topic is dealt in class]

3. Beginner Boxes : Collection of elementary sub–topic based questions. [Students should


attempt these on completion of each subtopic]

4. Golden Key Points : Important points/formulaes or concepts summarized at the end to have a
quick revision of the topic.

5. Solved examples : A collection of miscellaneous solved question based on different concepts


from the chapter at the end. [Students should refer this before exercise solving]

EXERCISE
ETBD (ESSENTIALLY TO BE DISCUSSED IN CLASS)
6. Exercise-1 : Collection of miscellaneous objective problems with single correct to enhance
speed & accuracy during problem solving. [Targeted towards JEE-Mains]

7. Exercise-2 : Collection of miscellaneous objective problems with one or more than one correct
options and comprehension based objective problems to test analytical, reasoning &
comprehension skills as per latest JEE pattern.[Targeted towards JEE-Advance]

8. Exercise-3 : Collection of Numerical answer based and Matrix Match questions to test
analytical, reasoning & problem solving skills as per latest JEE pattern.[Targeted towards JEE-
Advance]

9. Exercise-4A : Questions from 2008-2019 & 2020 (first attempt) AIEEE/JEE–mains both
offline & online. [Fundamental level]

10. Exercise-4B : Questions from 2008-2019 & 2020 (first attempt) AIEEE/JEE–mains both
offline & online. [Advance level]

11. Exercise-5: Questions from 2008-2019 IIT JEE/JEE advanced.

NTBD (NOT TO BE DISCUSSED IN CLASS)


12. Year Long Revision Exercise : A Large collection of subjective & objective problems to
attain subject expertise categorized in to different patterns for each chapter. [Students should
use this for yearlong revision of the topic and discuss in doubt classes]
CENTRE OF MASS

Recap of Early Classes


In this section, we describe the overall motion of a system in terms of a special point called the center
of mass of the system. The system can be either a group of particles, such as a collection of atoms in a
container, or an extended object, such as a gymnast leaping through the air. We shall see that the translational
motion of the center of mass of the system is the same as if all the mass of the system were concentrated at
that point. That is, the system moves as if the net external force were applied to a single particle located at the
center of mass. This behavior is independent of other motion, such as rotation or vibration of the system.

1.0 SYSTEM OF PARTICLES


1.1 System of Particles and Mass Center
1.2 Center of Mass Frame of Reference or Centroidal Frame
1.3 Application of Newton’s Laws of Motion to a System of Particles
2.0 IMPULSE OF A FORCE
2.1 Impulse Momentum Principle
2.2 Impulsive Motion
3.0 MOMENTUM AND KINETIC ENERGY
3.1 Internal and external Forces and System of interacting Particles
3.2 Principle of Conservation of linear momentum
3.3 Application of Methods of Impulse and Momentum to a System of Particles
3.4 Application of Methods of Work and Energy to a System of Particles
4.0 IMPACT BETWEEN TWO BODIES
4.1 Central and Eccentric Impact
4.2 Head–on (Direct) Central Impact
4.3 Oblique Central Impact
4.4 Oblique Central Impact when one or both the colliding bodies are constrained in motion
5.0 VARIABLE MASS SYSTEM
5.1 Rocket propulsion
EXERCISE-1
EXERCISE-2
EXERCISE-3
EXERCISE-4(A)
EXERCISE-4(B)
EXERCISE-5
f
Centre of Mass
Study of kinematics enables us to explore nature of translation motion without any consideration to forces and
energy responsible for the motion. Study of kinetics enables us to explore effects of forces and energy on
motion. It includes Newton’s laws of motion, methods of work and energy and methods of impulse and
momentum. The methods of work and energy and methods of impulse and momentum are developed using
r r
equation F = ma together with the methods of kinematics. The advantage of these methods lie in the fact that
they make determination of acceleration unnecessary. Methods of work and energy directly relate force, mass,
velocity and displacement and enable us to explore motion between two points of space i.e. in a space interval
whereas methods of impulse and momentum enable us to explore motion in a time interval. Moreover methods
of impulse and momentum provides only way to analyze impulsive motion.
The work energy theorem and impulse momentum principle are developed from Newton’s second law, and we
have seen how to apply them to analyze motion of single particle i.e. translation motion of rigid body. Now we
will further inquire into possibilities of applying these principles to a system of large number of particles or rigid
bodies in translation motion.

1.0 SYSTEM OF PARTICLES

By the term system of particles, we mean a well defined collection of several or large number of particles, which
may or may not interact or be connected to each other.
As a schematic representation, consider a system of n particles of
masses m1, m2,...mi...mj.... and mn respectively. They may be actual particles mi mn
r
of rigid bodies in translation motion. Some of them may interact with each f ij
m1
other and some of them may not. The particles, which interact with each m3
r
r r f ji

other, apply forces on each other. The forces of interaction fij and f ji between mj
m
a pair of ith and jth particles are shown in the figure. Similar to these other 2

particles may also interact with each other. These forces of mutual interaction System of n interacting particles.
between the particles of the system are internal forces of the system.
These internal forces always exist in pairs of forces of equal magnitudes and opposite directions. It is not
necessary that all the particles interact with each other; some of them, which do not interact with each other,
do not apply mutual forces on each other. Other than internal forces, external forces may also act on all or
some of the particles. Here by the term external force we mean a force that is applied on any one of the particle
included in the system by some other body out-side the system.
In practice we usually deal with extended bodies, which may be deformable or rigid. An extended body is also
a system of infinitely large number of particles having infinitely small separations between them. When a body
undergoes deformation, separations between its particles and their relative locations change. A rigid body is an
extended body in which separations and relative locations of all of its particles remain unchanged under all
circumstances.

1.1 System of Par ticles and Mass Center

Until now we have deal with translation motion of rigid bodies, where a rigid body can be treated as a particle.
When a rigid body undergoes rotation, all of its particles do not move in identical fashion, still we must treat it
a system of particles in which all the particles are rigidly connected to each other. On the other hand we may
have particles or bodies not connected rigidly to each other but may be interacting with each other through
internal forces. Despite the complex motion of which a system of particles is capable, there is a single point,
251\D\Allen(IIT-JEE Wing)\2020-21\Nurture\Phy\Unit-03\01-Centre of Mass

known as center of mass or mass center (CM), whose translation motion is characteristic of the system.
The existence of this special point can be demonstrated in the following examples dealing with a rigid body.
Consider two disks A and B of unequal masses connected by a very light rigid rod. Place it on a very smooth
table. Now pull it horizontally applying a force at different points. You will find a point nearer to the heavier
disk, on which if the force is applied the whole assembly undergoes translation motion. Furthermore you cannot
find any other point having this property. This point is the mass center of this system. We can assume that all
the mass were concentrated at this point. In every rigid body we can find such a point. If you apply the force
on any other point, the system moves forward and rotates but the mass center always translates in the direction
of the force.
1
JE E-Physics

A A A
F

F
C C C

F
B B B

Force applied on Force not applied Force not applied


the mass center on the mass center on the mass center
In another experiment, if two forces of equal magnitudes are applied on the disks in opposite directions, the
system will rotate, but the mass center C remains stationary as shown in the following figure.

F
A

C
F
B

Body rotates but the mass center remains stationary


under action of equal and opposite forces.

If the above experiment is repeated with both disks A and B of identical masses, the mass center will be the mid
point. And if the experiment is repeated with a uniform rod, the mass center again is the mid point.

F
A

2F
B

Body rotates and the mass center translates under action


of unbalanced forces applied at different points.

As another Illustration let us throw a uniform rod in air holding it from one of its ends so that it rotates also.
Snapshots taken after regular intervals of time are shown in the figure. The rod rotates through 360°. As the
rod moves all of its particles move in a complex manner except the mass center C, which follows a parabolic
trajectory as if it were a particle of mass equal to that of the rod and force of gravity were acting on it.

A A A B B
B B
A
B A
B A B
A B A

B A B
A
251\D\Allen(IIT-JEE Wing)\2020-21\Nurture\Phy\Unit-03\01-Centre of Mass

B A

C
A B A B

Thus mass center of a rigid body or system of particles is a point, whose translation motion under action of
unbalanced forces is same as that of a particle of mass equal to that of the body or system under action the
same unbalanced forces. And if different forces having a net resultant are applied at different particles, the
system rotates but the mass center translates as if it were a particle of the mass same as that of the system and
the net resultant were applied on it.

2
Centre of Mass
Concept of mass center provides us a way to look into motion of the system as a whole as superposition of
translation of the mass center and motion of all the particles relative to the mass center. In case of rigid bodies
all of its particles relative to the mass center can move only on circular paths because they cannot changes their
separations.
The concept of mass center is used to represent gross translation of the system. Therefore total linear momentum
of the whole system must be equal to the linear momentum of the system due to translation of its mass center.
Center of Mass of System of Discrete Particles
A system of several particles or several bodies having finite separations y
vi
m
between them is known as system of discrete particles. Let at an instant i

particles of such a system m1, m2, ….mi, ……and m n are moving with vc
r r r r r r r
velocitie s v1 , v 1 , v 2 , …… v i ,……and v n a t loc ations r1 , r2 , C
rc
r r th
…… ri ,……and rn respectively. For the sake of simplicity only i particle and O x
r z
the mass center C are shown in the figure. The mass center C located at rc is
r
moving with velocity vc at this instant.
As the mass center represents gross translation motion of the whole system, the total linear i.e. sum of linear
momenta of all the particles must be equal to linear momentum of the whole mass due to translation of the mass
center.
r r r r r
m1v 1 + m2v 2 + ............... + mi v i + .................... + mnv n = Mv c
We can write the following equation in terms of masses and position vectors as an analogue to the above
equation. This equation on differentiating with respect to time yields the above equation therefore can be
thought as solution of the above equation.
r r r r r
m1r1 + m2 r2 + ............... + mi ri + .................... + mn rn = Mrc
If M = Smi denotes total mass of the system, the above two equations can be written in short as
r r
Smi v i = Mv c (1)
r r
Smi ri = Mrc (2)
The above equation suggests location of mass center of a system of discreet particles.
r r r r r
r m r + m2 r2 + ............... + mi ri + .................... + mn rn Smi ri
rc = 1 1 = (3)
M M
r
Cartesian coordinate (xc, yc, zc) of the mass center are components of the position vector rc of the mass center..

S mi x i S mi y i S mi z i
xc = ; yc = ; zc = (4)
M M M

Illustration 1. Center of Mass of Two Particle System


(a) Find expression of position vector of mass center of a system of two particles of masses m 1 and
r r
m2 located at position vectors r1 and r2 .
(b) Express Cartesian coordinates of mass center, if particle m1 at point (x1, y1) and particle m2 at
point (x2, y2).
(c) If you assume origin of your coordinate system at the mass center, what you conclude regarding
251\D\Allen(IIT-JEE Wing)\2020-21\Nurture\Phy\Unit-03\01-Centre of Mass

location of the mass center relative to particles.


(d) Now find location of mass center of a system of two particles masses m 1 and m2 separated by
distance r.
r r
Solution. (a) Consider two particles of masses m1 and m2 located at position vectors r1 and r2 . Let their
r
mass center C at position vector rc .
From eq. , we have
r r r
r Smi ri r m1r1 + m2 r2
rc = ® rc =
M m1 + m2

3
JE E-Physics
(b) From result obtained in part (a), we have
m1 x1 + m2 x2 m y + m2 y 2
xc = and yc = 1 1
m1 + m2 m1 + m2
r
(c) If we assume origin at the mass center vector rc vanishes and we have
r r r
m1r1 + m2 r2 = 0 y
m1
Since either of the masses m1 and m2 cannot be negative, to r
r r r1
satisfy the above equation, vectors r1 and r2 must have opposite
C x
signs. It is geometrically possible only when mass center C lies r
r2
between the two particles on the line joining them as shown in m2
the figure.
r r
If we substitute magnitudes r1 and r2 of vectors r1 and r2 in the above equation, we have m1r1
= m2r2, which suggest
r1 m2
=
r2 m1
Now we conclude that mass center of two particle system lies between the two particles on the
line joining them and divide the distance between them in inverse ratio of masses of the
particles.

(d) Consider two particles masses m1 and m2 at distance r from each other. There mass center C
must lie in between them on the line joining them. Let distances of these particles from the mass
center are r1 and r2.
r
r1 r2

m1 C m2

Since mass center of two particle system lies between the two particles on the line joining them
and divide the distance between them in inverse ratio of masses of the particles, we can write
m2 r m1r
r1 = and r2 =
m1 + m2 m1 + m2

Illustration 2. Mass centre of several particles


Find position vectors of mass center of a system of three particle of masses 1 kg, 2 kg and 3 kg
r r r
located at position vectors r1 = ( 4iˆ + 2ˆj - 3kˆ ) m, r2 = ( iˆ - 4ˆj + 2kˆ ) m and r3 = ( 2iˆ - 2ˆj + kˆ ) m
respectively.
Solution. From eq. , we have
r
r Smi ri r 1 ( 4ˆi + 2 ˆj - 3kˆ ) + 2 ( ˆi - 4 ˆj + 2kˆ ) + 3 ( 2ˆi - 2 ˆj + kˆ ) 2
rc = ® rc = = 2iˆ - 2ˆj + kˆ
M 1+2+3 3
Center of Mass of an Extended Body or Continuous Distribution of Mass
An extended body is collection of infinitely large number of particles so closely located that we
neglect separation between them and assume the body as a continuous distribution of mass. A rigid
251\D\Allen(IIT-JEE Wing)\2020-21\Nurture\Phy\Unit-03\01-Centre of Mass

body is an extended body in which relative locations of all the particles remain unchanged under all
circumstances. Therefore a rigid body does not get deformed under any circumstances.
Let an extended body is shown as a continuous distribution of mass by the shaded object in the
figure. Consider an infinitely small portion of mass dm of this body. It is called a mass element and
is shown at position given by position vector rr . Total mass M of the body is M = ò dm . The mass
r
center C is assumed at position given by position vector rc . Position vector of centre of mass of
such a body is given by the following equation.

4
Centre of Mass
r y
r ò rdm
rc = (5)
M dm
Cartesian coordinate (xC, yC, zC) of the mass center are
r r
components of the position vector rc of the mass center..
C
rc
xc =
ò xdm ; yc =
ò ydm ; zc =
ò zdm (6)
M M M
O x
z

Illustration 3. Mass centre of uniform symmetrical bodies.


Show that mass center of uniform and symmetric mass distributions lies on axis of symmetry.
Solution. For simplicity first consider a system of two identical particles and then extend the idea obtained to
a straight uniform rod, uniform symmetric plates and uniform symmetric solid objects.
Mass Center of a system of two identical particles r
Mass center of a system of two identical particles lies at the r/2 r/2
midpoint between them on the lie joining them. A B
Mass Center of a system of a straight uniform rod m C m
dm C dm
Consider two identical particles A and B at equal distances
from the center C of the rod. Mass center of system these two particles is at C. The whole rod can
be assumed to be made of large number of such systems each having its mass center at the mid
point C of the rod. Therefore mass center of the whole rod must be at its mid point.
Mass Center of a system of a uniform symmetric curved rod
A B
Consider two identical particles A and B located at equal distances
C
from the line of symmetry. Mass center of system these two particles
is at C. The whole rod can be assumed to be made of large number
of such systems each having its mass center at the mid point C of
the joining them. Therefore mass center of the whole rod must be on Line of symmetry
the axis of symmetry. Line of symmetry

Mass Center of a uniform plate (lamina)


A B
Consider a symmetric uniform plate. It can be assumed composed C
of several thin uniform parallel rods like rod AB shown in the figure.
All of these rods have mass center on the line of symmetry, therefore
the whole lamina has its mass center on the line of symmetry.

Mass Center of a uniform symmetric solid object Line of symmetry


A uniform symmetric solid object occupies a volume that is made
by rotating a symmetric area about its line of symmetry though
180º. Consider a uniform symmetric solid object shown in the figure.
It can be assumed composed of several thin uniform parallel disks
shown in the figure. All of these disks have mass center on the line C
of symmetry, therefore the whole solid object has its mass center on
the line of symmetry.

Mass Center of uniform bodies


Following the similar reasoning, it can be shown that mass center of uniform bodies lies on their
geometric centers.
251\D\Allen(IIT-JEE Wing)\2020-21\Nurture\Phy\Unit-03\01-Centre of Mass

Illustration 4. Mass Center of a system of a segment of a uniform circular rod (arc)


Find location of mass center of a thin uniform rod bent into shape of an arc.
Solution. Consider a thin rod of uniform line mass density l y
(mass per unit length) and radius r subtending angle A
r r
2q on its center O.
The angle bisector OP is the line of symmetry, and da
a P
2q rcosa
mass center lies on it. Therefore if we assume the angle O x
bisector as one of the coordinate axes say x-axis,
y-coordinate of mass center becomes zero. B

5
JE E-Physics
Let two very small segments A and B located symmetric to the line of symmetry (x-axis). Mass
center of these two segments is on P at a distance x = r cos a from center O. Total mass of these
two elements is dm = 2lrda. Now using eq. , we have
q
ò xdm ® ò ( r cos a ) ( 2l rd a ) r sin q
xc = xc = -q =
M lrq q
Mass center of a thin uniform arc shaped rod of radius r subtending angle 2q at the center lies
on its angle bisector at distance OC from the center.
r sin q
OC =
q

Illustration 5. Find coordinates of mass center of a quarter ring of radius r placed in the first quadrant of a
Cartesian coordinate system, with centre at origin.
y
Solution. Making use of the result obtained in the previous example, distance

r sin (p / 4) 2 2r yc C
OC of the mass center from the center is OC = =
p /4 p
p/4
æ 2r 2r ö xc
Coordinates of the mass center (x c, yc) are çè , ÷ O x
p pø

Illustration 6. Find coordinates of mass center of a semicircular ring of radius r placed symmetric to the y-axis of
a Cartesian coordinate system.
Solution. The y-axis is the line of symmetry, therefore mass center of the ring
lies on it making x-coordinate zero. y

Distance OC of mass center from center is given by the result


C
obtained in Illustration 4. Making use of this result, we have
yc
r sin q r sin ( p / 2) 2r p/2
OC = ® yc = =
q p /2 p O x

Illustration 7. Mass Center of a sector of a uniform circular plate


Find location of mass center of a sector of a thin uniform plate.
Solution. Consider a sector of a thin uniform plate of surface mass density mass density s (mass per unit
area) and radius r subtending angle 2q on its center.
y
dr
R A

r
2q
O x

Let a thin arc of radius r and width dr be an infinitely small part of the sector. Mass dm of the arc
AB equals to product of mass per unit area and area of the arc.
dm = s ( 2rq dr ) = 2s r q dr
251\D\Allen(IIT-JEE Wing)\2020-21\Nurture\Phy\Unit-03\01-Centre of Mass

Due to symmetry mass center of this arc must be on the angle bisector i.e. on x-axis at distance
r sin q
x= .
q
Using above two information in eq. , we obtain the mass center of the sector.

r sin q ö
Ræ Ræ r sin q ö
xc =
ò xdm ® xc =
ò
0 è
ç
q ø
÷ ( 2srqdr )
=
ò 0
ç
è q ø
÷ ( 2srqdr ) 2r sin q
=
M s ( Area of the sector ) srq2 3q

6
Centre of Mass
Illustration 8. Find coordinates of mass center of a quarter sector of a uniform disk of radius r placed in the first
quadrant of a Cartesian coordinate system with centre at origin.
Solution. Making use of the result obtained in the previous example, distance OC y
of the mass center from the center is
yc C
2r sin ( p / 4) 4 2r
OC = =
3p / 4 3p p/4
O xc x
æ 4r 4r ö
Coordinates of the mass center (x c, yc) are çè , ÷
3p 3p ø

Illustration 9. Find coordinates of mass center of a uniform semicircular plate of radius r placed symmetric to the
y-axis of a Cartesian coordinate system, with centre at origin.
Solution. The y-axis is the line of symmetry, therefore mass center of the plate y
lies on it making x-coordinate zero.
Distance OC of mass center from center is given by the result obtained C
in Illustration 7. Making use of this result, we have yc
p/2
2r sin q 2r sin ( p / 2) 4r
OC = ® yc = = O x
3q 3p / 2 3p

Illustration 10. Find coordinates of mass center of a non-uniform rod of length L whose linear mass density l
varies as l=(ax + b), where x is the distance from the lighter end.
Solution. Assume the rod lies along the x-axis with its lighter end on the y
origin to make mass distribution equation consistent with dm = ldx
coordinate system. x
O xc
Making use of eq. , we have dx x=L
L L

ò xdm ® x c =
ò x ldx = ò x (ax + b ) dx = (2aL + 3b ) L
0 0
xc = L L
( )
M ò ldx ò (ax + b ) dx 3 aL + 2b
0 0

Illustration 11. Mass Center of composite bodies


A composite body is made of joining two or more bodies. Find mass center of the following composite
body made by joining a uniform disk of radius r and a uniform square plate of side r of the same
mass per unit area.
Solution. To find mass center the component bodies are assumed particle
y
of masses equal to corresponding bodies located on their
respective mass centers. Then we use equation to find coordinates
of the mass center of the composite body. O x
To find mass center of the composite body, we first have to
calculate masses of the bodies, because their mass distribution is
given.
If we denote surface mass density (mass per unit area) by s, masses of the bodies are
Mass of the disk md = Mass per unit area ´ Area = s ( p r 2 ) = sp r 2

Mass of the square plate mp = Mass per unit area ´ Area = s ( r 2 ) = s r 2


Location of mass center of the disk x d = Center of the disk = r and y d = 0
251\D\Allen(IIT-JEE Wing)\2020-21\Nurture\Phy\Unit-03\01-Centre of Mass

Location of mass center of the square plate x p = Center of the surface plate = 3r and y d = 0
Using eq. , we obtain coordinates (x c, yc) of the composite body.
md x d + m p x p r ( p + 3) md x d + m p x p
xc = = and y c = =0
md + m p ( p + 1) md + mp

æ r ( p + 3) ö
Coordinates of the mass center are çè ( , 0÷
p + 1) ø

7
JE E-Physics

Illustration 12. Mass Center of truncated bodies y


A truncated body is made by removing a portion of a body. Find
mass center of the following truncated disk made by removing disk
of radius equal to half of the original disk as shown in the figure. O x

Radius of the original uniform disk is r.

Solution. To find mass center of truncated bodies we can make use of superposition principle that is, if we
add the removed portion in the same place we obtain the original body. The idea is illustrated in the
following figure.

y y y

O x O x O x

The removed portion is added to the truncated body keeping their location unchanged relative to
the coordinate frame.
Denoting masses of the truncated body, removed portion and original body by mtb, mrp and mob
and location of their mass centers by xtb, xrp and xob, we can write mtb xtb + mrb x rp = mob xob
From the above equation we obtain position co-ordinate xtb of the mass center of the truncated
body.
mob x ob - mrb x rp
xtb = (1)
mtb
Denoting mass per unit area by s , we can express the masses mtb, mrp and mob.
ì æ r 2 ö ü 3sp r 2
Mass of truncated body mtb = s íp ç r 2 - ÷ ý =
î è 4 øþ 4

sp r 2
Mass of the removed portion mrp =
4
Mass of the original body mob = sp r 2
Mass center of the truncated body x tb

r
Mass center of the removed portion x rp =
2
Mass center of the original body x ob = 0
Substituting the above values in equation (1), we obtain the mass the center of the truncated
body.

(sp r 2 ) ´ 0 - æç sp r
öærö 2
251\D\Allen(IIT-JEE Wing)\2020-21\Nurture\Phy\Unit-03\01-Centre of Mass

mob xob - mrb x rp è 4 ÷ø çè 2 ÷ø r


xtb = = =-
mtb 3sp r 2
6
4

æ r ö
Mass center of the truncated body is at point çè - , 0÷ø
6

8
Centre of Mass

Calculation of Centre of Mass


1. The distance of centre of mass of solid cone from its base is–
h 3h h 2h
(A) (B) (C) (D)
4 4 3 3

2. Two point mass 36 g and 72 g are located at coordinates (2 cm, 0) and (5 cm, 0). The coordinate of centre
of mass will be at
(A) (4, 0) (B) (4.5, 0) (C) (3.5, 0) (D) (3, 0)

3. An infinite number of bricks are placed one over the other as shown in the figure. Each succeeding brick having
half the length and breadth of its preceding brick and the mass of each succeeding bricks being
1/4th of the preceding one, take ‘O’ as the origin, the x-coordinate of centre of mass of the system of bricks is:-

a 3a 3a 2a
(A) - (B) (C) - (D) -
7 7 7 7

4. From the uniform disc of radius R, an equilateral triangle of side 3 R is


cut as shown in the figure. The new position of centre of mass is :
(A) (0,0) (B) (0, R)

æ 3Rö
(C) çç 0, 2 ÷÷ (D) None of these
è ø

5 A hemisphere and a solid cone of same density have a common base. The centre of mass of the common
structure coincides with the centre of the common base. If R is the radius of hemisphere and h is height of
the cone, then :

h h 1 h h 1
(A) = 3 (B) = (C) =3 (D) =
R R 3 R R 3

6. A uniform metal disc of radius R is taken and out of it a disc of diameter R is cut off from the end. The centre
of mass of the remaining part will be:

R R
(A) from the centre (B) from the centre
4 3
251\D\Allen(IIT-JEE Wing)\2020-21\Nurture\Phy\Unit-03\01-Centre of Mass

R R
(C) from the centre (D) from the centre
5 6

7. The centre of mass of a system of two particles divides the distance between them
(A) In inverse ratio of square of masses of particles
(B) In direct ratio of square of masses of particles
(C) In inverse ratio of masses of particles
(D) In direct ratio of masses of particles
9
JE E-Physics

8. Find the position of centre of mass of the uniform planner section shown in
figure with respect to the origin (O)

9. Seven homogeneous bricks, each of length L, are arranged as shown


in figure. Each brick is displaced with respect to the one in contact
by L/10. Find the x-coordinate of the centre of mass relative to the
origin O shown.
O X

10. A uniform disc of radius R is put over another uniform disc of radius 2R of the same thickness and density.
The peripheries of the two discs touch each other. Locate the centre of mass of the system from the centre
of large disc.

1.2 Center of Mass Frame of Reference or Centroidal Frame

Center of mass frame of reference or centroidal frame is reference frame assume attached with the mass
center of the system at its origin. It moves together with the mass center.
It is a special frame and presents simple interpretations and solutions to several phenomena. Let us first
discuss some of its fundamental properties.
In centroidal frame center of mass is assumed at the origin, therefore position vector, velocity and
acceleration of the mass center in centroidal frame all become zero.
• Sum of mass moments in centroidal frame vanishes.
Mass moment of a particle is product of mass of the particle and its position vector.
r r r r r r r
Smi ri / c = 0 or m1r1 / c + m2 r2 / c + ............... + mi ri / c + .................... + mn rn / c = 0 (7)
• Total linear momentum of the system in centroidal frame vanishes..
r r r r r r r
Smi v i / c = 0 or m1v 1 / c + m2v 2 / c + ............... + mi v i / c + .................... + mn v n / c = 0 (8)

Illustration 13. Motion of Mass Center in One Dimension


A jeep of mass 2400 kg is moving along a straight stretch of road at 80 km/h. It is followed by a car
of mass 1600 kg moving at 60 km/h.
(a) How fast is the center of mass of the two cars moving?
(b) Find velocities of both the vehicles in centroidal frame.
r r
r m jeep v jeep + mcar v car
Solution. (a) Velocity of the mass center v c =
m jeep + mcar
251\D\Allen(IIT-JEE Wing)\2020-21\Nurture\Phy\Unit-03\01-Centre of Mass

Assuming direction of motion in the positive x-direction, we have


r r
r m jeep v jeep + mcar v car r 2400 ´ 80 + 1600 ´ 60
vc = ® vc = = 72 km/h
m jeep + mcar 2400 + 1600
(b) Velocity of the jeep in centroidal frame
v jeep / c = 80 - 72 = 8 km/h in positive x-direction.

Velocity of the car in centroidal frame v car / c = 60 - 72 = -12 km/h


12 km/h negative x-direction direction.
10
Centre of Mass
Illustration 14. Motion of Mass Center in Vector Form
r
A 2.0 kg particle has a velocity of v = ( 2.0iˆ - 3.0ˆj ) m/s, and a 3.0 kg particle has a velocity
1

r
v 2 = (1.0iˆ + 6.0 ˆj ) m/s.
(a) How fast is the center of mass of the particle system moving?
(b) Find velocities of both the particles in centroidal frame.
r r
Solution. (a) Velocity of the mass center vrc = m1v 1 + m2v 2
m1 + m2
r r
r m v + m2v 2 r 2 ( 2.0iˆ - 3.0ˆj ) + 3 (1.0iˆ + 6.0ˆj )
vc = 1 1
m1 + m2 ® vc = = (1.4iˆ + 2.4 ˆj ) m/s
2+3
(b) Velocity of the first particle in centroidal frame
r r r r
v 1 / c = v 1 - v c ® v 1 / c = ( 2.0iˆ - 3.0ˆj ) - (1.4ˆi + 2.4ˆj ) = 0.6iˆ - 5.4ˆj m/s
Velocity of the second particle in centroidal frame
r r r r
v 2 / c = v 1 - v c ® v 2 / c = (1.0iˆ + 6.0ˆj ) - (1.4ˆi + 2.4ˆj ) = - ( 0.4ˆi + 3.6ˆj ) m/s

1.3 Application of Newton’s Laws of Motion to a System of Par ticles

y r y
Fi
mi mi
r
mi ai
r
fij

O x O x

z z

In order to write equation of motion for a system of particles, we begin by applying Newton’s second law to an
individual particle.
r
Consider ith particle of mass mi. Internal force applied on it by j th particle is shown by fij . Other particles of the
system may also apply internal forces on it. One of them is shown in the figure by an unlabeled vector. In
addition to these internal forces, external forces may also be applied on it by bodies out side the system.
r
Resultant of all these external forces is shown by vector Fi . If under the action of these forces this particles has
r
acceleration ai relative to an inertial frame Oxyz, its free body diagram and kinetic diagram can be represented
by the following figure and Newton’s second law can be written by the following equation.
r r r
Fi + å fij = mi ai
In similar fashion, we can write Newton’s second law for all the particles of the system. These equations are
r r r
251\D\Allen(IIT-JEE Wing)\2020-21\Nurture\Phy\Unit-03\01-Centre of Mass

For 1st particle F1 + å f1 j = m1a1


r r r
For 2nd particle F2 + å f2 j = m2 a2
..................... .....................
r r r
For ith particle Fi + å fij = mi ai
..................... .....................
r r r
For nth particle Fn + å fnj = mn an

11
JE E-Physics
r r
Every internal force fij on particle mi due to particle mj and f ji on the particle mj due to particle mi constituting
Newton’s third law pair must be equal in magnitude and opposite in direction, therefore the sum all these
internal forces for all the particles must be zero. Keeping this fact in mind and denoting the mass of the whole
r
system by M and acceleration of the mass center C by aC relative to the inertial frame, Newton’s second law
representing translation motion of the system of particles particle can be represented by the following equation.
r r r
å Fi = å ( mi ai ) = MaC
r r
r dpc
å iF = Ma C =
dt

Illustration 15. Newton’ Laws of Motion and System of Particles


A ladder of mass 20 kg is hanging from ceiling as shown in figure. Three
men A, B and C of masses 40 kg, 60 kg, and 50 kg are climbing the
ladder. Man A is climbing with upward retardation 2 m/s 2, B is climbing A
up with a constant speed of 0.5 m/s and C is climbing with upward
acceleration of 1 m/s2. Find the tension in the string supporting the ladder.
Solution. External forces acting on the system are weights of the men, weight of B
the ladder and tension supporting the ladder. Denoting masses of men
A, B, C and ladder by mA, mB, mC and mL, acceleration due to gravity
by g , tension in the string by T and accelerations of the men A, B, C C
and ladder by aA, aB, aC and aL respectively, we can write the following
equation according to equation .
r r
SFi = S ( mi ai ) ® T - m A g - mB g - mC g - mL g = mA aA + mB aA + mC aA + mL a A
Substituting given values of masses mA = 40 kg, mB = 60 kg, mC = 50 kg, mL = 20 kg,
given values of accelerations g = 10 m/s2, a A = -2 m/s2, aB = 0 m/s2, aC = 1 m/s2, and
aL = 0 m/s2,
we obtain T - 400 - 600 - 500 - 200 = -80 + 0 + 50 + 0
T = 1670 N

Illustration 16. Simple Atwood Machine as System of Particles


The system shown in the figure is known as simple Atwood machine.
Initially the masses are held at rest and then let free. Assuming mass m 2
more than the mass m1, find acceleration of mass center and tension in
the string supporting the pulley.
m1 m2
Solution. We know that accelerations a 1 and a2 are given by the following equations.
m2 - m1 m2 - m1
a2 = g and a1 = m + m g ­
m2 + m1 ¯ 2 1

Making use of eq. , we can find acceleration a C of the mass center. We denote upward direction
positive and downward direction negative signs respectively.
r r
MaC = S ( mi ai ) ® ( m1 + m2 ) ac = m1a1 - m2a2
251\D\Allen(IIT-JEE Wing)\2020-21\Nurture\Phy\Unit-03\01-Centre of Mass

Substituting values of accelerations a1 and a2, we obtain


2m1m2 - ( m12 + m22 )
aC = g
( m1 + m2 )2
To find tension T in the string supporting the pulley, we again use eq. (9)
r r
SFi = MaC ® T - m1 g - m2 g = ( m1 + m2 ) ac
Substituting expression obtained for aC, we have
4m1 m2
T = g
m1 + m2
12
Centre of Mass
Illustration 17. Two blocks each of mass m, connected by an un-stretched spring are kept at rest on a frictionless
horizontal surface. A constant force F is applied on one of the blocks pulling it away from the other
as shown in figure.
F
A B

(a) Find acceleration of the mass center.


(b) Find the displacement of the centre of mass as function of time t.
(c) If the extension of the spring is xo at an instant t, find the displacements of the two blocks
relative to the ground at this instant.
Solution. (a) Forces in vertical direction on the system are weights of the blocks and normal reaction from
the ground. They balance themselves and have no net resultant. The only external force on the
system is the applied force F in the horizontal direction towards the right.
r r
SFi = MaC ® F = ( m + m ) ac

F
ac = towards right
2m
(b) The mass center moves with constant acceleration, therefore it displacement in time t is given
by equation of constant acceleration motion.
Ft 2
x = ut + 12 at 2 ® x c =
4m
(c) Positions xA and xB of particles A and B forming a system and position xC mass center are
obtained by following eq.
r r
Mrc = Smi ri
x A + xB
Substituting values wee obtain 2mxc = mx A + mx B xc =
2

Ft 2
Now using result obtained in part (b), we have x A + xB =
2m
Extension in the spring at this instant is x o = x B - x A
From the above two equations, we have

1 æ Ft 2 ö 1 æ Ft 2 ö
xA = ç - x 0 ÷ and x B = ç + x0 ÷
2 è 2m ø 2 è 2m ø

Motion of Centre of Mass


1. If no external force is applied on a system then choose incorrect statement–
(A) Velocity of centre of mass may be zero (B) Velocity of centre of mass may not be zero
(C) Acceleration of centre of mass is zero (D) Acceleration of centre of mass may not be zero

2. Two objects of masses 200 gm and 500 gm possess velocities 10iˆ m/s and ( 3iˆ + 5ˆj ) m/s respectively. The
velocity of their centre of mass in m/s is–
251\D\Allen(IIT-JEE Wing)\2020-21\Nurture\Phy\Unit-03\01-Centre of Mass

5ˆ 25 ˆ 5
(A) 5iˆ - 25ˆj (B) i - 25ˆj (C) 5iˆ + j (D) 5iˆ - ˆj
7 7 7

3. Two particles bearing mass ratio n : 1 are interconnected by a light inextensible string that passes over a
smooth pulley. If the system is released, then the acceleration of the centre of mass of the system is :
2 2
æ n + 1ö æ n - 1ö æ n + 1ö
(A) (n – 1)2 g (B) ç g (C) ç g (D) ç g
è n - 1÷ø è n + 1÷ø è n - 1÷ø

13
JE E-Physics
4. Consider
r a system of two identical particles. One of the particles is at rest and the other has an acceleration
a . The centre of mass has an acceleration.
1 r r r
(A) zero (B) a (C) a (D) 2 a
2

5. Consider a thin stick of length L, standing on one of its ends on a frictionless


surface. It is slightly pushed at the other end of the rod. Then, path of
centre of mass of the rod is :
L2
(A) x = 0 (B) x2 + y2 =
4
x2 y2
(C) y = 0 (D) 2
+ =1
L /4 L2

6. X and Y components of acceleration of C. M. are :


m1m 2g m1m 2 g
(A) (a CM )X = (B) (a CM )X =
m1 + m 2 (m1 + m 2 )2
2
æ m2 ö æ m2 ö
(C) (a CM )Y = ç ÷ g (D) (a CM )Y = ç m + m ÷ g
è m1 + m 2 ø è 1 2 ø

m
7. Two particles of equal masses have velocities V1 = 2 î m/s .The first particle has an acceleration a1 = (3 î + 3 ĵ ) .
s2
while the acceleration of the other particle is zero. The centre of mass of the particles moves in a :
(A) circle (B) parabola (C) straight line (D) ellipse

8. Two blocks A and B are connected by a massless string (shown in B


figure). A force of 30 N is applied on block B. The distance travelled A
10 kg 20 kg F = 30N
by centre of mass in 2 s starting from rest is
Smooth

2. 0 IMPULSE OF A FORCE

Net force applied on a rigid body changes momentum i.e. amount of motion of that body. A net force for a
longer duration cause more change in momentum than the same force acting for shorter duration. Therefore
duration in which a force acts on a body together with magnitude and direction of the force decide effect of the
force on the change in momentum of the body.
Linear impulse or simply impulse of a force is defined as integral of the force with respect to time.
r
If a force F acts on a body, its impulse in a time interval from t i to tf is given by the following equation.
r tf r
Imp = ò
ti
Fdt
r
If the force is constant, its impulse equals to product of the force vector F and time interval Dt.
r r F
Imp = F ( Dt )
For one-dimensional force, impulse equals to area between force-time
251\D\Allen(IIT-JEE Wing)\2020-21\Nurture\Phy\Unit-03\01-Centre of Mass

graph and the time axis. In the given figure is shown how a force F along
x-axis varies with time t. Impulse of this force in time interval ti to tf
ti tf t
equals to area of the shaded portion.
r r r r r
If several forces F1 , F2 , F3 ..... Fn act on a body in a time interval, the total impulse I mp of all these forces
equals to impulse of the net force.
r tf r tf r tf r tfr r r
Im p = ò ti
F1 dt + ò ti
F2 dt + .............. + ò ti
Fn dt = ò (F
ti
1 + F2 + ............. + Fn ) dt

Impulse is measured in newton-second.


Dimensions of impulse are MLT-1
14
Centre of Mass

r
Illustration 18. Calculate impulse of force F = ( 3t 2ˆi - ( 2t - 1)ˆj + 2kˆ ) N over the time interval from t = 1 s to
t = 3 s.
r tf r r
( 3t 2ˆi - ( 2t - 1)ˆj + 2kˆ ) dt
3
Solution. Imp = ò Fdt ® Imp =
t i ò1

= éë t 3ˆi - ( t 2 - t ) ˆj + 2tkˆ ùû1 = ˆi [ t 3 ]1 - ˆj éë( t 2 - t )ùû1 + kˆ [ 2t ]1 = ( 26iˆ - 6ˆj + 4kˆ ) N-s


3 3 3 3

Illustration 19. A one-dimensional force F varies with time according to the given F (N)
graph. Calculate impulse of the force in following time intervals. 10

(a) From t = 0 s to t = 10 s.
5 10 15 t (s)
(b) From t = 10 s to t = 15 s.
(c) From t = 0 s to t = 15 s.

Solution. For one-dimensional force, impulse equals to area between force-


F (N)
time graph and the time axis. A B
10
(a) I0®10 = Area of trapezium OABC = 75 N-s
C E
(b) I10®15 = – Area of triangle CDE = – 25 N-s O
5 10 15 t (s)
(c) I0®15 = Area of trapezium OABC – Area of triangle CDE = 50 N-s D

2.1 Impulse Momentum Principle

Consider body of mass m in translational motion. When it is moving with velocity vr , net external force acting
r
on it is F . Equation of motion as suggested by Newton’s second law can be written in the form m
r r
Fdt = d(mv)
r r
If the force acts during time interval from ti to tf and velocity of the body changes from v i to v f , integrating the
above equation with time over the interval from t i to tf, we have
tf r r r
ò ti
Fdt = mv f - mv i
r r
Here left hand side of the above equation is impulse Imp of the net force F in time interval from t i to tf, and
r r
quantities mv i and mv f on the right hand side are linear momenta of the particle at instants t i and tf. If we
r r
denote them by symbols pi and pf , the above equation can be written as
r r r
Imp = p f - pi
The idea expressed by the above equation is known as impulse momentum principle. It states that change in
the momentum of a body in a time interval equals to the impulse of the net force acting on the body during the
concerned time interval.
For the ease of application to physical situations the above equation is rearranged as
r r r
251\D\Allen(IIT-JEE Wing)\2020-21\Nurture\Phy\Unit-03\01-Centre of Mass

pi + I mp = p f
This equation states that impulse of a force during a time interval when added to momentum of a body at the
beginning of an interval of time we get momentum of the body at the end of the interval concerned.
Since impulse and momentum both are vector quantities, the impulse momentum theorem can be expressed
by there scalar equations making use of Cartesian components.
p1x + å I mp,x = p2 x p1y + å I mp,y = p2 y p1 z + å I mp,z = p2 z
The impulse momentum principle is deduced here for a single body moving relative to an inertial frame,
therefore impulses of only physical forces are considered. If we are using a non-inertial reference frame, impulse
of corresponding pseudo force must also be considered in addition to impulse of the physical forces.
15
JE E-Physics
How to apply Impulse Momentum Principle

The impulse momentum principle is deduced here for a single body, therefore it is recommended at present to
use it for a single body. To use this principle the following steps should be followed.
r r
(i) Identify the initial and final positions as position 1 and 2 and show momenta p1 and p2 of the body
at these instants.
(ii) Show impulse of each force acting on the body at an instant between positions 1 and 2.
r r r
(iii) Use the impulse obtained in step (ii) and momenta obtained in step (i) into equation pi + I mp = p f .
r
Consider a particle moving with momentum p1 in beginning. It is acted upon by two forces, whose impulses in
r r
a time interval are I mp1 and I mp 2 . As a result, at the end of the time interval, momentum of the particle
r
becomes p2 . This physical situation is shown in the following diagram. Such a diagram is known as impulse
momentum diagram.
r
I mp1
r
p2
r
p1 r
r I mp1
r p1 r
I mp 2 I mp 2

r
Illustration 20. A particle of mass 2 kg is moving with velocity v o = ( 2iˆ - 3ˆj ) m/s in free space. Find its velocity 3
r
s after a constant force F = ( 3iˆ + 4 ˆj ) N starts acting on it.
r r r r r r
Solution. pf = pi + I mp ® mv f = mv o + F Dt
Substituting given values, we have
r
2v f = 2 ( 2iˆ - 3ˆj ) + ( 3iˆ + 4 ˆj ) ´ 3 = 13iˆ + 6ˆj
r
v f = ( 6.5iˆ + 3ˆj ) m/s
F
Illustration 21. A box of mass m = 2 kg resting on a frictionless horizontal 20 N
ground is acted upon by a horizontal force F, which varies as 10 N
shown. Find speed of the particle when the force ceases to act.
r r r r r tf r
Solution. pf = pi + I mp ® mv f = mv i + ò Fdt ti
2s 4s t

2v = 2 ´ 0 + 12 ´ 20 ´ 4
v = 20 m/s

Illustration 22. Two boxes A and B of masses m and M interconnected by an ideal rope
and ideal pulleys are held at rest as shown. When it is released, box B
accelerates downwards. Find velocities of box A and B as function of
time t after system has been released.
251\D\Allen(IIT-JEE Wing)\2020-21\Nurture\Phy\Unit-03\01-Centre of Mass

A B

Solution. We first explore relation between accelerations aA and aB of the

boxes A and B, which can be written either by using constrained

relation or method of virtual work or by inspection. T T


T
vA = 2vB ...(i) vA vB
A B

16
Centre of Mass

Applying impulse momentum principle to box A y


Tt
mv
p2 y = p1y + å Imp,y ® Mv A = 0 + Tt - mgt ...(ii) zero x A

A
mgt
Applying impulse momentum principle to box B
y
p2 y = p1y + å Imp,y ® mv B = 0 + Mgt - 2Tt ...(iii)
2Tt x
zero

From equations (i), (ii) and (iii), we have B


MaB
æ M - 2m ö æ M - 2m ö Mgt
v A = 2ç gt and v B = ç gt
è M + 4m ÷ø è M + 4m ÷ø

2.2 Impulsive Motion

Sometimes a very large force acts for a very short time interval on a
particle and produces finite change in momentum. Such a force is known as
impulsive force and the resulting motion as impulsive motion. When a batsman
hits a ball by bat, the contact between the ball and the bat lasts for a very
small duration Dt, but the average value of the force F exerted by the bat on
the ball is very large, and the resulting impulse FDt is large enough to change
momentum of the ball.
During an impulsive motion, some other forces of magnitudes very small in comparison to that of an impulsive
force may also act. Due to negligible time interval of the impulsive motion, impulse of these forces becomes
negligible. These forces are known as non-impulsive forces. Effect of non-impulsive forces during an impulsive
motion is so small that they are neglected in analyzing impulsive motion of infinitely small duration.
Non-impulsive forces are of finite magnitude and include weight of a body, spring force or any other force of
finite magnitude. When duration of the impulsive motion is specified, care has to be taken in neglecting any of
the non-impulsive force. In analyzing motion of the ball for very small contact duration (usually in mili-
seconds), impulse of the weight of the ball has to be neglected. Unknown reaction forces may be impulsive or
non-impulsive; their impulse must therefore be included.

Illustration 23. A 100 gm ball moving horizontally with 20 m/s is struck by a bat,
as a result it starts moving with a speed of 35 m/s at an angle of 35 m/s
37° above the horizontal in the same vertical plane as shown in
the figure. 20 m/s
(a) Find the average force exerted by the bat if duration of impact is 0.30 s.
(b) Find the average force exerted by the bat if duration of impact is 0.03 s.
(c) Find the average force exerted by the bat if duration of impact is 0.003 s.
(d) What do you conclude for impulse of weight of the ball as duration of contact decreases?
Solution. The impulse momentum diagram of the ball is shown in the figure below. Here F, mg, and Dt
represent the average value of the force exerted by the bat, weight of the ball and the time interval.

y Pf = 3.5
mgDt = 1.0Dt
251\D\Allen(IIT-JEE Wing)\2020-21\Nurture\Phy\Unit-03\01-Centre of Mass

pix = -2 x pfy = 2.1


37°

pfx = 2.8
FDt
FxDt FyDt

Applying principle of impulse and momentum in x- direction, we have


pfx = pix + å I mp,x ® -2 + Fx Dt = 2.8

17
JE E-Physics

4.8
Fx = N ...(i)
Dt
Applying principle of impulse and momentum in y- direction, we have
pfy = piy + å I mp ,y ® 0.0 + Fy Dt - 1.0Dt = 2.1

æ 2.1 ö
Fy = ç + 1.0÷ N ...(ii)
è Dt ø
r
(a) Substituting Dt = 0.30 s, in equations (i) and (ii), we find F = 16iˆ + 8ˆj N
r
(b) Substituting Dt = 0.03 s, in equations (i) and (ii), we find F = 160iˆ + 71ˆj N
r
(c) Substituting Dt = 0.003 s, in equations 1 and 2, we find F = 1600iˆ + 701jˆ N
(d) It is clear from the above results that as the duration of contact between the ball and the bat
decreases, effect of the weight of the ball also decreases as compared with that of the force of
the bat and for sufficiently short time interval, it can be neglected.

3.0 MOMENTUM AND KINETIC ENERGY

A moving particle possesses momentum as well as kinetic energy. If a particle of mass m is moving with velocity
v, magnitude of its momentum p and its kinetic energy K bear the following relation.

p2 1
K= = pv
2m 2

Illustration 24. An object is moving so that its kinetic energy is 150 J and the magnitude of its momentum is 30.0
kg-m/s. With what velocity is it traveling?

p2 1 2 ´ 150
Solution. K= = pv ® v = = 10.0 m/s
2m 2 30.0

3.1 Internal and external Forces and System of interacting Particles

Bodies applying forces on each other are known as interacting bodies. If we consider them as a system, the
forces, which they apply on each other, are known as internal forces and all other forces applied on them by
bodies not included in the system are known as external forces.
Consider two blocks A and B placed on a frictionless horizontal floor.

Their weights W1 and W2 are counterbalanced by normal reactions


N1 and N2 on each of them from the floor. Push F by the hand is A B
applied on A. The forces of normal reaction Non-A and Non-B constitute
Newton’s third law action-reaction pair, therefore are equal in magnitude W2
W1
251\D\Allen(IIT-JEE Wing)\2020-21\Nurture\Phy\Unit-03\01-Centre of Mass

and opposite in direction. Among these forces weights W1 and W2 A Non-A Non-B B
F
applied by the earth, normal reactions N1 and N2 applied by the
ground and the push F applied by the hand are external forces and
N1
normal reactions Non-A and Non-B are internal forces. N2
If the blocks are connected by a spring and the block A is either
pushed or pulled, the forces W1, W2, N1 and N2 still remain external
forces for the two block system and the forces, which the spring applies

18
Centre of Mass

on each other are the internal forces. Here force of gravitational


interaction between them being negligible has been neglected.
r
We can conceive a general model of two interacting particles. In the m1 F12 r
F21
figure is shown a system of two particles of masses m1 and m2. Particle m2
r
m1 attracts m2 with a force F12 and m2 attracts (or pulls) m1 with a
Particles attracting each other
r r r
force F21 . These forces F12 and F21 are the internal forces of this r
F12 m
1
two-particle system and are equal in magnitude and opposite in m2 r
F21
directions. Instead of attraction may repeal each other. Such a system
of two particles repealing each other is also shown. Particles repealing each other
In similar way we may conceive a model of a system of n interacting
particles having masses m1,m2,...mi....mj.....and mn respectively. The
r r
forces of interaction Fij and F ji between mi and mj are shown in the mi r mn
Fij
figure. Similar to these other particles may also interact with each r
m3
m1
other. These forces of mutual interaction between the particles are Fji
internal forces of the system. Any of the two interacting particles mj
m2
always apply equal and opposite forces on each other. Here for
r r
simplicity only the forces Fij and F ji are shown. System of n interacting particles.

3.2 Principle of Conservation of linear momentum

The principle of conservation of linear momentum or simply conservation of momentum for two or more
interacting bodies is one of guiding principles of the classical as well as the modern physics.
To understand this principle, we first discuss a system of two interacting bodies, and then extend the ideas
developed to a system consisting of many interacting bodies.
Consider a system of two particles of mass m 1 and m 2. Particle m1 F12 r
r r F21
m
m1 attracts m2 with a force F12 and m2 attracts m1 with a force F21 . 2

These forces have equal magnitudes and opposite directions as shown in Particles attracting each other
the figure. If the bodies are let free i.e. without any external force acting r
F12 m
on any of them, each of them move and gain momentum equal to the 1
m2 r
impulse of the force of interaction. Since equal and opposite interaction F21
forces act on both of them for the same time interval, the momenta
Particles repelling each other
gained by them are equal in magnitude and opposite in direction resulting
no change in total momentum of the system.
However, if an external force acts on any one of them or different forces with a nonzero resultant act on both
of them, the total momentum of the bodies will certainly change. If the system undergoes an impulsive motion,
total momentum will change only under the action of external impulsive force or forces. Internal impulsive
forces also exist in pairs of equal and opposite forces and cannot change the total momentum of the system.
Non-impulsive forces if act cannot change momentum of the system by appreciable amount. For example,
gravity is a non-impulsive force, therefore in the process of collision between two bodies near the earth the total
momentum remains conserved.
251\D\Allen(IIT-JEE Wing)\2020-21\Nurture\Phy\Unit-03\01-Centre of Mass

The total momentum of a system of two interacting bodies remains unchanged under the action of the forces
of interaction between them. It can change only if a net impulse of external force is applied.
In similar way we may conceive a model of a system of n interacting
particles having masses m1, m2, ....mi....mj.....and mn respectively. The mi r mn
r r Fij
forces of interaction Fij and F ji between mi and mj are shown in the m1 m
r 3

figure. Since internal forces exist in pairs of equal and opposite forces, in F ji
mj
any time interval of concern each of them have a finite impulse but their
m2
total impulse is zero. Thus if the system is let free, in any time interval
System of n interacting particles.

19
JE E-Physics

momentum of every individual particle changes but the total momentum


of the system remains constant.
It can change only if external forces are applied to some or all the particles. Under the action of external forces,
the change in total momentum of the system will be equal to the net impulse of all the external forces.
Thus, total momentum of a system of particles cannot change under the action of internal forces and if net
impulse of the external forces in a time interval is zero, the total momentum of the system in that time interval
will remain conserved.
r r
å pinitial = å pfinal
The above statement is known as the principle of conservation of momentum. It is applicable only when the
net impulse of all the external forces acting on a system of particles becomes zero in a finite time interval. It
happens in the following conditions.
• When no external force acts on any of the particles or bodies.
• When resultant of all the external forces acting on all the particles or bodies is zero.
• In impulsive motion, where time interval is negligibly small, the direction in which no impulsive forces act,
total component of momentum in that direction remains conserved.
Since force, impulse and momentum are vectors, component of momentum of a system in a particular direction
is conserved, if net impulse of all external forces in that direction vanishes.

Illustration 25. Two blocks of masses m and M are held against a


compressed spring on a frictionless horizontal floor with
the help of a light thread. When the thread is cut, the
M m
smaller block leaves the spring with a velocity u relative to
the larger block. Find the recoil velocity of the larger block.
Solution. When the thread is cut, the spring pushes both the block, and impart them momentum. The forces
applied by the spring on both the block are internal forces of the two-block system. External forces
acting on the system are weights and normal reactions on the blocks from the floor. These external
forces have zero net resultant of the system. In addition to this fact no external force acts on the
system in horizontal direction, therefore, horizontal component of the total momentum of the
system remains conserved. v u-v
Velocities of both the objects relative to the ground
(inertial frame) are shown in the adjoining figure. M m
Since before the thread is cut system was at rest, its
total momentum was zero. Principle of conservation
of momentum for the horizontal direction yields
n

åp
i =1
horizontal =0 ® -Mv + m(u - v ) = 0

mu
v=
M +m

Illustration 26. A shell fired vertically up, when reaches its highest point, explodes North
into three fragments A, B and C of masses m A = 4 kg, mB = 2 kg
3 m/s
and mC = 3kg. Immediately after the explosion, A is observed
A 4.5 m/s
moving with velocity vA = 3 m/s towards north and B with a
251\D\Allen(IIT-JEE Wing)\2020-21\Nurture\Phy\Unit-03\01-Centre of Mass

velocity vB = 4.5 m/s towards east as shown in the figure. Find B East
the velocity vC of the piece C. C

Solution. Explosion takes negligible duration; therefore, impulse of gravity, which is a finite external force,
can be neglected. The pieces fly off acquiring above-mentioned velocities due to internal forces
developed due to expanding gases produced during the explosion. The forces applied by the expanding
gases are internal forces; hence, momentum of the system of the three pieces remains conserved
during the explosion and total momentum before and after the explosion are equal.
Assuming the east as positive x-direction and the north as positive y-direction, the momentum
r r
vectors p A and pB of pieces A and B become
20
Centre of Mass
r r
p A = mAv A ˆj = 12 kg-m/s and pB = mB v B iˆ = 9 kg-m/s
Before the explosion, momentum of the shell was zero, therefore from the principle of conservation
of momentum, the total momentum of the fragments also remains zero.
r r r r r
p + p + p = 0 ® p = -(9iˆ + 12ˆj )
A B C C
From the above equation, velocity of the piece C is
r
r p
v C = C = -(3iˆ + 4ˆj ) = 5 m/s, 53° south of west.
mC

Illustration 27. In free space, three identical particles moving with velocities v o iˆ , -3v o ˆj and 5v o kˆ collide
successively with each other to form a single particle. Find velocity vector of the particle formed.
Solution. Let m be the mass of a single particle before any of the collisions. The mass of particle formed after
collisions must be 3m. In free space, no external forces act on any of the particles, their total
momentum remains conserved.
Applying principle of conservation of momentum, we have
r r
å pinitial = å pfinal ® mv o iˆ - 3mv o ˆj + 5mv o kˆ = 3mvr
r
v = 13 v o ( iˆ - 3 ˆj + 5kˆ ) m/s

Illustration 28. A bullet of mass 50 g moving with velocity 600 m/s hits a block of
mass 1.0 kg placed on a rough horizontal ground and comes out of
the block with a velocity of 400 m/s. The coefficient of friction between 600 m/s
the block and the ground is 0.25. Neglect loss of mass of the block Block
as the bullet pierces through it.
(a) In spite of the fact that friction acts as an external force, can you apply principle of conservation
of momentum during interaction of the bullet with the block?
(b) Find velocity of the block immediately after the bullet pierces through it.
(c) Find the distance the block will travel before it stops.
Solution. (a) There is no net external force in the vertical direction and in the horizontal direction, only
external force friction is non-impulsive, therefore momentum of the bullet-block system during
their interaction remains conserved.
(b) Let us denote velocities of the bullet before it hits the block and immediately after it pierces
through the block by vbo and vb, velocity of the block immediately after the bullet pierces
through it is vB and masses of the bullet and the block by m and M respectively. These are
shown in the adjacent figure.
vbo vB
vb
m M M m

Immediately before the bullet hits the block Immediately after the bullet pierces the block

Applying principle of conservation of momentum for horizontal component, we have

m (v bo - v b )
mv bo = mv b + Mv B ® vB =
251\D\Allen(IIT-JEE Wing)\2020-21\Nurture\Phy\Unit-03\01-Centre of Mass

M
Substituting the given values, we have vB = 10 m/s
(c) To calculate distance traveled by the block before it stops, work kinetic energy theorem has to
be applied.

21
JE E-Physics

K1 = 12 Mv B2 Mg K2 = 0
vB
M Fk = mMg M

x
N = Mg

During sliding of the box on the ground only the force of kinetic friction does work.
W1®2 = K 2 - K 1 ® - mMgx = 0 - 12 Mv B2

v B2
x=
2m g
Substituting given values, we have x = 20 m

Illustration 29. Ballistic Pendulum : A ballistic pendulum is used to measure speed


of bullets. It consists of a wooden block suspended from fixed support.
A wooden block of mass M is suspended with the help of two threads to
prevent rotation while swinging. A bullet of mass m moving horizontally
with velocity vo hits the block and becomes embedded in the block. vo
Receiving momentum from the bullet, the bullet-block system swings to
a height h. Find expression for speed of the bullet in terms of given
quantities.
Solution. When the bullet hits the block, in a negligible time interval, it becomes embedded in the block and
the bullet-block system starts moving with horizontally. During this process, net force acting on the
bullet-block system in vertical direction is zero and no force acts in the horizontal direction. Therefore,
momentum of the bullet-block system remains conserved.

Before the bullet Immediately after the


hits the block bullet becomes
embedded in the block
vo
p

Let us denote momentum of the bullet-block system immediately after the bullet becomes embedded
in the block by p and apply principle of conservation of momentum to the system for horizontal
component of momentum.
p = mvo
Using equation K = p2 / 2m, we can find kinetic energy K1 of the bullet-block system immediately
after the bullet becomes embedded in the block.

K1 =
( mv o )2
2 (M + m)
During swing, only gravity does work on the bullet-block system. Applying work-kinetic energy
theorem during swing of the bullet-block system, we have
251\D\Allen(IIT-JEE Wing)\2020-21\Nurture\Phy\Unit-03\01-Centre of Mass

Immediately after the


bullet becomes
embedded in the block K2 = 0
K1
p h

22
Centre of Mass

W1®2 = K 2 - K 1 ® - ( M + m ) gh = 0 -
( mv o )2
2 (M + m)
(M + m)
Rearranging terms, we have v o = 2gh
m

3.3 Application of Methods of Impulse and Momentum to a System of Particles

In a phenomenon, when a system changes its configuration, some or all of its particles change their respective
locations and momenta. Sum of linear momenta of all the particles equals to the linear momentum due to
translation of mass center. Principle of impulse and moment suggests net impulse of all the external forces
equals to change in momentum of mass center.
r r r
å ò Fi dt = pcf - pci
Conservation of Linear momentum
The above event suggests that total linear momentum of a system of particle remains conserved in a time
interval in which impulse of external forces is zero.
Total momentum of a system of particles cannot change under the action of internal forces and if net impulse
of the external forces in a time interval is zero, the total momentum of the system in that time interval will
remain conserved.
r r r r
å pinitial = å pfinal or pci = pcf
The above statement is known as the principle of conservation of momentum.
Since force, impulse and momentum are vectors, component of momentum of a system in a particular direction
is conserved, if net impulse of all external forces in that direction vanishes.
During an event the net impulse of external forces in a direction is zero in the following cases.
• When no external force acts in a particular direction on any of the particles or bodies.
• When resultant of all the external forces acting in a particular direction on all the particles or bodies is zero.
• In impulsive motion, where time interval is negligibly small, the direction in which no impulsive forces act.

Illustration 30. No external force: Stationary mass relative to an inertial frame remains at rest
A man of mass m is standing at on end of a plank of mass
M. The length of the plank is L and it rests on a frictionless
horizontal ground. The man walks to the other end of the
plank. Find displacement of the plank and man relative to
the ground.
Solution. Denoting x-coordinates of the man, mass center of plank and mass center of the man-plank system
by xm, xp and xc, we can write the following equation.
r r r r r
( Smi ) rc = Smi ri ® ( m + M ) xc = mx m + Mx p
Net force on the system relative to the ground is zero. Therefore mass center of the system which is
r
at rest before the man starts walking, remains at rest ( Dxc = 0 ) after while the man walks on the plank.
r r r
( Dxc = 0) ® mDx m + M Dx p = 0 (1)
251\D\Allen(IIT-JEE Wing)\2020-21\Nurture\Phy\Unit-03\01-Centre of Mass

r
( )
The man walks displacement Dx m / p = -Liˆ relative to the plank. Denoting displacements of the
r r
man and the plank relative to the ground by Dx m and Dx p , we can write
r r r r r
Dx m / p = Dx m - Dx p ® Dx m - Dx p = -Liˆ (2)
From the above equations (1) and (2), we have
r MLiˆ
Dx m = -
m+M

23
JE E-Physics

ML
The man moves a distance towards left relative to the ground.
m+M
r mLiˆ
Dx p =
m+M
mL
The plank moves a distance towards right relative to the ground.
m+M

Illustration 31. No external force: Mass center moving relative to an inertial frame moves with constant
velocity
Two particles of masses 2 kg and 3 kg are moving under their mutual interaction in free space. At
an instant they were observed at points (-2 m, 1 m, 4 m) and (2 m, -3 m, 6 m) with velocities

(3iˆ - 2ˆj + kˆ ) m/s and ( -ˆi + ˆj - 2kˆ ) m/s respectively. If after 10 sec, the first particle passes the point
(6 m, 8 m, -6 m), find coordinate of the point where the second particle passes at this instant?
Solution. System of these two particles is in free, therefore no external forces act on them. There total linear
momentum remains conserved and their mass center moves with constant velocity relative to an
inertial frame.
Velocity of the mass center
r
vc =
( ) (
Smi v i 2 3iˆ - 2ˆj + kˆ + 3 - ˆi + ˆj - 2kˆ
=
)
=
3ˆi - ˆj - 4kˆ
m/s
S mi 2+3 5
r
Location rco of the mass center at the instant t = 0 s
r
® rr = 2 ( -2i + j + 4 k ) + 3 ( 2i - 3 j + 6 k ) = 2i - 7 j + 26k
r Smi ri ˆ ˆ ˆ ˆ ˆ ˆ ˆ ˆ ˆ
rc =
Smi co
2+3 5
r
New location rc of the mass center at the instant t = 10 s

r r r r 2ˆi - 7ˆj + 26kˆ 3ˆi - ˆj - 4kˆ 32ˆi - 17ˆj - 14kˆ


rc = rco + v c t ® rc = + ´ 10 =
5 5 5
New location (x, y, z) of the second particle.
r
® 32i - 17 j - 14k = 2 ( 6i + 8 j - 6k ) + 3 ( xi + yj + zk )
r Smi ri ˆ ˆ ˆ ˆ ˆ ˆ ˆ ˆ ˆ
rc =
Smi 5 2+3
Solving the above equation, we obtain the coordinates of the second particle (20/3, –11, –2/3)

3.4 Application of Methods of Work and Energy to a System of Particles

In a system of particles, all the particles occupy different locations at every instant of time and may change
their locations with time. At an instant of time set of locations of all the particles of a system is known as
configuration of the system. We say something has happened with the system only when some or all of its
particles change their locations. It means that in every event or phenomena the system changes its configuration.
Methods of work and energy equips us to analyze what happens when a particle moves from one point of space
to other. Now we will apply these methods to analyze a phenomenon in which a system of particle changes its
configuration.
251\D\Allen(IIT-JEE Wing)\2020-21\Nurture\Phy\Unit-03\01-Centre of Mass

Kinetic Energy of a System of Particle y


Kinetic energy of a system of particles is defined as sum of kinetic energies of mi
all the particles of the system. vi
If at an instant particles of masses m1, m2, ….mi….mj……….. and mn are
r r r r
observed moving with velocities v 1 , v 2 , …. v i , …….. v n respectively relative
O x
to a reference frame, the kinetic energy of the whole system relative to the
reference frame is given by the following equation. z

24
Centre of Mass

1
K=
2
å mi v i2 y
vi
If the system consists of continuous distribution of mass, instead of dm
discrete particles, expression of kinetic energy becomes
1 2

K= v dm O x
z
Kinetic Energy of a System of Particle using Centroidal Frame
Centroidal frame of reference or center of mass frame is reference frame attached with the mass center of the system.
r
Let velocity of ith particle of mass mi is moving with velocity v i y vi/c
vi
relative to frame Oxyz. Mass center C and hence the centroidal frame Cxyz
r mi
vc
is moving with velocity v c . Therefore velocity of ith particle relative to the
r vc
centroidal frame is v i / c .
Kinetic energy of the whole system is given by the following equation. O C
1 1 1 x
K=
2
å mi v i2 = å mi v c2 + å mi v i2/ c
2 2 z

Here the first term on the right hand side is kinetic energy due to translation of the mass center and the second
term is kinetic energy of the system relative to the centroidal frame.

Kinetic Energy of a Two Particle System using Centroidal Frame


y
A two particle system consists of only two particles. Let a two particle
m2
system consists of particles of masses m1 and m2 moving with velocities v1 vc v2
r r
v 1 and v 2 relative to a frame Oxyz. Their mass center C lies on the
m1 C
line joining them and divides separation between them in reciprocal
ratio of masses m1 and m2. The mass center and hence the centroidal O
r x
frame is moving with velocity vc . z
Kinetic energy of this two particle system relative to a frame O xyz is given by the following equation.
1 1 2
K=
2
( m1 + m2 ) v c2 + mv rel
2
The first term on the right hand side is kinetic energy due to translation of the mass center and the second term
is kinetic energy of the system relative to the centroidal frame.
Here symbol m is known as reduced mass of the two particle system and symbol v rel is magnitude of velocity of
either of the particles relative to the other.
m1 m2 r r r r
m=
m1 + m2 and v rel = v 1 - v 2 = v 2 - v 1
Work Energy Theorem for a System of Particles
The work energy theorem can be applied to each particle of the system. For ith particle of the system, we can write
K i ,i + Wi ,i ® f = K i ,f
r r
Here Wi,i®f is total work done by all the internal forces fij and resultant external force Fi on the ith particle,
when the system goes from one configuration to other.
Adding kinetic energies of all particles, we can write kinetic energies K i and Kf of the whole system in the initial
251\D\Allen(IIT-JEE Wing)\2020-21\Nurture\Phy\Unit-03\01-Centre of Mass

as well as the final configuration. Adding work done Wi ,i ®f by internal as well as external forces on every
particle we find total work done Wi ®f by all the internal as well as external forces on the system. Now we can
write work energy theorem.
K i + Wi ®f = K f
While applying the above equation to a system, care must be taken in calculating Wi ®f . In spite of the fact
r r
that the internal forces fij and f ji being equal in magnitude and opposite in direction, the work done by them
on the ith and the jth particles will not, in general, cancel out, since i th and the jth particles may undergo different
amount of displacements.
25
JE E-Physics

The above description at first presents calculating of Wi ®f as a cumbersome task. However for systems, which
we usually encounter are not as complex as a general system of large number of particles may be. Systems
which we usually face to analyze have limited number of particles or bodies interacting. For these systems we
can simplify the task by calculating work of conservative internal forces as decrease in potential energy of the
system. Total work of internal forces other than internal conservative forces vanishes, if these forces are due
connecting inextensible links or links of constant length. These forces include string tension and normal reaction
at direct contacts between the bodies included I the system. Work of internal forces of the kind other than these
and work of external forces, can be calculated by definition of work.
Conservation of Mechanical Energy
If total work of internal forces other than conservative is zero and no external forces act on a system, total
mechanical energy remains conserved.
Ki + Ui = Kf + Uf
Since external forces are capable of changing mechanical energy of the system, under their presence total
mechanical energy changes by amount equal to work W ext, i’!f done by all the external forces.
Wext , i ® f = E f - E i = ( K f + U f ) - ( K i + U i )

Illustration 32. Total work of pseudo forces in centroidal frame.


Show that total work done in centroidal frame on all the particles of a system by pseudo forces due
to acceleration of mass center is zero.
r
Solution. Let acceleration of mass centre relative to an inertial frame is a c . Pseudo force on ith particle of
r r
mass mi in centroidal frame is ( -mi ac ) . Let displacement of ith particle in a time interval is D ri
relative to the centroidal frame.
Total work of pseudo forces on all the particles in centroidal frame can now be expressed by the
expression
r r r r r r
S ( -mi ac ) × D ri = -a c × S ( mi D ri ) = -a c × 0 = 0

Illustration 33. Two blocks of masses m and M connected by a spring are placed on frictionless horizontal ground.
When the spring is relaxed, a constant force F is applied as shown. Find maximum extension of the
spring during subsequent motion.

F
M m

Solution. If we use ground as inertial frame as we usually do, solution of the problem becomes quite involved.
Therefore, we prefer to use the centroidal frame, in which mass center remains at rest.
CM
F
M m

mx Mx
x2 = x1 =
m+ M m+ M

F
M m

In the adjacent figure is shown horizontal position of mass center (CM) by dashed line. It remains
251\D\Allen(IIT-JEE Wing)\2020-21\Nurture\Phy\Unit-03\01-Centre of Mass

unchanged in centroidal frame.


Mass center of two particle system divides separation between them in reciprocal ratio of the
masses; therefore displacements x1 and x2 of the blocks must also be in reciprocal ratio of their
masses. The extension x is sum of displacements x 1 and x2 of the blocks as shown in the figure.
When extension of the spring achieves its maximum value, both the block must stop receding away
from the mass center, therefore, velocities of both the blocks in centroidal frame must be zero.
During the process when spring is being extended, total work done by pseudo forces in centroidal
frame become zero, negative work done by spring forces becomes equal to increase in potential
energy and work done by the applied force evidently becomes Fx 1.

26
Centre of Mass
Using above fact in applying work energy theorem on the system relative to the centroidal frame, we
obtain
K i + Wi ®f = K f ® 0 + Wi ® f ,springforce + Wi ® f ,F = 0
FMx
- ( 0 - 12 kx 2 ) + =0
m+m
2FM
x=
k (m + m)

Impulse and Momentum


1. Two blocks of masses m and M are moving with speeds v1 and v2 (v1 > v2) in the same direction on the
frictionless surface respectively, M being ahead of m. An ideal spring of force constant k is attached to
the backside of M (as shown). The maximum compression of the spring when the block collides is:
m M
(A) v 1 (B) v2
k k

mM
(C) (v1 – v2) (D) None of above is correct.
(M + m) K

2 In a vertical plane inside a smooth hollow thin tube a block of same mass
as that of tube is released as shown in figure. When it is slightly disturbed,
it moves towards right. By the time the block reaches the right end of the
tube then the displacement of the tube will be (where ‘R’ is mean radius
of tube). Assume that the tube remains in vertical plane.
2R 4R R
(A) (B) (C) (D) R
π π 2

3. Mr. Verma (50 kg) and Mr. Mathur (60 kg) are sitting at the two extremes of a 4 m long boat (40 kg)
standing still in water. To discuss a mechanics problem, they come to the middle of the boat. Neglecting
friction with water, how far does the boat move in the water during the process ?
(A) 40/3 cm (B) 50/3 cm (C) 110/3 cm (D) 10 cm

4. The balloon, the light rope and the monkey shown in figure are at rest in the air.
If the monkey reaches the top of the rope, by what distance does the balloon
descend? Mass of the balloon = M, mass of the monkey = m and the length of
the rope ascended by the monkey = L.
(A) ML/ (m + M) (B) mL/ (m + M) (C) L (D) 0

r
5. Velocity of a particle of mass 2 kg varies with time t according to the equation v = (2t î + 4 ĵ ) m / s . Here t is in
seconds. Find the impulse imparted to the particle in the time interval from t = 0 to t = 2 s.
(A) 8 ˆi N - s (B) 10 ˆi N - s (C) 12iˆ N - s (D) 16iˆ N - s
251\D\Allen(IIT-JEE Wing)\2020-21\Nurture\Phy\Unit-03\01-Centre of Mass

6. Particle 'A' moves with speed 10 m/s in a frictionless circular fixed horizontal pipe of radius 5 m and strikes
with 'B' of double mass that of A. Coefficient of restitution is 1/2 and particle 'A' starts its journey at
t = 0. The time at which second collision occurs is :

p 2p
(A) s (B) s
2 3

5p
(C) s (D) 4 p s
2

27
JE E-Physics
7. Two blocks of mass 3 kg and 6 kg respectively are placed on a
smooth horizontal surface. They are connected by a light spring
of force constant k = 200 N/m. Initially the spring is unstretched.
The indicated velocities are imparted to the blocks. Find the
maximum extension of the spring.

4.0 IMPACT BETWEEN TWO BODIES

Impact or collision is interaction of very small duration between two bodies in which the bodies apply relatively
large forces on each other.
Interaction forces during an impact are created due to either direct contact or strong repulsive force fields or
some connecting links. These forces are so large as compared to other external forces acting on either of the
bodies that the effects of later can be neglected. The duration of the interaction is short enough as compared
to the time scale of interest as to permit us only to consider the states of motion just before and after the event
and not during the impact. Duration of an impact ranges from 10-23 s for impacts between elementary particles to
millions of years for impacts between galaxies. The impacts we observe in our everyday life like that between
two balls last from 10-3 s to few seconds.

4.1 Central and Eccentric Impact

The common normal at the point of contact between the bodies is known as line of impact. If mass centers of the
both the colliding bodies are located on the line of impact, the impact is called central impact and if mass centers
of both or any one of the colliding bodies are not on the line of impact, the impact is called eccentric impact.

B B
A A
Common Common Normal Common Common Normal
Tangent or or
Tangent
Line of Impact Line of Impact

Central Impact Eccentric Impact

Central impact does not produce any rotation in either of the bodies whereas eccentric impact causes the body
whose mass center is not on the line of impact to rotate. Therefore, at present we will discuss only central
impact and postpone analysis of eccentric impact to cover after studying rotation motion.
Head–on (Direct) and Oblique Central Impact
If velocities vectors of the colliding bodies are directed along the line of impact, the impact is called a direct or
head-on impact; and if velocity vectors of both or of any one of the bodies are not along the line of impact, the
impact is called an oblique impact.
t
B B
A A
uA uB
n
n uA uB

Central Impact Oblique Impact

In this chapter, we discuss only central impact, therefore the term central we usually not use and to these
251\D\Allen(IIT-JEE Wing)\2020-21\Nurture\Phy\Unit-03\01-Centre of Mass

impacts, we call simply head-on and oblique impacts. Furthermore, use of the line of impact and the common
tangent is so frequent in analysis of these impacts that we call them simply t-axis and n-axis.

4.2 Head–on (Direct) Central Impact

To understand what happens in a head-on impact let us consider two balls A and B of masses m A and mB
moving with velocities uA and uB in the same direction as shown. Velocity uA is larger than uB so the ball A hits
the ball B. During impact, both the bodies push each other and first they get deformed till the deformation
reaches a maximum value and then they tries to regain their original shape due to elastic behaviors of the
materials forming the balls.

28
Centre of Mass

A B A B A B
uA uB u u vA vB

Deformation Restitution
Period Period

Instant when Instant of maximum Instant when


impact starts deformation impact ends

The time interval when deformation takes place is called the deformation period and the time interval in which
the ball try to regain their original shapes is called the restitution period. Due to push applied by the balls on
each other during period of deformation speed of the ball A decreases and that of the ball B increases and at
the end of the deformation period, when the deformation is maximum both the ball move with the same
velocity say it is u. Thereafter, the balls will either move together with this velocity or follow the period of
restitution. During the period of restitution due to push applied by the balls on each other, speed of the ball A
decrease further and that of ball B increase further till they separate from each other. Let us denote velocities
of the balls A and B after the impact by vA and vB respectively.
Equation of Impulse and Momentum during impact
Impulse momentum principle describes motion of ball A during deformation period.

mAuA ò Ddt mAu


mAu A - ò Ddt = mAu ...(i)

Impulse momentum principle describes motion of ball B during deformation period.

mBuB ò Ddt mBu


mBu B + ò Ddt = mBu ...(ii)

Impulse momentum principle describes motion of ball A during restitution period.

mAu ò Rdt mAvA


mAu - ò Rdt = mAv A ...(iii)

Impulse momentum principle describes motion of ball B during restitution period.

mBu ò Rdt mBvB


mBu + ò Rdt = mBv B ...(iv)

Conservation of Momentum during impact


From equations, (i) and (ii) we have mAu A + mBuB = ( mA + mB ) u ...(v)

From equations, (iii) and (iv) we have ( mA + mB ) u = mAv A + mBv B ...(vi)


From equations, (v) and (vi) we obtain the following equation.
251\D\Allen(IIT-JEE Wing)\2020-21\Nurture\Phy\Unit-03\01-Centre of Mass

mAv A + mB v B = mAuA + mBu B ...(vii)


The above equation elucidates the principle of conservation of momentum.

Coefficient of Restitution
Usually the force D applied by the bodies A and B on each other during period differs from the force R applied
by the bodies on each other during period of restitution. Therefore, it is not necessary that magnitude of

impulse ò Ddt of deformation equals to the magnitude of impulse ò Rdt restitution.


The ratio of magnitudes of impulse of restitution to that of deformation is called the coefficient of restitution
and is denoted by e.
29
JE E-Physics

e=
ò Rdt
...(viii)
ò Ddt
Now from equations (i), (ii), (iii) and (iv), we have
vB - vA
e= ...(ix)
u A - uB
Coefficient of restitution depend on various factors as elastic properties of materials forming the bodies, velocities
of the contact points before impact, state of rotation of the bodies and temperature of the bodies. In general,
its value ranges from zero to one but in collision where kinetic energy is generated its value may exceed one.
Depending on values of coefficient of restitution, two particular cases are of special interest.
Perfectly Plastic or Inelastic Impact For these impacts e = 0, and bodies undergoing impact stick to each
other after the impact.
Perfectly Elastic Impact For these impacts e = 1.

Strategy to solve problems of head-on impact


Write momentum conservation equation
mAv A + mB v B = mAuA + mBu B ...(A)
Write rearranging terms of equation of coefficient of restitution
v B - v A = e (u A - u B ) ...(B)
Use the above equations A and B.

Illustration 34. A ball of mass 2 kg moving with speed 5 m/s collides directly with another of mass 3 kg moving in
the same direction with speed 4 m/s. The coefficient of restitution is 2/3. Find the velocities after collision.
Solution. Denoting the first ball by A and the second ball by B velocities immediately before and after the
impact are shown in the figure.
uA = uB = 4 vA vB

A B A B
Immediately before Immediately after
impact starts impact ends
Applying principle of conservation of momentum, we have
mB v B + mAv A = mAuA + mBuB ® 3v B + 2v A = 2 ´ 5 + 3 ´ 4
3v B + 2v A = 22 ...(i)
Applying equation of coefficient of restitution, we have
v B - v A = e (u A - u B ) ® v B - v A = 23 ( 5 - 4)
3v B - 3v A = 2 ...(ii)
From equation (i) and (ii), we have v A = 4 m/s and vB = 4.67 m/s Ans.

Illustration 35. A block of mass 5 kg moves from left to right with a velocity of 2 m/s and collides with another
block of mass 3 kg moving along the same line in the opposite direction with velocity 4 m/s.
251\D\Allen(IIT-JEE Wing)\2020-21\Nurture\Phy\Unit-03\01-Centre of Mass

(a) If the collision is perfectly elastic, determine velocities of both the blocks after their collision.
(b) If coefficient of restitution is 0.6, determine velocities of both the blocks after their collision.
Solution. Denoting the first block by A and the second block by B velocities immediately before and after the
impact are shown in the figure.
A B vA A B
uA=2 uB = 4 vB

Immediately before Immediately after


impact starts impact ends

30
Centre of Mass
Applying principle of conservation of momentum, we have
mBv B + mAv A = mAu A + mBuB ® 3v B + 5v A = 5 ´ 2 + 3 ´ ( -4 )

3v B + 5v A = -2 ...(i)
Applying equation of coefficient of restitution, we have
v B - v A = e (u A - u B ) ® v B - v A = e {2 - ( -4)}

v B - v A = 6e ...(ii)
(a) For perfectly elastic impact e = 1. Using this value in equation (ii), we have
vB – v A = 6 ...(iia)
Now from equation (i) and (iia), we obtain
vA = – 2.5 m/s and vB = 3.5 m/s
(b) For value e = 0.6, equation 2 is modified as
vB – vA = 3.6 (iib)
Now from equation (i) and (iib), we obtain
vA = – 1.6 m/s and vB = 2.0 m/s
Block A reverse back with speed 1.6 m/s and B also move in opposite direction to its original
direction with speed 2.0 m/s.

Illustration 36. Two identical balls A and B moving with velocities uA and uB in the same direction collide. Coefficient
of restitution is e.
(a) Deduce expression for velocities of the balls after the collision.
(b) If collision is perfectly elastic, what do you observe?
Solution. Equation expressing momentum conservation is
v A + v B = u A + uB ...(A)
Equation of coefficient of restitution is
v B - v A = eu A - euB ...(B)
(a) From the above two equations, velocities v A and vB are

æ1 - eö æ1 + eö
vA = ç u + u
è 2 ÷ø A çè 2 ÷ø B ...(i)

æ1 + eö æ1 - eö
vB = ç u + u
è 2 ÷ø A çè 2 ÷ø B ...(ii)

(b) For perfectly elastic impact e = 1, velocities vA and vB are


vA = u B ...(iii)
vB = u A ...(iv)
Identical bodies exchange their velocities after perfectly elastic impact.

Conservation of kinetic energy in perfectly elastic impact


For perfectly elastic impact equation for conservation of momentum and coefficient of restitution are
251\D\Allen(IIT-JEE Wing)\2020-21\Nurture\Phy\Unit-03\01-Centre of Mass

mAv A + mB v B = mAuA + mBu B ...(A)

v B - v A = u A - uB ...(B)
Rearranging the terms of the above equations, we have
mA (v A - uA ) = mB (uB - v B )

u A + v A = v B + uB

31
JE E-Physics
Multiplying LHS of both the equations and RHS of both the equations, we have
mA (v A2 - u A2 ) = mB (u B2 - v B2 )
Multiplying by ½ and rearranging term of the above equation, we have
1
2 mAu A2 + 12 mBuB2 = 12 mAv A2 + 21 mBv B2
In perfectly elastic impact total kinetic energy of the colliding body before and after the impact are equal.
In inelastic impacts, there is always loss of kinetic energy.

Head on Collision
1. There are hundred identical sliders equally spaced on a frictionless track as shown in the figure. Initially all
the sliders are at rest. Slider 1 is pushed with velocity v towards slider 2. In a collision the sliders stick
together. The final velocity of the set of hundred stucked sliders will be :

v v
(A) (B) (C) zero (D) v
99 100

2. A ball of mass 2m impinges directly on a ball of mass m, which is at rest. If the velocity with which the larger ball
impinges be equal to the velocity of the smaller mass after impact then the coefficient of restitution
1 3 1 2
(A) (B) (C) (D)
3 4 2 5

3. In head on elastic collision of two bodies of equal masses


(A) the velocities are interchanged
(B) the speeds are interchanged
(C) the momenta are interchanged
(D) the faster body slows down and the slower body speeds up.

4. Two identical balls one move with 12 m/s and second is at rest collides elastically after collision velocitis of
second and first ball will be
(A) 6 m/s, 6 m/s (B) 12 m/s, 12 m/s (C) 12 m/s, 0 m/s (D) 0 m/s, 12 m/s

5. A body moving towards a finite body at rest collides with it. It is possible that
(A) both the bodies come to rest
(B) both the bodies move after collision
(C) the moving body comes to rest and the stationary body starts moving
(D) the stationary body remains stationary, the moving body changes its velocity.

6. In the arrangement shown, the pendulum on the left is pulled aside. It is


then released and allowed to collide with other pendulum which is at
rest. A perfectly inelastic collision occurs and the system rises to a height
251\D\Allen(IIT-JEE Wing)\2020-21\Nurture\Phy\Unit-03\01-Centre of Mass

1/4 h. The ratio of the masses of the pendulum is :


(A) 1 (B) 2
(C) 3 (D) 4

7. A ball of mass m approaches a moving wall of infinite mass with a speed 'v' along the normal to the wall. The
speed of the wall is 'u' toward the ball. The speed of the ball after ‘elastic’ collision with wall is:
(A) u + v away from the wall (B) 2u + v away from the wall
(C) | u – v| away from the wall (D) | v – 2u | away from the wall

32
Centre of Mass
8. A ball of mass 1 kg drops vertically on to the floor with a speed of 25 m/s. It rebounds with an initial velocity of
10 m/s. What impulse acts on the ball during contact?
(A) 35kg m/s downwards (B) 35 kg m/s upwards
(C) 30 kg m/s downwards (D) 30kg m/s upwards

4.3 Oblique Central Impact

In oblique central impact, velocity vectors of both or of any one of the bodies are not along the line of impact
and mass center of bodies are on the line of impact. Due to impact speeds and direction of motion of both the
balls change. In the given figure is shown two balls A and B of masses m A and mB moving with velocities uA
and uB collide obliquely. After the collision let they move with velocities v A and vB as shown in the next figure.

uB vA

uA vB
A B A B

Immediately before Impact Immediately after Impact

To analyze the impact, we show components of velocities before and after the impact along the common
tangent and the line of impact. These components are shown in the following figure.
t t
A B A B
uAn uBn vAn vBn
n n

uAt uBt vAt vBt


uA uB vA vB

Immediately before Impact Immediately before Impact

Component along the t-axis If surfaces of the bodies undergoing impact are smooth, they cannot apply any
force on each other along the t-axis and component of momentum along the t-axis of each bodies, considered
separately, is conserved. Hence, t-component of velocities of each of the bodies remains unchanged.
vAt = uAt and vBt = uBt ...(A)
Component along the n-axis For components of velocities along the n-axis, the impact can be treated same as
head-on central impact.
The component along the n-axis of the total momentum of the two bodies is conserved
mB v Bn + mAv An = mBuBn + mAu An ...(B)
Concept of coefficient of restitution e is applicable only for the n-component velocities.
v Bn - v An = e (u An - uBn ) ...(C)
The above four independent equation can be used to analyze oblique central impact of two freely moving
bodies.
251\D\Allen(IIT-JEE Wing)\2020-21\Nurture\Phy\Unit-03\01-Centre of Mass

Illustration 37. A disk sliding with velocity u on a smooth horizontal plane


A u
strikes another identical disk kept at rest as shown in the
figure. If the impact between the disks is perfectly elastic
B
impact, find velocities of the disks after the impact.
Solution. (a) We first show velocity components along the t and the n-axis immediately before and after the
impact. angle that the line of impact makes with velocity u is 30°.

33
JE E-Physics

uAn t vAn t
uAt vAt
30 A A
u B vA B
vBn
n n

Immediately before Impact Immediately after Impact

Component along t-axis Components of momentum along the t-axis of each disk, considered
separately, is conserved. Hence, t-component of velocities of each of the bodies remains unchanged.
u
v At = u At = and v Bt = u Bt = 0 ...(i)
2
Component along n-axisThe component along the n-axis of the total momentum of the two
bodies is conserved
u 3
mB v Bn + mA v An = mB uBn + m AuAn ® mv Bn + mv An = m ´ 0 + m
2

u 3
v Bn + v An = ...(ii)
2
Concept of coefficient of restitution e is applicable only for the n-component velocities.
u 3
v Bn - v An = e (u An - uBn ) ® v Bn - v An = ...(iii)
2

u 3
From equations (ii) and (iii), we have v An = 0 and v Bn = ...(iv)
2
From equations (i) and (iv) we can write velocities of both the disks.

Illustration 38. A ball collides with a frictionless wall with velocity u as shown in the
Normal
figure. Coefficient of restitution for the impact is e. q
(a) Find expression for the velocity of the ball immediately after the impact.
u
(b) If impact is perfectly elastic what do you observe?
Solution. (a) Let us consider the ball as the body A and the wall as the body B. Since the wall has infinitely
large inertia (mass) as compared to the ball, the state of motion of the wall, remains unaltered
during the impact i.e. the wall remain stationary.
Now we show velocities of the ball and its t and n-components immediately before and after
the impact. For the purpose we have assumed velocity of the ball after the impact v.

t
v
vt
un q'
n q n
vn
ut
u
t
251\D\Allen(IIT-JEE Wing)\2020-21\Nurture\Phy\Unit-03\01-Centre of Mass

Immediately before Impact Immediately after Impact

Component along t-axis : Components of momentum along the t-axis of the ball is conserved.
Hence, t-component of velocities of each of the bodies remains unchanged.
v t = ut = u sin q ...(i)
Component along n-axis : Concept of coefficient of restitution e is applicable only for the n-
component velocities.

34
Centre of Mass

v Bn - v An = e (u An - uBn ) ® -v n = eu n
v n = -eu cos q ...(ii)
From equations (i) and (ii), the t and n-components of velocity of the ball after the impact are
v t = u sin q and v n = eu sin q
(b) If the impact is perfectly elastic, we have v t = u sin q , v n = u sin q and q '= q
The ball will rebound with the same speed making the same angle with the vertical at which it
has collided. In other words, a perfectly elastic collision of a ball with a wall follows the same
laws as light follows in reflection at a plane mirror.

4.4 Oblique Central Impact when one or both the colliding bodies are constrained in motion

In oblique collision, we have discussed how to analyze impact of bodies that were free to move before as well
as after the impact. Now we will see what happens if one or both the bodies undergoing oblique impact are
constrained in motion.
Component along the t-axis : If surfaces of the bodies undergoing impact are smooth, the t-component of
the momentum of the body that is free to move before and after the impact remain conserved.
If both the bodies are constrained, the t-component of neither one remains conserved.
Momentum Conservation : We may find a direction in which no external force acts on both the bodies. The
component of total momentum of both the bodies along this direction remains conserved.
Coefficient of restitution : Concept of coefficient of restitution e is applicable only for the n-component
velocities.
v Bn - v An = e (u An - uBn )

Illustration 39. A 250 g ball moving horizontally with velocity 10.0 m/s strikes
10 m/s
inclined surface of a 720 g smooth wedge as shown in the
figure. The wedge is placed at rest on a frictionless horizontal
ground. If the coefficient of restitution is 0.8, calculate the 37°
velocity of the wedge after the impact.
Solution. Let us consider the ball as the body A and the wedge as the body B. After the impact, the ball
bounces with velocity vA and the wedge advances in horizontal direction with velocity vB. These
velocities and their t and n-components are immediately before and after the impact are shown in
the following figures.

n n
t
vA
t vAt
uAt uAn
vAn
37°
vB
uA
53°
37° 37°
vBn
251\D\Allen(IIT-JEE Wing)\2020-21\Nurture\Phy\Unit-03\01-Centre of Mass

n
Immediately before Impact
Immediately after Impact

Component along t-axis : The ball is free to move before and after the impact, therefore its
t-component of momentum conserved. Hence, t-component of velocities of the ball remains
unchanged.
v At = u At = 10 cos 37° = 8 m/s ...(i)

35
JEE-Physics
Momentum Conservation : In the horizontal direction, there is no external force on both the
bodies. Therefore horizontal component of total momentum of both the bodies remain conserved.
mAu A = mA (v At cos 37° - v An cos 53°) + mBv B

æ 4 3v ö
0.25 ´ 10 = 0.25 ç 8 ´ - An ÷ + 0.72v B ...(ii)
è 5 5 ø
Coefficient of restitution : Concept of coefficient of restitution e is applicable only for the
n-component velocities.
3v B
v Bn - v An = e (u An - uBn ) ® - v An = 0.8 ( 6 - 0) ...(iii)
5
From equations (i), (ii) and (iii), we obtain vB = 2.0 m/s

Oblique Collision
r
1. A particle of mass 1 g moving with a velocity u1 = (3 î – 2 ĵ )ms–1 experiences a perfectly inelastic
r
collision with another particle of mass 2 g and velocity u 2 = (4 ĵ – 6 k̂ )ms–1. The velocity of the
combined particle is :
(A) ˆi + 2jˆ - 4kˆ (B) ˆi - 2jˆ + 4kˆ (C) ˆi - 2jˆ - 4kˆ (D) ˆi + 3.33jˆ + 4kˆ

2. A ball collides with an inclined plane of inclination q after falling through a distance h. If it moves
horizontally just after the impact, the coefficient of restitution is :
(A) tan2q (B) cot2q (C) tan q (D) cot q

3. AB is an L shaped obstacle fixed on a horizontal smooth table. A ball strikes


it at A, gets deflected and restrikes it at B. If the velocity vector before
r
collision is v and coefficient of restitution of each collision is 'e', then the
velocity of ball after its second collision at B is :
r r
(A) e 2 v (B) - e 2 v
r
(C) -ev (D) data insufficient

4. In the figure shown a particle P strikes the inclined smooth plane horizontally and rebounds vertically. If
the angle q is 60º, then the co-efficient of restitution is :
1 1
(A) (B)
3 3

1
(C) (D) 1
2

5. A particle of mass m is moving along the x-axis with speed


v when it collides with a particle of mass 2 M initially at rest.
After the collision, the first particle has come to rest and the
251\D\Allen(IIT-JEE Wing)\2020-21\Nurture\Phy\Unit-03\01-Centre of Mass

second particle has split into two equal-mass pices that are
shown in the figure. Which of the following statements
correctly describes the speeds of the pieces? (q > 0)
(A) Each piece moves with speed v.
(B) Each piece moves with speed v/2.
(C) One of the peices moves with speed v/2, the other with speed greater than v/2
(D) Each piece moves with speed greater than v/2.

36
Centre of Mass
6. A mass m moves with a velocity v and collides inelastically with another identical mass. After collision the Ist
v
mass moves with velocity in a direction perpendicular to the initial direction of motion. Find the speed of
3
the second mass after collision-
2 v
(A) v (B) 3v (C) v (D)
3 3

7. A disk A of radius r moving on perfectly smooth surface at a speed v A


B
undergoes an elastic collision with an identical stationary disk B. Find v
the velocity of the disk B after collision if the impact parameter is r/2 r/2
as shown in the figure
15 v v 3v
(A) v (B) (C) (D)
4 4 2 2

5.0 VARIABLE MASS SYSTEM


r
If a mass is added or ejected from a system, at rate m kg/s and relative velocity v rel (w.r.t. the system),
r
then the force exerted by this mass on the system has magnitude m v rel .
r
Thrust Force ( Ft )

r r æ dm ö
Ft = v rel ç ÷
è dt ø
r
Suppose at some moment t = t mass of a body is m and its velocity is v . After some time at t = t + dt
r r
its mass becomes (m – dm) and velocity becomes v + dv . The mass dm is ejected with relative
r r r
velocity v r . Absolute velocity of mass ‘dm’ is therefore ( v + v r ). If no external forces are acting on
the system, the linear momentum of the system will remain conserved, or
r r
Pi = Pf
r r r r r
or m v = (m – dm) ( v + d v ) + dm ( v + v r )
r r r r r r r
or m v = m v + md v – (dm) v – (dm) (d v ) + (dm) v + v r dm
r
The term (dm) (d v ) is too small and can be neglected.
r r
\ md v = – v r dm
r
æ dv ö r æ dm ö
or mç ÷ = vr ç - ÷
è dt ø è dt ø
r
æ dv ö r
Here, mç -
è dt ø
÷ = thrust force F
t ( )
251\D\Allen(IIT-JEE Wing)\2020-21\Nurture\Phy\Unit-03\01-Centre of Mass

dm
and – = rate at which mass is ejecting
dt

r r æ dm ö
or Ft = v r ç ÷
è dt ø

37
JE E-Physics

Illustration 40. A flat car of mass m 0 starts moving to the right due to a
constant horizontal force F. Sand spills on the flat car from a
stationary hopper. The rate of loading is constant and equal
to m kg/s. Find the time dependence of the velocity and the m
acceleration of the flat car in the process of loading. The
m0 F
friction is negligibly small.

Solution Initial velocity of the flat car is zero. Let v be its velocity at time t and m its mass at that instant.
Then

At t = 0, v = 0 and m = m0 at t = t, v = v and m = m0 + mt
Here, vr = v (backwards)
dm
=m
dt
dm
\ Ft = v r = mv (backwards)
dt
Net fo rce on the flat car at time t is Fnet = F – Ft
dv
or m = F – mv ....(i)
dt
dv
or (m0 + mt) =F – mv
dt
v dv t dt
or ò 0 F - mv
= ò 0 m 0 + mt
1 1
\ – [ln (F – mv)]0v = [ln (m0 + mt)]0t
m m

æ F ö æ m + mt ö
Þ ln çç ÷÷ = ln çç 0 ÷
÷
è F - mv ø è m0 ø
F m + mt
\ = 0
F - mv m0
Ft
or v= Ans.
m 0 + mt
dv
From Eq. (i), = acceleration of flat car at time t
251\D\Allen(IIT-JEE Wing)\2020-21\Nurture\Phy\Unit-03\01-Centre of Mass

dt
F - mv
or =
m
æ Fmt ö
çF- ÷
ç m0 + mt ÷ Fm 0
a= ç ÷ or a = Ans.
m0 + mt (m 0 + mt ) 2
ç ÷
è ø

38
Centre of Mass
Illustration 41. A cart loaded with sand moves along a horizontal floor due to a constant force F coinciding
in direction with the cart’s velocity vector. In the process sand spills through a hole in the
bottom with a constant rate mkg/s. Find the acceleration and velocity of the cart at the
moment t, if at the initial moment t = 0 the cart with loaded sand had the mass m 0 and
its velocity was equal to zero. Friction is to be neglected.
Solution In this problem the sand spills through a hole in the bottom of the cart. Hence, the relative
velocity of the sand v r will be zero because it will acquire the same velocity as that of the
cart at the moment.
vr = 0
v
m
æ dm ö F
Thus, F t = 0 ç as Ft = v r ÷
è dt ø
and the net force will be F only. v
\ F net = F
æ dv ö
or mç ÷ = F ....(i)
è dt ø
But here m = m 0 – mt
dv
\ (m 0 – mt) =F
dt
v t F dt
or ò 0
dv = ò 0 m0 - mt

F
\ v= [ln(m0 - mt)] 0t
-m

F æ m0 ö
or v= ln ç ÷
m è m0 - mt ø
From eq. (i), acceleration of the cart
dv F F
a= = or a = m
dt m 0 - mt

5.1 Rocket propulsion

Let m 0 be the mass of the rocket at time t = 0. m its mass at any time t and v its velocity at that
moment. Initially, let us suppose that the velocity of the rocket is u.

æ - dm ö
Further, let ç ÷ be the mass of the gas ejected per unit time and v r the exhaust velocity of the
251\D\Allen(IIT-JEE Wing)\2020-21\Nurture\Phy\Unit-03\01-Centre of Mass

è dt ø
æ -dm ö
gases with respect to rocket. Usually ç and v r are kept constant throughout the journey of the
è dt ÷ø
rocket. Now, let us write few equations which can be used in the problems of rocket propulsion. At
time t = t,
æ -dm ö
1. Thrust force on the rocket F t = v r çè ÷ (upwards)
dt ø
2. Weight of the rocket W = mg (downwards)

39
JE E-Physics
3. Net force on the rocket F net = F t – W (upwards)
æ - dm ö
or F net = v r ç ÷ –mg
è dt ø
F
4. Net acceleration of the rocket a =
m
dv vr æ - dm ö
or = ç ÷ –g
dt m è dt ø
vr
or dv =
m
(- dm) – g dt

v m - dm t
or ò
u
dv = v r ò m0 m
–g ò 0
dt

æ m0 ö
Thus, v = u – gt + v r ln ç ...(i)
è m ÷ø

æ dm ö dm
Note : 1. F t = v r çè - ÷ø is upwards, as v r is downwards and is negative.
dt dt
æ m0 ö
2. If gravity is ignored and initial velocity of the rocket u = 0, Eq. (i) reduces to v = v r ln ç .
è m ÷ø

Illustration 42. A rocket, with an initial mass of 1000 kg, is launched vertically upwards from rest under
gravity. The rocket burns fuel at the rate of 10 kg per second. The burnt matter is ejected
vertically downwards with a speed of 2000 ms –1 relative to the rocket. If burning stops
after one minute. Find the maximum velocity of the rocket. (Take g as at 10 ms –2)
Solution Using the velocity equation

æ m0 ö
v = u – gt + v r ln ç ÷
è m ø
Here u = 0, t = 60s, g = 10 m/s 2, v r = 2000 m/s, m 0 = 1000 kg
and m = 1000 – 10 × 60 = 400 kg
æ 1000 ö
We get v = 0 – 600 + 2000 ln çè ÷
400 ø
or v = 2000 ln 2.5 – 600
The maximum velocity of the rocket is 200(10 ln 2.5 – 3) = 1232.6 ms –1 Ans.

Illustration 43. A uniform chain of mass m and length l hangs on a thread


and touches the surface of a table by its lower end. Find the
force exerted by the table on the chain when half of its length
has fallen on the table. The fallen part does not form heap.
Solution 1. Weight of the portion BC of the chain
251\D\Allen(IIT-JEE Wing)\2020-21\Nurture\Phy\Unit-03\01-Centre of Mass

mg
lying on the table, W = (downwards) Using v = 2gh
2

æ dm ö
2. Thrust force F t = v r ç ÷
è dt ø
vr = v
dm
= lv
dt
40
Centre of Mass
F t = lv2 (where, l = m/l, is mass per unit length of chain)

( gl )
2
v2 = ( = gl

æmö
\ F t = ç ÷ (gl) = mg (downwards)
èlø
\ Net force exerted by the chain on the table is
mg 3
F = W + Ft = + mg = mg
2 2
So, from Newton’s third law the force exerted by the table on the chain will be 3/2 mg
(vertically upwards).

Variable Mass System


1. A wagon filledr with sand has a hole so that sand leaks through the bottom at a constant rate l. An
external force F acts on the wagon in the direction of motion. Assuming instantaneous velocity of the wagon
r
to be v and initial mass of system to be m 0, the force equation governing the motion of the wagon is :
r r
r dv r r dv r
(A) F = m0 + lv (B) F = m0 – lv
dt dt
r r
r dv r dv r
(C) F = (m0 – lt) (D) F = (m0 – lt) + lv
dt dt

2. The carriage of mass M has constant initial velocity u along a straight horizontal
track when at t = 0, it starts raining. The rain drops have a vertical velocity u¢
and result into addition of mass m per second to the carriage. The velocity of
car after T second of start of rain is (Assume frictionless surface) :
Mu Mu + mu¢ (u + u¢) (M + m) M (u + U¢)
(A) (B) M + mt (C) (D)
M + mt M mt

3. A balloon having mass ' m ' is filled with gas and is held in hands of a boy. Then suddenly it get released and gas
starts coming out of it with a constant rate. The velocities of the ejected gases is also constant 2 m/s with respect
to the balloon. Find out the velocity of the balloon when the mass of gas is reduced to half. (neglect gravity)
(A) l n 2 (B) 2 ln 4 (C) 2 ln 2 (D) none of these

4. Sand drops from a stationary hopper at the rate of 5 kg/s on to a conveyor belt moving with a constant speed of
2 m/s. What is the force required to keep the belt moving and what is the power delivered by the motor moving
the belt?
v=1.2 m/s
5. If the chain is lowered at a constant speed v = 1.2 m/s, determine the
normal reaction exerted on the floor as a function of time. The chain has a 6m
mass of 80 kg and a total length of 6 m.

6. An open water tight railway wagon of mass 5 × 10 3 kg coasts at an initial velocity 1.2 m/s without friction on a
251\D\Allen(IIT-JEE Wing)\2020-21\Nurture\Phy\Unit-03\01-Centre of Mass

railway track. Rain drops fall vertically downwards into the wagon. The velocity of the wagon after it has
collected 103 kg of water will be
(A) 0.5 m/s (B) 2m/s (C) 1 m/s (D) 1.5 m/s

7. A uniform metallic spherical shell is suspended from ceiling. It has two holes A and B
A
at top and bottom respectively. Which of the following is/are true: sand
(A) If B is closed and sand is poured from A, centre of mass first rises and then falls
(B) If shell is completely filled with sand and B is opened then centre of mass falls initially
(C) If shell is slightly filled with sand and B is opened, then centre of mass falls.
(D) None of these B

41
JE E-Physics

l Kinetic energy of system of particles with respect to ground is equals to kinetic energy of centre of mass w.r. to
ground plus kinetic energy of system of particles w.r. to centre of mass.

l If net force on system of particles is equals to zero then minimum kinetic energy of that system is kinetic energy
of centre of mass.

l If momentum of system of particles is equals to zero then it is not necessary that kinetic energy of system of
particle is zero.

l During collision kinetic energy is not conserve whether collision is elastic or inclastic.

l If no external force acts on the system of two spheres which undergoes, a perfectly elastic head on collision, the
minimum kinetic energy of the system is zero, if net momentum of this system is zero.

Problems related to variable mass can be solved in following four steps

l Make a list of all the forces acting on the main mass and apply them on it.
r r æ dm ö
l Apply an additional thrust force Ft on the mass, the magnitude of which is v r çè ± ÷ and direc-
dt ø
r r
tion is given by the direction of v r in case the mass is increasing and otherwise the direction of – v r

if it is decreasing.
l Find net force on the mass and apply

r r
dv
Fnet = m (m = mass at the particular instant)
dt

l Integrate it with proper limits to find velocity at any time t.

251\D\Allen(IIT-JEE Wing)\2020-21\Nurture\Phy\Unit-03\01-Centre of Mass

42
Centre of Mass

SOME WORKED OUT ILLUSTRATIONS


Illustration 1.
A ball of mass 2 kg dropped from a height H above a horizontal surface rebounds to a height h after one bounce.
The graph that relates H to h is shown in figure. If the ball was dropped from an initial height of 81 m and made
ten bounces, the kinetic energy of the ball immediately after the second impact with the surface was

h(m)

40

H(m)
O 90

(A) 320 J (B) 480 J (C) 640 J (D) Can't be determined


Ans. (A)
Solution

h 40 2
From graph e = = =
H 90 3
Kinetic energy of the ball just after second bounce
4
1 1 æ 2ö
m ( e2 u ) = me 4 u 2 = ( e 4 ) ( mgH) = ç ÷ ( 2) (10 ) ( 81) = 320J
2
=
2 2 è 3ø
Illustration 2.
Consider an one dimensional elastic collision between a given incoming body A and body B, initially at rest. The
mass of B in comparison to the mass of A in order that B should recoil with greatest kinetic energy is
(A) mB>>mA (B) mB<<mA (C) mB=mA (D) can't say anything
Ans. (C)
Solution
mA mB mA mB
A u1 B A v1 B v2

Before collision After collision

2m A u1
Velocity of block B after collision v 2 = m + m
A B

1 1 é 4m2A u12 ù 2m2A mB


KE of block B = mB v 22 = m B ê ú = u2
2 2 (
ëê Am + m B ) 2
ûú ( m A + m B ) 2 1

which is maximum if mA = mB

Illustration 3.
An object is moving through air at a speed v. If the area of the object normal to the direction of velocity is
251\D\Allen(IIT-JEE Wing)\2020-21\Nurture\Phy\Unit-03\01-Centre of Mass

A and assuming elastic collision with the air molecules, then the resistive force on the object is proportional
to– (assume that molecules striking the object were initially at rest)
(A) 2Av (B) 2Av2 (C) 2Av1/2 (D) Can't be determined
Ans. (B)
Solution
Velocity of air molecule after collision = 2v . The number of air– molecules accelerated to a velocity 2v in time

Dp æ 2v ö
Dt is proportional to AvDt. Therefore F = µ (AvDt) çè ÷ø Þ F µ 2Av2
Dt Dt

43
JE E-Physics
Illustration 4.
The magnitude of acceleration of centre of mass of the system is

m=0.2 5kg

5kg

(A) 4 m/s2 (B) 10 m/s2 (C) 5 m/s2 (D) 2 Ö2 m/s2


Ans. (D)
Solution
r r
Net force on system 5g – m ( 5g) 50 (1 - 0.2 ) m1a1 + m2 a2 a
a= = = = 4 m/s ; a cm =
2
= = 2 2 m/s 2
total mass of system 5+5 10 m1 + m2 2

Illustration 5.
For shown situation find the maximum elongation in the spring. Neglect friction everywhere. Initially, the
blocks are at rest and spring is unstretched.
F K
3m 6m F
2

4F 3F 4F 2F
(A) (B) (C) (D)
3K 4K K K
Ans. (A)
Solution
At the instant of maximum extension, the relative velocity of one block w.r.t. another block must be zero.
® ® ®
a rel = a 2 - a 1
arel = a2 + a1
æ F - Kx ö æ F - 2Kx ö
arel = ç ÷+ç ÷
è 6m ø è 6m ø

dv rel æ 2F - 3Kx ö
v rel =ç ÷
dx è 6m ø

0 x max
(2F - 3Kx)
ò
0
v rel dv rel = ò
0
6m
dx

x 2max
0 = 2Fx max - 3K
2
4F
x max =
3K
251\D\Allen(IIT-JEE Wing)\2020-21\Nurture\Phy\Unit-03\01-Centre of Mass

Illustration 6.
A small sphere of mass 1kg is moving with a velocity (6i$ + $j) m/s. It hits a fixed smooth wall and rebounds with

velocity (4i$ + $j) m/s . The coefficient of restitution between the sphere and the wall is-

3 2 9 4
(A) (B) (C) (D)
2 3 16 9
Ans. (B)

44
Centre of Mass
Solution
Impulse = Change in momentum = 1(4i$ + $j) - 1(6i$ + $j) = -2i$
Which is perpendicular to the wall.
Component of initial velocity along $i = 6i$ Þ Speed of approach = 6 m/s

4 2
Similarly speed of separation = 4ms–1 Þ e = =
6 3

Illustration 7.
y
Two smooth balls A and B, each of mass m and radius R, have their
centre at (0, 0, R) and (5R, –R, R) respectively, in a coordinate system as
shown. Ball A, moving along positive x-axis, collides with ball B. Just x(m)
before the collision, speed of ball A is 4 m/s and ball B is stationary. The
collision between the balls is elastic. Velocity of the ball A just after the A
collision is B

(a) (ˆi + 3ˆj ) m/s (b) (ˆi - 3ˆj ) m/s (c) ( 2iˆ + 3ˆj ) m/s (d) ( 2iˆ + 2jˆ) m/s
Ans. (A)
Solution
4sin30°

A
A
4 m/s 60°
30° 30°
R R
R

B B 4cos30°

Before collision After collision

v A = 4 sin 30° éë cos 60iˆ + sin 60jˆ ùû = ˆi + 3jˆ

Illustration 8.
Find the center of mass (x,y,z) of the following structure of four identical cubes if the length of each side of a cube
is 1 unit.

(A) (1/2,1/2,1/2) (B)(1/3,1/3,1/3) (C) (3/4,3/4,3/4) (D)(1/2,3/4,1/2)


251\D\Allen(IIT-JEE Wing)\2020-21\Nurture\Phy\Unit-03\01-Centre of Mass

Ans. (C)
Solution
First we find the center of mass of each cube. It is located by symmetry: (0.5,0.5,0.5), (1.5,0.5,0.5), (0.5,1.5,0.5),
(0.5,0.5,1.5) . Now we find the center of mass by treating the COM of each cube as a point particle:
0.5 + 1.5 + 0.5 + 0.5 0.5 + 0.5 + 1.5 + 0.5
x COM = = 0.75 ; y COM = = 0.75
4 4
0.5 + 0.5 + 0.5 + 1.5
z COM = = 0.75
4

45
JE E-Physics

Illustration 9.
Two masses m and 2m are placed in fixed horizontal circular smooth hollow tube of radius r as shown. The
mass m is moving with speed u and the mass 2m is stationary. After their first collision, the time elapsed for
next collision. (coefficient of restitution e=1/2)
2p r 4 pr
(A) (B) 2m
u u
3pr 12pr m u
(C) (D)
u u
Ans. (B)
Solution
Let the speeds of balls of mass m and 2m after collision be v 1 and v2 as shown in figure. Applying conservation
u u
of momentum mv1 + 2mv2=mu & – v1+v2= . Solving we get v1=0 and v2=
2 2

u 2pr 4 pr
Hence the ball of mass m comes to rest and ball of mass 2m moves with speed .t = =
2 u/2 u

Illustration 10.
Find the x coordinate of the centre of mass of the bricks shown in figure :

l
l 6 m
l 4 m
2 m
m x

24 25 15 16
(A) l (B) l (C) l (D) l
25 24 16 15
Ans. (B)
Solution

æ lö æ l lö æ l l lö æ l l l lö
mç ÷ + mç + ÷ + mç + + ÷ + mç + + + ÷
è 2ø è 2 2ø è 2 4 2ø è 2 4 6 2 ø 25
X cm = = l
m+m+m+m 24

Illustration 11.
Object A strikes the stationary object B with a certain given speed u head–on in an elastic collision. The mass of
A is fixed, you may only choose the mass of B appropriately for following cases. Then after the collision :
(A) For B to have the greatest speed, choose mB = mA
(B) For B to have the greatest momentum, choose mB << mA
(C) For B to have the greatest speed, choose m B<<mA
(D) For the maximum fraction of kinetic energy transfer, choose mB = mA
Ans. (B,C,D)
Solution
251\D\Allen(IIT-JEE Wing)\2020-21\Nurture\Phy\Unit-03\01-Centre of Mass

vB - v A 2m A u
mAu = mAvA + mBvB and e = 1 = Þ vB =
u mA + mB
For mA >> mB , vB = 2u
For mA = mB, vB = u
For mA <<mB, vB = 0

1 2m B u2
kinetic energy K B = mB v 2B = 2
2 æ mB ö
çè 1 +
m A ÷ø
46
Centre of Mass
Illustration 12.
A man is sitting in a boat floating in water of a pond. There are heavy stones placed in the boat.
(A) When the man throws the stones in water from the pond, the level of boat goes down.
(B) When the man throws the stones in water from the pond, the level of boat rises up.
(C) When the man drinks some water from the pond, the level of boat goes down
(D) When the man drinks some water from the pond, the level of boat remains unchanged.
Ans. (B,D)
Solution
For (A/B) : Force of buoyancy increases. Therefore level of boat rises up.
For (C/D): When man drinks some water, the level of boat remains unchanged.

Illustration 13.
Two blocks A and B are joined together with a compressed spring. When the system is released, the two blocks
appear to be moving with unequal speeds in the opposite directions as shown in figure.
Select incorrect statement(s) : 15m/s
10m/s
(A) The centre of mass of the system will remain stationary. K=500Nm -1
(B) Mass of block A is equal to mass of block B. A B
(C) The centre of mass of the system will move towards right.
(D) It is an impossible physical situation.
Ans. (BCD)
Solution
As net force on system = 0 (after released)
So centre of mass of the system remains stationary.

Illustration 14.
In which of the following cases, the centre of mass of a rod may be at its centre?
(A) The linear mass density continuously decreases from left to right.
(B) The linear mass density continuously increases from left to right.
(C) The linear mass density decreases from left to right upto centre and then increases.
(D) The linear mass density increases from left to right upto centre and then decreases.
Ans. (CD)
Solution
By symmetry centre of mass will be at centre of ord for option C and D.

Illustration 15.
A man of mass 80 kg stands on a plank of mass 40 kg. The plank is lying on a smooth horizontal floor. Initially
both are at rest. The man starts walking on the plank towards north and stops after moving a distance of 6 m on
the plank. Then
(A) The centre of mass of plank-man system remains stationary.
(B) The plank will slide to the north by a distance 4 m
(C) The plank will slide to the south by a distance 4 m
(D) The plank will slide to the south by a distance 12 m
Ans. (AC)
Solution 6m
Let x be the displacement of the plank.
Since CM of the system remains stationary
so 80 (6–x) = 40 x Þ 12 – 2x = x Þx = 4m south x north
251\D\Allen(IIT-JEE Wing)\2020-21\Nurture\Phy\Unit-03\01-Centre of Mass

Illustration 16.
A body moving towards a body of finite mass at rest, collides with it. It is impossible that
(A) both bodies come to rest
(B) both bodies move after collision
(C) the moving body stops and body at rest starts moving
(D) the stationary body remains stationary and the moving body rebounds
Ans. (AD)
Solution
For (A) : Momentum can't destroyed by internal forces.
For (D) : If mass of stationary body is infinite then the moving body rebounds.

47
JE E-Physics
Illustration 17.
Three interacting particles of masses 100 g, 200 g and 400 g each have a velocity of 20 m/s magnitude along the
positive direction of x-axis, y-axis and z-axis. Due to force of interaction the third particle stops moving. The velocity
of the second particle is (10ˆj + 5kˆ ) . What is the velocity of the first particle?

(A) 20iˆ + 20jˆ + 70kˆ (B) 10iˆ + 20jˆ + 8kˆ (C) 30iˆ + 10jˆ + 7kˆ (D) 15iˆ + 5jˆ + 60kˆ
Ans. (A)
Solution
r r r
Initial momentum = m1 v1 + m2 v 2 + m3 v 3 = 2iˆ + 4jˆ + 8kˆ
r r r r r
When the third particle stops the final momentum = m1 v1 + m 2 v 2 + m 3 v 3 = 0.1v 1 + 0.2 (10ˆj + 5kˆ ) + 0
r r
By principle of conservation of momentum 0.1 v1 + 2jˆ + kˆ = 2iˆ + 4jˆ + 8kˆ ; v1 = 20iˆ + 20jˆ + 70kˆ

Illustration 18 to 20.
A bullet of mass m is fired with a velocity 10 m/s at angle q with the horizontal. At the highest point of its
trajectory, it collides head-on with a bob of mass 3m suspended by a massless string of length 2/5 m and gets
embedded in the bob. After the collision the string moves through an angle of 60°.

18. The angle q is


(A) 53° (B) 37° (C) 45° (D) 30°

19. The vertical coordinate of the initial position of the bob w.r.t. the point of firing of the bullet is
9 9 24
(A) m (B) m (C) m (D) None of these
4 5 5
20. The horizontal coordinate of the initial position of the bob w.r.t. the point of firing of the bullet isn
9 24 9
(A) m (B) m (C) m (D) None of these
5 5 4
Solution
18. Ans. (B) y
l
l 60°
Velocity of combined mass just after collision v=0

5 10ms–1 m 3m v
m (10 cos q) = 4mv Þ v = cos q
2
1 q
x
But from energy conservation (4m)v2= 4mgl(1– cos 60°)
2

5 2 2 2 4
Þ v = gl = cos q Þ cos q = gl = 10 ´ = Þ q = 37°
2 5 5 5 5
19. Ans. (B)

u 2 sin2 q (100)(9 / 25) 9


H max = = = m
2g 20 5
20. Ans. (B)
251\D\Allen(IIT-JEE Wing)\2020-21\Nurture\Phy\Unit-03\01-Centre of Mass

æ 3ö æ 4ö
(100) ç ÷ ç ÷
R 2u 2 sin q cos q è 5 ø è 5 ø 24
= = = m
2 2g 10 5

Illustration 21.
A ball moving vertically downward with a speed of 10 m/s collides with a heavy platform. The platform moves
with a velocity of 5 m/s in downward direction. If e = 0.8, find the speed (in m/s) of the ball just after collision.
Ans. (1)

48
Centre of Mass
Solution

10 m/s v

5 m/s 5 m/s
Just before collision Just after collision

v 2 - v1 v+5
By definition of e : e = ; we have 0.8 = Þ v = 1 m/s
u1 - u 2 10 - 5
\\\\\\\\\\\\\\\\

Illustration 22. 1kg


In the shown figure, the heavy block of mass 2 kg rests on the horizontal surface and

0.9m
the lighter block of mass 1 kg is dropped from a height of 0.9 m. At the instant the

2.2m
string gets taut, find the upward speed (in m/s) of the heavy block.
Ans. 2
Solution
2kg
Velocity of lighter block at the instant the string just gets taut \\\\\\\\\\\\\\\\\\\\\\\\\\\\\

v= 2gh = 2 ´ 10 ´ 1.8 = 6 m/s

v 6
Now by impulse - momentum theorem, let common speed be v 1 then (2+1) v1= (1) v Þ v 1 = = = 2 m/s
3 3
Illustration 23.
1
Two balls of equal mass have a head-on collision with speed 6 m/s. If the coefficient of restitution is , find the
3
speed of each ball after impact in m/s.
Ans. 2
Solution
6m/s 6m/s
Just before collision A B
\\\\\\\\\\\\\\\\\\\\\\\\\\\\\\\\\\\\\\\\\\\\\\\\\\\\\\\\\\\\\\\\\\\\\\\\\\\\\\\

v A B v
Just after collision
\\\\\\\\\\\\\\\\\\\\\\\\\\\\\\\\\\\\\\\\\\\\\\\\\\\\\\\\\\\\\\\\\\\\\\\\\\\\\\\

v 2 - v1 1 v+v
By definition of e : e= Þ = Þ v = 2 m/s
u1 - u 2 3 6+6

Illustration 24.
A thin rod of length 6 m is lying along the x-axis with its ends at x=0 and x = 6 m. Its linear density (mass/length)
varies with x as kx4. Find the position of centre of mass of rod in meters.
Ans. 5
Solution
6
æ x6 ö
ò xdm = ò x ( kx dx) = ò
6 6
4 5
x dx çè ÷ø
6 0
x cm = 0 0
= = 5m
251\D\Allen(IIT-JEE Wing)\2020-21\Nurture\Phy\Unit-03\01-Centre of Mass

ò dm ò ( kx dx) ò
6 6 6
4
x 4 dx æ x5 ö
0 0 çè ÷ø
5 0
Illustration 25.
The friction coefficient between the horizontal surface and blocks A and B
1 2 A B
are and respectively. The collision between the blocks is perfectly
15 15 m 4m/s m
elastic. Find the separation (in meters) between the two blocks when they \\\\\\\\\\\\\\\\\\\\\\\\\\\\\\\\\\\\\\\\\\\\\\\\\\\\\\\\\\\\\\\\\\\\\\\\\\\\\\\

come to rest. 2m
Ans. 5

49
JE E-Physics
Solution

æ 1ö 40
Velocity of block A just before collision v A = u A - 2mgx = 16 - 2 çè ÷ø (10 ) ( 2) =
2
15 3

40
Velocity of Block B just after collision vB = vA =
3
Velocity of Block A just after collision = 0

v B2 40 / 3
Total distance travelled by block B = = = 5m
2mg æ 2 ö( )
2 ç ÷ 10
è 15 ø

–1
u=10ms
Illustration 26.
A ball of mass 1 kg is projected horizontally as shown in figure. Assume that
collision between the ball and ground is totally inelastic. The kinetic energy of ball 5m
(in joules) just after collision is found to be 10a. Find the value of a.
Ans. 5
Solution
Vertical velocity just before collision vy = 2gh = 2 ´ 10 ´ 5 = 10 m/s
10ms–1 10ms–1
–1
10ms

1
Þ Kinetic energy of ball just after collision = × 1 × 102 = 50 J
2

Illustration 27. F
A body of mass 1 kg moving with velocity 1 m/s makes an elastic one dimensional
collision with an identical stationary body. They are in contact for brief time 1
sec. Their force of interaction increases from zero to F0 linearly in time 0.5 s and F0
decreases linearly to zero in further time 0.5 sec as shown in figure. Find the
magnitude of force F0 in newton.
t
O 0.5s 1s
Ans. 2
Solution
In the one dimensional elastic collision with one body at rest, the body moving initially comes to rest & the one
which was at rest earlier starts moving with the velocity that first body had before collision.
so, if m & V0 be the mass & velocity of body,
2mV0
the change in momentum = mV0 Þ ò Fdt = mV0 Þ ò Fdt = mV0 Þ F = = 2N
Dt

Illustration 28.
An 80 kg man is riding on a 40 kg cart travelling at a speed of 2.5 m/s on a frictionless horizontal plane. He
jumps off the cart, such that, his velocity just after jump is zero with respect to ground. The work done by him
251\D\Allen(IIT-JEE Wing)\2020-21\Nurture\Phy\Unit-03\01-Centre of Mass

A
on the system during his jump is given as KJ (AÎ integer). Find the value of A.
4
Ans. 3
Solution
By conservation of linear momentum (80 + 40) (2.5) = 80 (0) + 40 (v) Þ v = 7.5 m/s
1 1
40 ( 7.5) - (80 + 40) (2.5 ) = 750 J
2 2
work done = DKE =
2 2

50
Centre of Mass

ANSWERS
BEGINNER'S BOX-1
1. (A) 2. (A) 3. (C) 4. (B) 5. (A) 6. (D) 7. (C)
8. (5a/6, 5a/6) 9. 22L/35
10. At R/5 from the centre of the bigger disc towards the centre of the smaller disk.

BEGINNER'S BOX-2
1. (D) 2. (C) 3. (C) 4. (B) 5. (A) 6. (B,C) 7. (B) 8. (2m)

BEGINNER'S BOX-3
1. (C) 2. (C) 3. (A) 4. (B) 5. (A) 6. (C) 7. (30cm)

BEGINNER'S BOX-4
1. (B) 2. (C) 3. (ABCD) 4. (C) 5. (BC) 6. (A)
7. (B) 8. (D)

BEGINNER'S BOX-5
1. (A) 2. (A) 3. (C) 4. (A) 5. (D) 6. (C) 7. (A)

BEGINNER'S BOX-6
1. (C) 2. (A) 3. (C) 4. F ext = 10N; P = 20 watt.
5. (19.2 + 16 t) N 6. (C) 7. (B)
251\D\Allen(IIT-JEE Wing)\2020-21\Nurture\Phy\Unit-03\01-Centre of Mass

51
JE E-Physics

SINGLE CHOICE CORRECT QUESTIONS


1. The centre of mass of a non uniform rod of length L whose mass per unit length varies as r = kx2/L (where k is
a constant and x is the distance measured from one end) is at the following distance from the same end.
(A) 3L/4 (B) L/4 (C) 2L/3 (D) L/3

2. A uniform wire of length l is bent into the shape of 'V' as shown. The distance of its centre of mass from the
vertex A is
B
l 3
(A) l / 2 (B)
4 A 600

l 3
(C) (D) None of these C
8

3. A square plate of edge d and a circular disc of diameter d are placed touching each other at the midpoint of an
edge of the plate as shown in the figure. The centre of mass of the combination, assuming same mass per unit
area for the two plates from the centre of the disk is
4d 4d
(A) towards right (B) towards left
4+p 4+p
4d 4d
(C) towards right (D) towards left d d
4-p 4-p

4. Considering a system having two masses m 1 and m2 in which first mass is pushed towards centre of mass by
a distance a, the distance required to be moved for second mass to keep centre of mass at same position is

m1 m2
a

m1 m1 m2 m2 æ m2 m1 ö
(A) m a (B) (C) m a (D) ç a
2 a 1 è m1 + m2 ÷ø

5. An isolated particle of mass m is moving in horizontal plane (x–y), along the x–axis, at a certain height above
m 3m
the ground. It suddenly explodes into two fragment of masses and . An instant later, the smaller
4 4
fragment is at y = +15 cm. The larger fragment at this instant is at :–
(A) y = –5 cm (B) y = +20 cm (C) y = +5 cm (D) y = –20 cm

6. The velocity of centre of mass of the system as shown in the figure 1kg 2m/s
y
æ2 - 2 3ö æ 2 + 2 3ö
ˆi - 1 ˆj ˆi - 2 ˆj
(A) ç 3 ÷ø 3 (B) ç 3 ÷ø 3 x’ x 2 kg
è è 0
30
251\D\Allen(IIT-JEE Wing)\2020-21\Nurture\Phy\Unit-03\01-Centre of Mass

(C) 4iˆ (D) None of these y’


2m/s
7. The figure shows the positions and velocities of two particles. If the particles move under the mutual attraction
of each other, then the position of centre of mass at t =1 s is
5m/s 3m/s
1kg 1kg
x=2m x=8m
(A) x=5m (B) x=6m (C) x=3m (D) x=2m
52
Centre of Mass
8. A particle of mass 2m is connected by an inextensible string of length 1.2 m to a ring of mass m which is free to
slide on a horizontal smooth rod. Initially the ring and the particle are at the same level with the string taut. Both
are then released simultaneously. The distance in meters moved by the ring when the string becomes vertical is
(A) 0 (B) 0.4 (C) 0.8 (D) 1.2

9. A particle of mass m is made to move with uniform speed v0 along the perimeter of a regular hexagon, inscribed
in a circle of radius R. The magnitude of impulse applied at each corner of the hexagon is
(A) 2mv0sinp/6 (B) mv0sinp/6 (C)mv0sinp/3 (D) 2mv0sinp/3

10. Two balls of same mass are dropped from the same height h, on to the floor. The first ball bounces to a height
h/4 ,after the collision & the second ball to a height h/16. The impulse applied by the first & second ball on the
floor are I1 and I2 respectively. Then
(A) 5I1 = 6I2 (B) 6I1 = 5I2 (C) I1 = 2I2 (D) 2I1 = I2

11. A particle of mass 4m which is at rest explodes into masses m, m & 2m. Two of the fragments of masses m and
2m are found to move with equal speeds v each in opposite directions. The total mechanical energy released in
the process of explosion is
(A) mv2 (B) 2mv2 (C) 1/2 mv2 (D) 4mv2

12. Two blocks A(3 kg) and B(2 kg) resting on a smooth horizontal surface is connected by a spring of stiffness
480N/m. Initially the spring is undeformed and a velocity of 2 m/s is imparted to A along the line of the spring
away from B. The maximum extension in meters of the spring during subsequent motion is
1 1 1
(A) (B) (C) (D) 0.15
10 2 10 2 15

13. A shell is fired from a cannon with a velocity v (m/s) at an angle q with the horizontal direction. At the highest
point in its path it explodes into two pieces of equal mass. One of the pieces retraces its path to the cannon and
the speed (m/s) of the other piece immediately after the explosion is :–

3 3
(A) 3vcosq (B) 2vcosq (C) vcosq (D) vcosq
2 2

14. A particle moving horizontally collides with a fixed plane inclined at 60o to the horizontal. If it bounces vertically,
the coefficient of restitution is:
1 2 1
(A) (B) (C) (D) None of these
3 3 3

15. A ball of mass 1 kg strikes a heavy platform, elastically, moving upwards with a velocity 10 m/s
1kg
of 5m/s. The speed of the ball just before the collision is 10m/s downwards. Then the
impulse imparted by the platform on the ball is (e = 1)
5 m/s
(A) 15 N–s (B) 10 N–s
(C) 20 N–s (D) 30 N–s

16. Two particles of mass m, constrained to move along the circumference of a smooth
v0 v0
circular hoop of equal mass m, are initially located at opposite ends of a diameter
and given equal velocities v0 shown in the figure. The entire arrangement is located
in gravity free space. Their velocity just before collision is
251\D\Allen(IIT-JEE Wing)\2020-21\Nurture\Phy\Unit-03\01-Centre of Mass

1 3 2 7
(A) v0 (B) v (C) v0 (D) v
3 2 0 3 3 0

17. A projectile of mass 3m explodes at highest point of its path. It breaks into three equal parts. One part
retraces its path, the second one comes to rest. The range of the projectile was 100 m if no explosion
would have taken place. The distance of the third part from the point of projection when it finally lands on
the ground is–
(A) 100 m (B) 150 m (C) 250 m (D) 300 m

53
JE E-Physics
18. Two men 'A' and 'B' are standing on a plank. 'B' is at the middle of the plank
40kg 60kg
and 'A' is at the left end of the plank. Surface of the plank is smooth. A B
System is initially at rest and masses are as shown in figure. A and B starts
moving such that the position of 'B' remains fixed with respect to ground
then 'A' meets 'B'. Then the point where A meets B is located at–
(A) the middle of the plank smooth
40kg
(B) 30 cm from the left end of the plank
120cm
(C) the right end of the plank
(D) None of these

19. The diagram shows the velocity–time graph for two masses v(m/s)
R and S that collided elastically. Which of the following 1.2
R S
statements is true ?
0.8
I. R and S moved in the same direction after the collision
II. The velocities of R and S were equal at the mid time 0.4
of the collision.
1 2 3 4 t(s)
III. The mass of in was greater than mass of S.
(A) I only (B) II only (C) I and II only (D) I, II and III

20. A system of two blocks A and B are connected by an inextensible massless strings as shown. The pulley is
massless and frictionless. Initially the system is at rest when, a bullet of mass 'm' moving with a velocity 'u' as
shown hits the block 'B' and gets embedded into it. The impulse imparted by tension force to the block of mass
3m is–

5mu 4mu
(A) (B) m
4 5
u
2mu 3mu m B
(C) (D) A 3m
5 5

21. An arrow sign is made by cutting and rejoining a quarter part of a square plate of
side 'L' as shown. The distance OC, where 'C' is the centre of mass of the arrow, is

L L 3L
(A) (B) (C) (D) None of these
3 4 8

22. A block of mass M is tied to one end of a massless rope. The other
end of the rope is in the hands of a man of mass 2M as shown in the
figure. The block and the man are resting on a rough wedge of mass
M as shown in the figure. The whole system is resting on a smooth
2M
horizontal surface. The man pulls the rope. Pulley is massless and M
frictionless. What is the displacement of the wedge when the block 2m M
meets the pulley. (Man does not leave his position during the pull)
251\D\Allen(IIT-JEE Wing)\2020-21\Nurture\Phy\Unit-03\01-Centre of Mass

(A) 0.5m (B) 1m (C) Zero (D) 2/3 m

23. A continuous stream of particles of mass m and velocity v, is emitted from a source at a rate of n per second.
The particles travel along a straight line, collide with a body of mass M and get embedded in the body. If the
mass M was originally at rest, its velocity when it has received N particles will be

mvn mvN mv Nm + M
(A) (B) (C) (D)
Nm + n Nm + M Nm + M mv

54
Centre of Mass

24. The end ' A ' of a uniform rod AB of length ' l ' touches a horizontal smooth fixed surface. Initially the rod
makes an angle of 30° with the vertical. Find the magnitude of displacement of the end B just before it
touches the ground after the rod is released.

13 l 13 l l
(A) (B) (C) (D) l
2 4 2

25. A spherical ball of mass 1 kg moving with a uniform speed of 1 m/s collides symmetrically with two identical
spherical balls of mass 1 kg each at rest touching each other. If the two balls move with 0.5m/s in two directions
at the same angle of 60° with the direction of the first ball, the loss of kinetic energy on account of the collision is :
(A) 0.125 J (B) 0.5J (C) 1.0 J (D) 0.75J

26. If both the blocks as shown in the given arrangement are given together a 1kg µ=0.1
horizontal velocity towards right. If acm be the subsequent acceleration of µ=0.2
2 kg
the centre of mass of the system of blocks then acm equals
5 7
(A) 0 m/s2 (B) m/s2 (C) m/s2 (D) 2 m/s2
3 3

27. A bead of mass m and diameter d is sliding back and forth with velocity v on a wire held between two rigid walls
of length L. Assume that the collisions with the wall are perfectly elastic and there is no friction. The average
force that the bouncing bead exerts on the one of the walls is

mv 2 mv 2 2m v 2 2m v 2
(A) (B) (C) (D)
L-d L+d L-d L+d

A
28. A particle of mass m = 0.1 kg is released from rest from a point A of a wedge
of mass M = 2.4 kg free to slide on a frictionless horizontal plane. The particle
m
slides down the smooth face AB of the wedge. When the velocity of the wedge is
v
0.2 m/s the velocity of the particle in m/s relative to the wedge is V
M 60 0
B
(A) 4.8 (B) 5
(C) 7.5 (D) 10

® ®
29. Two particles of equal mass have velocities v 1 = 2iˆ m/s and v 2 = 2ˆj m/s. First particle has an acceleration

®
a1 = (3iˆ + 3ˆj) m/s 2, while the acceleration of the other particle is zero. The centre of mass of the two
particles moves in a-
(A) circle (B) parabola (C) straight line (D) ellipse

30. Two identical rods are joined at one of their ends by a pin. Joint is smooth
and rods are free to rotate about the joint. Rods are released in vertical plane
on a smooth surface as shown in the figure. The displacement of the joint
251\D\Allen(IIT-JEE Wing)\2020-21\Nurture\Phy\Unit-03\01-Centre of Mass

from its initial position to the final position is (i.e. when the rods lie straight on
the ground) :

L 17 5L
(A) (B) L (C) (D) none of these
4 4 2

55
JE E-Physics

SECTION - 1 : MULTIPLE CHOICE CORRECT QUESTIONS


1. A set of n–identical cubical blocks lies at rest parallel to each other along a line on a smooth horizontal surface.
The separation between the near surfaces of any two adjacent blocks is L. The block at one end is given a speed
v towards the next one at time t = 0. All collisions are completely inelastic, then
L
(A) The last block starts moving at t = n (n–1)
2v

L
(B) The last block starts moving at t = (n–1)
v

v
(C) The centre of mass of the system will have a final speed
n
(D) The centre of mass of the system will have a final speed v.

2. A ball moving with a velocity v hits a massive wall moving towards the ball with a velocity u. An elastic impact
lasts for a time Dt.
m(u + v)
(A) The average elastic force acting on the ball is
Dt
2m(u + v)
(B) The average elastic force acting on the ball is
Dt
(C) The kinetic energy of the ball increases by 2mu(u + v)
(D) The kinetic energy of the ball remains the same after the collision

3. A particle moving with kinetic energy = 3J makes an elastic head–on collision with a stationary particle which
has twice its mass. During the impact,
(A) the minimum kinetic energy of the system is 1J.
(B) the maximum elastic potential energy of the system is 2J.
(C) momentum and total energy are conserved at every instant.
(D) the ratio of kinetic energy to potential energy of the system first decreases and then increases.

4. A smooth sphere A of mass m collides elastically with an identical sphere B at rest. The velocity of A before
collision is 8 m/s in a direction making 60° with the line of centres at the time of impact.
(A) The sphere A comes to rest after collision.
(B) The sphere B will move with a speed of 8 m/s after collision.
(C) The directions of motion A and B after collision are at right angles.
(D) The speed of B after collision is 4 m/s.

5. In a one dimensional collision between two identical particles A and B, B is stationary and A has momentum p
before impact. During impact, B gives impulse J to A.
(A) The total momentum of the 'A plus B' system is p before and after the impact, and (p–1) during the impact.
(B) During the impact A gives impulse J to B
251\D\Allen(IIT-JEE Wing)\2020-21\Nurture\Phy\Unit-03\01-Centre of Mass

2J
(C) The coefficient of restitution is -1
p

J
(D) The coefficient of restitution is +1
p

56
Centre of Mass
6. When a block is placed on a wedge as shown in figure, the block starts sliding rough
down and the wedge also start sliding on ground. All surfaces are rough. The
centre of mass of (wedge + block) system will move Block
(A) leftward and downward.
Wedge
(B) right ward and downward.
(C) leftward and upwards. rough
(D) only downward.

7. A particle strikes a horizontal smooth floor with a velocity u making an angle q with the floor and rebounds with
velocity v making an angle f with the floor. If the coefficient of restitution between the particle and the floor is e,
then :
(A) the impulse delivered by the floor to the body is mu (1 + e) sin q.
(B) tan f = e tan q.

( 2
)
2
(C) v = u 1 - 1 - e sin q .
(D) the ratio of the final kinetic energy to the intial kinetic is (cos2q + e2 sin2q)

8. A ball is projected from a point in one of the two smooth parallel vertical walls
against the other in a plane perpendicular to both after being reflected at each B
wall impinge again on the second at a point in the same horizontal plane as is
started. The distance between two walls is a,b is the free range on a horizontal A
plane and e be the coefficient of restitution
v
a a
(A) The total time taken in moving from O to C is 2 (e 2 + e + 1) O u C
e u
2uv
(B) The free range on the horizontal plane b = g
(C) be2 = a (e 2 + e + 1)
(D) All above options are correct

9. Two blocks A and B each of mass m, are connected by a massless spring of natural length L and spring constant
k. The blocks are initially resting on a smooth horizontal floor with the spring at its natural length, as shown in
figure. A third identical block C, also of mass m, moves on the floor with a speed v along the line joining A and
B, and collides elastically with A. Then :

(A) the kinetic energy of the A-B system, at maximum compression of the spring, is zero
(B) the kinetic energy of the A-B system, at maximum compression of the spring, is mv 2/4
(C) the maximum compression of the spring is v m / K

m
(D) the maximum compression of the spring is v
2K
251\D\Allen(IIT-JEE Wing)\2020-21\Nurture\Phy\Unit-03\01-Centre of Mass

10. Three blocks A, B and C each of mass m are placed on a surface as shown in the figure. Blocks B and C are
initially at rest. Block A is moving to the right with speed v. It collides with block B and sticks to it. The A–B
combination collides elastically with block C. Which of the following statement is (are) true about the velocity, of
block B and C. m m m
(A) Velocity of the block C after collision is 2/3 v towards right A B C
(B) Velocity of the A–B combination after collision is v/3 towards left
(C) Velocity of the A–B combination after collision is 2/3 v towards left
(D) Velocity of the block C after collision is v/3 towards right.

57
JE E-Physics
SECTION - 2 : COMPREHENSION BASED QUESTIONS
(SINGLE CHOICE CORRECT QUESTION)
Comprehension-1
When two bodies collide normally they exert equal and opposite impulses on each other. Impulse = change in
linear momentum. Coefficient of restitution between two bodies is given by :–
|Re lative velocity of separation|
e= = 1, for elastic collision
|Re lative velocity of approach|
2kg
1kg
6m/s 4m/s

Two bodies collide as shown in figure. During collision they exert impulse of magnitude J on each other.
11. If the collision is elastic, the value of J is ............... N–s :
(A) 10/3 (B) 5/4 (C) 8/3 (D) 3/2
12. For what values of J (in N–s) the 2 kg block will change its direction of velocity :
(A) J < 12 (B) J > 12 (C) J < 10 (D) J > 10

Comprehension-2
An initially stationary box on a frictionless floor explodes into two pieces, piece A with mass m A and piece B
with mass mB. Two pieces then move across the floor along x–axis. Graph of position versus time for the two
pieces are given.
x A x x x x
x A
A A A
q 2q q
t t t t q t
B q q q
2q q 2q B
q t A
B B B B
(I) (II) (III) (IV) (V) (VI)
13. Which graphs pertain to physically possible explosions ?
(A) II, IV, V (B) VI (C) I, III (D) I, IV

14. Based on the above question, Match column A with the column B.
Column B (Graph number)
Column A
(P) mA = mB I
(Q) mA > mB II
(R) mA < mB III
IV
V
VI
(A) P – VI, Q – III, R – I (B) P – II, Q – V, R – IV
(C) P – II, Q – IV, R – V (D) P – VI, Q – II, R – IV

15. If all the graphs are possible then, in which of the following cases external force must be acting on
the box :-
(A) Only II (B) Only V (C) Only VI (D) I and VI
251\D\Allen(IIT-JEE Wing)\2020-21\Nurture\Phy\Unit-03\01-Centre of Mass

Comprehension-3
Two blocks A and B of masses m and 2m respectively are connected by a spring
A B C
of spring constant k. The masses are moving to the right with a uniform velocity k
m 2m 2m
v0 each, the heavier mass leading the lighter one. The spring is of natural length
during this motion. Block B collides head on with a third block C of mass 2m.
at rest, the collision being completely inelastic.

16. The velocity of block B just after collision is-


v 3v 0 2v 0
(A) v0 (B) 0 (C) (D)
2 5 5
58
Centre of Mass
17. The velocity of centre of mass of system of block A, B & C is-

3v 0 2v 0 v0
(A) v0 (B) (C) (D)
5 5 2

18. The maximum compression of the spring after collision is -

mv 20 mv 20 mv 20
(A) (B) (C) (D) None of these
12k 5k 10k

Comprehension-4
If net force on a system in a particular direction is zero (say in horizontal
m
direction) we can apply:
åmRxR = åmLxL, åmRvR = åmLvL and åmRaR = åmLaL
Here R stands for the masses which are moving towards right and L for 4m
the masses towards left, x is displacement, v is velocity and a the M 2m
acceleration (all with respect to ground). A small block of mass m = 1 kg
is placed over a wedge of mass M = 4 kg as shown in figure. Mass m is O +x-axis
4m
released from rest. All surfaces are smooth. Origin O is as shown.

19. Final velocity of the wedge is ................. m/s :–


1 1
(A) 3 (B) 2 (C) (D)
2 3

20. The block will strike the x–axis at x = ................m :–


(A) 4.2 (B) 7.6 (C) 5.6 (D) 6.8

21. Normal reaction between the two blocks at an instant when absolute acceleration of m is 5 3 m/s2 at 60° with
horizontal is ........... N. Normal reaction at this instant is making 30° with horizontal :
(A) 6 (B) 10 (C) 4 (D) 5

22. At the same instant reaction on the wedge from the ground is .............. N.
(A) 42.5 (B) 40 (C) 43.46 (D) None of these
251\D\Allen(IIT-JEE Wing)\2020-21\Nurture\Phy\Unit-03\01-Centre of Mass

59
JE E-Physics

SECTION - 1 : NUMERICAL ANSWER BASED QUESTIONS

1. A man has constructed a toy as shown in fig. The density of the material of the sphere
is 12 times of the cone. The position of the centre of mass is at a distance of nR from O.
h
Find the value of n [Centre of mass of a cone of height h is at height of from its
4
base.]

2. The figure shows a square metal plate of side l from which a square plate of side y
a has been cut as shown in the figure. The centre of mass of the remaining

æ n - 1ö
L–shaped plate coincides with the point A. The ratio of (a/l) is ç 2 ÷ . Find the A
è ø a
o x
value of n. a

3. A hemisphere of radius R and of mass 4m is free to slide with its base on a smooth
horizontal table. A particle of mass m is placed on the top of the hemisphere. The m
angular velocity of the particle relative to hemisphere, when angular displacement of
xv 4m
particle is q and velocity of hemisphere has become v, is equal to . Find the q R
R cos q
value of x.

4. In the figure shown a small block B of mass m is released from the top of a
smooth movable wedge A of the same mass m. The height of wedge A
shown in figure is h = 100 cm. B ascends another movable smooth wedge C
of the same mass. Neglecting friction any where the maximum height (in cm)
attained by block B on wedge C is 20 + h . Find h :

5. Two bodies of same mass tied with an inelastic string of length l lie together. One of them is projected vertically
upwards with velocity 6gl .The maximum height up to which the centre of mass of system of the two masses

l
rises is equal to x × . Find value of x
2

6. Two bodies A and B of masses m and 2m respectively are placed on a smooth floor. They are connected by a
spring. A third body C of mass m moves with velocity v 0 along the line joining A and B and collides elastically
with A as shown in fig. At a certain instant of time t 0 after collision, it is found that the instantaneous velocities
of A and B are the same. Further at this instant the compression of the spring is found to be x 0. The common

v0 2mv 20
velocity of A and B at time t 0 is and the spring constant is . Find the value of n.
n n x 20
251\D\Allen(IIT-JEE Wing)\2020-21\Nurture\Phy\Unit-03\01-Centre of Mass

C A B

7. A sphere of mass m1 in motion hits directly another sphere of mass m 2 at rest and sticks to it, the total kinetic
energy after collision is 2/3 of their total K.E. before collision. If the ratio of m1 : m2 is equal to n : 1, find the
value of n.

60
Centre of Mass
8. A simple pendulum is suspended from a peg on a vertical wall. The pendulum
is pulled away from the wall to a horizontal position (see fig.) and released. L

2
The ball hits the wall, the coefficient of restitution being . What is the
5
minimum number of collisions after which the amplitude of oscillations
becomes less than 60 degrees?

9. Two masses A & B each of 5 kg are suspended by a light inextensible string


passing over a smooth massless pulley such that mass A rest on smooth table
& B is held at the position shown. Mass B is now gently lifted up to the pulley 1m

and allowed to fall from rest. Determine up to what height will A rise for the 2m B
ensuing motion.
A

10. A ball of mass m is released from A inside a smooth wedge of mass m as shown in the figure. The speed of the
1/2
æ gR ö
wedge when the ball reaches point B is ç . Find the value of x
è x 2 ÷ø

450
B

smooth

11. A small ring of mass m attached at an end of a light string the other end of
which is tied to a small block B of mass 2 m. The ring is free to move on a
fixed smooth horizontal rod. The velocity of the ring when the string becomes

x gl
vertical is . Find the value of x
3

SECTION - 2 : MATRIX - MATCH QUESTIONS


12. A particle of mass m, kinetic energy K and momentum p collides head on elastically with another particle of
mass 2 m at rest. After collision :
Column I Column II
(A) Momentum of first particle (p) 3/4 p
(B) Momentum of second particle (q) – K/9
(C) Kinetic energy of first particle (r) – p/3
8K
(D) Kinetic energy of second particle (s)
9
(t) None
251\D\Allen(IIT-JEE Wing)\2020-21\Nurture\Phy\Unit-03\01-Centre of Mass

13. In each situation of column–I, a system involving two bodies is given. All strings and pulleys are light and
friction is absent everywhere. Initially each body of every system is at rest. Consider the system in all situation
of column I from rest till any collision occurs. Then match the statements in column – I with the corresponding
results in column–II
Column I m Column II
(A) The block plus wedge system is placed over (p) Shifts towards right
smooth horizontal surface. After the system M
is released from rest, the centre of mass of
system

61
JE E-Physics

(B) The string connecting both the blocks m m (q) Shifts downwards
of mass m is horizontal. Left block is
placed over smooth horizontal table
as shown. After the two block system is released
from rest, the centre of mass of system

(C) The block and monkey have same mass. The (r) Shifts upwards
monkey starts climbing up the rope. After the
monkey starts climbing up, the centre of mass
of monkey + block system

(D) Both block of mass m are initially at rest. The (s) Does not shift
left block is given initial velocity u downwards.
Then, the centre of mass of two block system
afterwards
m m

14. Two blocks A and B of mass 2m and m respectively are connected by a massless spring of spring constant k.
This system lies over a smooth horizontal surface. At t = 0 the block A has velocity u towards right as shown
while the speed of block B is zero, and the length of spring is equal to its natural length at that instant. In each
situation of column–I, certain statements are given and corresponding results are given in column II.
B k A
m 2m u

smooth horizontal surface


Column I Column II
(A) The velocity of block A (p) Can never be zero
(B) The velocity of block B (q) May be zero at certain instants of time
(C) The kinetic energy of system of two blocks (r) is minimum at maximum compression of spring
(D) The potential energy of spring (s) Is maximum at maximum extension of spring

15. Collision between ball and block A is perfectly Rigid support


inelastic as shown. If impulse on ball Y
1kg
(at the time of collision) is J then
Column- I Column-II A 1kg

(A) Net impulse on block A is (p) J


(B) Net impulse on block B is (q) 4J/9
(C) Impulse due to rigid support Y is (r) 16J/9 B 2kg

(D) Impulse due to rigid support X is (s) 2J/9 Rigid support


X
(t) J/9

16. A smooth ball A of mass m is attached to one end of a light


inextensible string, and is suspended from fixed point O. Another
B
identical ball B, is dropped from a height h, so that the string just
251\D\Allen(IIT-JEE Wing)\2020-21\Nurture\Phy\Unit-03\01-Centre of Mass

h
touches the surface of the sphere.
Column I Column II

A
3m
(A) If collision between balls is completely elastic then (p) 2gh
5
speed of ball A just after collision is

62
Centre of Mass

6gh
(B) If collision between balls is completely elastic then (q)
5
impulsive tension provided by string is

6m
(C) If collision between balls is completely inelastic then (r) 2gh
5
speed of ball A just after collision is

2 6gh
(D) If collision between balls is completely inelastic then (s)
5
impulsive tension provided by string is
(t) None of these
251\D\Allen(IIT-JEE Wing)\2020-21\Nurture\Phy\Unit-03\01-Centre of Mass

63
JEE-Physics

SINGLE CHOICE CORRECT QUESTIONS


1. A block of mass 0.50 kg is moving with a speed of 2.00 ms–1 on a smooth surface. It strikes another mass of
1.00 kg and then they move together as a single body. The energy loss during the collision
is : [AIEEE - 2008]
(1) 0.16 J (2) 1.00 J (3) 0.67 J (4) 0.34 J

2. Consider a rubber ball freely falling from a height h = 4.9 m onto a horizontal elastic plate. Assume that the
duration of collision is negligible and the collision with the plate is totally elastic.Then the velocity as a function
of time and the height as a function of time will be :- [AIEEE - 2009]
v y
v y
+v1 +v1
h
h
(1) O t1 2t1 3t1 4t1
t (2) O t1 2t1 3t1 4t1
t

–v1 t
–v1 t

v
y v y
+v1
h +v1
h
(3) O t
t (4) O t

–v1 t

3. Directions : This Questions contain Statement-1 and Statement-2.


Of the four choices given after the statements, choose the one that best discribes the two statements.
Statement-1 : Two particles moving in the same direction do not lose all their energy in a completely inelastic
collision.
Statement-2 : Principle of conservation of momentum holds true for all kinds of collisions. [AIEEE - 2010]
(1) Statement–1 is true, Statement–2 is false
(2) Statement–1 is true, Statement–2 is true; Statement–2 is the correct explanation of Statement–1
(3) Statement–1 is true, Statement–2 is true; Statement–2 is not the correct explanation of Statement–1
(4) Statement–1 is false, Statement–2 is true

4. A projectile moving vertically upwards with a velocity of 200 m/s breaks into two equal parts at a height of
490 m. One part starts moving vertically upward with a velocity of 400 m/s. How much time after the break up
will the other part hit the ground ? [AIEEE - 2012, Online]
(1) 5 s (2) 2 10 s (3) 10 s (4) 10 s
5. A moving particle of mass m, makes a head on elastic collision with another particle of mass 2m, which is
initially at rest. The percentage loss in energy of the colliding particle on collision, is close to :-
[AIEEE - 2012, Online]
(1) 10% (2) 90% (3) 33% (4) 67%

6. Sand is being dropped on a conveyor belt at the rate of 2 kg per second. The force necessary to keep the belt
moving with a constant speed of 3 ms–1 will be :- [AIEEE - 2012, Online]
(1) 12 N (2) 6 N (3) 18 N (4) Zero
251\D\Allen(IIT-JEE Wing)\2020-21\Nurture\Phy\Unit-03\01-Centre of Mass

7. A projectile of mass M is fired so that the horizontal range is 4 km. At the highest point the projectile explodes in
two parts of masses M/4 and 3M/4 respectively and the heavier part starts falling down vertically with zero initial
speed. The horizontal range (distance from point of firing) of the lighter part is :- [JEE (Main) - 2013, Online]
(1) 10 km (2) 2 km (3) 16 km (4) 1 km
8. An engine pumps water continuously through a hose. Water leaves the hose with velocity v and m is mass per
unit length of the water jet. If this jet hits a surface and came to rest instantaneously, the force on the surface is:
[AIEEE ONLINE - 2012]
1 1
(1) mv2 (2) mv2 (3) mv3 (4) mv3
2 2
64
Centre of Mass
9. A boy of mass 20 kg is standing on a 80 kg free to move long cart. There is negligible friction between cart and
ground. Initially, the boy is standing 25 m from a wall. If he walks 10 m on the cart towards the wall, then the
final distance of the boy from thewall will be :- [JEE (Main) - 2013, Online]
(1) 15.5 m (2) 15 m (3) 12.5 m (4) 17 m

10. Directions : This Questions contain Statement-1 and Statement-2.


Of the four choices given after the statements, choose the one that best discribes the two statements.
This question has statement I and statement II. Of the four choices given after the statements, choose the one
that best describes the two statements. [JEE(Main) - 2013]
Statement-I : A point particle of mass m moving with speed n collides with stationary point particle of mass

M. If the maximum energy loss possible is given as f æç 1 mv 2 ö÷ then f = æç m ö÷ .


è2 ø è M + mø
Statement-II : Maximum energy loss occurs when the particles get stuck together as a result of the collision.
(1) Statement–1 is true, Statement–2 is false; Statement-II is the correct explanation of Statement-I.
(2) Statement–1 is true, Statement–2 is true; Statement–2 is not the correct explanation of Statement–2
(3) Statement–1 is true, Statement–2 is fasle
(4) Statement–1 is false, Statement–2 is true

11. A wind - powered generator converts wind energy into electrical energy. Assume that the generator converts a
fixed fraction of the wind energy intercepted by its blades into electrical energy. For wind speed u, the electrical
power output will be most likely proportional to : [JEE (Main) - 2013, Online]
(1) u (2) u4 (3) u2 (4) u3

12. A 4 g bullet is fired horizontally with a speed of 300 m/s into 0.8 kg block of wood at rest on a table. If the
coefficient of friction between the block and the table is 0.3, how far will the block slide approximately :-
[JEE (Main) - 2014, Online]
(1) 0.758 m (2) 0.19 m (3) 0.569 m (4) 0.379 m

13. Three masses m, 2m and 3m are moving in x-y plane with speed 3u, 2u, and u respectively as shown in figure.
The three masses collide at the same point at P and stick together. The velocity of resulting mass will be :-
[JEE (Main) - 2014, Online]
y
2m, 2u

60º x
m, 3u
P 60º

3m, u

(1)
12
(
u ˆ
- i - 3 ˆj ) (2)
12
(
u ˆ
)
- i + 3 ˆj (3)
12
(
u ˆ
i - 3 ˆj ) (4)
12
(
u ˆ
i + 3 ˆj )
14. A large number (n) of identical beads, each of mass m and radius r are strung on a thin smooth rigid horizontal
rod of length L (L>>r) and are at rest at random positions. The rod is mounted between two rigid supports (see
251\D\Allen(IIT-JEE Wing)\2020-21\Nurture\Phy\Unit-03\01-Centre of Mass

figure). If one of the beads is now given a speed v, the average force experienced by each support after a long time
is (assume all collisions are elastic) :- [JEE (Main) - 2015, Online]

mv 2 mv 2
(1) (2)
2 ( L - nr ) L - nr L

mv 2
(3) zero (4)
L - 2nr

65
JE E-Physics

15. Distance of the centre of mass of a solid uniform cone from its vertex is z0. If the radius of its base is R and its
height is h then z0 is equal to [JEE(Main) - 2015]

h2 3h 5h 3h2
(1) (2) (3) (4)
4R 4 8 8R

16. A block of mass m = 0.1 kg is connected to a spring of unknown spring constant k. It is compressed to a

æ xö
distance x from its equilibrium position and released from rest. After approaching half the distance ç ÷ from
è 2ø
equilibrium position, it hits another block and comes to rest momentarily, while the other block moves with a
velocity 3 ms–1. The total initial energy of the spring is :-
[JEE (Main) - 2015, Online]
(1) 0.8 J (2) 0.3 J (3) 1.5 J (4) 0.6 J

BC
17. In the figure shown ABC is a uniform wire If centre of mass of wire lies vertically below point A, then
is close
AB
to : [JEE (Main) - 2016, Online]
A

B 60° C
(1) 1.85 (2) 3 (3) 1.5 (4) 1.37

18. The mass of a hydrogen molecule is 3.32 × 10–27 kg. If 1023 hydrogen molecules strike, per second, a fixed wall
of area 2 cm2 at an angle of 45° to the normal, and rebound elastically with a speed of 103 m/s, then the
pressure on the wall is nearly : [JEE (Main) - 2018]
(1) 2.35 × 103 N/m2 (2) 4.70 × 103 N/m2 (3) 2.35 × 102 N/m2 (4) 4.70 × 102 N/m2

19. An alpha-particle of mass m suffers 1-dimensional elastic coolision with a nucleus at rest of unknown mass. It
is scattered directly backwards losing, 64% of its initial kinetic energy. The mass of the nucleus is :-
[JEE (Main) - 2019, Online]
(1) 4 m (2) 3.5 m (3) 2 m (4) 1.5 m

20. A liquid of density r is coming out of a hose pipe of radius a with horizontal speed v and hits a mesh. 50% of
the liquid passes through the mesh unaffected. 25% looses all of its momentum and 25% comes back with the
same speed. The resultant pressure on the mesh will be : [JEE (Main) - 2019, Online]
3 2 1 2 1 2
(1) pv2 (2) pv (3) pv (4) pv
4 2 4

21. Four particles A, B, C and D with masses mA = m, mB = 2m, mC = 3m a


Y
and mD = 4m are at the corners of a square. They have accelerations of equal
251\D\Allen(IIT-JEE Wing)\2020-21\Nurture\Phy\Unit-03\01-Centre of Mass

magnitude with directions as shown. The acceleration of the centre of mass B a


C
of the particles is : [JEE (Main) - 2019, Online]
X
(1)
5
( )
a ˆ ˆ
i-j (2)
5
( )
a ˆ ˆ
i+ j
D
a A
(3) Zero ( )
(4) a ˆi + ˆj a

66
Centre of Mass
22. A uniform rectangular thin sheet ABCD of mass M has length a and breadth b, as shown in the figure. If the
shaded portion HBGO is cut-off, the coordinates of the centre of mass of the remaining portion will be :-
[JEE (Main) - 2019, Online]

(0, b) H (a, b)
A B
a b
2 2
E G
O

D C
(0, 0) F (a, 0)

æ 2a 2b ö æ 5a 5b ö æ 3a 3b ö æ 5a 5b ö
(1) ç , ÷ (2) ç , ÷ (3) ç , ÷ (4) ç , ÷
è 3 3 ø è 3 3 ø è 4 4 ø è 12 12 ø

23. A wedge of mass M = 4m lies on a frictionless plane. A particle of mass m approaches the wedge with speed
v. There is no friction between the particle and the plane or between the particle and the wedge. The maximum
height climbed by the particle on the wedge is given by : [JEE (Main) - 2019, Online]

2v2 v2 2v2 v2
(1) (2) (3) (4)
7g g 5g 2g

24. A man (mass = 50 kg) and his son (mass = 20 kg) are standing on a frictionless surface facing each other.
The man pushes his son so that he starts moving at a speed of 0.70 ms –1 with respect to the man. The speed
of the man with respect to the surface is : [JEE (Main) - 2019, Online]
(1) 0.20 ms–1 (2) 0.14 ms–1 (3) 0.47 ms–1 (4) 0.28 ms–1

25. Three particles of masses 50 g, 100 g and 150 g are placed at the vertices of an equilateral triangle of side 1
m (as shown in the figure). The (x, y) coordinates of the centre of mass will be :
[JEE (Main) - 2019, Online]
Y

m3 = 150 g

50 g = m1 60º m2 = 100 g
0 0.5m 1.0m

æ 7 3 ö æ 3 5 ö æ 7 3 ö æ 3 7 ö
(1) çç 12 m, 8 m ÷÷ (2) çç 4 m, 12 m ÷÷ (3) çç 12 m, 4 m ÷÷ (4) çç 8 m, 12 m ÷÷
è ø è ø è ø è ø
251\D\Allen(IIT-JEE Wing)\2020-21\Nurture\Phy\Unit-03\01-Centre of Mass

26. Three point particles of masses 1.0 kg, 1.5 kg and 2.5 kg are placed at three
corners of a right angle triangle of sides 4.0 cm, 3.0 cm and 5.0 cm as shown 2.5 kg
in the figure. The center of mass of the system is at a point:
[JEE (Main) - 2020, Online] 5 cm
4 cm
(1) 1.5 cm right and 1.2 cm above 1 kg mass
(2) 0.9 cm right and 2.0 cm above 1 kg mass 1.0 kg 1.5 kg
(3) 0.6 cm right and 2.0 cm above 1 kg mass 3 cm
(4) 2.0 cm right and 0.9 cm above 1 kg mass

67
JE E-Physics
27. As shown in figure, when a spherical cavity (centred at O) of radius 1 is cut out of R
a uniform sphere of radius R (centred at C), the centre of mass of remaining (shaded)
part of sphere is at G, i.e, on the surface of the cavity. R can be detemined 1
G
by the equation : [JEE (Main) - 2020, Online] CO
(1) (R2 – R + 1) (2 – R) = 1 (2) (R2 + R – 1) (2 – R) = 1
(3) (R2 + R + 1) (2 – R) = 1 (4) (R2 – R – 1) (2 – R) = 1

28. The coordinates of centre of mass of a uniform flag shaped lamina (thin flat plate)
of mass 4kg. (The coordinates of the same are shown in figure) are : (0, 3) (2, 3)
[JEE (Main) - 2020, Online]
(1) (1.25m, 1.50m) (2, 2)
(1, 2)
(2) (1m, 1.75m)
(3) (0.75m, 0.75m)
(4) (0.75m, 1.75m) (0, 0) (1, 0)

2
æxö
29. A rod of length L has non-uniform linear mass density given by r(x) = a + b ç ÷ , where a and b are constants
èLø
and 0 £ x £ L. The value of x for the centre of mass of the rod is at :
[JEE (Main) - 2020, Online]

4æ a +b ö 3æ a + b ö 3 æ 2a + b ö 3 æ 2a + b ö
L L L L
3 çè 2a + 3b ÷ø 2 çè 2a + b ÷ø 2 çè 3a + b ÷ø 4 çè 3a + b ÷ø
(1) (2) (3) (4)

æ ˆi + ˆj ö
30. Two particles of equal mass m have respective initial velocities uiˆ and u ç ÷ . They collide completely
è 2 ø
inelastically. The energy lost in the process is : [JEE (Main) - 2020, Online]
3 2 1 2 2 2 1 2
(1) mu (2) mu (3) mu (4) mu
4 8 3 3

251\D\Allen(IIT-JEE Wing)\2020-21\Nurture\Phy\Unit-03\01-Centre of Mass

68
Centre of Mass

SINGLE CHOICE CORRECT QUESTIONS


1. A thin rod of length ‘L’ is lying along the x-axis with its ends at x = 0 and x = L. It linear density (mass/length)
n
æ xö
varieswith x as k ç ÷ where n can be zero or any positive number. If the position x CM of the centre of mass of
è Lø
the rod is plotted against ‘n’, which of the following graphs best approximates the depence of x CM on n?
[AIEEE - 2008]

(1) (2) (3) (4)

2. The figure shows the position-time (x-t) graph of one dimensional motion of a body of mass 0.4 kg. The
magnitude of each impulse is [JEE(Main) - 2010]

2
x(m)

0 2 4 6 8 10 12 14 16
t(s)

(1) 0.4 Ns (2) 0.8 Ns (3) 1.6 Ns (4) 0.2 Ns

dM
3. = av
A satellite moving with velocity v in a force free space collects stationary interplanetary dust at a rate of
dt
where M is the mass of (satellite + dust) at the instance. The instantaneous acceleration of the satellite is :
[AIEEE - 2012, Online]
av 2 2av 2 av 2
(1) –av2 (2) - (3) - (4) -
M M 2M

4. Two bodies A and B of mass m and 2 m respectively are placed on a smooth floor. They are connected by a
spring of negligible mass. A third body C of mass m is placed on the floor. The body C moves with a velocity v 0
along the line joining A and B and collides elastically with A. At a certain time after the collision it is found that
the instantaneous velocities of A and B are same and the compression of the spring is x 0. The spring constant k
will be :- [AIEEE - 2012, Online]
2
v0 2 æ v0 ö v0 v 02
(1) 2m x (2) m ç ÷ (3) m 2x (4) m
0 3 è x0 ø 0 x 02

5. Two point masses of mass m1 = fM and m2 = (1 – f)M (f < 1) are in outer space (far from gravitational
251\D\Allen(IIT-JEE Wing)\2020-21\Nurture\Phy\Unit-03\01-Centre of Mass

influence of other objects) at a distance R from each other. They move in circular orbits about their centre of
mass with angular velocities w1 for m1 and w2 for m2. In tha t case :- [AIEEE - 2012, Online]
(1) w1 = w2 and independent of f (2) fw1 = (1 – f) w2
(3) w1 = w2 and depend of f (4) (1 – f) w1 = fw2

6. A particle of mass m moving in the x direction with speed 2v is hit by another particle of mass 2m moving in the
y direction with speed v. If the collision is perfectly inelastic, the percentage loss in the energy during the collision
is close to [JEE(Main) - 2015]
(1) 44% (2) 50% (3) 56% (4) 62%

69
JE E-Physics
7. Two particles A and B of equal mass M are moving with the same speed v as
shown in the figure. They collide completely inelastically and move as a single Y
particle C. The angle q that the path of C makes with the X-axis is given by :
[JEE (Main) - 2017, Online]
C
1- 3 1- 2 q
tan q =
(1) tan q =
1+ 2
(2)
(
2 1+ 3 ) A 3 45°
X
0° B
3+ 2 3- 2
(3) tan q = (4) tan q =
1- 2 1- 2

8. In a collinear collision, a particle with an initial speed v0 strikes a stationary particle of the same mass. If the final
total kinetic energy is 50% greater than the original kinetic energy, the magnitude of the relative velocity between
the two particles, after collision, is: [JEE (Main) - 2018]

v0 v0 v0
(1) (2) 2 v0 (3) (4)
4 2 2
®
9. The position vector of the centre of mass r cm of an symmetric uniform bar of
negligible area of cross-section as shown in figure is : [JEE (Main) - 2019, Online]
® 13 5 ® 11 3
(1) r cm = Lxˆ + Ly
ˆ (2) r cm = L xˆ + Ly
ˆ
8 8 8 8
L
® 3 11 ® 5 13
(3) r cm = L xˆ + Lyˆ (4) r cm = L xˆ + ˆ
Ly L 2L 3L
8 8 8 8

10. A piece of wood of mass 0.03 kg is dropped from the top of a 100 m height building. At the same time, a bullet of
mass 0.02 kg is fired vertically upward, with a velocity 100 ms–1, from the ground. The bullet gets embedded in the
wood. Then the maximum height to which the combined system reaches above the top of the building before
falling below is : (g =10ms–2) [JEE (Main) - 2019, Online]
(1) 30 m (2) 10 m (3) 40 m (4) 20 m

11. Three blocks A, B and C are lying on a smooth horizontal surface, as shown in the figure. A and B have equal
masses, m while C has mass M. Block A is given an brutal speed v towards B due to which it collides with B
perfectly inelastically. The combined mass collides with C, also perfectly inelastically 5/6th of the initial kinetic
energy is lost in whole process. What is value of M/m ? [JEE (Main) - 2019, Online]
A B C
m m m
(1) 4 (2) 5 (3) 3 (4) 2

12. A particle of mass 'm' is moving with speed '2v' and collides with a mass '2m' moving with speed 'v' in the same
direction. After collision, the first mass is stopped completely while the second one splits into two particles each
of mass 'm', which move at angle 45° with respect to the origianl direction.
The speed of each of the moving particle will be : [JEE (Main) - 2019, Online]

(1) (
v/ 2 2 ) (2) 2 2v (3) 2v (4) v / 2
251\D\Allen(IIT-JEE Wing)\2020-21\Nurture\Phy\Unit-03\01-Centre of Mass

13. A particle of mass m is dropped from a height h above the ground. At the same time another particle of the
same mass is thrown vertically upwards from the ground with a speed of 2gh . If they collide head-on completely
h
inelastically, the time taken for the combined mass to reach the ground, in units of is :
g
[JEE (Main) - 2020, Online]
1 1 3 3
(1) (2) (3) (4)
2 2 4 2

70
Centre of Mass

r r
1. Two balls, having linear momenta p1 = piˆ and p2 = -piˆ , undergo a collision in free space. There is no
r r
external force acting on the balls. Let p1' and p2' be their final momenta. The following option(s) is (are)
NOT ALLOWED for any non–zero value of p, a1, a2, b1, b2, c1 and c2. [IIT-JEE 2008]
r ˆ ˆ ˆ r ˆ r ˆ r
(A) p1' = a1 i + b1 j + c1k (B) p1' = c1k (C) p1' = a1ˆi + b1ˆj + c1k (D) p1' = a1ˆi + b1ˆj
r r r r
p2' = a2ˆi + b2ˆj p2' = c2 kˆ p2' = a2ˆi + b2ˆj - c2kˆ p2' = a2ˆi + b1ˆj

Comprehension based questions* [IIT-JEE 2008]


A small block of mass M moves on a frictionless surface of an inclined plane, as shown in figure. The angle of
the incline suddenly changes from 60° to 30° at point B. The block is initially at rest at A. Assume that collisions
between the block and the incline are totally inelastic (g = 10 m/s 2)

A
M

60° B

30° C

3m 3 3m

2. The speed of the block at point B immediately after it strikes the second incline is :–
(A) 60 m/s (B) 45 m/s (C) 30 m/s (D) 15 m/s

3. The speed of the block at point C, immediately before it leaves the second incline is :–
(A) 120 m/s (B) 105 m/s (C) 30 m/s (D) 75 m/s

4. If collision between the block and the incline is completely elastic, then the vertical (upward) component of the
velocity of the block at point B, immediately after it strikes the second incline is :–
251\D\Allen(IIT-JEE Wing)\2020-21\Nurture\Phy\Unit-03\01-Centre of Mass

(A) 30 m/s (B) 15 m/s (C) 0 (D) – 15 m/s

5. Two small particles of equal masses start moving in opposite directions from a point A in V A 2V
a horizontal circular orbit. Their tangential velocities are v and 2v, respectively, as shown in
the figure. Between collisions, the particles move with constant speeds. After making how
many elastic collisions, other that at A, these two particles will again reach the point A ?
[IIT-JEE 2009]
(A) 4 (B) 3 (C) 2 (D) 1

71
JE E-Physics
6. Look at the drawing given in the figure which has been drawn with ink of uniform line–
y
thickness. The mass of ink used to draw each of the two inner circles, and each of the
two line segments is m. The mass of the ink used to draw the outer circle is 6m. The
coordinates of the centres of the different parts are : outer circle (0, 0), left inner circle x
(– a, a) , right inner circle (a, a), vertical line (0, 0) and horizontal line (0, – a). The
y–coordinates of the centre of mass of the ink in this drawing is : [IIT-JEE 2009]
a a a a
(A) (B) (C) (D)
10 8 12 3

7. Three objects A, B and C are kept in a straight line on a frictionless horizontal B


surface. These have masses m, 2m and m, respectively. The object A moves A C
m 2m m
towards B with a speed 9 m/s and makes an elastic collision with it. Thereafter,
B makes completely inelastic collision with C. All motions occur on the same
straight line Find the final speed (in m/s ) of the object C. [IIT-JEE 2009]

8. A point mass of 1 kg collides elastically with a stationary point mass of 5 kg. After their collision, the 1 kg mass
reverses its direction and moves with a speed of 2 ms –1. Which of the following statement(s) is (are) correct for
the system of these two masses ? [JEE(Adv.) 2010]
–1
(A) Total momentum of the system is 3 kg ms
(B) Momentum of 5 kg mass after collision is 4 kg ms –1
(C) Kinetic energy of the centre of mass is 0.75 J
(D) Total kinetic energy of the system is 4J

9. A ball of mass 0.2 kg rests on a vertical post of height 5 m. A bullet of mass 0.01 kg, traveling with a velocity V
m/s in a horizontal direction, hits the centre of the ball. After the collision, the ball and bullet travel indepen-
dently. The ball hits the ground at a distance of 20 m and the bullet at a distance of 100 m from the foot of the
post. the velocity V of the bullet is- [IIT-JEE 2011]

V m/s

0 20 100

(A) 250 m/s (B) 250 2 m/s (C) 400 m/s (D) 500 m/s

10. A particle of mass m is projected from the ground with an initial speed u 0 at an angle a with the horizontal. At
the highest point of its trajectory, it makes a completely inelastic collision with another identical particle, which
was thrown vertically upward from the ground with the same initial speed u 0. The angle that the composite
system makes with the horizontal immediately after the collision is :- [IIT-JEE 2013]
p p p p
(A) (B) +a (C) -a (D)
251\D\Allen(IIT-JEE Wing)\2020-21\Nurture\Phy\Unit-03\01-Centre of Mass

4 4 2 2

11. A bob of mass m, suspended by a string of length l1, is given a minimum velocity required to complete a full
circle in the vertical plane at the highest point it collides elastically with another bob of mass m suspended by a
string of length l2, which is initially at rest. Both the strings are mass-less and inextensible. If the second bob,
l1
after collision acquires the minimum speed required to complete a full circle in the vertical plane, the ratio l
2

is [JEE(Adv.) 2013]

72
Centre of Mass

12. A tennis ball is dropped on a horizontal smooth surface. It bounces back to its original position after hitting the
surface. The force on the ball during the collision is proportional to the length of compression of the ball. Which
one of the following sketches describes the variation of its kinetic energy K with time t most appropriately ? The
figures are only illustrative and not to the scale. [JEE(Adv.) 2014]

(A) (B)

(C) (D)

13. Consider regular polygons with number of sides n = 3, 4, 5 ........ as shown in the figure. The centre of mass of
all the polygons is at height h from the ground. They roll on a horizontal surface about the leading vertex without
slipping and sliding as depicted. The maximum increase in height of the locus of the center of mass for each
polygon is D. Then D depends on n and h as [JEE(Adv.) 2017]

2 æ pö æ 2p ö æ 1 ö 2æ p ö
(A) D = h sin çè ÷ø (B) D = h sin çè ÷ø (C) D = h ç -1
÷ (D) D = h tan çè ÷ø
n n æ pö 2n
çè cos çè ÷ø ÷ø
n

14. A block of mass M has a circular cut with a frictionless


surface as shown. The block rests on the horizontal R
frictionless surface of a fixed table. Initially the right edge
of the block is at x = 0, in a co-ordinate system fixed to the y m
table. A point mass m is released from rest at the topmost R
point of the path as shown and it slides down. When the M
x
mass loses contact with the block, its position is x and the
velocity is v. At that instant, which of the following options
x=0
is/are correct?
mr
(A) The x component of displacement of the center of mass of the block M is -
M+m
251\D\Allen(IIT-JEE Wing)\2020-21\Nurture\Phy\Unit-03\01-Centre of Mass

mR
(B) The position of the point mass is x = - 2
M+m

2gR
(C) The velocity of the point mass m is v =
m
1+
M

m
(D) the velocity of the block M is V = - 2gR [JEE(Adv.) 2017]
M

73
JE E-Physics

15. A ball is projected from the ground at an angle of 45° with the horizontal surface. It reaches a maximum height
of 120 m and returns to the ground. Upon hitting the ground for the first time, it loses half of its kinetic energy.
Immediately after the bounce, the velocity of the ball makes an angle of 30° with the horizontal surface. The
maximum height it reaches after the bounce, in metres, is ___________.
[JEE (Adv.) - 2018]

16. A ball is thrown from ground at an angle q with horizontal and with an initial speed u0. For the resulting
projectile motion, the magnitude of average velocity of the ball up to the point when it hits the ground for
the first time is V1. After hitting the ground, ball rebounds at the same angle q but with a reduced speed
of u0/a. Its motion continues for a long time as shown in figure. If the magnitude of average velocity of
the ball for entire duration of motion is 0.8 V1, the value of a is______ . [JEE (Adv.) - 2019]
u0
u0/a u0/a2 u0/am

q q q q

251\D\Allen(IIT-JEE Wing)\2020-21\Nurture\Phy\Unit-03\01-Centre of Mass

74
Centre of Mass

ANSWERS
EXERCISE-1
Que. 1 2 3 4 5 6 7 8 9 10 11 12 13 14 15
Ans. A C B A A B B C A A B A A C D
Que. 16 17 18 19 20 21 22 23 24 25 26 27 28 29 30
Ans. D C C D D B A B B A D A D C B

EXERCISE-2
l MULTIPLE CHOICE CORRECT QUESTIONS
1. (AC) 2. (BC) 3. (ABCD) 4. (CD) 5. (BC) 6. (B) 7. (ABCD)
8. (ABCD) 9. (BD) 10. (A)

l COMPREHENSION BASED QUESTIONS


11. (C) 12. (B) 13. (A) 14. (B) 15. (D) 16. (B) 17. (B) 18. (B)
19. (B) 20. (D) 21. (D) 22. (A)

EXERCISE-3
l NUMERICAL ANSWER BASED QUESTIONS
1. (4) 2. (5) 3. (5) 4. (5) 5. (2) 6. (3) 7. (2)
8. (4) 9. (1.25 m) 10. (3) 11. (8)
l MATRIX MATCH QUESTIONS
12. (A)-r, (B)–t, (C)–t, (D)–s 13. (A)–q, (B)–p,q (C)–r, (D)–s
14. (A)–p, (B)–q, (C)–p, (D)–q,s 15. (A) t (B) q (C) r (D) q
16. (A) s, (B) r, (C) q, (D) p

EXERCISE-4(A)
Que. 1 2 3 4 5 6 7 8 9 10 11 12 13 14 15
Ans. 3 1 2 3 2 2 1 1 4 4 4 4 1 4 2
Que. 16 17 18 19 20 21 22 23 24 25 26 27 28 29 30
Ans. 4 4 1 1 2 1 4 3 1 4 2 3 4 4 2

EXERCISE-4(B)
Que. 1 2 3 4 5 6 7 8 9 10 11 12 13
Ans. 1 2 2 2 1 3 3 2 1 3 1 2 4

EXERCISE-5
251\D\Allen(IIT-JEE Wing)\2020-21\Nurture\Phy\Unit-03\01-Centre of Mass

1. (AD) 2. (B) 3. (B) 4. (C) 5. (C) 6. (A) 7. 4 m/s 8. (AC)


9. (D) 10. (A) 11. (5) 12. (B) 13. (C) 14. (A, C) 15. (30) 16. (4.00)

75
76
JE E-Physics

IMPORTANT NOTES

251\D\Allen(IIT-JEE Wing)\2020-21\Nurture\Phy\Unit-03\01-Centre of Mass


RIGID BODY
DYNAMICS
Recap of Early Classes
When an extended object such as a wheel rotates about its axis, the motion cannot be analyzed by
modeling the object as a particle because at any given time different parts of the object have different linear
velocities and linear accelerations. We can, however, analyze the motion of an extended object by modeling it as
a collection of particles, each of which has its own linear velocity and linear acceleration.
In dealing with a rotating object, analysis is greatly simplified by assuming the object is rigid. A rigid
object is one that is non-deformable; that is, the relative locations of all particles of which the object is composed
remain constant. All real objects are deformable to some extent; our rigid-object model, however, is useful in
many situations in which deformation is negligible.

1.0 RIGID BODY


1.1 Rotation Motion of a Rigid Body
1.2 Types of Motions involving Rotation
1.3 Angular displacement, angular velocity and angular acceleration
1.4 Kinematics of rotation about fixed axis
2.0 MOMENT OF INERTIA (I)
2.1 Moment of Inertia of System of Particles
2.2 Moment of inertia of a rigid body
2.3 Theorems on Moment of Inertia
2.4 Radius of Gyration
3.0 TORQUE
3.1 Torque about a point
3.2 Torque about an axis
3.3 Force Couple
3.4 Point of Application of Force
3.5 Equilibrium
4.0 ROTATION ABOUT A FIXED AXIS
5.0 COMBINED TRANSLATIONAL AND ROTATIONAL MOTION OF A RIGID BODY
5.1 Instantaneous Axis of Rotation (IAR)
5.2 Rolling as Rotation about an Axis in Translation
6.0 TOPPLING
7.0 ENERGY METHODS
7.1 Concept of Work in rotation motion
7.2 Potential Energy of a rigid body
7.3 Kinetic Energy of a rigid body in rotation motion
7.4 Power
7.5 Work and Energy Theorem
7.6 Conservation of Mechanical Energy
8.0 METHODS OF IMPULSE AND MOMENTUM
8.1 Angular Impulse
8.2 Angular momentum of a particle
8.3 Angular Momentum of a Rigid Body
8.4 Conservation of Angular Momentum
8.5 Eccentric Impact
EXERCISE-1
EXERCISE-2
EXERCISE-3
EXERCISE-4(A)
EXERCISE-4(B)
EXERCISE-5
j

dsfa
Rigid Body Dynamics

Till so far we have learnt kinematics and kinetics of translation motion in which all the particles of a body
undergo identical motions i.e. at any instant of time all of them have equal velocities and equal accelerations
and in any interval of time they all follow identical trajectories. Therefore kinematics of any particle of a body
or of its mass center in translation motion is representative of kinematics of the whole body. But when a body
is in rotation motion, all of its particles and the mass center do not undergo identical motions. Newton’s laws
of motion, which are the main guiding laws of mechanics, are applicable to a point particle and if applied to
a rigid body or system of particles, they predict motion of the mass center. Therefore, it becomes necessary to
investigate how mass center and different particles of a rigid body move when the body rotates. In kinematics
of rotation motion we investigate relations existing between time, positions, velocities and accelerations of
different particles and mass center of a rigid body in rotation motion.

1.0 RIGID BODY

A rigid body is an assemblage of a large number of material particles, which do not change their mutual
distances under any circumstance or in other words, they are not deformed under any circumstance.
Actual material bodies are never perfectly rigid and are deformed under action of external forces. When these
deformations are small enough to be considered during their course of motion, the body is assumed a rigid
body. Hence, all solid objects such as stone, ball, vehicles etc are considered as rigid bodies while analyzing
their translation as well as rotation motion.
To analyze rotation of a body, relative motion between its particles cannot be neglected and size of the body
becomes a considerable factor. This is why study of rotation motion is also known as mechanics of rigid
bodies.

1.1 Rotation Motion of a Rigid Body

Any kind of motion of a body is identified by change in position or change in orientation or change in both.
If a body changes its orientation during its motion it said to be in rotation motion.
In the following figures, a rectangular plate is shown moving in the x-y plane. The point C is its mass center.
In the first case it does not changes orientation, therefore is in pure translation motion. In the second case it
changes its orientation during its motion. It is a combination of translation and rotation motion.

y y New
q orientation
t+dt A q A
t C
C C B
C
B t+dt
Original
t
orientation

O x O x
Pure Translation Combination of translation and rotation

Rotation i.e. change in orientation is identified by the angle through which a linear dimension or a straight line
drawn on the body turns. In the figure this angle is shown by q.
251\D\Allen(IIT-JEE Wing)\2020-21\Nurture\Phy\Unit-03\02-Rigid Body Dynamics

Illustration 1. Identify Translation and rotation motion


A rectangular plate is suspended from the ceiling by two parallel rods
each pivoted at one end on the plate and at the other end on the
ceiling. The plate is given a side-push to oscillate in the vertical plane
containing the plate. Identify motion of the plate and the rods.
Solution. Neither of the linear dimensions of the plate turns during the motion.
Therefore, the plate does not change its orientation. Here edges of the
body easily fulfill our purpose to measure orientation; therefore, no
line is drawn on it.
The plate is in curvilinear translation motion and the rods are in rotation motion.

77
JEE-Physics
1.2 Types of Motions involving Rotation

Motion of body involving rotation can be classified into following three categories.
I Rotation about a fixed axis.
II Rotation about an axis in translation.
III Rotation about an axis in rotation

Rotation about a fixed axis


Rotation of ceiling fan, potter’s wheel, opening and closing of doors and needles of a wall clock etc. come into
this category.
When a ceiling fan it rotates, the vertical rod supporting it remains stationary and all the particles on the fan
move on circular paths. Circular path of a particle P on one of its blades is shown by dotted circle. Centers of
circular paths followed by every particle are on the central line through the rod. This central line is known as
axis of rotation and is shown by a dashed line. All the particles on the axis of rotation are at rest, therefore the
axis is stationary and the fan is in rotation about this fixed axis.

Ceiling Fan Door

Axis of rotation
Axis of rotation

A door rotates about a vertical line that passes through its hinges. This vertical line is the axis of rotation. In the
figure, the axis of rotation is shown by dashed line.

Axis of rotation
An imaginary line perpendicular to plane of circular paths of particles Axis of rotation
of a rigid body in rotation and containing the centers of all these
circular paths is known as axis of rotation.
r
It is not necessary that the axis of rotation pass through the body.
Consider system shown in the figure, where a block is fixed on a
rotating disc. The axis of rotation passes through the center of the
disc but not through the block.

Important observations P
Let us consider a rigid body of arbitrary shape rotating about a fixed
axis PQ passing through the body. Two of its particles A and B shown
are moving on their circular paths.
A
l All of its particles, not on the axis of rotation, move on circular
paths with centers on the axis of rotation. All these circular
paths are in parallel planes that are perpendicular to the axis B
of rotation.
l All the particles of the body cover same angular displacement
Axis of rotation
in the same time interval, therefore all of them move with the Q
251\D\Allen(IIT-JEE Wing)\2020-21\Nurture\Phy\Unit-03\02-Rigid Body Dynamics

same angular velocity and angular acceleration.


l Particles moving on circular paths of different radii move with different speeds and different magnitudes
of linear acceleration. Furthermore, no two particles in the same plane perpendicular to the axis of
rotation have same velocity and acceleration vectors.
l All the particles on a line parallel to the axis of rotation move circular paths of the same radius therefore
have same velocity and acceleration vectors.
l Consider two particles in a plane perpendicular to the rotational axis. Every such particle on a rigid
body in rotation motion moves on circular path relative to another one. Radius of the circular path
equals to the distance between the particles. In addition, angular velocity and angular acceleration
equals to that of rotation motion of the body.

78
Rigid Body Dynamics
Rotation about an axis in translation
Rotation about an axis in translation includes a broad category of motions. Rolling is an Illustration of this
kind of motion. A rod lying on table when pushed from its one end in its perpendicular direction also executes
this kind of motion. To understand more let us discuss few Illustrations.
Consider rolling of wheels of a vehicle, moving on straight level road. Relative
to a reference frame, moving with the vehicle wheel appears rotating about its
stationary axel. The rotation of the wheel from this frame is rotation about C
fixed axis. Relative to a reference frame fixed with the ground, the wheel appears
rotating about the moving axel, therefore, rolling of a wheel is superposition of
two simultaneous but distinct motions – rotation about the axel fixed with the
vehicle and translation of the axel together with the vehicle.
Important observations
l Every particle of the body always remains in a plane perpendicular to the rotational axis. Therefore,
this kind of motion is also known as general plane motion.
l Relative to every particle another particle in a plane perpendicular to axis of rotation moves on circular
path. Radius of the circular path equals to the distance between the particles, angular velocity and
angular acceleration equals to that of rotation motion of the body.
l Rotation about axis in translation is superposition of pure rotation about the axis and simultaneous
translation motion of the axis.

Rotation about an axis in rotation


In this kind of motion, the body rotates about an axis that also rotates about some other axis. Analysis of
rotation about rotating axes is not in the scope of JEE, therefore we will discuss it to have an elementary idea
only.

Rotation about Precession of the


central axis central axis

As an Illustration consider a rotating top. The top rotates about


its central axis of symmetry and this axis sweeps a cone about a vertical axis. The central axis continuously
changes its orientation, therefore is in rotation motion. This type of rotation in which the axis of rotation also
rotates and sweeps out a cone is known as precession.
Another Illustration of rotation about axis in rotation is a table-fan swinging while rotating. Table-fan rotates
about its horizontal shaft along which axis of rotation passes. When the rotating table-fan swings, its shaft
rotates about a vertical axis.

1.3 Angular displacement, angular velocity and angular acceleration

Rotation motion is the change in orientation of a rigid body with time.It is measured by turning of a linear
dimension or a straight line drawn on the body.

New orientation
251\D\Allen(IIT-JEE Wing)\2020-21\Nurture\Phy\Unit-03\02-Rigid Body Dynamics

t=0 t
q
A A

B
B
Original orientation

In the figure is shown at two different instants t = 0 and t a rectangular plate moving in its own plane. Change
in orientation during time t equals to the angle q through which all the linear dimensions of the plate or a line
AB turns.

79
JEE-Physics
If the angle q continuously changes with time t, instantaneous angular velocity w and angular acceleration a
for rotation of the body are defined by the following equations.
dq
w= [1]
dt
d 2q dw dw
a= = =w [2]
dt 2
dt dq

Direction of angular motion quantities


Angular displacement, angular velocity and angular acceleration are known as angular motion quantities.
Infinitesimally small angular displacement, instantaneous angular velocity and angular acceleration are vector
quantities. Direction of infinitesimally small angular displacement and instantaneous angular velocity is given
by the right hand rule. For a disc rotating as shown in the figure, the angular velocity points upwards along the
axis of rotation.

r r
dq w

Axis of rotation Axis of rotation

The direction of angular acceleration depends on whether angular velocity increases or decreases with time.
For increasing angular velocity, the angular acceleration vector points in the direction of angular velocity
vector and for decreasing angular velocity, the angular acceleration vector points opposite to the angular
velocity vector.
r
w
r
w
r
a

r
a

Angular acceleration: Increasing angular speed Angular acceleration: Decreasing angular speed

In rotation about fixed axis and rotation about axis in translation, the axis of rotation does not rotate and
angular velocity and acceleration always point along the axis of rotation. Therefore, in dealing these kinds of
motions, the angular motion quantities can used in scalar notations by assigning them positive sign for one
direction and negative sign for the opposite direction.
These quantities have similar mathematical relations as position coordinate, velocity, acceleration and time
have in rectilinear motion.
l A body rotating with constant angular velocity w and hence zero angular acceleration is said to be
uniform rotation. Angular position q is given by equation
q = q o + wt [3]
Thus for a body rotating with uniform angular acceleration a, the angular position q and angular
251\D\Allen(IIT-JEE Wing)\2020-21\Nurture\Phy\Unit-03\02-Rigid Body Dynamics

velocity w can be expressed by the following equation.


w = w o + at [4]

q = q o + w o t + 12 at 2 = q o + 1
2 (w o + w ) t [5]

w 2 = w o2 + 2a (q - q o ) [6]
Angular motion quantities in rotation and assumption of axis of rotation
Rotation is identified by change in orientation, which is measured by turning of a linear dimension of the
body or a line drawn on the body. It remains unchanged relative to all inertial frames.
Therefore, if we assume axis of rotation anywhere but parallel to the original one, angular displacement,
angular velocity and angular acceleration of rotation motion remain the same.
80
Rigid Body Dynamics

Illustration 2. A wheel is rotating with angular velocity 2 rad/s. It is subjected to uniform angular acceleration 2.0
rad/s2.
(a) How much angular velocity does the wheel acquire after 10 s?
(b) How many complete revolution it makes in this time interval?
Solution. The wheel is in uniform angular acceleration,
w = wo + at ® Substituting values of wo, a and t, we have
w = 2 + 2 ´ 10 = 22 rad/s
q = qo + 1
2 (wo + w ) t ® Substituting q o = 0
for initial position, and wo from above equation, we have
q = 0 + 12 ( 2 + 22) 10 = 120 rad.
In one revolution, the wheel rotates through 2p radians. Therefore, number of complete revolutions
n is
q 60
n= = rev.
2p p

Illustration 3. A disc rotates about a fixed axis. Its angular velocity w varies with time according to equation
w = at + b . At the instant t = 0 its angular velocity is 1.0 rad/s and at angular position is 2 rad and
at the instant t = 2 s, angular velocity is 5.0 rad/s. Determine angular position q and angular
acceleration a when t = 4 s.
Solution The given equation w = at + b has form similar to eq.[4], therefore motion is rotation with uniform
m
angular acceleration.
Initial angular velocity = wo = b = 1.0 rad/s
Angular acceleration a = a
Substituting these values in eq.[5], we get

q = 12 at 2 + w0 t + c
Since at t = 0, w = 1.0 rad/s, we obtain the constant c.
Initial angular position = qo = c = 2.0 rad
Since at t = 2.0 s angular velocity is 5.0 rad/s, from given expression of angular velocity, we have
w = at + b ® Substituting b = 1.0 rad/s, t = 2.0 s and w = 5.0 rad/s, we have
2
a = 2.0 rad/s
Now we can write expressions for angular position, angular velocity and angular acceleration.
q = t 2 + t + 2.0 ....(1)
w = 2.0t + 1.0 ....(2)
From the above equations, we can calculate angular position, angular velocity and angular
251\D\Allen(IIT-JEE Wing)\2020-21\Nurture\Phy\Unit-03\02-Rigid Body Dynamics

acceleration at t =4.0 s
q 4 = 22 rad, w 4 = 9.0 rad/s, a = 2.0 rad/s2

Illustration 4. An early method of measuring the speed of light makes use of a rotating slotted wheel. A beam of
light passes through slot at the outside edge of the wheel, as shown in figure below, travels to a
distant mirror, and returns to the wheel just in time to pass through the next slot in the wheel. One
such slotted wheel has a radius of 5.0 cm and 500 slots at its edge. Measurements taken when the
mirror is L = 500 m from the wheel indicate a speed of light of 3.0 × 10 5 km/s.
(a) What is the (constant) angular speed of the wheel ?
(b) What is the linear speed of a point on the edge of the wheel?
81
JEE-Physics

Light Light beam


source

Mirror perpen
dicular
to light beam
Rotating
slotted wheel
Solution (a) During the time light goes from the wheel to the mirror and comes back again, the wheel turns
2p
through an angle of q = = 1.26 × 10–2 rad.
500

2L 2 ( 500m)
That time is t = = = 3.34 × 10–6 s
c 2.998 ´ 108 m/ s
q 1.26x10 -2 rad
So the angular speed of the wheel is w = = = 3.8 × 103 rad/s
t 3.34x10 -6 s
(b) Linear speed of a point on the edge of a wheel v = wr = 3.8 × 103 × 0.05 = 1.9 × 102 m/s

Illustration 5. A pulsar is rapidly rotating neutron star that emits a radio beam like a lighthouse emits a light beam.
We receive a radio pulse for each rotation of the star. The period T of rotation is found by measur-
ing the time between pulses. The pulsar in the Crab nebula has a period of rotation of T = 0.033
s that is increasing at the rate of 1.26 x 10–5 second/year.
(a) What is the pulsar's angular acceleration?
(b) If its angular acceleration is constant, how many years from now will the pulsar stop rotating?
(c) The pulsar originated in a supernova explosion seen in the year 1054. What was the initial T
for the pulsar? (Assume constant angular acceleration since the pulsar originated.).
Solution (a) The angular velocity in rad/s w = 2p / T .
dw 2p dT
The angular acceleration = a = =– 2
dt T dt

dT 1.26 ´ 10-5 s / y
For the pulsar described = = 4.00 × 10–13
dt 3.16 ´ 107 s / y

é 2p ù
So a = - ê
( 0.033s )
(
2 ú 4.00 ´ 10
- 13
)
= – 2.3 × 10–3 rad/s2
ë û
The negative sign indicates that the angular acceleration is opposite the angular velocity and
the pulsar is slowing down.
(b) w = w 0 + at for the time t when w=0.
251\D\Allen(IIT-JEE Wing)\2020-21\Nurture\Phy\Unit-03\02-Rigid Body Dynamics

w 2p 2p
t=– =– =– = 8.3 ×1010 s .
a aT (
-2.3 ´ 10 red / s 2 ( 0.033s)
- 9
)
This is about 2600 years.
(c) The pulsar was born 1992– 1054 = 938 years ago.
This is equivalent to (938 y) (3.16 × 107 s/y) = 2.96 x 1010 s. Its angular velocity was then
2p 2p
w = w 0 + at = + at = + (–2.3 x 10–9 rad/s2) (–2.96 x 1010s) = 258 rad/s.
T 0.033s
2p
Its period was T = = 2.4 × 10–2 s.
w
82
Rigid Body Dynamics
Illustration 6. A turn table is rotating in a horizontal plane about the vertical axis passing through its centre with
an angular velocity 20 rad/s. It carries upon it a flywheel rotating with an angular velocity 40 rad/s
about a horizontal axle mounted in bearings. Find the angular velocity of the wheel as seen by an
observer in the room.
Solution As the axis of the turn table is vertical its angular velocity wT is directed vertical. The axis of flywheel
is horizontal therefore its angular velocity wF is directed horizontal, hence the resultant angular
r r r
velocity is w R = w F + w T
r
|w R| = w2F + w2T = 402 + 202 = 20 5 rad/s wR
wT
r
wR lies in a plane which makes an angle q with the horizontal
æ wT ö æ 1ö q
plane, given by q = tan–1 ç = tan-1 ç ÷
è w F ÷ø è 2ø wF

1.4 Kinematics of rotation about fixed axis

z
In figure is shown a rigid body of arbitrary shape rotating about the
z-axis. In the selected frame (here the coordinate system) all the
three axes are at rest, therefore the z-axis that is the axis of rotation
is at rest and the body is in fixed axis rotation. All of its particles other r
w
than those on the z-axis move on circular paths with their centers on
the z-axis. All these circular paths are parallel to the x-y plane. In the C r
figure, one of its particles P is shown moving with velocity vr on a
r
q r
v
r P
circular path of radius r and center C. Its position vector is R . It weree s
x
at the line Cx at t = 0 and at the position shown at the instant t.
r
During time interval t, it covers the circular arc of length s and its R
radius vector turns through angle q.
In an infinitesimally small time interval dt let, the particle covers
O
infinitesimally small distance ds along its circular path.
r r r r r y
ds = dq ´ r = dq ´ R [7] x
r r r
r ds dq r dq r
v= = ´r = ´R [8]
dt dt dt
From eq. [7] and [8] we have
r r r r r [9]
v = w ´r = w ´R
The above equation tells us the relation between the linear and angular velocity. Now we explore relation
between the linear and angular accelerations. For the purpose, differentiate the above equation with respect
to time.
r r r
r dv d w r r dr r r r r
a= = ´r +w ´ =a ´r +w ´v [10]
dt dt dt
The first term on the RHS points along the tangent in the direction of the velocity vector and it is known as
r
tangential acceleration aT same as we have in circular motion. In addition, the second term point towards the
r
center C. It is known as centripetal acceleration or normal component an of acceleration same as in circular
motion. Now we have
251\D\Allen(IIT-JEE Wing)\2020-21\Nurture\Phy\Unit-03\02-Rigid Body Dynamics

r r r
Tangential acceleration aT = a ´ r [11]
r r r r
Normal acceleration an = w ´ v = -w 2 r [12]

2.0 MOMENT OF INERTIA (I)

Definition : Moment of Inertia is defined as the capability of system to oppose the change produced in
the rotational motion of a body. As “Inertia” plays an important role in definition of Newton's first law
which is also called as inertia law, moment of inertia is the key concept in defining the state of rotation.
It is the property of a body rotating or which can rotate about an axis and which resists the change in
state of body's rotational motion.
83
JEE-Physics
Moment of Inertia depends on
(i) density of the material of body
(ii) shape & size of body
(iii) axis of rotation
Combinedly we can say that it depends upon distribution of mass relative to axis of rotation.
Note : Moment of inertia does not change if the mass is rotated with constant radius about axis of
rotation.

2.1 Moment of Inertia of System of Particles

Moment of inertia of system of particles is given by


n
2 2
I = mr1 + m2 r2 +......................... = åm r
i=1
i i
2

= I1 + I2 + I3 +.........................
SI units of Moment of Inertia is Kgm 2.
Moment of Inertia of a single particle : I = mr2
where m = mass of the particle
r = perpendicular distance of the particle from the axis about which
moment of Inertia is to be calculated

Illustration 7. Four particles each of mass m are kept at the four corners of a
square of edge a. Find the moment of inertia of the system about a
line perpendicular to the plane of the square and passing through
the centre of the square.

Solution The perpendicular distance of every particle from the given line is a/ 2 . The moment of inertia of
1
one particle is, therefore, m(a/ 2 )2 = ma2. The moment of inertia of the system is, therefore, e,
2
1
4 × ma2 = 2ma2.
2

Illustration 8. Two heavy particles having masses m 1 & m2 are situated in a plane perpendicular to line AB
at a distance of r 1 and r2 respectively.
(i) What is the moment of inertia of the system about axis AB ?
(ii) What is the moment of inertia of the system about an axis passing through m 1 and
perpendicular to the line joining m 1 and m2 ?
(iii) What is the moment of inertia of the system about an axis passing through m 1 and m 2 ?
Solution (i) Moment of inertia of particle on left is I1 = m1r12.
Moment of Inertia of particle on right is I2 = m2r22.
Moment of Inertia of the system about AB is
251\D\Allen(IIT-JEE Wing)\2020-21\Nurture\Phy\Unit-03\02-Rigid Body Dynamics

I = I1+ I2 = m 1r22 + m 2r22


(ii) Moment of inertia of particle on left is I1 = 0
Moment of Inertia of particle on right is I2 = m2(r1 + r2)2.
Moment of Inertia of the system about AB is
I = I1+ I2 = 0 + m 2(r1 + r 2)2
(iii) Moment of inertia of particle on left is I1 = 0
Moment of Inertia of particle on right is I2 = 0
Moment of Inertia of the system about AB is I = I1+ I2 = 0 + 0

84
Rigid Body Dynamics
Illustration 9. Four point masses are connected by a massless rod as shown in figure. Find out the moment
of inertia of the system about axis CD ?

Solution I = m(2a) 2 + 2m(a)2 + 3m(0) 2 + 4m(a)2 = 10 ma 2

Illustration 10. Three particles, each of mass m, are situated at the vertices of an
equilateral triangle ABC of side L (figure). Find the moment of inertia
of the system about the line AX perpendicular to AB in the plane of
ABC.
Solution Perpendicular distance of A from AX = 0
Perpendicular distance of B from AX = L
Perpendicular distance of C from AX = L/2
Thus, the moment of inertia of the particle at A = 0, of the particle B = mL2, and of the
particle C = m(L/2) 2. The moment of inertia of the three-particle system about AX is

5 mL2
I = 0 + mL2 + m(L/2)2 =
4

Illustration 11. Four point masses each of mass m kept at the four corners
of a square of side length ‘a’ find the moment of system
inertia about axis CD.
2 2
æ a ö æ a ö
( )
2
Solution I = 0 + mç ÷ + mç ÷ +m 2a
è 2ø è 2ø
= 3ma 2

2.2 Moment of inertia of a rigid body

A rigid body is continuous distribution of mass and can be assumed consisting of infinitely large number of
point particles. If one of the point particle of infinitely small mass dm is at a distance r from the axis of rotation
OO’, the moment of inertia of this point particle is given by
O
dI o = r 2 dm
The moment of inertia of the whole body about the axis OO’ can now be r
obtained by integrating term of the above equation over the limits to cover dm
whole of the body. O’
I o = ò dI o = ò r dm
2
[3]
Expression for moment of inertia contains product of two terms. One of them is the mass of the body and the
other is a characteristic dimension, which depends on the manner how mass of the body is distributed relative
251\D\Allen(IIT-JEE Wing)\2020-21\Nurture\Phy\Unit-03\02-Rigid Body Dynamics

to the axis of rotation. Therefore moment of inertia of a rigid body depends on the mass of the body and
distribution of the mass relative to the axis of rotation. Obviously for uniform bodies expression of moment of
inertia depends on their shape and location and orientation of the axis of rotation. Based on these facts we
can conclude.

Illustration 12. Calculate the moment of inertia of a ring having mass M, radius R and
having uniform mass distribution about an axis passing through the centre
of ring and perpendicular to the plane of ring ?

85
JEE-Physics

ò (dm)r
2
Solution I=

Because each element is equally distanced from the axis so r = R = R 2 dm = MR 2 ò


I = MR2
(Note : Answer will remain same even if the mass is nonuniformly distributed because

ò dm = M always.)
Illustration 13. Calculate the moment of inertia of a uniform rod of mass M and length l about an axis
passing through an end and perpendicular to the rod.
l
æM ö Ml 2
ò ò çè l dx ÷ø x
2 2
Solution I= (dm)r = =
0 3

Illustration 14. Determine the moment of Inertia of a uniform disc having mass M, radius R about an axis
passing through centre & perpendicular to the plane of disc ?

Solution I= ò dI ring

element - ring dI = dmr2


M
dm = 2prdr (here we have used the uniform mass
pR 2
distribution)
R
M MR 2
\ I= ò pR
0
2 . (2prdr).r2 Þ I=
2

Moment of Inertia for some commonly used bodies

Body Axis Moment of Iner-


tia

Uniform thin rod bent into shape Passing through center and I C = mr 2
r
of an arc of mass m C
perpendicular to the plane
containing the arc

Uniform ring of mass m Passing through center and I C = mr 2


perpendicular to the plane
C r
containing the arc or the
centroidal axis.
251\D\Allen(IIT-JEE Wing)\2020-21\Nurture\Phy\Unit-03\02-Rigid Body Dynamics

C
mL2
Straight uniform rod L
Passing through center and IC =
12
perpendicular to the rod or
the centroidal axis.

r
mr 2
Sector of a uniform disc of mass m C Passing through center and IC =
2
perpendicular to the plane
containing the sector.
86
Rigid Body Dynamics

mr 2
Uniform disc of mass m Passing through center and IC =
C r 2
perpendicular to the plane
containing the disc or the
centroidal axis.
mr 2
Homogeneous cylinder of mass m IC
Axis of the cylinder or the IC =
2
centroidal axis.

Homogeneous sphere of mass m Diameter or the I C = 52 mR 2


centroidal axis

Spherical shell of mass m Diameter or the I = 23 mR 2


centroidal axis

2.3 Theorems on Moment of Inertia

Moment of inertias of a rigid body about different axes may be different. There are two theorems known as
theorem of perpendicular axes and theorem of parallel axes, which greatly simplify calculation of moment of
inertia about an axis if moment of inertia of a body about another suitable axis is known.
Theorem of Perpendicular Axes
This theorem is applicable for a rigid body that lies entirely within a plane
z
i.e. a laminar body or a rod bent into shape of a plane curve. The moment
of inertia Ix, Iy and Iz of the body about the x, y and z-axis can be expressed
by the following equations.
Iz = Iy + Ix O
x
For a planar body, the moment of inertia about an axis perpendicular to the
plane of the body is the sum of the moment of inertias about two
perpendicular axes in the plane of the object provided that all the three y
axes are concurrent.

Illustration 15. Find moment of inertia of a uniform disc of mass m and radius r about one of its diameter.
Solution. In the adjoining figure a disc is shown with two of its
diameter perpendicular to each other. These diameters
are along the x and the y-axis of a coordinate system.
251\D\Allen(IIT-JEE Wing)\2020-21\Nurture\Phy\Unit-03\02-Rigid Body Dynamics

The x-axis is perpendicular to the plane of the disc and


passes through its center is also shown.
Since the disc is symmetric about both the diameters,
moment of inertias about both the diameters must be
equal. Thus substituting this in the theorem of
perpendicular axes, we have
I z = I y + I x ® I z = 2I x = 2I y
Moment of inertia of the disc about the z-axis is I z = 12 mr 2 . Substituting it in the above equation,
we have
I x = I y = 12 I z = 14 mr 2

87
JEE-Physics
Theorem of Parallel Axes
This theorem also known as Steiner’s theorem can be used to determine the moment of inertia of a rigid body
about any axis, if the moment of inertia of the body about a parallel axis passing through mass center of the
body and perpendicular distance between both the axes is known.
IC
Consider a body of arbitrary shape and mass m shown in the figure. Its Io
moment of inerta Io and IC are defined about two parallel axes. The axis
about which moment of ienertia I C is defined passes through the mass center C
C. Seperation between the axes is r. These two moment of inertias are
related by the following equation.
r
I O = I C + Mr 2
The above equation is known as the theorem of parallel axes or Steiner’s theorem.
l The moment of inertia about any axis parallel to an axis through the mass center is given by sum of
moment of inertia about the axis through the mass center and product term of mass of the body and
square of the distance between the axes.
l Among all the parallel axes the moment of inertia of a rigid body about the axis through the mass
center is the minimum moment of inertia.
The second term added to the moment of inertia I C about the centroidal axis in the above equation can be
recognized as the moment of inertia of a particle of mass equal to that of the body and located at its mass
center. It again reveals that the plane motion of a rigid body is superposition of pure rotation about the mass
center or centroidal rotation and translation of its mass center.

Illustration 16. Find moment of inertia of a uniform ring, uniform disc, uniform
cylinder and uniform sphere each of mass m and radius r about an
axis passing through point P and perpendicular to plane of paper.

Solution. IC is the moment of inertia about an axis passing through centre and perpendicular to plane of
paper.
Ring I P = I C + Mr 2 = Mr 2 + Mr 2 = 2Mr 2

Disc I P = I C + Mr 2 = 12 Mr 2 + Mr 2 = 32 Mr 2

Cylinder I P = I C + Mr 2 = 12 Mr 2 + Mr 2 = 32 Mr 2

Sphere I P = I C + Mr 2 = 25 Mr 2 + Mr 2 = 57 Mr 2

Illustration 17. Find expression for moment of inertia of a uniform disc of mass m, radius r about one of its secant
making an angle q with one of its diameter.
E
Solution. A disc, the secant OB and diameter OA are shown in the adjoining
figure. The secant OB is parallel to another diameter DE. Moment
A C
of inertia of the disc about one of its diameter is 1
4
mr 2 and hence q O

moment of inertia I about the diameter DE. Distance between the


251\D\Allen(IIT-JEE Wing)\2020-21\Nurture\Phy\Unit-03\02-Rigid Body Dynamics

D
1
B
secant OB and the parallel diameter DE is rsinq. I 1 = 4 mr 2
1 r sin q
Substituting above information in the theorem of parallel axes, we I2
have

I 2 = I 1 + m ( r sin q ) ® I 2 = mr
2 2
( 1
4
+ sin2 q )
Illustration 18. Find moment of inertia about centroidal axis of a bobbin, which Centroidal
is constructed by joining coaxially two identical discs each of
mass m and radius 2r to a cylinder of mass m and radius r as
shown in the figure.

88
Rigid Body Dynamics
Solution. The bobbin is a composite body made by joining two identical discs coaxially to cylinder. The
moment of inertia I of the bobbin equals to the sum of moment of inertias of the two discs and
moment of inertia of the cylinder about their centroidal axes. Using expressions for the moment
of inertia for disc and cylinder, we have

I = 2I disk + Icylinder ® I = 2 { 1
2
m ( 2r )2 } + 21 mr 2 = 92 mr 2

Illustration 19. Find moment of inertia about one of diameter of a hollow sphere of mass m, inner radius r and
outer radius R.
Solution. The hollow sphere is assumed as if a concentric smaller sphere of radius r is removed from a larger
sphere of radius R. Thus the moment inertia of the hollow sphere about any axes can be obtained
by subtracting moment of inertia of the smaller sphere from that of the larger sphere. As shown in
the following figure.
y
Let the mass of the hollow sphere is m. y
I I
y 1
2

Density of the material used is I 2

O x O x O x
3m
r=
(
4p R 3 - r 3 )
I1 - I2 = I
Masses m and m of the smaller spheres are
1 2
mR 3 mr 3
m1 = r ( 4
3
pR3 =
R3 - r 3
)and m2 = r ( 4
3 )
pr 3 =
R3 - r 3
Subtracting I from I we have I.
2 1

2m 1 R 2 2m 2 r 2 2m R 5 - r 5 ( )
I = I1 - I 2 ® I = - =
5 5 5 R3 - r3 ( )
2.4 Radius of Gyration

It is the radial distance from a rotation axis at which the mass of an object could be concentrated without
altering the moment of inertia of the body about that axis.
If the mass m of the body were actually concentrated at a distance k from the axis, the moment of inertia
about that axis would be mk2.
I
k=
m
The radius of gyration has dimensions of length and is measured in appropriate units of length such as
meters.

Calculation of Moment of Inertia


1. A uniform rod of mass 6M and length 6l is bent to make an equilateral hexagon. Its M.I. about an axis
251\D\Allen(IIT-JEE Wing)\2020-21\Nurture\Phy\Unit-03\02-Rigid Body Dynamics

passing through the centre of mass and perpendicular to the plane of hexagon is:
(A) 5ml2 (B) 6ml2 (C) 4ml2 (D) ml2/12

2. The moment of inertia of an uniform thin sheet of mass M of the given


shape about the specified axis is (axis and sheet both are in
same plane:)
7 5
(A) Ma 2 (B) Ma 2
12 12
1 1
(C) Ma 2 (D) Ma 2
3 12

89
JEE-Physics
3. Three rings each of mass m and radius r are so placed that they touch each other. The radius of gyration of the
system about the axis as shown in the figure is :

5 5
(A) r (B) r
3 6

7 7
(C) r (D) r
3 6

4. Find the moment of inertia of a solid cylinder of mass M and radius R about a line parallel to the axis of
the cylinder and on the surface of the cylinder.

5. Find the moment of inertia of the two uniform joint rods about an axis passing
through P and perpendicular to plane of paper (mass of each rod = m and
length of each rod = l)

6. Three identical uniform rods, each of length l, are joined to form a rigid equilateral triangle. Find radius
of gyration about an axis passing through a corner and perpendicular to the plane of the triangle.

7. Find moment of inertia of a solid hemisphere of mass M shown in


figure, about an axis AA’ passing through its centre of mass.

8. Find the moment of inertia of a uniform square plate of mass M, edge of


length 'l' about its axis passing through P and perpendicular to it.

3.0 TORQUE

Torque represents the capability of a force to produce change


in the rotational motion of the body.

3.1 Torque about a point

r ® ® ®
Torque of force F about a point t = r´F
r
Where F = force applied
P = point of application of force
Q = Point about which we want to calculate the torque.
251\D\Allen(IIT-JEE Wing)\2020-21\Nurture\Phy\Unit-03\02-Rigid Body Dynamics

r
r = position vector of the point of application of force w.r.t. the point about which we want
to determine the torque.
r
t = r F sinq = r ^F = rF^
Where q = angle between the direction of force and the position vector of P wrt. Q.
r^ = r sin q = perpendicular distance of line of action of force from point Q ,it is also called
force arm.
® r
F ^ = F sin q = component of F perpendicular to r
SI unit of torque is N-m
Torque is a vector quantity and its direction is determined using right hand thumb rule and its always
perpendicular to the plane of rotation of the body.

90
Rigid Body Dynamics

Illustration 20. A particle of mass M is released in vertical plane from a point P at x = x 0 on the x-axis it falls
vertically along the y-axis. Find the torque t acting on the particle at a time t about origin ?
Solution Torque is produced by the force of gravity.
r
t = r F sin q k̂
or t = r^F = x 0mg
®
t = mgx0 k̂

Illustration 21. A particle having mass m is projected with a velocity v 0


from a point P on a horizontal ground making an angle q V0
with horizontal. Find out the magnitude of torque about q
the point of projection acting on the particle when it is at
P Q
its maximum height ?
2 2
R v sin 2q mv 0 sin 2q
Solution t = rFsinq = mg = 0 mg Þ t =
2 2g 2
®
Illustration 22. Find the torque about point O and A due to given force F .

r r r r ˆ ˆ r
Solution Torque about point O, t = r0 ´ F, r0 = i + j , F = 5 3 ˆi + 5ˆj
r
t = (iˆ + ˆj) × ( 5 3 ˆi + 5jˆ ) = 5(1 - 3)kˆ
r r r r ˆ r
Torque about point A , t = ra ´ F, ra = j , F = 5 3 ˆi + 5ˆj
r
t = ĵ × ( 5 3 ˆi + 5jˆ ) = 5(- 3)kˆ
®
Illustration 23. Find out torque about point A, O and B due to given force F .
251\D\Allen(IIT-JEE Wing)\2020-21\Nurture\Phy\Unit-03\02-Rigid Body Dynamics

r r r r r
Solution Torque about point A , t A = rA ´ F , rA = 3 î , F =10 î
r
t A = 3 î ´ 10 î = 0
r r r r r
Torque about point B , tB = rB ´ F , rB = 5 ĵ , F =10 î
r
tB = 5 ĵ ´ 10 î = –50 k̂
r r r r r
Torque about point O , tO = rO ´ F , rO = 3 î + 5 ĵ , F =10 î
r
tO = (3 î + 5 ĵ ) ´ 10 î = – 50 k̂
91
JEE-Physics
3.2 Torque about an axis

®
The torque of a force F about an axis AB is defined as the component of
®
torque of F about any point O on the axis AB, along the axis AB.
® ® ® ®
In the given figure torque of F about O is t0 = r ´ F
® r
The torque of F about AB, tAB is component of t0 along line AB.
There are four cases of torque of a force about an axis.:
r
Case I : Force is parallel to the axis of rotation, F || AB . AB is the axis of rotation about which torque is
r r r r r r
required r ´ F is perpendicular to F , but F || AB , hence r ´ F is perpendicular to AB . The component
r r
of r ´ F along AB is, therefore, zero..

Case II : The line of force intersects the axis of rotation (F intersect AB)
r r r
F intersects AB, then F and r are along the same line.
The torque about O is
r r
r ´ F = 0.
Hence component this torque along line AB is also zero.
r r
Case III : F perpendicular to AB but F and AB do not intersect.
In the three dimensions, two lines may be perpendicular without
intersecting each other.
Two nonparallel and nonintersecting lines are called skew lines.
Figure shows the plane through the point of application of force P
that is perpendicular to the axis of rotation AB. Suppose the plane
intersects the axis at the point O. The force F is in this plane (since F
is perpendicular to AB). Taking the origin at O,
r r r
Torque = r ´ F = OP × F .
Thus, torque = rF sin q = F(OS)

r
where OS is the perpendicular from O to the line of action of the force F . The line OS is also perpendicular
to the axis of rotation. It is thus the length of the common perpendicular to the force and the axis of
rotation.
r r r
The direction of t = OP × F is along the axis AB because AB ^ OP and AB ^ F . The torque about
r
251\D\Allen(IIT-JEE Wing)\2020-21\Nurture\Phy\Unit-03\02-Rigid Body Dynamics

AB is, therefore, equal to the magnitude of t that is F.(OS).


Thus, the torque of F about AB = magnitude of the force F × length of the common perpendicular to the
force and the axis. The common perpendicular OS is called the lever arm or moment arm of this torque.
r
Case IV : F and AB are skew but not perpendicular..
r
Here we resolve F into two components, one is parallel to axis and other is perpendicular to axis. Torque
of the perallel part is zero and that of the perpendicular part may be found, by using the result of case
(III).

92
Rigid Body Dynamics

Illustration 24. Find the torque of weight about the axis passing through point P.

r r r r
Solution t = r ´ F, r = R
r and mg sin q both are at perpendicular so torque about point
P = mgRsinq

Illustration 25. A bob of mass m is suspendend at point O by string of length l.


Bob is moving in a horizontal circle find out (i) magnitude of
torque of gravity and tension about point O and O'. (ii) Magnitude
of net torque about axis OO'.
Solution (i) Torque about point O
Torque of tension (T), tten = 0
(tension is passing through point O)
Torque of gravity tmg = lmgsin q
Torque about point O'
Torque of gravity tmg = mgr, r = lsin q
Torque of tension
mg
tten = (lsinq ) cos q = mgl sinq (along positive ĵ )
cos q
(ii) Torque about axis OO'
Torque of gravity about axis OO' tmg = 0 (force mg parallel to axis OO')
Torque of tension about axis OO' tten = 0 (force T is passing through the axis OO')
Net torque about axis OO' tnet = 0

3.3 Force Couple

A pair of forces each of same magnitude and acting in opposite


direction is called a force couple.
Torque due to couple = Magnitude of one force × distance
between their lines of action.
Magnitude of torque = t = F (2d)
l A couple does not exert a net force on an object even though it exerts a torque.
l Net torque due to a force couple is same about any point.
l A consequence is that, if F net = 0 and tnet = 0 about one point, then tnet = 0 about any point.

3.4 Point of Application of Force

Point of Application of force is the point at which, if net force is assumed to be acting, then it will produce
same translational as well as rotational effect, as was produced earlier.
251\D\Allen(IIT-JEE Wing)\2020-21\Nurture\Phy\Unit-03\02-Rigid Body Dynamics

We can also define point of application of force as a point about which torque of all the forces is zero.

93
JEE-Physics
r r r
Consider three forces F1,F2 ,F3 acting on a body if D is point of application of force then torque of
r r r
F1 + F2 + F3 acting at a point D about O is same as the original torque about O
r r r r r r r r r r
[ ]
r1 ´ F1 + r2 ´ F2 + r3 ´ F3 = r ´ (F1 + F2 + F3 )

Illustration 26. Determine the point of application of force, when forces of


20 N & 30 N are acting on the rod as shown in figure.

Solution Net force acting on the rod F net = 10N


Net torque acting on the rod about point C
tc = (20 × 0) + ( 30 × 20) = 600 clockwise
Let the point of application be at a distance x from point C
600 = 10 x Þ x = 60 cm
\ 70 cm from A is point of Application
Note :
(i) Point of application of gravitational force is known as the centre of gravity.
r
(ii) Centre of gravity coincides with the centre of mass if value of g is assumed to be constant.
(iii) Concept of point of application of force is imaginary, as in some cases it can lie outside the body.

3.5 Equilibrium

A system is in mechanical equilibrium if it is in translational as well as rotational equilibrium.

For this : Fnet = 0


Y
tnet = 0 (about every point) F1

if Fnet = 0 then tnet is same about every point

Hence necessary and sufficient condition for equilibrium is F2


Fnet = 0 , F5
X
® ® ® ® ®
Þ F1 + F2 + F3 + F4 + F5 = 0

tnet = 0 about any one point, which we can choose as per our F4 F3

convenience. ( tnet will automatically be zero about every point)


251\D\Allen(IIT-JEE Wing)\2020-21\Nurture\Phy\Unit-03\02-Rigid Body Dynamics

l The equilibrium of a body is called stable if the body tries to


regain its equilibrium position after being slightly displaced
and released. It is called unstable if it gets further displaced
after being slightly displaced and released. If it can stay in
unstable stable Neutral
equilibrium even after being slightly displaced and released, it
equilibrium equilibrium equilibrium
is said to be in neutral equilibrium.

94
Rigid Body Dynamics

Illustration 27. Two small kids weighing 10 kg and 15 kg are trying to


balance a seesaw of total length 5.0 m, with the fulcrum
at the centre. If one of the kids is sitting at an end, where
should the other sit ?

Solution It is clear that the 10 kg kid should sit at the end and the 15 kg kid should sit closer to the
centre. Suppose his distance from the centre is x. As the kids are in equilibrium, the normal
force between a kid and the seesaw equals the weight of that kid. Considering the rotational
equilibrium of the seesaw, the torque of the forces acting on it should add to zero. The forces are
(a) (15 kg) g downward by the 15 kg kid,
(b) (10 kg) g downward by the 10 kg kid,
(c) weight of the seesaw and
(d) the normal force by the fulcrum.
Taking torques about the fulcrum,
(15 kg)g x = (10 kg)g (2.5 m) or x = 1.7 m.

Illustration 28. A uniform ladder of mass m = 10 kg leans against a smooth vertical wall making an angle
q = 53° with it. The other ends rests on a rough horizontal floor. Find the normal force and the
friction force that the floor exerts on the ladder.
Solution The forces acting on the ladder are shown in figure. They are
(a) Its weight W,
(b) normal force N1 by the vertical wall,
(c) normal force N2 by the floor and
(d) frictional force f by the floor.
Taking horizontal and vertical components,
N1 = f ....(i)
and N2 = mg ....(ii)
Taking torque about B,
N1(AO) = mg(CB)

AB
or, N1(AB) cosq = mg sin q
2

3 W 4
or N1 =
5 2 5

2
or, N1 = W ....(iii)
3
The normal force by the floor is N2 = W = (10 kg) (9.8 m/s2) = 98 N.
251\D\Allen(IIT-JEE Wing)\2020-21\Nurture\Phy\Unit-03\02-Rigid Body Dynamics

2
The frictional force is f = N1 = W = 65 N.
3

Illustration 29. The ladder shown in figure has negligible mass and rests on
a frictionless floor. A crossbar connects the two legs of the
ladder at the centre as shown. The angle between the two
legs is 60°. The fat person sitting on the ladder has a mass of
80 kg. Find the constant forces exerted by the floor on each
leg and the tension in the crossbar.

95
JEE-Physics
Solution The forces acting on different parts are shown in figure.
Consider the vertical equilibrium of “the ladder plus the
person” system. The forces acting on this system are its
weight (80 kg)g and the contact force N + N = 2 N due to
the floor. Thus
2 N = (80 kg) g
or N = (40 kg) (9.8 m/s2) = 392 N.
Next consider the equilibrium of the left leg of the ladder. Taking torques of the forces acting
on it about the upper end,

2 2
N (2m) tan 30° = T(1 m) or T = N = (392 N) × = 450 N.
3 3

Illustration 30. A uniform rod of length l, mass m is hung from two strings of equal
length from a ceiling as shown in figure . Determine the
tensions in the strings ?
Solution TA + TB = mg ............(i)
Torque about point A is zero
3l l
So, TB × = mg ............(ii)
4 2
From eq. (i) & (ii),
TA = mg/3, TB = 2mg/3.

Problems Based Upon Rotational Equilibrium


1. A force F = 2 î + 3 ˆj - k̂ acts at a point (2, - 3, 1). Then magnitude of torque about point (0, 0, 2) will be:
(A) 6 unit (B) 3 5 unit (C) 6 5 unit (D) none of these

2. Two uniform rods of equal length but different masses are rigidly joined to
form an L-shaped body, which is then smoothly pivoted about O as shown. If m O
in equilibrium the body is in the shown configuration, ratio M/m will be :
90° M
(A) 2 (B) 3 30°

(C) 2 (D) 3

3. Figure shows an arrangement of masses hanging from a ceiling. In equilibrium, each rod is horizontal, has
negligible mass and extends three times as far to the right of the wire supporting it as to the left. If mass m 4 is
251\D\Allen(IIT-JEE Wing)\2020-21\Nurture\Phy\Unit-03\02-Rigid Body Dynamics

48 kg then mass m1 is equal to

m4
m3
m2 m1
(A) 1 kg (B) 2 kg (C) 3 kg (D) 4 kg

96
Rigid Body Dynamics

4. A uniform rod of mass M and length L leans against a frictionless


wall, with quarter of its length hanging over a corner as shown.
Friction at corner is sufficient to keep the rod at rest. Then the
ratio of magnitude of normal reaction on rod by wall and the
magnitude of normal reaction on rod by corner is

1 2 1 2
(A) (B) (C) (D)
2 sin q sin q 2 cos q cos q

5. In figure the uniform gate weighs 300 N and is 3 m wide & 2 m high. It is
supported by a hinge at the bottom left corner and a horizontal cable at the
top left corner, as shown. Find :
(a) the tension in the cable and
(b) the force that the hinge exerts on the gate (magnitude & direction).

r
6. If F = 3î +5 ĵ-2k̂ N acting at 7 î - 2 ĵ + 5k̂ , then find the torque about the point (0, - 1, 0).

7. A uniform horizontal meter scale of mass m is suspended by two vertical strings attached to its two ends.
A body of mass 2 m is placed on the 75 cm mark. Find the ratio of tensions in the strings.

4.0 ROTATION ABOUT A FIXED AXIS

Consider a rigid body rotating about a fixed axis AB (figure). Consider a particle P of mass m rotating in a circle
of radius r.

v2
The radial acceleration of the particle = = w2 r .
r
(Where v is speed of particle, w is angular speed of body at r is distance
of particle from axis of rotation
2
Thus, the radial force on it = mw r

dv
The tangential acceleration of the particle =
dt

dv dw
Thus, the tangential force on it = m = mr = mra
dt dt

The torque of mw2r about AB is zero as it intersects the axis and that of mra is mr2a as the force and the axis
are skew and perpendicular. Thus, the torque of the resultant force acting on P is mr 2a. Summing over all the
particles, the total torque of all the forces acting on all the particles of the body is
251\D\Allen(IIT-JEE Wing)\2020-21\Nurture\Phy\Unit-03\02-Rigid Body Dynamics

ttotal = å m r a = Ia
i
i i
2
....(i)

where
I= åm r
i
i i
2

The quantity I is called the moment of inertia of the body about the axis of rotation. Note that m i is the mass of
the ith particle and ri is its perpendicular distance from the axis.

97
JEE-Physics

Illustration 31. A wheel of radius r and moment of inertia I about its axis is
fixed at the top of an inclined plane of inclination q as shown
in figure. A string is wrapped round the wheel and its free
end supports a block of mass M which can slide on the plane.
Initially, the wheel is rotating at a speed w in a direction such
that the block slides up the plane. How far will the
block move before stopping ?
Solution Suppose the deceleration of the block is a. The linear deceleration of the rim of the wheel is
also a. The angular deceleration of the wheel is a = a/r. If the tension in the string is T, the
equations of motion are as follows:
Mg sin q – T = Ma and Tr = Ia = Ia/r.
Eliminating T from these equations,
a
Mg sinq – I 2 = Ma
r
Mg r 2 sin q
giving, a =
I + Mr 2
The initial velocity of the block up the incline is v = w r. Thus, the distance moved by the
block before stopping is

v2 w2r 2 (I + Mr 2 ) (I + Mr 2 )w2
x= = =
2a 2M r 2 sin q 2M g sin q

////////////////////////////
Illustration 32. The pulley shown in figure has a moment of inertia I about its axis
I
and its radius is R. Find the magnitude of the acceleration of the
R
two blocks. Assume that the string is light and does not slip on the
pulley. m
Solution Suppose the tension in the left string is T 1 and that in the right string M
in T2. Suppose the block of mass M goes down with an acceleration
a and the other block moves up with the same acceleration.
This is also the tangential acceleration of the rim of the wheel as the string does not slip over
the rim. The angular acceleration of the wheel is, therefore, a = a/R. The equations of motion
for the mass M, the mass m and the pulley are as follows :
Mg – T1 = Ma .........(i)
T2 – mg = ma .........(ii)
T1R – T2R = Ia = Ia /R ......(iii)
Putting T1 and T2 from (i) and (ii) into (iii),
a (M - m)gR 2
[(Mg – a) – m(g + a)] R = I which gives a = .
R I + (M + m)R 2

Illustration 33. A uniform rod of mass m and length l can rotate in vertical plane
about a smooth horizontal axis hinged at point H.
251\D\Allen(IIT-JEE Wing)\2020-21\Nurture\Phy\Unit-03\02-Rigid Body Dynamics

(i) Find angular acceleration a of the rod just after it is released from initial horizontal
position from rest ?
(ii) Calculate the acceleration (tangential and radial) of point A at this moment.
(iii) Calculate net hinge force acting at this moment.
Solution (i) tH = I H a
l ml 2 3g
mg. = a Þ a=
2 3 2l
3g 3g
(ii) a tA = al = .l =
2l 2
a CA = w2 r =0 ( Q w = 0 just after release)
98
Rigid Body Dynamics

(iii) Suppose hinge exerts normal reaction in component form as shown

In vertical direction F ext = ma CM


3g l mg
Þ mg – N 1 = m. Þ a cm = a Þ N1 =
4 2 4
In horizontal direction
F ext = ma CM Þ N 2 = 0 ( Q a CM in horizontal = 0 as w = 0 just after release).

Illustration 34. A uniform rod of mass m and length l can rotate in vertical plane
about a smooth horizontal axis hinged at point H. Find angular
acceleration a of the rod just after it is released from initial position
making an angle of 370 with horizontal from rest ? Find force exerted
by the hinge just after the rod is released from rest.

Solution Torque about hinge = tH = I a

l ml 2
mg cos 37° = a
2 3
a = 6g / 5l
l 3g
at = a =
2 5
mg cos 37° – N 1 = mat
mg
N1 =
5
angular velocity of rod is zero. so N 2 = mgsin37° = 3mg/5

2 2
æ mg ö æ 3mg ö mg 10
N= N12 + N 2 2 = ç ÷ +ç ÷ =
è 5 ø è 5 ø 5

Rotation about Fixed Axis


1. A uniform disc of mass m, radius r and a point mass m are arranged as shown
in the figure. The acceleration of point mass is: (Assume there is no slipping
between pulley and thread and the disc can rotate smoothly about a fixed
horizontal axis passing through its centre and perpendicular to its plane)
g g
(A) (B)
2 3
2g
251\D\Allen(IIT-JEE Wing)\2020-21\Nurture\Phy\Unit-03\02-Rigid Body Dynamics

(C) (D) none of these


3

2. A nearly massless rod is pivoted at one end so that it can swing freely as a pendulum. Two masses 2m and
m are attached to it at distances b and 3b respectively from the pivot. The rod is held horizontal and then
released. The angular acceleration of the rod at the instant it is released is
b 5g
(A) bg (B) (C) gb + 5 (D)
g 11b

99
JEE-Physics
3. In the pulley system shown, if radii of the bigger and smaller pulley are 2 m and 1 m
respectively and the acceleration of block A is 5 m/s2 in the downward direction, then
the acceleration of block B will be:
(A) 0 m/s2 (B) 5 m/s2 upwards
(C) 10 m/s2 upwards (D) 5/2 m/s2 upwards

4. A uniform thin rod of mass ‘m’ and length L is held horizontally by two vertical strings attached to the two ends.
One of the string is cut. Find the angular acceleration soon after it is cut :
g g 3g 2g
(A) (B) (C) (D)
2L L 2L L

5. Uniform rod AB is hinged at the end A in a horizontal position as shown in the


figure. The other end is connected to a block through a massless string as shown.
The pulley is smooth and massless. Masses of the block and the rod are same
and are equal to ' m '. Then acceleration of the block just after release from this
position is:
(A) 6 g/13 (B) g/4
(C) 3 g/8 (D) none

6. A uniform disc of mass M and radius R is released from the shown position.
PQ is a string, OP is a horizontal line, O is the centre of the disc and
distance OP is R/2. Then tension in the string just after the disc is released
will be :
Mg Mg
(A) (B)
2 3
2Mg
(C) (D) none of these
3

7. Figure shows two blocks of masses m and M connected by a string


passing over a pulley. The horizontal table over which the mass m
slides is smooth. The pulley has a radius r and moment of inertia I
about its axis and it can freely rotate about this axis. Find the
acceleration of the mass M assuming that the string does not slip on
the pulley.

5.0 COMBINED TRANSLATIONAL AND ROTATIONAL MOTION OF


A RIGID BODY

The general motion of a rigid body can be thought of as a sum of two independent motions. A translation
of some point of the body plus a rotation about this point . A most convenient choice of the point is the
centre of mass of the body as it greatly simplifies the calculations.
Consider a fan inside a train, and an observer A on the platform.
251\D\Allen(IIT-JEE Wing)\2020-21\Nurture\Phy\Unit-03\02-Rigid Body Dynamics

If the fan is switched off while the train moves, the motion of fan is pure translation as each point on the
fan undergoes same translation in any time interval.
If fan is switched on while the train is at rest the motion of fan is pure rotation about axle ; as each point
on the axle is at rest, while other points revolve about it with equal angular velocity.
if the fan is switched on while the train is moving, the motion of fan to the observer on the platform is
neither pure translation nor pure rotation. This motion is an Illustration of general motion of a rigid body.
Now if there is an observer B inside the train, the motion of fan will appear to him as pure rotation.
Hence we can see that the general motion of fan w.r.t. observer A can be resolved into pure rotation of
fan as observed by observer B plus pure translation of observer B (w.r.t. observer A)
Such a resolution of general motion of a rigid body into pure rotation & pure translation is not restricted
to just the fan inside the train, but is possible for motion of any rigid system.

100
Rigid Body Dynamics
5.1 Instantaneous Axis of Rotation (IAR)

It is a mathematical line about that a body in combined translation and rotation can be conceived in pure
rotation at an instant. It continuously changes its location. This axis is always perpendicular to the plane used
to represent the motion and the intersection of the axis with this plane defines the location of instantaneous
centre of zero velocity (IC).
Location of the Instantaneous Centre of Rotation
If the location of the IC is unknown, it may be determined by using the fact that the relative position vector
extending from the IC to a point is always perpendicular to the velocity of the point. Following three possibilities
exist.
(i) Given the velocity of a point (normally the centre of mass) on the body and the angular
velocity of the body.
If v and w are known, the IC is located along the line drawn perpendicular
v
to ®v at P, such that the distance from P to IC is,
r = . Note that IC lie on
w
that side of P which causes rotation about the IC, which is consistent
® ®
with the direction of motion caused by w and v

(ii) Given the lines of action of two non-parallel velocities


Consider the body shown in figure where the line of action of the velocities
® ®
v A and v B are known. Draw perpendiculars at A and B to these lines of
action. The point of intersection of these perpendiculars
as shown locates the IC at the instant considered.

(iii) Given the magnitude and direction of two parallel velocities


When the velocities of points A and B are parallel and have known magnitudes v A and vB then the
location of the IC is determined by proportional triangles as shown in figure.

vA vB
In both the cases, rA = and rB =
w w
In fig. (a) rA + rB = d
and in fig. (b) rB – rA = d
As a special case, if the body is translating, vA = vB and the IC would be located at infinity, in which
case w = 0.
251\D\Allen(IIT-JEE Wing)\2020-21\Nurture\Phy\Unit-03\02-Rigid Body Dynamics

Illustration 35. A 100 cm rod is moving on a horizontal surface. At an instant, y


30 cm/s
when it is parallel to the x-axis its ends A and B have velocities x
30 cm/s and 20 cm/s as shown in the figure.
A B
(a) Find its angular velocity and velocity of its center.
20 cm/s
(b) Locate its instantaneous axis of rotation.
Solution. Velocity vector of point B
r r r uuur
v B = v A + w ´ AB ® w = 0.5 rad/s
Velocity vector of the center C of the rod.
r r r uuuur r
v C = v A + w ´ AC ® v C = -20ˆj + 0.5kˆ ´ 50ˆi = 5.0ˆj cm/s
101
JEE-Physics
B

30 cm/s

A B
20 cm/s P
A
’P
(b) Here velocity vectors of the particles A and B are antiparallel, therefore the instantaneous
axis of rotation passes through intersection of the common perpendicular to their velocity vectors
and a line joining tips of the velocity vectors. The required geometrical construction is shown in
the following figure.
Since triangles AA’P and BB’P are similar and AB = 100 cm, we have AP = 40 cm.
The instantaneous axis of rotation passes through the point P, which is 40 cm from A.
Analytical Approach.
The instantaneous center of rotation is at instantaneous rest.
r r r uuur r
v P = v A + w ´ AP ® 0 = -20ˆj + 0.5kˆ ´ ( AP ) ˆj Þ AP = 40 cm

Illustration 36. Can you suggest a quick way to find angular velocity of a rod, if velocities of two of its points are
known?
Solution. when distance between two points and their velocity
components perpendicular to the lining joining them are vB^
known.
vA||
Angular velocity of the rod
A
r r r B vB||
v BA v B^ - v A^ vA^
w= =
AB AB

Illustration 37. A 50 cm long rod AB is in combined translation and


rotation motion on a table. At an instant velocity
vBx
component of point A perpendicular the rod is 10 cm/s, A
velocity component of point B parallel to the rod is 6.0 vAy
B
cm/s and angular velocity of the rod is 0.4 rad/s in
anticlockwise sense as shown in the given figure.
(a) Find velocity vectors of point A and B.
(b) Locate the instantaneous axis of rotation.
Solution. Let x-y plane of a coordinate system coincides with the tabletop and the rod is parallel to the x-
axis at the instant considered. The rod is shown in this coordinate frame in the following figure.
y
x vBy
v
Ax
A
B vBx
vAy

(a) Since distance between any two points remains unchanged, the velocity components of any
251\D\Allen(IIT-JEE Wing)\2020-21\Nurture\Phy\Unit-03\02-Rigid Body Dynamics

two points parallel to the line joining them must be equal. Therefore, we have
v Ax = v Bx = 6.0 cm/s (1)
Velocities of points A and B must satisfy following equation
r r r uuur
v B = v A + w ´ AB ® 6.0iˆ + v By ˆj = 6.0iˆ - 10ˆj + 0.4kˆ ´ 50iˆ
Equating y-components of both the sides, we have
v By = 10 cm/s (2)
From eq. (1), (2) and the given information, we can express the velocity vectors of the points A and B.
r r
v A = 6.0iˆ - 10ˆj cm/s, and v B = 6.0iˆ + 10ˆj cm/s

102
Rigid Body Dynamics
(b) The instantaneous axis of rotation is at instantaneous rest. Let the end A of the rod is at the
origin and coordinates of the point P in the x-y plane through which the IAR passes is (x, y).
r r r uuur r
(
v P = v A + w ´ AP ® 0 = 6.0ˆi - 10ˆj + 0.4kˆ ´ xiˆ + yjˆ )
Equating coefficients of x and y-components of both the sides, we have
x = 25 cm and y = 15 cm
Therefore, coordinates of the point P through which IAR passes the x-y plane are (25, 15).

5.2 Rolling as Rotation about an Axis in Translation

Wheels of a moving vehicle roll on road. A ball rolls on ground when pushed. In fact, a body of round section
can roll smoothly under favorable conditions. On the other hand, objects with corners, such as dice, roll by
successive rotations about the edge or corner that is in contact with the ground. This type of motion is usually
known as toppling.
If the point of contact of the of the rolling body does not slide it is known as rolling without slipping or pure
rolling or simply rolling and if the point of contact slides it is known as rolling with slipping.
All kind of rolling motion is examples of rotation about an axis in translation.
Rolling without slipping on stationary surface.
We first discuss velocity relations and thereafter accelerations relations of two points of a body of round
section rolling on a stationary surface. For the purpose, we can use any of the following methods.
I Analytical Method: By using relative motion equations.
II Superposition Method: By superimposing translation of a point and pure rotation about that point.
III Use of ICR.
Velocity relations by Analytical Method
Its point of contact P does not slide on the surface, therefore velocity of the point of contact relative to the
surface is zero. In the next figure, velocity vectors of its center C and top point A are shown.

A vA
C
y
B rB
C x C vC
q
w

vP = 0

P P

Velocity of the center C can be obtained with the help of relative motion equation.
r r r uuur r r
v C = v P + w ´ PC ® v C = 0 + ( -w kˆ ) ´ Rjˆ
251\D\Allen(IIT-JEE Wing)\2020-21\Nurture\Phy\Unit-03\02-Rigid Body Dynamics

r
v C = w Riˆ

The above equation is used as condition of rolling without slipping on stationary surface.
Velocity of the top point A can be obtained by relative motion equation.
r r r uuur r r
v A = v P + w ´ PA ® v A = 0 + ( -wkˆ ) ´ (2Rjˆ )
r r
v A = 2w Riˆ = 2v C
Once velocity of the center is obtained, we can use relative motion between A and C as well.

103
JEE-Physics
r r r uuur r
v A = v C + w ´ CA ® v C = wRiˆ + ( -wkˆ ) ´ ( Rjˆ)
r r
v A = 2w Riˆ = 2v C
In similar fashion, velocity vector of an arbitrarily chosen point B.
r r r uuur r
v B = v C + w ´ CB ® v B = v C ˆi + ( -w kˆ ) ´ ( -r cos q iˆ + r sin q ˆj )
r
v = (v + w r sin q ) iˆ + w r cos q ˆj
B C

Velocity relations by Superposition Method


Now we will see that the above velocity relation can also be obtained by assuming rolling of the wheel as
superposition of translation of its center and simultaneous rotation about the center.
v C = wR v A / C = wR y
A A A v A = 2w R
x

v C = wR v C = wR
C C w C

P v C = wR v P / C = wR P P

Translation of the center Pure rotation about the center Rolling

Velocity of an arbitrary point B as superposition of translation of the center and rotation about the center.

vBC=wr vC r
v B = ( v C + wr sin q) ˆi + wr cos qˆj
q

B r C
vC y
q
w x
w

vP=0
P

Velocity relations by Use of ICR vA=2wR


In rolling without slipping on stationary surface the point of contact
is at instantaneous rest, therefore the ICR and the IAR passes through
it. We will see how velocity of the center C, the top point A and an B r
C y
arbitrarily chosen point B can be calculated by assuming the body in q
state of pure rotation about the ICR. w vC=wR x
r r uuur R
Velocity of center C v C = w ´ PC = w Riˆ B
r uuur
P w
r
Velocity of the top point A v A = w ´ PA = 2wRiˆ
r r uuur
Velocity of the point B v B = w ´ PB = (v C + w r sin q )iˆ + w r cos q ˆj
251\D\Allen(IIT-JEE Wing)\2020-21\Nurture\Phy\Unit-03\02-Rigid Body Dynamics

Illustration 38. A cylinder of radius 5 m rolls on a horizontal surface. Velocity of its center is 25 m/s. Find its
angular velocity and velocity of the point A.
A
Solution In rolling the angular velocity wr and velocity of the center of a y

round section body satisfy condition 37 vC x


r r r r
º C
v C = w ´ rC / P ® 25ˆi = w kˆ ´ 5ˆj Þ w = -5kˆ rad/s
Angular velocity vector points in the negative z-axis so the P
cylinder rotates in clockwise sense.
104
Rigid Body Dynamics
Velocity of the point A can be calculated by either analytical method, superposition method or by
using method of ICR.
Analytical Method
r r r uuur r
v A = v C + w ´ CA ® v A = 25ˆi + ( -5kˆ ) ´ ( -5 cos 37°ˆi + 5 sin 37°ˆj )
r
v A = ( 40iˆ + 20ˆj ) m/s
Superposition Method vAC vC

In rolling v C = v AC = wR = 25m / s . The superposition i.e. vA


37
r r r º y
vector addition of the terms of equation v A = v C + v AC aree A
37 vC x
shown in the following figure. Resolving v AC = wR = 25m / s
º
r C
into its Cartesian components and adding to v C , we obtain
r r r r
v A = 25iˆ + 15iˆ + 20ˆj = ( 40iˆ + 20ˆj ) m/s
P
v A = vC + v A /C ®

Use of ICR
vA
The contact point P is the ICR in rolling. The cylinder is in
A y
pure rotation about the ICR at the instant under
37
consideration, so from the relative motion equation, we have º C x
r r r r r uuur uuur
{ }
v A = w ´ rAP ® v A = w ´ ( PC + CA ) = ( -5kˆ ) ´ ( 5 ˆj ) + ( -4iˆ + 3ˆj )
r
v A = ( 40iˆ + 20ˆj ) m/s
P

Illustration 39. A disc of radius r is rolling down a circular track of radius R. There is
O
no slipping between the disc and the track. When line OC is at angle q
q down the horizontal, center of the disc has velocity vC. Assume
C
center O of the track as origin of reference frame, find angular velocity
of translation motion of the center of the disc and angular velocity of
rotation motion.
Solution. The angular velocity of translation motion of the center of the disc equals to the rate of change is
q . Let us denote it by wo.
O
dq q
wo =
dt
The center of the disc moves on circular path of radius R - r. Relation C
between velocity vC of the center of the disc, radius R of the circular
vC
track and radius of the disc is
v C = w o (R - r )
vC
Therefore, angular velocity of translation motion of center of the disc is w o =
R -r
Since the disc is rolling without slipping on the circular track, its angular velocity of rotation w is
given by the following equation.
vC = wr
251\D\Allen(IIT-JEE Wing)\2020-21\Nurture\Phy\Unit-03\02-Rigid Body Dynamics

v
Therefore, angular velocity of rotation of the disc is w = C
r

Acceleration relations by Analytical Method


The point of contact P does not slide on the surface, therefore y
component of its acceleration parallel to the surface must be zero.
x
However, it has an acceleration component towards the center. The C
aC = a R
center always moves parallel to the horizontal surface and does not
changes direction of its velocity; therefore, its acceleration can only aPy = w 2R
be parallel to the surface. P aPx = 0

105
JEE-Physics
Relation between acceleration of acceleration vector of the center C
and point of contact P.
r uuur uuur
r r
aC = aP + a ´ PC - w 2 PC ® ( )
aC iˆ = aPy ˆj + -a kˆ ´ Rjˆ - w 2 Rjˆ = a Py ˆj + a Riˆ - w 2Rjˆ

Equating coefficients of x and y-components on both the sides of the above equation, we have
r r
aC = a Riˆ ; a P = w 2 Rjˆ

Condition of pure rolling on fixed surface is a cm = ±aR


In the given figure, acceleration vectors the point of contact; center and the top point are shown. Now we will
see how these accelerations can be calculated by using relative motion equation.
A a Ax = 2a R
Once velocity of the center is obtained, we can use relative motion
aA
between A and C as well. Now we calculate acceleration of the top
a Ay = w 2R
point A. y

r r r uuur uuur r
a A = a Riˆ + ( -a kˆ ) ´ Rjˆ - w 2 Rjˆ
C x
a A = aC + a ´ CA - w 2 CA ® aC = a R

r
a A = 2a Riˆ - w 2 Rjˆ

Acceleration vector of point A and its components are shown in the given figure.

Acceleration relations by Superposition Method


Now we see how acceleration relations are expressed for a rolling wheel by assuming its rolling as superposition
of its translation with the acceleration of center and simultaneous rotation about the centre.

a aR a+aR

wR
2
wR
2

aR aR
a a a w R
2
w 2R w 2R+a a
wR
2

aR aR
wR
2
wR2

a aR aR a

Acceleration relations by use of ICR A 2a R


Acceleration relations can also be obtained by assuming the body in aA
pure rotation about the ICR. Here we will use relative motion w 2R

equation. Always keep in mind that the acceleration of the ICR is aC = a R


y
not zero, it has value w2R and points towards the center of the body. C
x
Now we will see how acceleration of the center C, the top point A
and an arbitrarily chosen point B can be calculated by assuming the
251\D\Allen(IIT-JEE Wing)\2020-21\Nurture\Phy\Unit-03\02-Rigid Body Dynamics

P
body in state of pure rotation about the ICR.
r r r uuur uuur
Acceleration of center C aC = aP + a ´ PC - w2 PC = aRiˆ
r
aC = aRiˆ r
v B = ( v C + wr sin q) ˆi + wr cos qˆj
r uuur uuur B r
r r
Acceleration of the top point A a A = ap + a ´ PA - w 2 PAjˆ q C
y
w x
r
a A = 2a Riˆ - w 2 Rjˆ R

w vP=0
P

106
Rigid Body Dynamics
r r r uuur uuur
Acceleration of the point B aB = aP + a ´ PB - w 2 PB
r
{ } { }
aB = a ( R + r sin q ) + w 2 iˆ + a r cos q - w 2 r sin q iˆ

A
Illustration 40. A body of round section of radius 10 cm starts rolling on a y
horizontal stationary surface with uniform angular a
C x
acceleration 2 rad/s2.
(a) Find initial acceleration of the center C and top point A.
(b) Find expression for acceleration of the top point A as P
function of time.
r r r uuur
Solution. Initially when the body starts, it has no angular velocity; therefore, a A = a B + a ´ BA
The angular acceleration vector is ar = -2kˆ rad/s2.
(a) Acceleration of the center C is obtained by using condition for rolling without slipping.
r r uuur r
aC = a ´ PC ® aC = -2kˆ ´ 10ˆj = 20iˆ cm/s2
Acceleration of the point A can be obtained either by analytical method, superposition method or
by use of ICR. These methods for calculation of acceleration of the top point are already described;
therefore, we use the result.
r r
a A = 2a Riˆ ® a A = 40iˆ cm/s2
(b) Initially at the instant t = 0, when the body starts, its angular velocity is zero. At latter time it
r
acquires angular velocity w , therefore acceleration of any point on the body, other than its center,,
has an additional component of acceleration.
Angular velocity acquired by the body at time t is
r r r r
w = wo + at ® Substituting wo = 0 , we have w = -2tkˆ

Analytical Method
Using the relative motion equation for the pair of points C and A, we have
r uuur uuur r
r r
a = a + a ´ CA - w 2 CA ®
A C A ( )
a = a Riˆ + -a kˆ ´ Rjˆ - w 2 Rjˆ = 2a Riˆ - w 2 Rjˆ
Substituting the known values
r 2 r
a = -2kˆ rad/s , w = -2tkˆ rad/s and R = 10 cm
r
we have a A = 40iˆ - 40t 2 ˆj cm/s
A aC a AC t = a R
Superposition Method
We superimpose translation motion of the center and rotation w R
2 y

motion about the center. In fact it is vector addition of terms of aC = 20 x


w
above equation used in analytical method. a
C
From the above figure, we have
P
r
a A = ( aC + a R ) iˆ - w 2 Rjˆ
r r
Substituting known values a = -2kˆ rad/s2, w = -2tkˆ rad/s and R=10 cm,
r
we have a A = 40iˆ - 40t 2 ˆj cm/s2
251\D\Allen(IIT-JEE Wing)\2020-21\Nurture\Phy\Unit-03\02-Rigid Body Dynamics

Use of ICR
The point of contact P is the ICR, because the body is rolling without slipping. We use relative
motion equation for pairs of points P and A.
r r r uuur uuur r
a A = aP + a ´ PA - w 2 PA ® a A = w 2 Rjˆ + 2a Riˆ - 2w 2 Rjˆ = 2a Riˆ - w 2Rjˆ
Substituting known values
2
a = 2 rad/s , w = 4t rad/s and R = 10 cm,
r
we have a A = 40iˆ - 40ˆj cm/s2

107
JEE-Physics

Illustration 41. Two identical discs, each of radius r, are connected by a cord as I
shown in the figure. The disc I rotates with constant angular
acceleration a in anticlockwise direction. Find acceleration of II
the center of disc II and its angular acceleration. B C A

Solution. As the disc I rotates, the thread unwrap it. Acceleration of a point on the portion of the thread
between the two discs equals to tangential acceleration of any point on the periphery of the disc
I. The extreme left point A of the disc II must also descend with the same acceleration.
Downward acceleration aB of point B = Tangential acceleration of a point on the periphery of
disc I.
aB = a I r ® aB = a r
The point B on the thread is at rest relative to the ground; therefore, it can be assumed that the
second disc is in a motion similar to rolling without slipping on a vertical surface.
Now applying conditions of rolling without slipping we have
Acceleration of the center of the disc II aC = 12 a B = 12 a r
aC 1
Angular acceleration of the disc II a II = = 2a
r

Illustration 42. A wheel of radius r rolls (rolling without slipping) on a level


road as shown in figure. Find out velocity of point A and B

Solution Contact surface is in rest for pure rolling velocity of point A is zero.
so v = w r
velocity of point B = v + wr = 2v
B F
Illustration 43. A force F acts tangentially at the highest point of a r
C
sphere of mass m kept on a rough horizontal plane. If
the sphere rolls without slipping, find the acceleration
of the centre (C). A rough surface
Solution The situation is shown in figure. As the force F rotates the sphere, the point of contact has a
tendency to slip towards left so that the static friction on the sphere will act towards right. Let
r be the radius of the sphere and a be the linear acceleration of the centre of the sphere. The
angular acceleration about the centre of the sphere is a = a/r, as there is no slipping.
For the linear motion of the centre,
F + f = ma ..........(i)
and for the rotational motion about the centre,

æ2 2ö æaö 2
Fr – f r = I a = ç mr ÷ ç ÷ or, F–f= ma, ...........(iii)
è5 ø èrø 5

7 10 F
From (i) and (ii), 2F = ma or a= .
251\D\Allen(IIT-JEE Wing)\2020-21\Nurture\Phy\Unit-03\02-Rigid Body Dynamics

5 7m

Illustration 44. A circular rigid body of mass m, radius R and radius of gyration (k) rolls without slipping on an
inclined plane of a inclination q. Find the linear acceleration of the rigid body and force of friction
on it. What must be the minimum value of coefficient of friction so that rigid body may roll without
sliding?
Solution If a is the acceleration of the centre of mass of the rigid body and f the force of friction between
sphere and the plane, the equation of translatory and rotatory motion of the rigid body will be
mg sin q – f = ma (Translatory motion)

108
Rigid Body Dynamics
fR = I a (Rotatory motion)
Ia
f=
R
I = mk2 , due to pure rolling a = aR
Ia
mg sin q – = maR
R
Ia
mg sin q = m aR +
R
mk 2 a
mg sin q = m a R +
R
mk 2 a
mg sin q = ma +
R
éR 2 + k 2 ù
mg sin q = a ê 2 ú
êë R úû

g sin q
a=
é R2 + k 2 ù
ê 2 ú
ëê R ûú

g sin q
a=
æ k2 ö
çç 1 + R 2 ÷÷
è ø
Ia
f=
R
mk 2 a mg k 2 sin q
f= Þ
R2 R2 + k 2
f £ mN

mk 2
a £ m £ mg cos q
R2
k2 g sin q
R2 × £ µg cos q
R 2 (k 2 + R 2 )

tan q

é R2 ù
ê1 + 2 ú
ë k û

tan q
mmin. =
é R2 ù
ê1 + 2 ú
251\D\Allen(IIT-JEE Wing)\2020-21\Nurture\Phy\Unit-03\02-Rigid Body Dynamics

ë k û
From above Illustration if rigid bodies are solid cylinder, hollow cylinder, solid sphere and hollow
sphere.
Increasing order of acceleration : asolid sphere > asolid cylinder > ahollow sphere > ahollow cylinder

Illustration 45. A block of mass m is attached at one end of a thin light cord, which passes over an ideal pulley. At
the other end, it is wrapped around a cylinder of mass M, which can roll without slipping over a
horizontal plane.
(a) What is the acceleration of the block?
(b) What is the friction force on the cylinder?

109
JEE-Physics
Solution. The problem requires solution of the force and the torque equations consistent with the condition
of rolling, so it is not necessary to decide the direction of friction. To start with let the static friction
f acts in the forward direction.
s
(a) Let the block descend with acceleration a. Since the cord y A a
is inextensible the top point A of the cylinder also moves x a C

with the same acceleration. Applying relative motion C


a
equation with the condition required for rolling that the
P
particle of the cylinder at the point of contact has no a

acceleration parallel to the horizontal plane.


r r r
( ) ( )
a A = a ´ rA / P ® a A iˆ = -a kˆ ´ 2rjˆ Þ a =
a
2r
(1)

From eq. (1) and relative motion equation for P and the center C, we have
r r r
( ) ( )
aC = a ´ rC / P ® aC ˆi = -a kˆ ´ rjˆ Þ aC = 12 a (2)
T

The block is in translation motion under the action of its weight mg and
tension T in the string. These forces and the effective force ma are shown mg ma
in the adjacent figure.
Applying Newton’s second law for translation of mass center, we have
r r
å Fi = MaC ® mg - T = ma (3)
The cylinder is in rolling under the action of its weight Mg, normal reaction N from the ground;
tension T in the cord and force of static friction f . These forces, the effective force Ma and the
s C
effective torque I a are shown in the adjacent figure.
C
Applying Newton’s second law for translation of mass center, we have

åF x = MaCx ® T + fs = MaC Mg T
y

x
Substituting a from eq. (2), we have I C a Ma
C C
C

T + fs = 12 Ma (4) P C
fs
Applying torque equation for centroidal rotation, we have
N
r r
å tC = I C a ® Tr - fs r = I C a
Substituting 1
2
mr 2 for I , and value of a form eq. (1), we have
C

T - fs = 14 Ma (5)
From eq. (3), (4) and (5), we have
8mg
Acceleration of the block a=
3M + 8m
(b) From eq. (4), (5) and above value of acceleration a, we have
Mmg
Force of static friction fs =
3M + 8m
251\D\Allen(IIT-JEE Wing)\2020-21\Nurture\Phy\Unit-03\02-Rigid Body Dynamics

Illustration 46. A uniform sphere of mass m and radius r is projected along a


rough horizontal floor with linear velocity vo and no angular
velocity. The coefficients of kinetic and static frictions are vo

represented by ms and mk respectively.


(a) How long the sphere will slide on the floor before it starts
rolling.
(b) How far the sphere will slide on the floor before it starts
rolling.
(c) Find the linear and angular velocities of the sphere when
it starts rolling.
110
Rigid Body Dynamics

Solution When the sphere touches the floor it is on translation motion. mg


All the points including the bottom one are moving with the y
same velocity v . Thus the bottom point which makes the contact x
o
with the floor slide on it causing kinetic friction to act in backward C

direction. In the adjacent figure the forces acting on the sphere


are shown. Here mg represent weight, N the normal reaction
from the ground and f . fk
k
N
Since the sphere has no vertical component of acceleration, by applying
Newton’s law we have

åF y = 0® N = mg
The kinetic friction fk = mN = m mg (1)
The only force which applied torque about the centroidal axis is the kinetic friction. Angular
impulse of torque of kinetic friction increases the angular velocity w and impulse of kinetic friction
decreases the mass center velocity v till both bear following condition required for rolling. Thereafter
C
the sphere will continue to roll with the unfirom velocity.
vC = wr (2)
In the adjacent figure of impulse momentum diagram the impulse of kinetic friction is shown

y
p1 = mv o x p2 = mv C
C C C
LC 1 = 0 LC 2 = I C w

ò f dt = f t
k k

Translation of mass center:Applying linear impulse momentum principle in x direction, we have


r r r
p1 + å I mp1® 2 =p2 ® p1 - fkt = p2
Substituting p , p and f from eq. (1), we have
1 2 k
v C = v o - m gt (3)
Centroidal rotation: Angular impulse momentum principle about the centroidal axis.
r r r
LC 1 + å J C ,1®2 = LC 2 ® 0 + fk rt = I C w

Substituting 2
5
mr 2 for I and f from eq. (1), we have
C k

5m gt
w= (4)
2r
(a) Substituting values of v and w form eq. (3) and (4) into eq. (2), we have
C

2v o
Time when rolling starts t= (5)
251\D\Allen(IIT-JEE Wing)\2020-21\Nurture\Phy\Unit-03\02-Rigid Body Dynamics

7m g
(b) Eq. (3) reveals that the mass center is in uniformly retarded motion. So its displacement in
time t, when it starts rolling is given by the following equation.
x= 1
2 (v o + v C ) t ® Substituting values for vC and t from eq. (3) and (5) respectively we have
12v o2
x=
49m g

111
JEE-Physics
(c) Linear and angular velocities of the sphere when it starts rolling can be obtained by substituting
t from eq. (5) into (3) and (4) respectively.
Linear velocity when rolling starts v C = 57 v o

5v o
Angular velocity when rolling starts w=
7r

Combined translations and Rotational Motion


1. A sphere of mass m rolls without slipping on an inclined plane of inclination q. The linear acceleration of the
sphere is :-
1 2 3 5
(A) g sin q (B) g sin q (C) g sin q (D) g sin q
7 7 7 7

2. A solid sphere rolls on an rough inclined plane and reaches at bottom with speed v 1. Now the sphere is
released from similar typed but smooth inclined plane then reaches at bottom with speed v 2. Then–
(A) v1 = v2 (B) v1 > v2 (C) v1 < v2 (D) v1 = 2v2

3. A thin uniform disc of mass M and radius R is rolling over a horizontal surface. Speed of it’s top most point
is found to be V wrt surface over which it is rolling. Total kinetic energy of the disc will be given by–
3 5 7 3
(A) MV 2 (B) MV 2 (C) MV 2 (D) MV 2
4 6 10 16

4. A solid sphere of mass m is lying at rest on a rough horizontal surface. The coefficient of friction between the
ground and sphere is m. The maximum value of F, so that the sphere will not slip, is equal to :-
7 4
(A) mmg (B) mmg
5 7 F
5 7
(C) mmg (D) mmg
7 2

5. A uniform circular disc of radius r placed on a horizontal rough surface has


initially a velocity v 0 and an angular velocity w0 as shown in the figure. The
disc comes to rest after moving some distance in the direction of motion. Then
v0/w0 is:
(A) r/2 (B) r
(C) 3 r/2 (D) 2

6. A sphere of mass ' m ' is given some angular velocity about a horizontal axis
through its centre and gently placed on a plank of mass ' m '. The co-efficient
of friction between the two is m. The plank rests on a smooth horizontal surface.
The initial acceleration of the centre of sphere relative to the plank will be:
251\D\Allen(IIT-JEE Wing)\2020-21\Nurture\Phy\Unit-03\02-Rigid Body Dynamics

(A) zero (B) mg (C) (7/5) mg (D) 2 mg

7. A sphere S rolls without slipping, moving with a constant speed on a


plank P. The friction between the upper surface of P and the sphere is
sufficient to prevent slipping, while the lower surface of P is smooth and
rests on the ground. Initially, P is fixed to the ground by a pin N. If N is
suddenly removed:
(A) S will begin to slip on P
(B) P will begin to move backwards
(C) the speed of S will decrease and its angular velocity will increase
(D) there will be no change in the motion of S and P will still be at rest.
112
Rigid Body Dynamics

8. Find the acceleration of solid right circular roller A, weighing 12 kg when it


is being pulled by another weight B (6 kg) along the horizontal plane as in
figure (pulley is massless). The weight B is attached to the end of a string
wound around the circumference of roller. Assume there is no slipping
of the roller and the string is inextensible.

9. A uniform cylinder of mass m and radius R as shown in figure starts


descending at a moment t = 0 due to gravity. Neglecting the mass of the
thread, find the tension of each thread and the angular acceleration of the
cylinder.

6.0 TOPPLING

In many situations an external force is applied to a body to cause it to slide along a surface. In certain
cases, the body may tip over before sliding ensues. This is known as toppling.
(1) There is a no horizontal force so pressure at bottom is uniform and normal is collinear with mg.

(2) If a force is applied at COM, pressure is not uniform Normal reaction shifts right so that torque of
N can counter balance torque of friction.

(3) If F is continuously increased N keeps shifting towards right until it reaches the right most point D.
Here we have assumed that the surface is sufficiently rough so
that there is no sliding as F is increase to F max. If force is
increased any further, then torque of N can not counter balance
torque of friction f r & body will topple.The value of force now
is the max value for which toppling will not occur F max.
F max = f r
N = mg
torque about COM
f r . b/2 = N . a/2 Þ f r = Na/b = mg a/b, F max = mg a/b
251\D\Allen(IIT-JEE Wing)\2020-21\Nurture\Phy\Unit-03\02-Rigid Body Dynamics

(4) If surface is not sufficiently rough and the body slides before F is
increased to F max = mg a/b then body will slide before toppling.
Once body starts sliding friciton becomes constant and hence no
toppling. This is the case if
F max > flimit Þ mg a/b > mmg
m < a/b
Condition for toppling when m ³ a/b in this case body will topple if F > mg a/b
but if m < a/b, body will not topple any value of F applied a COM

113
JEE-Physics

a
Illustration 47. Find the minimum value of F to topple about an edge. F
b M
N
Solution In case of toppling
F
Taking torque about O
b
a/2
æ aö Mga
F (b) = Mg çè ÷ø Þ Fmin =
2 2b
Mg

Illustration 48. A uniform cylinder of height h and radius r is placed with its circular face on a rough inclined
plane and the inclination of the plane to the horizontal is gradually increased. If m is the coefficient
of friction, then under what conditions the cylinder will (a) slide before toppling (b) topple before
sliding. N
f
Solution (a) The cylinder will slide if mg sin q>mmg cosq Þ tanq > m ...(i)
q
sin mgcosq
h 2r mg
The cylinder will topple if ( mg sin q) >(mgcosq)r Þ tanq> ... (ii) q
2 h

2r
Thus, the condition of sliding is tanq>m & condition of toppling is tanq> .
h
2r
Hence, the cylinder will slide before toppling if m <
h
2r
(b) The cylinder will topple before sliding if m >
h

Toppling
1. An equilateral prism of mass m rests on a rough horizontal surface with
coefficient of friction m. A horizontal force F is applied on the prism as shown
in the figure. If the coefficient of friction is sufficiently high so that the prism
does not slide before toppling, then the minimum force required to
topple the prism is :
mg mg mmg mmg
(A) (B) (C) (D)
3 4 3 4

2. In the figure shown, a cubical block is held stationary against a rough wall by
251\D\Allen(IIT-JEE Wing)\2020-21\Nurture\Phy\Unit-03\02-Rigid Body Dynamics

applying force ‘F’ then incorrect statement among the following is


(A) frictional force, f = Mg
(B) F = N, N is normal reaction
(C) F does not apply any torque about centre of mass
(D) N does not apply any torque about centre of mass

3. The torque acting on a body about centre of mass due to normal reaction–
(A) Is always zero
(B) Is always non-zero
(C) Is non-zero if body in acceleration
(D) May be non-zero on stationary or moving body

114
Rigid Body Dynamics
4. A uniform cube of side a & mass m rests on a rough horizontal table. A horizontal
force F is applied normal to one of the faces at a point that is directly above the
centre of the face, at a height 3 a/4 above the base. Find the minimum value of
F for which the cube begins to tip about an edge.
(assume that cube does not slide).

5. A uniform cube of side a and mass m rests on a rough horizontal table. A horizontal force f is applied
normal to one of the faces at a point that is directly above the centre of the face, at a height of 3a/4 above
the base. Find the minimum value of f for which the cube begins to tip about an edge? (Assume that the
cube does not slide)

7.0 ENERGY METHODS

Newton’s laws of motion tell us what is happening at an instant, while method of work and energy equips us
to analyze what happens when a body moves from one place to other or a system changes its configuration.
In this section, we introduce how to use methods of work and energy to analyze motion of rigid bodies.

7.1 Concept of Work in rotation motion

Work of a force is defined as the scalar product of the force vector and displacement vector of the point of
r r r
application of the force. If during the action of a force F its point of application moves from position r1 to r2 ,
the work W1®2 done by the force is expressed by the following equation.
r2 r r
r
r Q
W1®2 = òr F × dr F
r1

Either we can use of this idea to calculate work of a force or its modified P
version in terms of torque and angular displacement. r q
The work done by a torque during a finite rotation of the rigid body from r
w
initial value qi of the angle q to final value qf, can be obtained by integrating O
both the sides of the equation given
qf r r
Wi ®f = ò t o × dq
qi

Illustration 49. A thin light cord is wound around a uniform cylinder placed on a rough F
horizontal ground. When free end of the cord is pulled by a constant force F
the cylinder rolls. Denote radius of the cylinder by r and obtain expression for
work done by each of the forces acting on the cylinder when center of
the cylinder shifts by distance x.
Solution. Forces acting on the cylinder are its weight W, the normal reaction from the ground N, the tension
T in the cord and the force of static friction f . The tension in the cord equals to the applied force
s
F. These forces are shown in the adjacent figure.

mg F mg F
251\D\Allen(IIT-JEE Wing)\2020-21\Nurture\Phy\Unit-03\02-Rigid Body Dynamics

A A
C x C
P P
fs fs
N N

In rolling point of contact P is at instantaneous rest, the center C moves with velocity v C = w r and
the top point moves with velocity v A = 2v C = 2w r both parallel to the surface on which body rolls.
Since the cord is inextensible displacement of the top point equals to the displacement of the free
end of the cord. These fact suggests that during displacement x of the center the free end of the
cord shift through a distance 2x.
115
JEE-Physics
Work done by the weight of the cylinder
Wg = 0 The weigh is assumed to act on the center of gravity which coincides with the mass center in uniform
m
gravitation field near the ground. The displacement x of the mass center and weight both are perpendicular
to each other so the work done by gravity is zero.

Work done by the normal reaction on the cylinder


WN = 0 The normal reaction acts on the particle of the body which is in contact with the ground. The particles
making contact continuously change and remain at instantaneous rest during contact. Therefore normal
reaction does no work.

Work done by the force of static friction.


Wfs = 0 The force of static friction f acts on the particle of the body which is in contact with the ground. The
s
particles making contact continuously change and remain at instantaneous rest during contact. Therefore
force of static friction f does no work.
s
Work done by the tension in the cord
v A dt
WT = WF The particle of the wheel on which the tension in the cord acts is at the top A T =F
point. Though this particle is also continuously changing but it is not in instantaneous
rest and has velocity v . So in every infinitesimally small time interval displacement of
A
this particle is v A dt = 2v C dt , thus work done dW by the tension during a time interval dt
T

dWT = T (2v C dt ) = 2F (v C dt )
When the center shifts by a distance x the work done by the tension becomes
WT = WF = 2Fx

7.2 Potential Energy of a rigid body

Since potential energy of a system is function of its configuration and does not depend on the manner in
which the system is brought into a particular configuration, hence it does not depends on motion involved
whether it is translation, rotational or their combination.

7.3 Kinetic Energy of a rigid body in rotation motion

A rigid body can be represented as a system of large number of particles, which keep their mutual distances
unchanged in all circumstances. Kinetic energy of the whole body must be sum of kinetic energies of all of its
particles. In this section we develop expressions for kinetic energy of a rigid body.

Kinetic Energy of a rigid body in plane motion


y
In the figure shown a body is in plane motion. Its mass center at an instant
r
is moving with velocity v C and rotating with angular velocity wr . Both r
r
vC
w
these motions are shown superimposed in the given figure.
251\D\Allen(IIT-JEE Wing)\2020-21\Nurture\Phy\Unit-03\02-Rigid Body Dynamics

C
(
Kinetic energy too can be written as sum of kinetic energy 1
2
Mv C2 ) due to

translation motion of the mass center and kinetic energy ( I w 2 ) due to


x
1 O
2 C

centroidal rotation.
K = 12 Mv C2 + 12 I C w 2
If location of the instantaneous axis of rotation (IAR) is known, making use of the parallel axis theorem we can
write kinetic energy by the following equation also.
K = 12 I IAR w 2

116
Rigid Body Dynamics
Kinetic Energy of a rigid body in rotation about fixed axis not passing through the mass centre
In this kind of motion the mass center is in circular motion about the axis of
rotation. In the figure is shown a body rotation with angular velocity w about a
fixed axis through point P and perpendicular to plane of the paper. Mass center
P r r
moves with speed vC = wr . Kinetic energy of the body can now be expressed rC / P vC

by the following equation. w


C

K = 12 Mv + 12 I C w
2
C
2

Making use of the parallel axis theorem (I P = MrP2 / C + I C ) we can write kinetic energy by the following
equation also.
K = 12 I P w 2

Kinetic Energy of a rigid body in pure centroidal rotation


In pure centroidal rotation the mass center remain at rest; therefore kinetic energy due to translation of mass
center vanishes.
K = 12 I C w 2

Illustration 50. A rod of mass m and length l is pivoted to a fixed support at one w
of its ends O. It is rotating with constant angular velocity w. O
Write expression for its kinetic energy.
Solution If the point C is the mass center of the rod, from theorem of parallel axes, the moment of inertia
I of the rod about the fixed axis is 1
l
O 2

I O = I C + m (OC ) ® I O = I C + 14 ml2
2 O
w C

Substituting 1
12
ml for I , we have
2
C

I O = ml
1
3
2

Kinetic energy of the rod equals to kinetic energy due to rotation about the fixed axis.
K = 12 I o w 2 ® Using above expression for I , we have
O

K = 16 ml2w 2

Illustration 51. A uniform rigid body of mass m and round section of radius r is w
C
rolling on horizontal ground with angular velocity w. Its radius of
gyration about the centroidal axis is k.
(a) Write expression of its kinetic energy.
(b) Also express the kinetic energy as sum of kinetic energy due to translation of mass center and
kinetic energy due to simultaneous centroidal rotation.
Solution (a) The point of contact with ground of a body rolling on the ground is its ICR. Let the point P is
the ICR as shown in the adjacent figure. The geometrical center C of a uniform body and the
mass center coincide. Therefore moment of inertia I of the body about the ICR can be
251\D\Allen(IIT-JEE Wing)\2020-21\Nurture\Phy\Unit-03\02-Rigid Body Dynamics

P
written by using the theorem of parallel axes.
I P = IC + m ( PC ) ®
2
I P = IC + mr 2 v C = wr
C
Substituting I C = mk 2 , we have w

(
IP = m k 2 + r 2 ) (1)
P

Kinetic energy of a rigid body equals to kinetic energy due to rotation about the ICR.
K = 12 I P w 2 ® Substituting I from eq. (1), we have
P

K = m k +r w
1
2 ( 2 2
) 2

117
JEE-Physics
(b) Kinetic energy of the body also equals to sum of kinetic energy due to translation of its mass
center and kinetic energy due to simultaneous centroidal rotation.
K = 12 mv C2 + 12 I C w 2 ®Substituting condition for rolling v C = w r and I C = mk 2 , we have

( )
K = 12 m (wr ) + 12 mk 2w 2 = 12 m r 2 + k 2 w 2
2

Illustration 52. A thin meter scale is kept vertical by placing its one end on floor, keeping the end in contact stationary, it
is allowed to fall. Calculate the velocity of its upper end when it hit the floor .

Solution Loss in PE = gain in rotational l


2
CM
mgl 1 2 1 ml2 v 2
KE = = Iw = ´ 2 Þ v = 3gl
2 2 2 3 l

7.4 Power

Power defined as the time rate of work done, takes into account the duration in which work is done. To
calculate power we make use of the following equation.
dW
P =
dt
Instantaneous power of a force can be expressed by the following equation. Here velocity vr is the velocity of
r
the point of application of the force F .
r r
P = F ×v

7.5 Work and Energy Theorem

Work energy theorem can be applied in similar fashion as it was applied to analyze translation motion of a
single body or a system of several bodies.
The work energy theorem relates kinetic energy K1 and K2 of a body in its initial and final position with work
W1’!2 done by all the external forces acting on the body to carry it form the initial position to the final position
according to the following equation.
W1®2 = K 2 - K 1
This equation is applicable in all inertial as well as noninertial frames. To write equation of work energy
theorem kinetic energy must be written relative to the frame under consideration. To calculate work consider
only all the physical forces in inertial frame and all the physical forces as well as pseudo force in non-inertial
frame and displacement of point of applications of these forces relative to the frame under consideration.
For a system of several bodies the corresponding equation of work energy theorem can be obtained by
applying the theorem for each individual body and then adding all of them. In this way we obtain an equation
of the following form.
251\D\Allen(IIT-JEE Wing)\2020-21\Nurture\Phy\Unit-03\02-Rigid Body Dynamics

SW1®2 = K 2,s - K1,s

Here the term SW1®2 equals to the work of all the forces acting on various bodies irrespective of whether the
force are internal or external from point of view of the system under consideration. In systems of several
bodies interconnected by links of constant length e.g. inextensible cords, rods etc or body in direct contact the
total work of internal forces vanishes. The work done by internal conservative forces can be accounted for by
decrease in corresponding potential energies. The terms K 2,s and K 1,s are total kinetic energies of all the
bodies in initial and final configurations of the system.

118
Rigid Body Dynamics
7.6 Conservation of Mechanical Energy

The work of conservative forces equals to decrease in potential energy. When a single rigid body moves or a
system of rigid body changes its configuration under the action of conservative forces and nonconservative
forces are either not present or if present do no work, the work energy principle can be expressed as
U 1,s + K 1,s = U 2,s + K 2,s
The above equation expresses the law of conservation of mechanical energy and states that if a rigid body
moves or a system consisting of several rigid bodies changes its configuration under action of conservative
forces the mechanical energy i.e. sum of kinetic and potential energy remain constant; provided that
nonconservative, if present, do no work,
Though the work energy principle and the law of conservation of mechanical energy are equivalent, we
prefer to use the former to account for nonconservative forces easily

Illustration 53. A uniform rod AB of mass m and length l is pivoted at a point (O) to
rotate in the vertical plane. The rod is held in horizontal position and A O C B
released. Find the distance x of the pivot from the mass center (C) of
x
the rod, so that angular speed w of the rod as it passes through
the vertical position is maximum.
Solution. The problem involves change in angular velocity with change in A Initial
position, therefore demands application of principle of work and OO Position
energy.
The rod when released rotates about a fixed horizontal axis x

passing through the point O. Its initial and final positions are C
shown in the adjacent figure.
Moment of inertia of the rod about the pivot O can be calculated Initial
by theorem of parallel axes. Position B

I o = I C + mx 2 ® Substituting 1
12
ml2 for I , we have
C

Io = 1
12 (
m l + 12x 2 2
) (1)
Kinetic energy in the initial position.
K = 12 I o w 2 ® K 1 = 0 (2)
Kinetic energy in the final position.
K = 12 I o w 2 ® Substituting for I form eq. (1), we have
o

K2 = 1
24 (
m l2 + 12x 2 w 2 ) (3)
Only gravity does work when the rod moves from the initial to final position.
r r
W = ò F × dr ® W1®2 = mgx (4)
Substituting values form eq. (2), (3) and (4) in equation of work energy principle, we have
251\D\Allen(IIT-JEE Wing)\2020-21\Nurture\Phy\Unit-03\02-Rigid Body Dynamics

W1®2 = K 2 - K 1 ® w = 24 gx l2 + 12x 2 ( ) (5)


The above equation expresses angular velocity of the rod when it passes the vertical position. For
it to be maximum

dw l
= 0® x =
dx 12

119
JEE-Physics
Illustration 54. A uniform rigid body of mass m and round section of radius r rolls
down a slope inclined at an angle q to the horizontal. The radius of
gyration of the body about it central axis of symmetry is k.
(a) Derive suitable expressions for angular velocity and velocity
of its mass center after it covers a distance x.
(b) Obtain expression for its angular acceleration and acceleration
of the mass center.
Solution The problem involves change in angular velocity with change in position, therefore demands
application of principle of work and energy
The geometrical center and mass center for uniform bodies coincide; therefore center C is the
mass center.
In rolling the point of contact P must always be at instantaneous rest and angular velocity w,
velocity of center C, angular acceleration a and acceleration of the center must bear the following
relations.
v C = w r and aC = a r (1)
(a) The rolling motion can be analyzed as superposition of translation of the mass center and
simultaneous centroidal rotation.
Kinetic energy in the initial position.
K1 = 0 (2)
Kinetic energy in the final position.
K = 12 mv C2 + 12 I C w 2 ® Substituting for I C = mk 2 and v form eq. (1), we have
C

(
K 2 = 12 m r 2 + k 2 w 2 ) (3)
The forces acting on the body are its weight mg, the normal reaction N from the slope and the force
of static friction f . These forces and displacement are shown in the adjacent figure. The normal
s
reaction and the force of static friction do no work in rolling, it is the weight, which does work.
r r
W = ò F × dr ® W1®2 = mgx sin q (4)
Substituting values form eq. (2), (3) and (4) in equation of mg sin q
work energy principle, we have E
mg cos q
q C
2gx sin q
W1®2 = K 2 - K 1 ® w=
k2 + r 2 x fs
Substituting v from eq. (1), we have
C N

2gx sin q q
vC =
(k 2
r 2
) +1
Acceleration a, velocity v and position coordinate x bear the relation a = v ( dv dx ) . Thereforee
acceleration of mass center of the body.
251\D\Allen(IIT-JEE Wing)\2020-21\Nurture\Phy\Unit-03\02-Rigid Body Dynamics

dv C g sin q
aC =
aC = v C
dx
® ( )
k r2 +1
2

Substituting a in eq. (1), we have the angular acceleration of the body.


C

g sin q
a=
aC = a r ® ( )
k2 r + r

120
Rigid Body Dynamics

Energy method in C.T.R.M.


1. A solid cylinder rolls down on inclined plane which has friction sufficient to prevent sliding. The ratio of rotational
energy to total kinetic energy is
(A) 1/2 (B) 1/3 (C) 2/3 (D) 3/4

2. A uniform rod is hinged as shown and is released from a hori-


zontal position. The angular velocity of the bar as it passes
the vertical position is:

12g 2g 24g 3g
(A) (B) (C) (D)
3l 3l 7l 7l

3. A plank P is placed on a solid cylinder S, which rolls on a horizontal surface.


The two are of equal mass. There is no slipping at any of the surfaces in
contact. The ratio of the kinetic energy of P to the kinetic energy of S is:
(A) 1: 1 (B) 2: 1
(C) 8: 3 (D) 1: 4

4. A uniform cylinder of mass M and radius R rolls without slipping down a slope
of angle q to the horizontal. The cylinder is connected to a spring constant K
while the other end of the spring is connected to a rigid support at P. The cylinder
is released when the spring is unstretched. The maximum distance that the
cylinder
travels is

3 Mg sin q Mg tan q 2Mg sin q 4 Mg sin q


(A) (B) (C) (D)
4 K K K 3 K

5. In the figure shown a ball rolls without sliding. On a horizontal


surface. It ascends a curved track upto height h and returns.
Value of h is h1 for sufficiently rough curved track to avoid
sliding and h 2 for smooth curved track, then:
(A) h1 = h 2 (B) h1 < h 2
(C) h1 > h 2 (D) h2 = 2 h1

6. A solid cylinder (initially at rest) of mass 'M' and radius 'R' rolls down an inclined
plane without slipping and reaches the bottom as shown in the figure. Its speed at
the instant it reaches the bottom is (h >> R)

10 3
(A) gh (B) gh
7 4
251\D\Allen(IIT-JEE Wing)\2020-21\Nurture\Phy\Unit-03\02-Rigid Body Dynamics

2 4
(C) gh (D) gh
3 3

7. A small solid sphere of mass m is released from a point A at a height h


above the bottom of a rough track as shown in the figure. If the sphere rolls
down the track without slipping, its rotational kinetic energy when it comes
to the bottom of track is

10 5 2
(A) mgh (B) mgh (C) mgh (D) mgh
7 7 7

121
JEE-Physics
8. A thin rod of length ‘L’ rotates in a horizontal plane about a vertical axis passing through its one end with
angular velcoity w. If A is cross-sectional area and r be density of the rod then rotational kinetic energy is–

1 3 2 1 3 2 1 1
(A) AL rw (B) AL rw (C) AL3rw 2 (D) AL3rw 2
3 6 24 18

9. Torque 1.2 Nm acts on an object which is rotating. Instantaneous rotational power of the torque at an
instant when the object is rotating with angular velocity 60 rpm is equal to–
(A) 2.4 p watt (B) 2.4 watt (C) 1.2 watt (D) 72 watt

10. A uniform solid sphere is in pure rolling motion over a horizontal surface. Ratio of translatial kinetic energy
to total kinetic energy is equal to–
2 5 2 1
(A) (B) (C) (D)
7 7 5 2

8.0 M E T H O D S OF IMPULSE AND MOMENTUM

Methods of impulse and momentum describe what happens over a time interval. When motion of a body
involves rotation we have to consider angular impulse as well as angular momentum. In this section we
discuss concept of angular impulse, angular momentum of rigid body, angular impulse momentum principle
and conservation of angular momentum.

8.1 Angular Impulse

Like impulse of a force angular impulse of a constant torque equals to product of the torque and concerned
time interval and if the torque is not constant it must be integrated with time over the concerned time interval.
r
If torque t o about an axis passing through O is constant, its angular impulse during a time interval from t 1 to
r
t2 denoted by J o,1®2 is given by the following equation.
r r
J o ,1®2 = t o (t 2 - t1 )
r
If torque t o about an axis passing through O is time varying, its angular impulse during a time interval from
r
t1 to t2 denoted by J o,1®2 is given by the following equation.
r t2 r
J o ,1®2 = ò t o dt
t1

8.2 Angular momentum of a particle


y
r
Angular momentum Lo about the origin O of a particle of mass m moving v
251\D\Allen(IIT-JEE Wing)\2020-21\Nurture\Phy\Unit-03\02-Rigid Body Dynamics

r r
with velocity vr is defined as the moment of its linear momentum p = mv q
r
about the point O. r P
r r r
Lo = r ´ ( mv )
O x

Q
8.3 Angular Momentum of a Rigid Body

Angular momentum is quantity of rotation motion in a body. The angular momentum of a system of particles
is the sum of angular momentum all the particles within the system. A rigid body is an assemblage of large

122
Rigid Body Dynamics
number of particles maintaining their mutual distances intact under all circumstances, therefore angular
momentum of a rigid body must be sum of angular momenta of all of its particles.

Angular Momentum about a point and about an axis


Angular momentum of a particle is not defined about an axis instead it is defined about a point. Therefore
above idea of summing up angular momenta of all the particles about a point gives angular momentum of the
rigid body about a point. But while dealing with fixed axis rotation or rotation about axis in translation we
need angular momentum about an axis.
Angular momentum about an axis is calculated similar to torque about an axis. To calculate angular momentum
of a particle of rigid body about an axis we take moment of momentum of the particle about the point where
plane of motion of the point of application of the force intersects the axis.
r r r
In the following figure is shown angular momentum dLz = r ´ ( dmv ) = r 2dmw of a particle P of a rigid body
rotating about the z-axis. It is along the z-axis i.e. axis of rotation. In the next figure total angular momentum
r r r
L z = ò dLz = I z w about the axis of rotation is shown. It is also along the axis of rotation.
z
z r r
r Lz = I Zw
w

r
dLz
r y
r
C
dmv = dmwr
P
x
r
rP O y
O y
x
x

Angular Momentum in general plane motion


y
Angular momentum of a body in plane motion can also be written similar
r
to torque equation or kinetic energy as sum of angular momentum about vC
w
the axis due to translation of mass center and angular momentum of
C
centroidal rotation about centroidal axis parallel to the original axis.
Consider a rigid body of mass M in plane motion. At the instant shown its r
mass center has velocity vr and it is rotating with angular velocity wr about
rC
O x
r
an axis perpendicular to the plane of the figure. It angular momentum Lo
about an axis passing though the origin and parallel to the original is
expressed by the following equation.
r r r r
Lo = rC ´ ( Mv C ) + I C w
The first term of the above equation represent angular momentum due to translation of the mass center and
the second term represents angular momentum in centroidal rotation.
251\D\Allen(IIT-JEE Wing)\2020-21\Nurture\Phy\Unit-03\02-Rigid Body Dynamics

Angular momentum in rotation about fixed axis y


Consider a body of mass M rotating with angular velocity w about a
fixed axis perpendicular to plane of the figure passing through point P r r
rC / P vC
P. Making use of the parallel axis theorem I P = MrC2 / P + I C and w
r r r r C
equation vC = w ´ rC / P we can express the angular momentum LP of
the body about the fixed rotational axis.
r r O x
LP = I P w

123
JEE-Physics
The above equation reveals that the angular momentum of a rigid body in plane motion can also be expressed
in a single term due to rotation about the instantaneous axis of rotation.

Angular momentum in pure centroidal rotation


In pure centroidal rotation, mass center remains at rest, therefore angular
momentum due to translation of the mass center vanishes. w
r r C
LC = I C w

Rotational Equivalent of the Newton’s Laws of Motion


r r r r
Differentiating terms on both the sides of equation Lo = rC ´ ( Mv C ) + I C w with respect to time, and making
r r r r
substitution of v C = drC dt , aC = dv C dt and ar = dwr dt we have
r
dLo r r r r r
= v C ´ ( Mv C ) + rC ´ MaC + I C a
dt
The first term on the right hand side vanishes, so we can write
r
dLo r r r
= rC ´ MaC + I C a
dt
r r r r
Now comparing the above equation with torque equation St o = rC ´ MaC + I C a , we have
r
r dLo
å t o = dt
The above equation though developed for plane motion only yet is valid for rotation about an axis in rotation
also. It states that the net torque about the origin of an inertial frame equals to the time rate of change in
angular momentum about the origin and can be treated as a parallel to Newton’s second law which states that
net external force on a body equals to time rate of change in its linear momentum.

Angular Impulse Momentum Principle


Rearranging the terms and integrating both the sides obtained form previous equation, we can write
t2 r r r
S ò t o dt = Lo 2 - Lo1
t1

The left hand side of the above equation is the angular impulse of torque of all the external forces in the time
interval in the time interval t 1 to t2.
r r r
SJ o ,1®2 = Lo 2 - Lo1

The idea expressed by the above equation is known as angular impulse momentum principle and states that
increment in the angular momentum of a body about a point in a time interval equals to the net angular
impulse of all the external forces acting on it during the concerned time interval.
For the ease of application the above equation is rearranged as
r r r
Lo1 + å J o ,1®2 = Lo 2
251\D\Allen(IIT-JEE Wing)\2020-21\Nurture\Phy\Unit-03\02-Rigid Body Dynamics

Like linear impulse momentum principle, the angular impulse momentum principle provides us solution of
problems concerned with change in angular velocity in a time interval or change in angular velocity during
very short interval interactions.

Method of Impulse Momentum Principle for Plane motion of a Rigid Body


Linear momentum and angular momentum serve as measures of amount of translation and rotation motion
respectively. The external forces acting on a rigid body can change its state of translation as well as rotation
motion which is reflected by change in linear as well as angular momentum according to the principles of
linear impulse and momentum and angular impulse and momentum.

124
Rigid Body Dynamics
r
ò F dt
1
r
r r ò F dt
i
r
I C w1 Mv C 1 IC w2 r
Mv C 2
C C
r
ò F2dt r
ò Fndt
Linear and angular momenta Impulse of all the forces during Linear and angular momenta
at the instant t1 time interval t1 to t2 at the instant t2

In the above figure is shown strategy to apply method of impulse and momentum. Consider a rigid body of
mass M in plane motion. Its moment of inertial about the centroidal axis perpendicular to plane of motion is
r r
IC. Let v C 1 and w1 represent velocity of its mass center and its angular velocity at the beginning of a time
r r r r
interval t1 to t2. Under the action of several forces F1 , F2 ……. Fi ….. Fn during the time interval its mass
r r
center velocity and angular velocity become v C 2 and w2 respectively. The adjacent figure shows strategy
representing how to write equations for linear and angular impulse momentum principles.
While applying the principle it becomes simpler to consider translation of the mass center and centroidal
rotation separately. Thus in an alternative way we apply linear impulse momentum principle for translation of
the mass center and angular impulse momentum principle for centroidal rotation.
Translation of mass center:Linear impulse momentum principle.
r r r
p1 + å I mp1®2 =p2
r r r r
Here p1 = Mv C 1 and p2 = Mv C 2 represent linear momentums at the beginning and end of the time interval
r
and å I mp1®2 stands for impulse of all the external forces during the time interval.

Centroidal rotation: Angular impulse momentum principle.


r r r
LC 1 + å J C ,1®2 = LC 2
r r r r
Here LC 1 = I C w1 and LC 2 = I C w 2 represent angular momentums about the centroidal axis at the beginning
r
and end of the time interval and å J C ,1®2 stands for angular impulse of all the external forces about the
centroidal axis during the time interval.

Illustration 55. A uniform disc of mass M and radius R rotating with angular velocity
w about a vertical axis passing through its center and perpendicular
o
to its plane is placed gently on a rough horizontal ground, where
coefficient of friction is m. How long it will take to stop.
251\D\Allen(IIT-JEE Wing)\2020-21\Nurture\Phy\Unit-03\02-Rigid Body Dynamics

Solution The torque of friction forces is

t C = 23 mMgR (1)
The angular impulse of the torque of friction is responsible to stop the disc. Applying angular
impulse momentum principle, we have
r r r
LC 1 + å J C ,1®2 = LC 2 ® I C wo - t C t = 0

3Rwo
Substituting I C = 12 MR 2 and tC from eq. (1), we have t =
4m g

125
JEE-Physics
Illustration 56. A body of radius R and mass m is placed on horizontal rough
surface with linear velocity v0 , after some time it comes in w
the condition of pure rolling then determine : m v0
v
(i) Time t at which body starts pure rolling. v= wR
m
(ii) Linear velocity of body at time t.
(iii) Work done by frictional force in this time t.
Solution For translatory motion v = u + at
Initial velocity u = v0
Let after time t pure rolling starts and at this time t final velocity = v and acceleration = a
From FBD :
FBD
Normal Reaction N = mg N
Friction force f = m N = m mg Þ ma = m mg [ Q f = ma]
Retardation a=mg
f= m N
v = v0 – at (–ve sign for retardation)
mg
v = v0 – m gt ...(i)
For rotatory motion w = w0 + a t (Initial angular velocity w0 = 0)
Þ w=at ...(ii)
t fR mmgR mgR
Q t = Ia Þ a = = 2 = 2
a = 2 ...(iii)
I mK mK K
mgR
From eqn. (ii) and eqn. (iii) w = t ...(iv)
K2
v
Q For pure rolling v = wR Þ w = ...(v)
R
v mgR mgR 2 t
From eq. (iv) and (v) = 2 t or v = ...(vi)
R K K2
mgR 2 t v0
substitute v from eqn. (vi) into eqn.(i) = v 0 - mgt Þ t =
K2 é R2 ù
mg ê1 + 2 ú
ë K û

v0 v0 v0
Putting the value of t in equation (i) v = v 0 - mg = v0 - =
é R2 ù R2 K2
mg ê1 + 2 ú 1+ 2 1+ 2
ë K û K R

Work done in sliding by frictional force = Initial kinetic energy – Final kinetic energy
2
1 1 æ K2 ö 1 1 Mv 20 Mv 0
2
Work done by friction Wf = Mv 02 - Mv 2 ç 1 + 2 ÷ = Mv 0 - = æ
R2 ö
2 2 è R ø 2 2æ K ö
2
2 1 +
çè 1 + R2 ÷ø çè K 2 ÷ø

8.4 Conservation of Angular Momentum


251\D\Allen(IIT-JEE Wing)\2020-21\Nurture\Phy\Unit-03\02-Rigid Body Dynamics

If angular impulse of all the external forces about an axis in time interval vanishes, the angular momentum of
the system about the same axis in that time interval remain unchanged.
t2 r r r
If S òt t o dt = 0 , we have Lo1 = Lo 2
1

The condition of zero net angular impulse required for conservation of angular momentum can be fulfilled in
the following cases.
l If no external force acts, the angular impulse about all axes will be zero and hence angular momentum
remains conserved about all axes.
l If net torque of all the external forces or torques of each individual force about an axis vanishes the
angular momentum about that axes will be conserved.
126
Rigid Body Dynamics
l If all the external forces are finite in magnitude and the concerned time interval is infinitely small, the
angular momentum remain conserved.
l If a system of rigid bodies changes its moment of inertia by changing its configuration due to internal
forces only its angular momentum about any axes remains conserved. If we denote the moment of
inertias in two configurations by I 1 and I2 and angular velocities by w1 and w2, we can write
r r
I 1w1 = I 2w2

The principle of conservation of angular momentum governs a wide range of physical processes from subatomic
to celestial world. The following Illustration s explicate some of these applications.

Spinning Ice Skater


A spinning ice skater and ballet dancers can control her moment of inertia by spreading
or bringing closer her hands and make use of conservation of angular momentum to
perform their spins. In doing so no external forces is needed and if we ignore effects of
friction from the ground and the air, the angular momentum can be assumed conserved.
When she spreads her hand or leg away, her moment of inertia decreases therefore her
angular velocity decreases and when she brings her hands or leg closer her moment of
inertia increases therefore her angular velocity increases.

Student on rotating turntable


The student, the turntable and dumbbells make an isolated
system on which no external torque acts, if we ignore friction
in the bearing of the turntable and air friction. Initially the
student has his arm stretched on rotating turntable. When he
pulls dumbbells close to his body, angular velocity increases
due to conservation of angular momentum. Larger moment of Smaller moment of
inertia and smaller inertia and larger
angular velocity angular velocity

Illustration 57. Consider the disc A of moment of inertia I1 rotating freely in


horizontal plane about its axis of symmetry with angular velocity
wo. Another disc B of moment of inertia I 2 held at rest above the B
disc A. The axis of symmetry of the disc B coincides with that of wo

the disc A as shown in the figure. The disc B is released to land on A


the disc A. When sliding stops, what will be the angular velocity
of both the discs?
Solution Both the discs are symmetric about the axis of rotation therefore does not require any external
torque to keep the axis stationary. When the disc B lands on A slipping starts. The force of friction
provides an internal torque to system of both the disc. It slows down rotation rate of A and
increases that of B till both acquire same angular velocity w.
251\D\Allen(IIT-JEE Wing)\2020-21\Nurture\Phy\Unit-03\02-Rigid Body Dynamics

Since there is no external torques on the system of both the discs about the axis of rotation, the
total angular momentum of the system remains conserved. The total angular momentum of the
system is the sum of angular momentum of both discs. Denoting the angular momentum of the
r r r
disc A before B lands on it and long after slipping between them stops by symbols LA1 , LB1 , LA 2
r
and LB2 respectively, we can express conservation of angular momentum by the following equation.
r r r r I1wo
LA1 + LB1 = LA 2 + LB 2 ® I1wo + 0 = I1w + I 2w Þ w =
I1 + I 2

127
JEE-Physics
Illustration 58. A cube of mass m and edge length l can slide freely on a
smooth horizontal floor. Moving on the floor with velocity
vo, it strikes a long obstruction PP of small height. The vo
P
obstruction is parallel to the leading bottom edge of the cube.
The leading bottom edge gets pivoted with the obstruction
P
and the cube starts rotating. Determine angular velocity of
the cube immediately after the impact.
Solution Before the impact, there is no external force in the horizontal direction and the cube slides with
uniform velocity vo, and during the impact reaction forces of the obstruction stops its leading
bottom edge and cause it to rotate about its leading bottom.
During the impact external forces acting on the cube are its weight, the normal reaction from the
ground and reaction from the obstruction. The weight and the normal reaction from the ground
both are finite in magnitude and the impact ends in infinitesimally small time interval so their
impulses and angular impulses about any axes are negligible. It is the reaction from the obstruction
which has finite impulse during the impact. Its horizontal component changes the momentum of
the cube during the impact, but its angular impulse about the obstruction is zero, therefore the
angular momentum of the cube about an axis coincident with the leading bottom edge remain
conserved. v o

vo
C C
P w P

Immediately before the Immediately after the


impact. impact.
Let the velocity of the mass center and angular velocity of the cube immediately after the impact
are v and w . These velocities are shown in the adjacent figure.
Co o
We denote the angular momentum of the cube about axis coincident with the obstruction edge
before and after the impact by L and L .
P1 P2
Applying principle of conservation of angular momentum about an axis coincident with the
obstruction, we have
r r r r r
LP 1 = LP 2 ® rC / P ´ mvo = I P w
Using theorem of parallel axes for moment of inertia I about the leading bottom edge, we get
P
I P = IC + mrC2 / P = 23 ml2 . Substituting this in the above equation, we have

3vo
Angular velocity immediately after the impact wo =
4l

Angular momentum of a body in combined translational and rotational motion


Suppose a body is rotating about an axis passing through its centre of mass with an angular velocity w cm
and moving translationally with a linear velocity v. Then, the angular momentum of the body about a
r r r r
point P outside the body in the lab frame is given by, L P = L cm + r ´ pcm where r is the position vector
r r r
of the centre of mass with respect to point P. Hence, L P = Icm + r ´ mv cm
251\D\Allen(IIT-JEE Wing)\2020-21\Nurture\Phy\Unit-03\02-Rigid Body Dynamics

Illustration 59. A solid sphere rolls without slipping on a rough surface and the centre of mass has constant
speed v . If mass of the sphere is m and its radius is R, then find the angular momentum of the
0
sphere about the point of contact.
r r r r r r r w
Solution Q L P = L cm + r ´ pcm = Icmw + R ´ mv cm
v0
Since sphere is in pure rolling motion hence
r æ2 2 v0 ö
w = v 0R Þ L p = ç MR
è5 R ÷ø ( -k̂ ) +Mv R= 75 Mv R ( -k̂ )
0 0

128
Rigid Body Dynamics
8.5 Eccentric Impact

In eccentric impact the line of impact which is the common normal drawn at the point of impact does not
passes through mass center of at least one of the colliding bodies. It involves change in state of rotation
motion of either or both the bodies.
Consider impact of two A and B such that the mass center C B of B does
CB Line of
not lie on the line of impact as shown in figure. If we assume bodies to be
Impact
frictionless their mutual forces must act along the line of impact. The reaction
force of A on B does not passes through the mass center of B as a result A B
state of rotation motion of B changes during the impact. CA

Problems of Eccentric Impact


Problems of eccentric impact can be divided into two categories. In one category both the bodies under going
eccentric impact are free to move. No external force act on either of them. There mutual forces are responsible
for change in their momentum and angular momentum. In another category either or both of the bodies are
hinged.

Eccentric Impact of bodies free to move


Since no external force acts on the two body system, we can use principle of conservation of linear momentum,
principle of conservation of angular momentum about any point and concept of coefficient of restitution.
The coefficient of restitution is defined for components of velocities of points of contacts of the bodies along
the line of impact.
While applying principle of conservation of angular momentum care must be taken in selecting the point
about which we write the equation. The point about which we write angular momentum must be at rest
relative to the selected inertial reference frame and as far as possible its location should be selected on line of
velocity of the mass center in order to make zero the first term involving moment of momentum of mass
center.

Eccentric Impact of hinged bodies


When either or both of the bodies are hinged the reaction of the hinge during the impact act as external force
on the two body system, therefore linear momentum no longer remain conserved and we cannot apply
principle of conservation of linear momentum. When both the bodies are hinged we cannot also apply
conservation of angular momentum, and we have to use impulse momentum principle on both the bodies
separately in addition to making use of coefficient of restitution. But when one of the bodies is hinged and
other one is free to move, we can apply conservation of angular momentum about the hinge.

Illustration 60. A uniform rod of mass M and length l is suspended from a fixed
support and can rotate freely in the vertical plane. A small ball of O
mass m moving horizontally with velocity vo strikes elastically the
lower end of the rod as shown in the figure. Find the
angular velocity of the rod and velocity of the ball immediately
after the impact. vo
Solution. The rod is hinged and the ball is free to move. External forces
251\D\Allen(IIT-JEE Wing)\2020-21\Nurture\Phy\Unit-03\02-Rigid Body Dynamics

acting on the rod ball system are their weights and reaction from
the hinge. Weight of the ball as well as the rod are finite and O w' O

contribute negligible impulse during the impact, but impulse of


reaction of the hinge during impact is considerable and cannot
be neglected. Obviously linear momentum of the system is not
vo v'
conserved. The angular impulse of the reaction of hinge about
the hinge is zero. Therefore angular momentum of the system Before the impact Immediately after
about the hinge is conserved. Let velocity of the ball after the the impact

impact becomes v'B and angular velocity of the rod becomes


w'.

129
JEE-Physics
We denote angular momentum of the ball and the rod about the hinge before the impact by L
B1
and L and after the impact by L and L .
R1 B2 R2
Applying conservation of angular momentum about the hinge, we have
r r r r
L B 1 + L R1 = L B 2 + L R 2 ® mv o l + 0 = mv B¢ l + I o w ¢

Substituting 1
3 M l2 for Io, we have 3mv B¢ + M lw ¢ = 3mv o ....(1)
The velocity of the lower end of the rod before the impact was zero and immediately after the
impact it becomes lw' towards right. Employing these facts we can express the coefficient of
restitution according to eq.

v Qn
¢ - v Pn
¢
e= lw ¢ - v B¢ = ev o
v pn - v Qn ® ....(2)

From eq. (1) and (2), we have

Velocity of the ball immediately after the impact v B¢ =


(3m - eM ) v o
3m + M

3 (1 + e ) mv o
Angular velocity of the rod immediately after the impact w¢ =
(3m + M ) l

Illustration 61. A uniform rod AB of mass M and length l is kept at rest on a smooth vo
A
horizontal plane. A particle P of mass mo moving perpendicular to the
rod with velocity vo strikes the rod at one of its ends as shown in the
figure. Derive suitable expressions for the coefficient of restitution,
velocity of mass center of the rod and angular velocity of the rod
immediately after the impact. Assume e is the coefficient of restitution. B
Solution. Both the bodies can move freely in the horizontal plane, therefore no
horizontal external force acts on the particle-rod system. The linear
v P' A
momentum as well as angular momentum about any axis normal to the
plane is conserved.
Let the velocity of the particle, angular velocity of the rod and velocity of the v C'
mass center of the rod immediately after the impact are v' P towards right, w' w'
C
in clockwise sense and v'C towards right as shown in the adjacent figure.
Using relative motion equation, we can express the velocity of the end A of B
the rod.
v A¢ = v C¢ + 12 w ¢l (1)
r r
We denote linear momentum of the particle and rod before the impact by pP 1 and pC 1 and
r r
immediately after the impact by pP 2 and pC 2 respectively..
Applying conservation of linear momentum, we have
251\D\Allen(IIT-JEE Wing)\2020-21\Nurture\Phy\Unit-03\02-Rigid Body Dynamics

r r r r
pP 2 + pC 2 = pP 1 + pC 1 ® mv P¢ + Mv C¢ = mv o (2)
The above equation shows that the mass center of the rod will move toward the right. If we write
angular momentum of the rod about a stationary point O, which is in line with the velocity v'C, the
first term involving moment of momentum of rod vanishes and only angular momentum due to
its centroidal rotation remains in the expression.
We denote angular momentum of the particle and the rod about the point O before the impact by
L and L and after the impact by L and L .
P1 R1 P2 R2
Applying conservation of angular momentum about the hinge,

130
Rigid Body Dynamics
we have A v P' A
r r r r vo
LP 2 + LR 2 = LP 1 + LR 1 ® 1
2
mv P¢ l + I C w ¢ = mv o l
1
2

Substituting 1
M l2 for IC, we have w' v C'
12
C
6mv P¢ + M lw ¢ = 6mv o (3)
The velocity of the end A of the rod before the impact was zero B B
Before the impact Immediately after
and immediately after the impact it becomes v towards right. '
A the impact
Employing these facts we can express the coefficient of restitution as
' '
v Qn - v Pn
e=
v pn - v Qn ® v A¢ - v P¢ = ev o

Substituting v'A form eq. (1), we have


2v C¢ + w ¢l - 2v P¢ = 2ev o (4)
Eq. (2), (3) and (4) involves three unknowns v'C, w' and v'P, which can be obtained by solving
these equation.
æ 4m - eM ö
Velocity of the ball immediately after the impact v P¢ = ç v
è 4m + M ÷ø o

ïì m (1 + e ) ïü
Velocity of mass center of rod immediately after the impact v C¢ = í ýv o
ïî 4m + M ïþ
Angular velocity of the rod immediately after the impact
ìï 6m (1 + e ) üï v o
w¢ = í ý
îï 4m + M ïþ l

Angular Momentum
1. A particle of mass m is projected with speed u at an angle q with the horizontal. Find angular momentum of
particle about point of projection when particle again at same horizontal level :
mu3 sin q × sin 2q mu 3 cos q × sin 2q mu3 sin q × sin 2q mu3 sin q × cos 2q
(A) (B) (C) (D)
g g 2g g

2. A particle of mass 1 kg is projected at an angle q with horizontal. Its co-ordinates at any instant are
(5m, 5m) and it is having velocity components along X-axis and Y-axis as 8 m/s and 4 m/s respectively. Its
angular momentum about origin is
(A) – 20 N-m k̂ (B) + 20 N-m k̂ (C) – 60 N-m k̂ (D) + 60 N-m k̂

3. A constant torque acting on a uniform circular wheel changes its angular momentum from A0 to 4A0 in 4
seconds. The magnitude of this torque is-
251\D\Allen(IIT-JEE Wing)\2020-21\Nurture\Phy\Unit-03\02-Rigid Body Dynamics

3A 0 4A 0
(A) (B) A0 (C) (D) 12A0
4 3

4. In the given figure a ball strikes a rod elastically and rod is hinged smoothly at
point A. Then which of the statement(s) is/are correct for the collision?
(A) linear momentum of system (ball + rod) is conserved
(B) angular momentum of system about hinged point A is conserved
(C) kinetic energy of system is conserved
(D) linear momentum of ball is conserved.

131
JEE-Physics
5. A uniform hollow sphere is resting on a smooth plane. A horizontal impulse I
is applied at a height ' h ' above its centre. Immediately after the impulse is
removed it starts pure rolling. Then ' h ' is equal to:
(A) R (B) (1/3) R
(C) (2/3) R (D) none of these

6. A playground merry-go-round is at rest, pivoted about a frictionless axis. A child of mass m runs along a path
tangential to the rim with speed v and jumps on to the merry-go-round. If R is the radius of the merry-go-round
and I is its moment of inertia, then the angular velocity of the merry-go-round and the child is -
mvR mvR mR 2 + I I
(A) 2 (B) (C) (D)
mR + I I mvR mvR

7. A disc of mass ‘m’ and radius R is free to rotate in horizontal plane about a
vertical smooth fixed axis passing through its centre. There is a smooth groove
along the diameter of the disc and two small balls of mass m/2 each are placed
in it on either side of the centre of the disc as shown in fig. The disc is given
initial angular velocity w0 and released. The angular speed of the disc
when the balls reach the end of disc is :
w0 w0 2w0 w0
(A) (B) (C) (D)
2 3 3 4

8. A thin circular disk of mass M and radius R rotating about its axis with a constant angular speed w. Four
small blocks, each of mass m are attached gently to opposite ends of two diameters of the disc. The angular
speed of the disc will be–

4Mw æ M - 2m ö Mw (M + 2m)w
(A) (B) çè ÷w (C) (D)
M + 2m M + 2m ø M + 8m M

9. A disc of mass m and radius r is rolling with angular speed w on a horizontal surface. The magnitude of
angular momentum about the origin O is–

w
m r

O x

3 2 1 2
(A) mr .w (B) mr .w (C) m.r2w (D) 2mr2.w
2 2

10. A ring (m, r) is rotating about it’s axis with angular momentum L. It’s rotational kinetic energy will be–
251\D\Allen(IIT-JEE Wing)\2020-21\Nurture\Phy\Unit-03\02-Rigid Body Dynamics

L L2 L2
(A) (B) (C) (D) 2mL2 r 2
2mr 2 2mr 2 mr 2

132
Rigid Body Dynamics

l If mass distribution is similar for two bodies about an axis, expressions of their moment of inertia must be of
the same form about that axis.
l If the whole body or any of its portions is shifted parallel to the axis of rotation keeping perpendicular distance
from axis of rotation same, moment of inertia remains unchanged.

® ®
l Total momentum of body is given by P = M V cm (where M = Total mass of body)

® ®
l F net = M a cm

l If body rotating about an axis passing through centre of mass, then net force on the body is zero. In this
situation it is possible that angular velocity of body is variable.

l If body rotating about an axis which is not passing through centre of mass, then net force on the body is
not equal to zero in this situation it is possible that angular velocity of body is constant.

l In case of pure rolling on horizontal surface distance travelled by particle at the circumference of sphere in one
complete rotation is 8 R and path of particle is known as cycloid.

l In case of pure rolling (in the presence of gravitational and contact force with inclined) on inclined plane the
direction of frictional force is upward along the inclined plane whether body is moving up the inclined or
down the inclined.

l In case of pure rolling on fixed surface acceleration of point of contact of body with surface, is not equal to
zero.

F
l For shown situation (A) & (B), more chances of toppling in (A).
In case of toppling, normal reaction must passes through A F
end points. B

l If block kept on smooth horizontal surface and a force F is applied at centre of mass in horizontal direction,
for any value of force F, block can't topple.

l If net momentum of system of particle is zero then angular momentum of that system about any point is
same.

l If uniform smooth rod hinged about one and free to rotate without friction in gravity free space and the
particle collide with this rod at a distance 2L/3 (L is length of rod) then impulsive force from hinge at the
time of collision is zero.
251\D\Allen(IIT-JEE Wing)\2020-21\Nurture\Phy\Unit-03\02-Rigid Body Dynamics

133
JEE-Physics

SOME WORKED OUT ILLUSTRATIONS


Illustration 1.
2N
A uniform solid disc of mass 1 kg and radius 1m is kept on a rough horizontal
surface. Two forces of magnitude 2 N and 4 N have been applied on the disc as
shown in the figure. Linear acceleration of the centre of mass of the disc is if there 4N
is no slipping.
(A) 4 m/s2 (B) 2 m/s2 (C) 1 m/s2 (D) zero
Ans. (D)
Solution
Taking torque about contact point, t =4 × R – 2 × 2R = 0, Fnet = 0

Illustration 2.
A light rod carries three equal masses A, B and C as shown in figure. What
will be velocity of B in vertical position of rod, if it is released from horizontal
position as shown in figure ?

8gl 4gl
(A) (B)
7 7

2gl 10gl
(C) (D)
7 7
Ans. (D)
Solution
Loss in P.E. = Gain in K.E.
1æ æ lö ö 2
2 2
l æ 2l ö æ 2l ö
mg + mg çè 3 ÷ø + mgl = 2 ç m çè 3 ÷ø + m çè 3 ÷ø + ml ÷ w
2

3 è ø

36g 2l 36g 8gl


Þw= Þ v B = wl B = =
14l 3 14l 7

Illustration 3.
Figure shows the variation of the moment of inertia of a uniform rod, about an
0.6
axis passing through its centre and inclined at an angle q to the length. The I (kg-m2)
moment of inertia of the rod about an axis passing through one of its ends and
p
making an angle q = will be p q(rad)
3
(A) 0.45 kg–m2 (B) 1.8 kg–m2 (C) 2.4 kg–m2 (D) 1.5 kg–m2
Ans. (B)
Solution
q
2 2
ML ML æ pö
I= sin2 q Þ 0.6 = sin ç ÷ Þ ML2 = 7.2
12 12 è 2ø
251\D\Allen(IIT-JEE Wing)\2020-21\Nurture\Phy\Unit-03\02-Rigid Body Dynamics

ML2 p ML2 æ 3 ö ML2 7.2 q


I= sin2 q , at q = ,I = = = = 1.8 kg-m2
3 3 3 çè 4 ÷ø 4 4

134
Rigid Body Dynamics
Illustration 4.
A child's top is spun with angular acceleration a = 4t3 – 3t2 + 2t where t is in seconds and a is in radian per
second-squared. At t =0, the top has angular velocity w0 = 2 rad/s and a reference line on it is at angular
position q0 = 1 rad.
Statement I : Expression for angular velocity w = ( 2 + t2 - t 3 + t 4 ) rad/s
Statement II : Expression for angular position q = (1 + 2t - 3t 2 + 4t 3 ) rad
(A) Only statement-I is true (B) Only statement-II is true
(C) Both of them are true (D) None of them are true
Ans. (A)
Solution
w t

ò dw = ò adt Þ w - 2 = ( t - t + t )0 Þ w = 2 + t - t + t
4 3 2 t 2 3 4

2 0

q t t
æ t3 t 4 t5 ö 3 4 5

ò1 d q = ò0 w dt Þ q - 1 = çè 2t + - + ÷ Þ q = 1 + 2t + t - t + t
3 4 5 ø0 3 4 5

Illustration 5.
Figure shows a uniform disk, with mass M = 2.4 kg and radius R = \\\\\\\\\\\\\\\\\\\\\\\\\\\\\\\\\\\\\\\\\\\\\\\\\\\\\\\\\\\\\\\\\\\\\\

20 cm, mounted on a fixed horizontal axle. A block of mass m = 1.2


kg hangs from a massless cord that is wrapped around the rim of the
disk. The tension in cord is
(A) 12 N (B) 20 N
(C) 24 N (D) None of these

Ans. (D)
Solution.

For block : mg – T = ma .......(i)


a
R
MR2
For disk (pulley) TR = Ia = a M
2
T
a Ma T
But a = so T = ...(ii)
R 2
m a
(1.2 ´ 10 )
Therefore mg - T = 2m Þ mg = 2m + 1 Þ t = mg
= = 6N
T M T M æ 2m ö æ 211.2 ö mg
çè + 1÷ ç + 1÷
M ø è 2.4 ø

Illustration 6.
y
The figure shows a uniform rod lying along the x-axis. The locus of all the points
lying on the xy-plane, about which the moment of inertia of the rod is same as
that about O is :
(A) an ellipse (B) a circle
(C) a parabola (D) a straight line O x
251\D\Allen(IIT-JEE Wing)\2020-21\Nurture\Phy\Unit-03\02-Rigid Body Dynamics

Ans. (B)
Solution y
P (x,y)
ML2 éæ Lö
2
ù
IP= ICM + Mr = 2
+ M êç x - ÷ + y 2 ú r y
12 ëè 2ø û
x
2 2
ML2 æ Lö æ Lö L
,0
I0 = Þ ç x - ÷ + y2 = ç ÷ Þ Locus is a circle 2
3 è 2 ø è 2ø

135
JEE-Physics
Illustration 7.
p
A small block of mass 'm' is rigidly attached at 'P' to a ring of mass '3m' and radius q
'r'. The system is released from rest at q = 90° and rolls without sliding. The
angular acceleration of hoop just after release is–
g g
(A) (B)
4r 8r
g g
(C) (D)
3r 2r
Ans. (B)
Solution
f = 4 ma ...(i) (mg – f) r = (3mr2 + mr2) a
mg – f = 4 ma ....(ii)
from (i) and (ii) Þ 8 ma = mg Þ a =g/8 Þ a = g/8r
mg
f
Illustration 8.
An impulsive force F acts horizontally on a solid sphere of radius R placed on a F
horizontal surface. The line of action of the impulsive force is at a height h h
above the centre of the sphere. If the rotational and translational kinetic energies R
of the sphere just after the impulse are equal, then the value of h will be-
2 2 2
(A) R (B) R (C) R (D) None of these
5 3 5
Ans. (C)
Solution
2
FDt = Mv ; FhDt = Iw Þ Mvh = I w = MR2w
5
1 1 1 2 2
Also. Mv2 = Iw2 = × MR2w2 Þ h = R
2 2 2 5 5

Illustration 9.
A bicycle is in motion. The force of friction exerted by the ground on its wheel is such that it acts:
(A) in backward direction on front wheel and in forward direction on rear wheel when it is accelerating
(B) in forward direction on front wheel and in backward direction on rear wheel when brakes are applied on
rear wheel only
(C)in backward direction on front wheel and in forward direction on rear wheel when brakes are applied on
rear wheel only
(D)in backward direction on both the wheels when brakes are applied on front wheel
Ans. (A,B)
Solution
251\D\Allen(IIT-JEE Wing)\2020-21\Nurture\Phy\Unit-03\02-Rigid Body Dynamics

Illustration 10.
In the figure, the blocks have unequal masses m 1 and m2 (m1 > m2). m1 has a downward
acceleration a. The pulley P has a radius r, and some mass. The string does not slip on P
the pulley–
(A) The two sections of the string have unequal tensions.
(B) The two blocks have accelerations of equal magnitude.
(C) The angular acceleration of P is a/r m2
æ m1 - m2 ö m1 a
(D) a < ç m + m ÷ g
è 1 2ø

136
Rigid Body Dynamics

Ans. (A,B,C,D)
Solution
a P
a
In this situation T1 ¹ T2 Þ a = Þ a =
(m1 - m2 ) g
r I
m1 + m2 +
r2 T2 T1

a m2 m1 a

Illustration 11.
A uniform rod AB of length l is free to rotate about a horizontal axis passing through A. A
The rod is released from rest from horizontal position. If the rod gets broken at midpoint
C when it becomes vertical, then just after breaking of the rod : C
(A) Angular velocity of upper part starts to decrease while that of lower part remains
constant. B
(B) Angular velocity of upper part starts to decrease while that of lower part starts to
increase
(C) Angular velocity of both the parts is identical at the time of breaking.
(D) Angular velocity of lower part becomes equal to zero
Ans. (A,C)
Solution
On upper part torque of mg about A will decrease the angular velocity.
v/2
v/2 Lower part of rod will not experience any couple hence its angular velocity can't change.
v
Initially both parts are having same angular velocities.

Illustration 12 to 14.
A uniform hollow sphere is released from the top of a fixed inclined plane of inclination 37° and height 3m. It
rolls without sliding.

37° fixed

12. The acceleration of the centre of mass of the hollow sphere is


30 18 9 15
(A) m/s2 (B) m/s2 (C) m/s2 (D) m/s2
7 5 5 7

13. The speed of the point of contact of the sphere with the inclined plane when the sphere reaches half–way of
the incline is
251\D\Allen(IIT-JEE Wing)\2020-21\Nurture\Phy\Unit-03\02-Rigid Body Dynamics

(A) 42 m/s (B) 21 m/s (C) 84 m/s (D) zero

14. The time taken by the sphere to reach the bottom is


3 5 5
(A) s (B) s (C) s (D) None of these
5 3 4
Solution
12. Ans. (B)
g sin q (10 ) ( 3 / 5) 18
a= = = ms -2
K2 2 5
1+ 2 1+
R 3
137
JEE-Physics
13. Ans. (D)
Speed of point of contact in pure rolling is always zero
14. Ans. (B)
1 2 3 1 æ 18 ö 5
s = ut + at Þ = ç ÷ ( t2 ) Þ t = s
2 sin 37° 2 è 5 ø 3

Illustration 15 to 17.
A mouse, searching for food, jumped onto the rim of a stationary circular disk mounted on a vertical axle.
The disk is free to rotate without friction. The velocity of the mouse was tangent to the edge of the disk before
it landed. When the mouse landed, it gripped the surface, remained fixed on the outer edge of the disk at
a distance R from the center, and set it into rotation. The sketch indicates the situation.
v0

R
w1

Before After
The mass of the mouse is m = 0.10 kg, the radius of the disk is R = 0.20 m, and the rotational inertia
of the disk is I = 0.0080 kg·m². The speed of the mouse, just before it landed on the disk is vo = 1.5 m/s.
15. Magnitude of the angular velocity of the disk plus mouse, after it landed becomes
(A) 0.25 rad/s (B) 2.5 rad/s (C) 0.375 rad/s (D) 3.75 rad/s

16. Find the magnitude of the impulse received by the mouse as it landed on the disk.
(A) 0.01 kg.m/s opposite to direction of motion (B) 0.01 kg.m/s in the direction of motion
(C) 0.10 kg.m/s opposite to direction of motion (D) 0.10 kg.m/s in the direction of motion

17. The mouse, still searching for food, crept to the center of the disk (where r = 0). Find angular velocity of the
disk plus mouse, when the mouse was at the center of the disk.
(A) 0.25 rad/s (B) 2.5 rad/s (C) 0.375 rad/s (D) 3.75 rad/s
Solution
15. Ans. (B)
mv 0R ( 0.1) (1.5) ( 0.2)
By conservation of angular momentum mv0R = (I + mR2)w Þ w = 2
= = 2.5 rad/s
I + mR 0.008 + 0.004
16. Ans. (C)
Impulse received by mouse = change in momentum = 0.1 (2.5 × 0.2 – 1.5) = – 0.1 kg m/s
17. Ans. (D)
( 0.1) (1.5 ) ( 0.2)
By conservation of angular momentum : mv0R = IwÞ w = = 3.75 rad/s
0.008

Illustration 18 to 19. m
A hollow sphere is released from the top of a movable wedge, friction is
sufficient for pure rolling of sphere on the wedge. There is no friction between
251\D\Allen(IIT-JEE Wing)\2020-21\Nurture\Phy\Unit-03\02-Rigid Body Dynamics

the wedge and the ground. Radius of sphere is R. At the instant it leaves the m
wedge horizontally.

18. Velocity of centre of mass of sphere w.r.t. ground is-

5 3 11
(A) gh (B) 2gh (C) gh (D) gh
7 7 7

19. Angular velocity of sphere w is-

12gh 27 gh 20gh 44gh


(A) (B) (C) (D)
7R 2 7 R2 7R 2 7R 2
138
Rigid Body Dynamics
Solution
18. Ans. (C)
m
2v
Rw - v = v Þ w =
w R
m v Energy conservation
v
ö æ 4v ö
2
1 1 1æ2 3
mgh = mv 2 + mv 2 + ç mR 2 ÷ ç 2 ÷ Þ v = gh
2 2 2è3 øè R ø 7
19. Ans. (A)
2v 4 3 12gh
w= = 2
´ gh =
R R 7 7R2

Illustration 20.
A non uniform rod OA of liner mass density l = l 0 x ( l 0 = co nst.) is suspended from ceiling with hinge joint
O & light string as shown in figure. Find the angular acceleration of rod just after the string is cut
2g g
(A) (B)
L L O g

4g x
(C) (D) None of these A
3L
Ans. (C)
Solution
t = Ia dI = dmx2

ò dI = ò dm x
2
f ×RC = Ia dm = l0 × dx
RC centre of mass

ò dm x L
RC =
ò ò
dI = l 0 x 3dx
ò dm 0

RC =
ò l x dx
0
2

L l 0L4
I=
ò l x dx
0
0 4

2L
RC = F = mg
3
L

ò
F = g dm
0

L
251\D\Allen(IIT-JEE Wing)\2020-21\Nurture\Phy\Unit-03\02-Rigid Body Dynamics

ò
F = g l 0 x dx
0

L2
F = l0g
2
By putting these values
l 0 L4 l L2 2L
a = 0 ´g´
4 2 3
4g
of for solving a =
3L
139
JEE-Physics

ANSWERS
BEGINNER'S BOX-1

3 5 ml 2
1. (A) 2. (A) 3. (D) 4. MR2 5. 6. l/ 2
2 3

é 83 ù
MR 2 ú 2ml 2
7. ê 8.
ë 320 û 3

BEGINNER'S BOX-2
1. (C) 2. (D) 3. (A) 4. (A)
5. (a) T = 225N, (b) FX = 225N, F Y = 300N 6. -23iˆ + 29ˆj + 38kˆ 7. 1 : 2

BEGINNER'S BOX-3
1. (C) 2. (D) 3. (D) 4. (C) 5. (C) 6. (C)

Mg
7.
M + m + I / r2

BEGINNER'S BOX-4
1. (D) 2. (C) 3. (D) 4. (D) 5. (A) 6. (D)
7. (D) 8. 2g/7 9. T = 1/6 mg, a = 2g/3R

BEGINNER'S BOX-5
1. (A) 2. (D) 3. (D) 4. 2mg/3 5. (2/3) mg

BEGINNER'S BOX-6
1. (B) 2. (C) 3. (C) 4. (C) 5. (C) 6. (D)
7. (D) 8. (B) 9. (A) 10. (B)

BEGINNER'S BOX-7
1. (A) 2. (A) 3. (A) 4. (B,C) 5. (C) 6. (A)
7. (B) 8. (C) 9. (A) 10. (B)
251\D\Allen(IIT-JEE Wing)\2020-21\Nurture\Phy\Unit-03\02-Rigid Body Dynamics

140
Rigid Body Dynamics

SINGLE CHOICE CORRECT QUESTIONS


1. Two rods of equal mass m and length l lie along the x axis and y axis with their centres origin. What is the
moment of inertia of both about the line x=y :
ml2 ml2 ml2 ml2
(A) (B) (C) (D)
3 4 12 6

2. A solid sphere (mass 2 M) and a thin hollow spherical shell (mass M) both of the same size, roll down an
inclined plane, then
(A) Solid sphere will reach the bottom first (B) Hollow spherical shell will reach the bottom first
(C) Both will reach at the same time (D) None of these

3. A wheel is rotating about an axis through its centre at 720 rpm. It is acted on by a constant torque opposing its

motion for 8 second to bring it to rest finally. The value of torque in Nm is :– (given I = 24 kg - m2 )
p
(A) 48 (B) 72 (C) 96 (D) 120

4. A rod of mass M and length L is placed in a horizontal plane with one end hinged about the vertical axis. A
Mg 5L
horizontal force of F= is applied at a distance from the hinged end. The angular acceleration of the
2 6
rod will be :-
4g 5g 3g 4g
(A) (B) (C) (D)
5L 4L 4L 3L

5. A string is wrapped around the rim of a wheel of moment of inertia 0.20 kg-m 2 and 20N
radius 20 cm. The wheel is free to rotate about its axis and initially the wheel is rest. The
string is now pulled by a force of 20N. The angular velocity of the string after 5 seconds
will be :–
(A) 90 rad/s (B) 70 rad/s
(C) 95 rad/s (D) 100 rad/s

6. In the following figure r1 and r2 are 5 cm and 30 cm respectively. If the moment of


10N
inertia of the wheel is 5100 kg-m2 then its angular acceleration will be :- °
30 r1
(A) 10–4 rad/sec2 (B) 10–3 rad/sec2
(C) 10–2 rad/sec2 (D) 10–1 rad/sec2 12N
9N
r2

1
7. If the earth were to suddenly contract to th of its present radius without any change in its mass then the
n
duration of the new day will be nearly :–
24 24
(A) hour (B) 24n hour (C) hour (D) 24n2 hour
251\D\Allen(IIT-JEE Wing)\2020-21\Nurture\Phy\Unit-03\02-Rigid Body Dynamics

n n2

8. The angular velocity of a body changes from w1 to w2 without applying torque. The ratio of initial radius of
gyration to the final radius of gyration is :–
(A) w2 : w1 (B) w1 : w2 (C) w2: w1 (D) w1 : w2

9. A sphere rolls down without slip on an inclined plane of inclination q. What is the linear acceleration as the
sphere reaches bottom
5 3 2 2
(A) g sin q (B) g sin q (C) g sin q (D) g sin q
7 5 7 9

141
JEE-Physics
10. A rod is hinged at its centre and rotated by applying a constant torque starting from rest. The power developed
by the external torque as a function of time is :–

Pext Pext

(A) (B)

time time

Pext Pext

(C) (D)

time time

11. A solid cylinder of mass M and radius R released from rest on smooth inclined plane of length L and height h.
What is the speed of its centre of mass when the cylinder reaches its bottom :–

3 4
(A) 2 gh (B) gh (C) gh (D) 4 gh
4 3

12. There is rod of length l. The velocities of its two ends are v1 and v2 in opposite directions normal to the rod.
The distance of the instantaneous axis of rotation from v 1 is :
v2 v 1l
(A) Zero (B) v + v l (C) v + v (D) 2l
1 2 1 2

A
13. A uniform rod AB of mass m and length l at rest on a smooth horizontal
surface. An impulse P is applied to the end B. The time taken by the rod to
turn through a right angle is :- ll

2pml pml pml 2pml


(A) (B) (C) (D)
P 3P 12P 3P P
B

14. A particle of mass m is projected with a velocity v making an angle of 45° with the horizontal. The magnitude
of the angular momentum of the projectile about the point of projection when the particle is at its maximum
height h is :

mv 3 mv 3
(A) zero (B) (C) (D) m 2gh 3
(4 2g) 2g
251\D\Allen(IIT-JEE Wing)\2020-21\Nurture\Phy\Unit-03\02-Rigid Body Dynamics

15. Two spheres each of mass M and radius R/2 are connected with a massless rod Y
of length 2R as shown in the-figure. What will be the moment of inertia of the M M
P
system about an axis passing through the centre of one of the spheres and Q
2
R/

perpendicular to the rod


21 2
(A) MR2 (B) MR2
5 5 Y’

5 5
(C) MR2 (D) MR2
2 21

142
Rigid Body Dynamics
16. A cord is wound over a cylinder of radius r and moment of inertia I. A mass m is attached to the free end of
the cord. The cylinder is free to rotate about its own horizontal axis. If mass m is released from rest, then the
velocity of the mass after it had fallen through a distance h will be-
1/2 1/2 1/ 2
æ 2mghr² ö æ 2mghr² ö æ mghr² ö
(A) (2gh)1/2 (B) ç ÷ø (C) ç (D) ç
è I è I + mr² ÷ø è I + 2mr² ÷ø

17. A solid sphere of radius R is placed on smooth horizontal surface. A


horizontal force ‘F’ is applied at height ‘h’ from the lowest point. For the
maximum acceleration of centre of mass, which is correct-
(A) h = R
(B) h = 2R
(C) h = 0
(D) No relation between h and R

18. A solid sphere is placed on a smooth horizontal plane. A horizontal impulse I is


applied at a distance h above the central line as shown in the figure. Soon after
giving the impulse the sphere starts rolling.
The ratio h/R would be-
1 2 1 1
(A) (B) (C) (D)
2 5 4 5

19. The moment of inertia of semicircular plate of radius R and mass M about axis A
AA' in its plane passing through its centre is
MR 2 MR 2 q
(A) (B) cos2 q
2 4
A'
MR 2 MR 2
(C) sin2 q (D)
4 4

20. A ring of mass m and radius R has three particles attached to the ring as shown in the figure. The centre of
the ring has a speed v0. The kinetic energy of the system is : (slipping is absent)

m
2m m

(A) 6 mv02 (B) 12 mv02 (C) 4 mv02 (D) 8 mv02


251\D\Allen(IIT-JEE Wing)\2020-21\Nurture\Phy\Unit-03\02-Rigid Body Dynamics

21. A thin rod of mass M and length L is struck at one end by a ball of clay of mass m, moving with speed v as
shown in figure. The ball sticks to the rod. After the collision, the angular momentum of the clay-rod system
about A, the midpoint of the rod, is

æ M ö æ vL ö æ M ö æ vL ö 900
(A) ç m + ÷ ç ÷ (B) çè m + ÷ø çè ÷ø
è 3ø è 2 ø 12 2 A
mvL
(C) (D) mvL
2

143
JEE-Physics
22. A rigid lamina is rotating about an axis passing perpendiuclar to its plane through point O as shown in figure.

5m w =10rad/s
3m

37°
O 4m A

The angular velocity of point B w.r.t. A is


(A) 10 rad/s (B) 8 rad/s (C) 6 rad/s (D) 0

23. A uniform thin stick of length l and mass m is held horizontally with its end B hinged at a point B on the edge
of a table. Point A is suddenly released. The acceleration of the centre of mass of the stick at the time of release,
is :

B A

3 3 2 1
(A) g (B) g (C) r g (D) g
4 7 7 7

24. A disc of radius R = 2m moves as shown in the figure, with a velocity of


2v0
2v 0 R R
translation of 6v0 of its centre of mass and an angular velocity of . The 6v 0
R
distance (in m) of instantaneous axis of rotation from its centre of mass is
(A) 3 (B) 4 (C) 5 (D) 6

25. Two equal masses each of mass M are joined by a massless rod of length L.
M
Now an impulse MV is given to the mass M making an angle of 30° with the
30°
length of the rod. The angular velocity of the rod just after imparting the impulse
is : MV
(A) v/L (B) 2v/L
(C) v/2L (D) none
251\D\Allen(IIT-JEE Wing)\2020-21\Nurture\Phy\Unit-03\02-Rigid Body Dynamics

26. A uniform disc of radius R is placed on a smooth horizontal surface. At any time
2v 0
angular velocity of point Q on the disc is clockwise. Translational velocity R
R
of centre of mass is v0. The speed of highest point A and lowest point B are
respectively
(A) 3v0, v0 (B) 2v0, v0
(C) v0, 2v0 (D) v0, 3v0

144
Rigid Body Dynamics
27. A disc of mass m is rolling without slipping on a rough surface. The velocity of
centre of mass of disc is 10 m/s. Find the velocity of point A which makes an A
angle 30o from horizontal as shown in figure. 30
o

(A) v A = 15iˆ - 3 5ˆj (B) v A = 15iˆ + 5 3jˆ O vcm

(C) v A = 15iˆ - 5 3jˆ (D) v A = -15iˆ - 5 3jˆ

28. A body is rolling down an inclined plane. Its translational and rotational kinetic energies are equal. The body
is a
(A) Solid sphere (B) Hollow sphere (C) Solid cylinder (D) Hollow cylinder

29. A solid sphere, a hollow sphere and a ring are released from top of an inclined plane (frictionless) so that they
slide down the plane. Then maximum acceleration down the plane is for (no rolling)
(A) Solid sphere (B) hollow sphere (C) Ring (D) All same
251\D\Allen(IIT-JEE Wing)\2020-21\Nurture\Phy\Unit-03\02-Rigid Body Dynamics

145
JEE-Physics

SECTION - 1 : MULTIPLE CHOICE CORRECT QUESTIONS


1. In the figure shown, the plank is being pulled to the right with a constant speed
v. If the cylinder does not slip then R
(A) the speed of the centre of mass of the cylinder is 2v
(B) the speed of the centre of mass of the cylinder is zero v
(C) the angular velocity of the cylinder is v/R
(D) the angular velocity of the cylinder is zero
2. A cylinder rolls without slipping over a horizontal plane with constant velocity. The
radius of the cylinder is equal to r. The curvature radii of trajectories traced out by
the points A and B in figure.
(A) RA = 4r (B) RB = 2 2 r
(C) RA = r (D) RB = 4r
3. A thin rod of mass m and length l is hinged to a ceiling and it is free to rotate in a vertical plane. A particle of
mass m, moving with speed v strikes it as shown in the figure and gets stick with the rod. The value of v , for
which the rod becomes horizontal after collision is
168 l/2
(A) The value of v, for which rod becomes horizontal after collision is gl
9
60°
53 v
(B) The value of v, for which rod becomes horizontal after collision is gl m
3 m, l

(C) Angular momentum of (rod + particle) system will remain constant about hinge just before and after
collision
(D) Angular momentum of (rod + particle) system will remain same about centre of mass just before and after
collision

4. A solid sphere is in pure rolling motion on an inclined surface having inclination q :-


(A) frictional force acting on sphere is ƒ = µ mgcosq
(B) ƒ is disspative force
(C) friction will increase its angular velocity and decreases its linear velocity
(D) if q decrease, friction will decrease q

5. A plank with a uniform sphere placed on it, rests on a smooth horizontal plane.
Plank is pulled to right by a constant force F. If the sphere does not slip
over the plank F
(A) Acceleration of centre of sphere is less than that of the plank
(B) Acceleration of centre of sphere is greater than the plank because friction acts rightward on the sphere
(C) Acceleration of the centre of sphere may be towards left
(D) Acceleration of the centre of sphere relative to plank may be greater than that of the plank relative to floor
6. A rod AC of length L and mass m is kept on a horizontal smooth
251\D\Allen(IIT-JEE Wing)\2020-21\Nurture\Phy\Unit-03\02-Rigid Body Dynamics

plane.It is free to rotate and move.A particle of same mass m mov-


ing on the plane with velocity v strikes the rod at point B making an
angle 37° with the rod.The collision is elastic.After collision :
72v
(A) Angular velocity of the rod will be
55L
pL
(B) The centre of the rod will travel a distance in the
3
time in which it makes half rotation
24mV
(C) Impulse of the impact force is
55
(D) None of these

146
Rigid Body Dynamics
7. A cylinder and a variable mass M are arranged on a fixed wedge using a Mass less pulley
light string and a massless pulley. There is enough friction between
cylinder and the wedge to prevent any slipping.
(A) Only one value of M is possible for which cylinder can remain in
equilibrium. Cylinder
(B) There is a range of value of M for which cylinder can remain in M
equilibrium.
Fixed Wedge
(C) For a certain value of M, the cylinder starts to roll up the plane.
In this situation, magnitude of friction force on the cylinder
by the wedge will be greater than tension in the string
(D) For a certain value of M, the cylinder starts to roll down the
plane. In this situation, magnitude of friction force on the cylinder
by the wedge will be greater than tension in the string

8. A uniform rigid rod of length l and mass m is hinged at O in such a way that it
y
can rotate in vertical plane. Initially it is kept vertically as shown in figure. When
it is released, it rotates about O in verticle plane. (consider all surfaces x
are frictionless).
(A) The magnitude of force given by the hinge to rod at instant when rod
O
37
becomes horizontal is mg
4
(B) The centre of mass of rod undergoes uniform circular motion
(C) The direction of hinge force is along +y-axis at instant when centre of mass of rod is at lowest position.
(D) The mechanical energy of rod is not conserved during the motion.

9. A horizontal uniform rod of mass ‘m’ has its left end hinged to the fixed incline Hinge
plane, while its right end rests on the top of a uniform cylinder of mass ‘m’ which m
in turn is at rest on the fixed inclined plane as shown. The coefficient of friction
between the cylinder and rod, and between the cylinder and inclined plane, is
sufficient to keep the cylinder at rest. fixed inclined plane

mg
(A) The magnitude of normal reaction exerted by the rod on the cylinder is
2
(B) The ratio of magnitude of frictional force on the cylinder due to the rod and the magnitude of frictional
force on the cylinder due to the inclined plane is 1 : 1
3mg
(C) The magnitude of normal reaction exerted by the inclined plane on the cylinder is
2
(D) The magnitude of normal reaction exerted by the inclined plane on the cylinder is 2 mg

10. The disc of radius r is confined to roll without slipping at A and B. If the plates have the velocities shown, then
(A) linear velocity v0 = v
251\D\Allen(IIT-JEE Wing)\2020-21\Nurture\Phy\Unit-03\02-Rigid Body Dynamics

A
3v v
(B) angular velocity of disc is
2r
v0
2v w0
(C) angular velocity of disc is
r 3v
(D) None of these B

147
JEE-Physics
11. A thin uniform rod of mass m and length l is free to rotate about its upper end. When it is at rest, it receives
an impulse J at its lowest point, normal to its length. Immediately after impact
3J
(A) the angular momentum of the rod is Jl. (B) the angular velocity of the rod is
ml

3J2 3J
(C) the kinetic energy of the rod is (D) the linear velocity of the midpoint of the rod is
2m 2m

SECTION - 2 : COMPREHENSION BASED QUESTIONS


(SINGLE CHOICE CORRECT QUESTION)
Paragraph for Q.no. (12 to 14)
A uniform rigid rod of mass m and length l is supported by a hinge and a spring as shown in the figure. The
entire arrangement is in vertical plane. The rod makes an angle q to the horizontal while spring is in extended
state along vertical direction. The rod is in equilibrium and spring constant of the spring is K N/m
q

12. The extension in the spring is


mg 2mg mg
(A) (B) (C) (D) none of these
k k 2k

13. The magnitude of hinge reaction is


mg
(A) mg (B) 2mg (C) (D) none of these
2

14. If at any instant t = t0, the spring is cut. Then the magnitude of hinge reaction at t = t 0 is
7mg 3 mg
(A) cos q (B) mg (C) mg cos q (D) 1 + 15 sin 2 q
4 4 4

Paragraph for Q.no. (15 to 18)


m, r
In the figure shown a plank of mass m is lying at rest on a smooth horizontal
surface. A disc of same mass m and radius r is rotated to an angular speed w0 w0
and then gently placed on the plank. If we consider the plank and the disc as a
system then frictional force between them is an internal force. Momentum of m
the system changes due to external force only. It is found that finally slipping
cease, and 50% of total kinetic energy of the system is lost. Assume that plank
is long enough. m is coefficient of friction between disc and plank.

15. Final velocity of the plank is


251\D\Allen(IIT-JEE Wing)\2020-21\Nurture\Phy\Unit-03\02-Rigid Body Dynamics

rw 0 rw 0 rw 0 rw 0
(A) (B) (C) (D)
4 10 2 2 10

16. Time when slipping ceases


rw 0 rw 0 rw 0 rw 0
(A) (B) (C) (D)
2mg 10mg 4mg 2 10mg

17. Magnitude of the change in angular momentum of disc about centre of mass of disc
3 2 1 2 1 2
(A) mr w 0 (B) mr w 0 (C) zero (D) mr w 0
4 4 2

148
Rigid Body Dynamics
18. Distance moved by the plank from the placing of disc on the plank till the slipping ceases between disc and plank

r 2 w 20 r 2 w 02 r2 w20 r 2 w 02
(A) (B) (C) (D)
16mg 8 mg 32mg 200mg

Paragraph for Q.no. (19 to 20)


w0
A solid sphere has linear velocity v0 = 4 m/s and angular velocity w0 =9 rad/s
as shown. Ground on which it is moving, is smooth. It collides elastically with v0
a rough wall of coefficient of friction µ. Radius of the sphere is 1 m and mass is
2 kg.

19. If the sphere after colliding with the wall roll without slipping in opposite direction, then coefficient of friction
µ is :
1 2 1 1
(A) (B) (C) (D)
2 3 3 4

20. What is net linear impulse imparted by the wall on the sphere during impact :-

(A) 32 N-s (B) 4 17 N-s (C) 4 5 N-s (D) 15 2 N-s

Paragraph for Q.no. (21 & 22)


A thin rod of mass m and length L lies on a frictionless horizontal surface and is m
hinged at one of its ends such that it can rotate freely about a vertical axis passing
through the hinge. A particle of same mass m hits the rod at the extreme end (as v0
shown) with velocity v0 and does not stick with rod and come to rest immediately m
such that the rod rotates with a constant angular velocity w after the collision.

21. Then angular velocity just after collision is equal to


2v 0 3v 0 5v 0 5v 0
(A) (B) (C) (D)
5L L 7L 9L

22. Magnitude of hinge reaction just after the collision is


7mv 02 5mv 02 9mv 02 9mv 02
(A) (B) (C) (D)
2L 2L 2L 4L

y
Paragraph for Q.no. (23 & 24)
A rod AB of length 2 m and mass 2 kg is lying on smooth horizontal x- y plane A
J
with its centre at origin O as shown figure. An impulse J of magnitude 10 N-s is
applied perpendicular to AB at A. O x

23. The distance of point P from centre of the rod which is at rest just after the impact is :- B
2 1 1 1
(A) m (B) m (C) m (D) m
3 3 2 4
251\D\Allen(IIT-JEE Wing)\2020-21\Nurture\Phy\Unit-03\02-Rigid Body Dynamics

p
24. Co-ordinates of point A of the rod after time t = s will be :-
45

éæ p 3ö 1 ù éæ 3 3 öù éæ p 1 ö 1 ù é1 1 ù
(A) êç 9 + 2 ÷ m, 2 m ú (B) ê ç m, m ÷ ú (C) êçè + ÷ø m, m ú (D) ê m, m ú
ëêè ø ûú ëè 4 2 øû ë 6 2 2 û ë 2 2 û

149
JEE-Physics

SECTION - 1 : NUMERICAL ANSWER BASED QUESTIONS


1. A solid uniform cylinder of mass m = 6 kg and radius r = 0.1 m is kept in balance on F
a slope of inclination a = 37° with the help of a thread fastened to its jacket. The m
r
cylinder does not slip on the slope. The minimum required coefficient of friction to
keep the cylinder in balance when the thread is held vertically is given as m. Find the
value of 4m. a

A
2. A uniform rod ABC of mass M is placed vertically on a rough horizontal
surface. The coefficient of kinetic friction between the rod and the surface
is m. A force F is applied on the rod at point B at distance l/3 below centre
l/3
of the rod as shown in Figure. The initial acceleration of point A, B F
F
(if the rod slips on the surface) is xmg – . Find the value of x C
M
F
3. A tangential force F acts at the top of a thin spherical shell of mass m and
R
xF
radius R. The acceleration of the shell if it rolls without slipping is . Find x
5m f

4. S2 is a fixed rough sphere and S1 is a solid sphere. S1 is given a negligible


velocity, so that it starts moving on the sphere S2. It rolls without slipping. The
angle formed with the vertical, by the line joining the centres of S1 and S2

when S1 leaves S2 is q = cos -1 æç 10 ö÷ . Find x


è xø

5. A rod of length l and mass M held vertically is let go down, without slipping at the point of contact. The velocity
of the top end at the time of touching the ground is x gl . Find the value of x.

6. An initial momentum is imparted to a homogeneous solid cylinder as a result of which it begins to roll without
slipping up an inclined plane at speed v0 = 4ms–1. The plane makes an angle of 30° with the horizontal. What
time (in sec) does the cylinder take before stopping.

7. A stick of length L and mass M lies on a frictionless horizontal surface on which


it is free to move in anyway. A ball of mass m moving with speed v as shown in
fig. The mass of the ball so that it remains at rest immediately after collision is

ML2
(Collision is elastic). Find the value of x
(L
2
+ 6 x d2 )
251\D\Allen(IIT-JEE Wing)\2020-21\Nurture\Phy\Unit-03\02-Rigid Body Dynamics

8. A uniform solid cylinder of mass M and radius 2R rests on a horizontal


table top. A string attached to it runs over a pulley (disc) of mass M and
radius R that is mounted on a frictionless axle through its centre. A
block of mass M is suspended from the free end of the string. The
string doesn’t slip over the pulley surface, and the cylinder rolls
xg
without slipping on the table top. The acceleration of the block is .
6
Find x.

150
Rigid Body Dynamics

9. A uniform metre scale of mass m is suspended by two vertical string attached


to its two ends as shown in figure. A body of mass m is placed on the 80 cm
x
mark. The ratio of tension in the strings is . Find the value of x
13

10. A carpet of mass 'M' made of inextensible material is rolled along its
length in the form of a cylinder of radius 'R' and is kept on a rough
floor. The carpet starts unrolling without sliding on the floor when a
negligibly small push is given to it. The horizontal velocity of the axis
of the cylindrical part of the carpet when its radius reduces to R/2 is

x ´ 7gR
.
3

SECTION - 2 : MATRIX - MATCH QUESTIONS


11. In the adjacent figure a uniform rigid body of mass m and radius R is kept at rest on F
a rough horizontal surface. A constant horizontal force F is applied at the top most
point of the body. The body starts rolling without slipping. Different shapes of bodies acm
R
are given in the column I and based on this problem some physical quantities related
to them are given in column II. Rough
Column I Column II
(A) Solid sphere (P) Friction force is zero
(B) Ring (Q) Magnitude of friction force is maximum
(C) Hollow sphere (R) Acceleration of C. O. M. is 4F/3 m
(D) Disc (S) Magnitude of friction force is F/5

12. Column I Column II

1
(A) A ring of mass m is projected on rough horizontal plane with velocity v 0 . (P) mv 20
3
The magnitude of work done by frictional force to start rolling

n0

1
(B) Kinetic energy of pivoted rod of mass m, velocity of centre of mass is v0. (Q) mv 20
8

n0
251\D\Allen(IIT-JEE Wing)\2020-21\Nurture\Phy\Unit-03\02-Rigid Body Dynamics

1
(C) Kinetic energy of translation of a smooth rod of mass m, (R) mv 20
4
where velocity of one end is v0.
v0

v=0

151
JEE-Physics

2
(D) Kinetic energy of a rod of mass m, as shown in figure. (S) mv 20
3

1
(T) mv 20
9
45° v0

13. A disc of radius R is rolling without slipping with an angular acceleration


a, on a horizontal plane. Four points are marked at the end of horizon-
tal and vertical diameter of a circle of radius r (<R) on the disc. If hori- w
zontal and vertical direction are chosen as x and y axis as shown in the a
1
r r r r R
figure, then acceleration of points 1, 2, 3 and 4 are a1 ,a2 ,a 3 and a4
respectively, at the moment when angular velocity of the disc is w . y 4
r 2
Match the following
Column-I Column-II x 3
r
(A) a1 (P) ( )
( Ra - ra ) ˆi + rw 2 ˆj
r
(B) a2 (Q) ( Ra + ra ) ˆi - ( rw ) ˆj
2

r
(C) a3 (R) ( Ra - rw ) ˆi - ( ra ) ˆj
2

r
(D) a4 (S) ( Ra + rw ) ˆi + (ra ) ˆj
2

(T) None of these

14. Four different situations of a moving disc are shown in column I and predictions about its final motion and
forces acting on it are given in column - II.
Column I Column II

v
2R
v
(A) (P) finally disc will roll along the initial direction of velocity (v)

2v
R

(B) v (Q) finally, disc will roll in direction opposite to the initial direction of velocity (v)

2v
R
v
(C) (R) finally, disc stops
251\D\Allen(IIT-JEE Wing)\2020-21\Nurture\Phy\Unit-03\02-Rigid Body Dynamics

4v
R
(D) v (S) Initially friction force acts in the direction opposite to that of initial velocity

(T) None of these

152
Rigid Body Dynamics

SINGLE CHOICE CORRECT QUESTIONS


1. Consider a uniform square plate of side ‘a’ and mass ‘m’ the moment of inertia of this plate about an axis
perpendicular to its plane and passing through one of its corners is [AIEEE - 2008]
5 1 7 2
(1) ma2 (2) ma2 (3) ma2 (4) ma2
6 12 12 3

2. A thin horizontal circular disc is rotating about a vertical axis passing through its centre. An insect is at rest at a
point near the rim of the disc. The insect now moves along a diameter of the disc to reach its other end. During
the journey of the insect, then angular speed of the disc :- [AIEEE-2011]
(1) continuously increases (2) first increases and then decreases
(3) remains unchanged (4) continuously decreses

3. This question has statement 1, statement 2. Of the four choices given after the statements, choose the one that
best describes the two statements.
Statement-1 : When moment of inertia I of a body rotating about an axis with angular speed w increases, its
angular momentum L is unchanged but the kinetic energy K decreases if there is no torque applied on it.
1
Statement-2 : L = I w, kinetic energy of rotation = Iw2. [AIEEE - 2012, Online]
2
(1) Statement-1 is false, Statement-2 is true.
(2) Statement-1 is true, Statement-2 is true and Statement-2 is not the correct explanation of statement-1.
(3) Statement-1 is true, Statement-2 is false.
(4) Statement-1 is true, Statement-2 is true and Statement-2 is the correct explanation of Statement-1.

4. A solid sphere is rolling on a surface as shown in figure, with a translation velocity v m/s. If it is to climb the
inclined surface continuing to roll without slipping, then minimum v for this to happen is :-
[AIEEE - 2012, Online]

v h

10 7 7
(1) gh (2) gh (3) gh (4) 2gh
7 2 5

5. A circular hole, of diameter R is cut from a disc of mass M and radius R ; the circumference of the cut passes
through the centre of the disc. The moment of inertia of the remaining portion of the disc about an axis
perpendicular to the disc and passing through its centre is : [AIEEE - 2012, Online]

æ 13 ö æ 15 ö æ 3ö æ 1ö
(1) çè ÷ø MR2 (2) çè ÷ø MR2 (3) çè ÷ø MR2 (4) çè ÷ø MR2
251\D\Allen(IIT-JEE Wing)\2020-21\Nurture\Phy\Unit-03\02-Rigid Body Dynamics

32 32 8 8

6. A solid sphere having mass m and radius r rolls down an inclined plane. Then its kinetic energy is :
[AIEEE - 2012, Online]
1 1 2 5
(1) rotational and translational (2) rotational and translational
2 2 7 7

2 3 5 2
(3) rotational and translational (4) rotational and translational
5 5 7 7

153
JEE-Physics
7. A ring of mass M and radius R is rotating about its axis with angular velocity w. Two identical bodies each of
mass m are now gently attached at the two ends of a diameter of the ring. Because of this, the kinetic energy
loss will be: [JEE (Main) - 2013, Online]
(M + m)M 2 2 Mm Mm m(M + 2m) 2 2
(1) w R (2) w2R2 (3) w 2R 2 (4) w R
(M + 2m) (M + 2m) (M + m) M

8. A ball of mass 160 g is thrown up at an angle of 60° to the horizontal at a speed of 10 ms–1. The angular
momentum of the ball at the highest point of the trajectory with respect to the point from which the ball is
thrown is nearly (g = 10 ms–2) [JEE (Main) - 2014, Online]
(1) 3.0 kg m /s
2 (2) 3.46 kg m /s
2 (3) 1.73 kg m /s
2 (4) 6.0 kg m 2/s

9. A mass ‘m’ is supported by a massless string wound around a uniform hollow


cylinder of mass m and radius R. If the string does not slip on the cylinder, with
what acceleration will the mass fall on release ? [JEE(Main) 2014]
2g g
(1) (2)
3 2
5g
(3) (4) g
6

10. A bob of mass m attached to an inextensible string of length l is suspended from a vertical support. The bob
rotates in a horizontal circle with an angular speed w rad/s about the vertical. About, the point of suspension:
(1) angular momentum is conserved [JEE(Main) 2014]
(2) angular momentum changes in magnitude but not in direction
(3) angular momentum changes in direction but not in magnitude
(4) angular momentum changes both in direction and magnitude

11. A particle of mass 2 kg is on a smooth horizontal table and moves in a circular path of radius 0.6 m. The height
of the table from the ground is 0.8 m. If the angular speed of the particle is 12 rad s–1, the magnitude of its
angular momentum about a point on the ground right under the centre of the circle Is :-
[JEE (Main) - 2015, Online]
(1) 20.16 kg m2s–1 (2) 8.64 kg m2s–1 (3) 14.4 kg m2s–1 (4) 11.52 kg m2s–1

12. Concrete mixture is made by mixing cement, stone and sand in a rotating cylindrical drum. If the drum rotates
too fast, the ingredients remain stuck to the wall of the drum and proper mixing of ingredients does not take
place. The maximum rotational speed of the drum in revolutions per minute(rpm) to ensure proper mixing is
close to : (Take the radius of the drum to be 1.25 m and its axle to be horizontal) :
[JEE (Main) - 2016, Online]
(1) 8.0 (2) 0.4 (3) 1.3 (4) 27.0

13. If the earth has no rotational motion, the weight of a person on the equator is W. Determine the speed with
3
which the earth would have to rotate about its axis so that the person at the equator will weighW. Radius
251\D\Allen(IIT-JEE Wing)\2020-21\Nurture\Phy\Unit-03\02-Rigid Body Dynamics

4
of the earth is 6400 km and g = 10 m/s2. [JEE (Main) - 2017, Online]
(1) 0.83 × 10–3 rad/s (2) 0.28 × 10–3 rad/s (3) 1.1 × 10 rad/s
–3 (4) 0.63 × 10–3 rad/s

14. In a physical balance working on the principle of moments, when 5 mg weight is placed on the left pan, the
beam becomes horizontal. Both the empty pans of the balance are of equal mass. Which of the following
statements is correct ? [JEE (Main) - 2017, Online]
(1) Left arm is longer than the right arm
(2) Every object that is weighed using this balance appears lighter than its actual weight.
(3) Both the arms are of same length
(4) Left arm is shorter than the right arm

154
Rigid Body Dynamics

R
15. A circular hole of radius is made in a thin uniform disc having mass M and radius R, as shown in figure. The
4
moment of inertia of the remaining portion of the disc about an axis passing through the point O and
perpendicular to the plane of the disc is :- [JEE (Main) - 2017, Online]

R
R
4
O O'

3R/4

219 MR 2 237 MR 2 19 MR 2 197 MR 2


(1) (2) (3) (4)
256 512 512 256

16. Seven identical circular planar disks, each of mass m and radius R are welded
symmetrically as shown. The moment of inertia of the arrangement about the axis P
normal to the plane and passing through the point P is: [JEE (Main) - 2018]
19 55
(1) MR 2 (2) MR 2 O
2 2

73 181
(3) MR 2 (4) MR 2
2 2

R
17. From a uniform circular disc of radius R and mass 9M, a small disc of radius is removed as shown in the
3
figure. The moment of inertia of the remaining disc about an axis perpendicular to the plane of the disc and
passing through centre of disc is: [JEE (Main) - 2018]

2R
3

R
251\D\Allen(IIT-JEE Wing)\2020-21\Nurture\Phy\Unit-03\02-Rigid Body Dynamics

42 37
(1) 4 MR2 (2) MR 2 (3) 10 MR2 (4) MR 2
9 9

18. Let the moment of inertia of a hollow cylinder of length 30 cm (inner radius 10 cm and outer radius 20 cm),
about its axis be I. The radius of a thin cylinder of the same mass such that its moment of inertia about its axis
is also I, is: [JEE (Main) - 2019, Online]
(1) 12 cm (2) 18 cm (3) 16 cm (4) 14 cm

155
JEE-Physics
19. The moment of inertia of a solid sphere, about an axis parallel to its diameter and at a distance of x from it, is
I(x)'. Which one of the graphs represents the variation of I(x) with x correctlv?
[JEE (Main) - 2019, Online]

(1) (2) (3) (4)

20. An equilateral triangle ABC is cut from a thin solid sheet of wood. (see figure) D, E and F are the mid-points of
its sides as shown and G is the centre of the triangle. The moment of inertia of the triangle about an axis
passing through G and perpendicular to the plane of the triangle is I0. It the smaller triangle DEF is removed
from ABC, the moment of inertia of the remaining figure about the same axis is I. Then:
[JEE (Main) - 2017, 2019, Online]
A
9 3
(1) I = I (2) I = I 0
16 0 4
D E
I0 15 G
(3) I = (4) I = I0
4 16 C
B F
r r r
21. A slob is subjected to two forces F1 and F2 of same magnitude F as shown in the figure. Force F2 is in XY--
r r
( )
plane while force F1 acts along z-axis at the point 2 i + 3 j . The moment of these forces about point O will be:
[JEE (Main) - 2019, Online]
z r
F1
r
F2 y
O
4m 30°

6m
x

( )
(1) 3iˆ - 2ˆj - 3kˆ F ( )
(2) 3iˆ + 2ˆj + 3kˆ F ( )
(3) 3iˆ + 2ˆj - 3kˆ F ( )
(4) 3iˆ - 2ˆj + 3kˆ F

22. The magnitude of torque on a particle of mass 1kg is 2.5 Nm about the origin. If the force acting on it is 1 N, and
the distance of the particle from the origin is 5m, the angle between the force and the position vector is (in
radians) :- [JEE (Main) - 2019, Online]
p p p p
(1) (2) (3) (4)
8 6 4 3
251\D\Allen(IIT-JEE Wing)\2020-21\Nurture\Phy\Unit-03\02-Rigid Body Dynamics

23. A circular disc D1 of mass M and radius R has two identical discs D2 and D3 of the same mass M and radius R
attached rigidly at its opposite ends (see figure). The moment of inertia of the system about the axis OO',
passing through the centre of D1, as shown in the figure, will be:- [JEE (Main) - 2019, Online]
O’
2
(1) 3MR2 (2) MR2
3

4
(3) MR2 (4) MR2 O
5 D2 D3
D1

156
Rigid Body Dynamics
24. A homogeneous solid cylindrical roller of radius R and mass M is pulled on a cricket pitch by a horizontal force.
Assuming rolling without slipping, angular acceleration of the cylinder is: [JEE (Main) - 2019, Online]
3F F 2F F
(1) 2m R (2) 3m R (3) 3m R (4) 2m R

25. Two identical spherical balls of mass M and radius R each are stuck on two ends of a rod of length 2R and mass
M (see figure). The moment of inertia of the system about the axis passing perpendicularly through the centre
of the rod is : [JEE (Main) - 2019, Online]

2R
R R

152 17 137 209


(1) MR 2 (2) MR 2 (3) MR 2 (4) MR 2
15 15 15 15

26. A thin circular plate of mass M and radius R has its density varying as r(r) = r0r with r0 as constant and
r is the distance from its centre. The moment of Inertia of the circular plate about an axis perpendicular to
the plate and passing through its edge is I = aMR2. The value of the coefficient a is :
[JEE (Main) - 2019, Online]
3 1 3 8
(1) (2) (3) (4)
2 2 5 5

27. A stationary horizontal disc is free to rotate about its axis. When a torque is applied on it, its kinetic energy
as a function of q, where q is the angle by which it has rotated, is given as kq2. If its moment of inertia is
I then the angular acceleration of the disc is : [JEE (Main) - 2019, Online]
k k k 2k
(1) q (2) q (3) q (4) q
2I I 4I I

28. A thin disc of mass M and radius R has mass per unit area s(r) = kr2 where r is the distance from its centre.
Its moment of inertia about an axis going through its centre of mass and perpendicular to its plane is :
[JEE (Main) - 2019, Online]

MR 2 MR 2 2MR 2 MR 2
(1) (2) (3) (4)
6 3 3 2

I1
29. Two coaxial discs, having moments of inertia I1 and , are rotating with respective angular velocities w1
2
251\D\Allen(IIT-JEE Wing)\2020-21\Nurture\Phy\Unit-03\02-Rigid Body Dynamics

w1
and , about their common axis. They are brought in contact with each other and thereafter they rotate
2
with a common angular velocity. If Ef and Ei are the final and initial total energies, then (Ef – Ei) is :
[JEE (Main) - 2019, Online]

I1w12 3 2 I1w12 I1w12


(1) (2) I1w1 (3) (4)
12 8 6 24

157
JEE-Physics
30. A metal coin of mass 5 g and radius 1 cm is fixed to a thin stick AB of negligible A
mass as shown in the figure. The system is initially at rest. The constant torque,
that will make the system rotate about AB at 25 rotations per second in 5 s, is close 1cm
to : [JEE (Main) - 2019, Online]
(1) 4.0 × 10–6 Nm (2) 2.0 × 10–5 Nm
(3) 1.6 × 10–5 Nm (4) 7.9 × 10–6 Nm B

31. A uniform rod of length l is being rotated in a horizontal plane with a constant angular speed about an axis
passing through one of its ends. If the tension generated in the rod due to rotation is T(x) at a distance x
from the axis, then which of the following graphs depicts it most closely? [JEE (Main) - 2019, Online]

T(x) T(x)

(1) (2)
x x
l l

T(x) T(x)

(3) (4)
x x
l l

32. A thin ring of 10 cm radius carries a uniformly distributed charge. The ring rotates at a constant angular speed
of 40 p rad s–1 about its axis, perpendicular to its plane. If the magnetic field at its centre is 3.8 × 10–9 T,
then the charge carried by the ring is close to (m0 = 4p × 10–7 N/A2) :
[JEE (Main) - 2019, Online]
(1) 2 × 10–6 C (2) 3 × 10–5 C (3) 4 × 10–5 C (4) 7 × 10–6 C

33. Mass per unit area of a circular disc of radius a depends on the distance r from its centre as
s (r) = A + Br. The moment of inertia of the disc about the the axis, perpendicular to the plane and passing
through its centre is : [JEE (Main) - 2020, Online]

æ A aB ö æA aB ö æ aA Bö æA Bö
+ + +
(1) 2pa4 ç + ÷ (2) pa4 çè 4 ÷
5 ø (3) 2pa4 çè 4 ÷
5ø (4) 2pa4 çè 4 ÷

è4 5 ø
m
34. As shown in the figure, a bob of mass m is tied by a massless string whose other
r
end portion is wound on a fly wheel (disc) of radius r and mass m. When released
from rest the bob starts falling vertically. When it has covered a distance of h, the
angular speed of the wheel will be : [JEE (Main) - 2020, Online]
m
251\D\Allen(IIT-JEE Wing)\2020-21\Nurture\Phy\Unit-03\02-Rigid Body Dynamics

1 2gh 3 1 4gh 3
(1) (2) r (3) (4) r
r 3 4gh r 3 2gh

35. A uniform sphere of mass 500 g rolls without slipping on a plane horizontal surface with its centre moving
at a speed of 5.00 cm/s. Its kinetic energy is : [JEE (Main) - 2020, Online]
(1) 8.75 × 10–4 J (2) 8.75 × 10–3 J (3) 6.25 × 10–4 J (4) 1.13 × 10–3 J

158
Rigid Body Dynamics
36. A uniformly thick wheel with moment of inertia I and radius R is free to rotate about /////////////////////
its centre of mass (see fig). A massless string is wrapped over its rim and two blocks
of masses m1 and m2 (m1 > m2) are attached to the ends of the string. The system
is released from rest. The angular speed of the wheel when m1 descents by a distance
h is : [JEE (Main) - 2020, Online]
1 1
é m1 + m 2 ù2 é 2(m 1 - m 2 )gh ù 2
(1) ê ú gh (2) ê ú
ë (m 1 + m 2 )R + I û ë (m 1 + m 2 )R + I û
2 2

m2
1 1
é 2(m 1 + m 2 )gh ù 2 é (m 1 - m 2 ) ù 2 m1
(3) ê ú (4) ê ú gh
ë (m 1 + m 2 )R + I û ë (m 1 + m 2 )R + I û
2 2
251\D\Allen(IIT-JEE Wing)\2020-21\Nurture\Phy\Unit-03\02-Rigid Body Dynamics

159
JEE-Physics

SECTION-1 : SINGLE CHOICE CORRECT QUESTIONS


1. A small particle of mass m is projected at an angle q with the x-axis with an initial velocity v0 in the x-y plane
v 0 sin q
as shown in the figure. At a time t < , the angular momentum of the particle is: [AIEEE - 2010]
g
1
(1) mg v0 t2 cos q $i (2) – mg v0 t2 cos q $j
2
1
(3) mg v0 t cos q k$ (4) – mg v0 t2 cos q k$
2

Where $i , $j and k$ are unit vectors along x, y and z-axis respectively..

2. A pulley of radius 2 m is rotated about its axis by a force F = (20t – 5t2) newton (where t is measured in
seconds) applied tangentially. If the moment of inertia of the pulley about its axis of rotation is 10 kg m 2, the
number of rotations made by the pulley before its direction of motion it reversed, is :- [AIEEE-2011]
(1) more than 6 but less than 9 (2) more than 9
(3) less than 3 (4) more than 3 but less than 6

3. A particle of mass 'm' is projected with a velocity v making an angle of 30° with the horizontal. The magnitude
of angular momentum of the projectile about the point of projection when the particle is at its maximum height
'h' is :- [AIEEE-2011]

3 mv 2 mv 3 3 mv 3
(1) (2) zero (3) (4)
2 g 2g 16 g

4. A thick-walled hollow sphere has outside radius R0. It rolls down an incline without slipping and its speed at the
bottom is V0. Now the incline is waxed, so that it is practically frictionless and the sphere is observed to slide
down (without any rolling). Its speed at the bottom is observed to be 5V 0/4. The radius of gyration of the
hollow sphere about an axis through its centre is :- [AIEEE - 2012, Online]
(1) 3R0/2 (2) 9R0/16 (3) 3R0/4 (4) 3R0

5. A stone of mass m, tied to the end of a string, is whirled around in a circle on a horizontal frictionless table. The
length of the string is reduced gradually keeping the angular momentum of the stone about the centre of the
circle constant. Then, the tension in the string is given by T = Ar n, where A is a constant, r is the instantaneous
radius of the circle. The value of n is equal to :- [AIEEE - 2012, Online]
(1) –3 (2) –1 (3) –4 (4) –2

6. A bullet of mass 10 g and speed 500 m/s is fired into a door and gets embedded exactly at the centre of the
door. The door is 1.0 m wide and weighs 12 kg. It is hinged at one end and rotates about a vertical axis
practically without friction. The angular speed of the door just after the bullet embeds into it will be :
[JEE (Main) - 2013, Online]
(1) 6.25 rad/sec (2) 0.625 rad/sec (3) 3.35 rad /sec (4) 0.335 rad/sec
251\D\Allen(IIT-JEE Wing)\2020-21\Nurture\Phy\Unit-03\02-Rigid Body Dynamics

7. A tennis ball (treated as hollow spherical shell) starting from


O rolls down a hill. At point A the ball becomes air borne leaving
at an angle of 30° with the horizontal. The ball strikes the ground
at B. What is the value of the distance AB ?
[JEE (Main) - 2013, Online]
(Moment of inertia of a spherical shell of mass m
and radius R about its diameter = 2/3 mR2) 30°
0.2m A B
(1) 2.08m (2) 1.87m
(3) 1.57m (4) 1.77m

160
Rigid Body Dynamics
8. A particle is moving in a circular path of radius a, with a constant velocity v as shown in the figure. The center
of circle is marked by 'C. The angular momentum from the origin O can be written as :-
[JEE (Main) - 2014, Online]
y

a
O q x
C

(1) va cos 2q (2) va (1 + cos q) (3) va (1 + cos 2q) (4) va

9. Consider a cylinder of mass M resting on a rough horizontal rug that is pulled out from under it with acceleration
'a' perpendicular to the axis of the cylinder. What is F friction at point P ? It is assumed that the cylinder does not
slip. [JEE (Main) - 2014, Online]

w
v
O

P
a
Ffriction

Ma Ma
(1) Mg (2) (3) Ma (4)
3 2

10. Consider a thin uniform square sheet made of a rigid material. If its side is 'a', mass m and moment of inertia
I about one of its diagonals, then :- [JEE (Main) - 2015, Online]

ma 2 ma 2 ma 2 ma 2 ma 2
(1) I > (2) I = (3) <I< (4) I =
12 24 24 12 12

11. From a solid sphere of mass M and radius R a cube of maximum possible volume is cut. Moment of inertia of
cube about an axis passing through its center and perpendicular to one of its faces is [JEE(Main) - 2015]

MR 2 MR 2 4MR 2 4MR 2
(1) (2) (3) (4)
32 2p 16 2p 9 3p 3 3p

12. A uniform solid cylindrical roller of mass 'm' is being pulled on a horizontal surface with force F parallel to the
surface and applied at its centre. If the acceleration of the cylinder is 'a' and it is rolling without slipping then the
value of 'F' is :- [JEE (Main) - 2015, Online]
5 3
251\D\Allen(IIT-JEE Wing)\2020-21\Nurture\Phy\Unit-03\02-Rigid Body Dynamics

(1) ma (2) ma (3) 2 ma (4) ma


3 2

13. The moment of inertia of a uniform cylinder of length l and radius R about its perpendicular bisector is I. What
is the ratio l/R such that the moment of inertia is minimum ? [JEE(Main) - 2017]

3 3 3
(1) (2) 1 (3) (4)
2 2 2

161
JEE-Physics
14. A uniform disc of radius R and mass M is free to rotate only about its axis. A M Wall
string is wrapped over its rim and a body of mass m is tied to the free end of the
string as shown in the figure. The body is released from rest. Then the acceleration
of the body is : [JEE (Main) - 2017, Online]
R
2mg 2Mg g¯
(1) (2)
2M + m 2M + m

2mg 2Mg m
(3) (4)
2m + M 2m + M

15. The machine as shown has 2 rods of length 1 m connected by a pivot at the top. The end of one rod is
connected to the floor by a stationary pivot and the end of the other rod has a roller that rolls along the floor in
a slot. As the roller goes back and forth, a 2 kg weight moves up and down. If the roller is moving towards right
at a constant speed, the weight moves up with a :- [JEE (Main) - 2017, Online]

3 2 kg
(1) speed which is th of that of the roller when the weight is 0.4 m
4
above the ground
(2) decreasing speed
(3) constant speed F Fixed pivot
x
(4) increasing speed
Movable roller

16. A slender unform rod of mass M and length l is pivoted at one end so that it can
rotate in a vertical plane (see figure.) There is neglifible friction at the pivot. The
free end is held vertically above the pivot and then released. The angular
acceleration of the rod when it make an angle q with the vertical is :
[JEE(Main) - 2017]
3g 3g
(1) sin q (2) cos q
3l 2l

2g 3g
(3) cos q (4) sin q
3l 2l

17. A particle of mass 20 g is released with an initial velocity 5 m/s along the
curve from the point A, as shown in the figure. The point A is at height h
from point B. The particle slides along the frictionless surface. When the
particle reaches point B, its angular momentum about O will be :
(Take g= 10 m/s2) [JEE (Main) - 2019, Online]
(1) 8kg-m2/s (2) 6kg-m2/s
(3) 3kg-m2/s (4) 2kg-m2/s

18. A string is wound around a hollow cylinder of mass 5 kg and radius 0.5 m. If the string is now pulled with a
251\D\Allen(IIT-JEE Wing)\2020-21\Nurture\Phy\Unit-03\02-Rigid Body Dynamics

horizontal force of 40 N, and the cylinder is rolling without slipping on a horizontal surface (see figure), then
the angular acceleration of the cylinder will be (Neglect the mass and thickness of the string) :-
[JEE (Main) - 2019, Online]
40N

(1) 12 rad/s2 (2) 16 rad/s2 (3) 10 rad/s2 (4) 20 rad/s2

162
Rigid Body Dynamics
19. To mop-clean a floor, a cleaning machine presses a circular mop of radius R vertically down with a total force
F and rotates it with a constant angular speed about its axis. If the force F is distributed uniformly over the mop
and if coefficient of friction between the mop and the floor is µ, the torque, applied by the machine on the mop
is : [JEE (Main) - 2019, Online]
2
(1) mFR (2) µFR/3 (3) µFR/2 (4) µFR/6
3

20. A rigid massless rod of length 3l has two masses attached at each end as shown in the figure. The rod is pivoted
at point P on the horizontal axis (see figure). When released from initial horizontal position, its instantaneous
angular acceleration will be : [JEE (Main) - 2019, Online]

g 7g
(1) (2) l 2l
2l 3l

g g 5 M0 P 2 M0
(3) (4)
13l 3l

21. An L-shaped object, made of thin rods of uniform mass density, is suspended
A
with a string as shown in figure. If AB = BC, and the angle made by AB with
q z
downward vertical is q, then : [JEE (Main) - 2019, Online]
2 1
(1) tan q = (2) tan q = B 90º x
3 3

1 1
(3) tan q = (4) tan q = C
2 2 3

22. A rod of length 50cm is pivoted at one end. It is raised such that if makes an angle of 30° from the horizontal
as shown and released from rest. Its angular speed when it passes through the horizontal (in rad s–1) will
be (g = 10ms–2) [JEE (Main) - 2019, Online]

30
(1) 30 (2)
2
30 20 30°
(3) (4)
2 3

23. A solid sphere and solid cylinder of identical radii approach an incline with the same linear velocity (see figure).
Both roll without slipping all throughout. The two climb maximum heights hsph and hcyl on the incline. The
h sph
ratio h is given by : [JEE (Main) - 2019, Online]
cyl

14 4
(1) (2)
15 5
2
251\D\Allen(IIT-JEE Wing)\2020-21\Nurture\Phy\Unit-03\02-Rigid Body Dynamics

(3) 1 (4)
5

24. A circular disc of radius b has a hole of radius a at its centre (see figure). If the
b
æ s0 ö
mass per unit area of the disc varies as ç ÷ , then the radius of gyration of the a
è r ø
disc about its axis passing through the centre is : [JEE (Main) - 2019, Online]

a+b a+b a 2 + b2 + ab a 2 + b2 + ab
(1) (2) (3) (4)
2 3 2 3

163
JEE-Physics

25. A rectangular solid box of length 0.3 m is held horizontally, with one of its sides l
on the edge of a platform of height 5m. When released, it slips off the table
in a very short time t = 0.01s, remaining essentially horizontal. The angle by
which it would rotate when it hits the ground will be (in radians) close to :
h
[JEE (Main) - 2019, Online]
(1) 0.02 (2) 0.28
(3) 0.5 (4) 0.3

26. The following bodies are made to roll up (without slipping) the same inclined plane from a horizontal plane.
R R
: (i) a ring of radius R, (ii) a solid cylinder of radius and (iii) a solid sphere of radius . If in each case,
2 4
the speed of the centre of mass at the bottom of the incline is same, the ratio of the maximum heights they
climb is : [JEE (Main) - 2019, Online]
(1) 4 : 3 : 2 (2) 14 : 15 : 20 (3) 10 : 15 : 7 (4) 2 : 3 : 4

27. Moment of inertia of a body about a given axis is 1.5 kg m 2. Initially the body is at rest. In order to produce
a rotational kinetic energy of 1200 J, the angular accleration of 20 rad/s2 must be applied about the axis for
a duration of : [JEE (Main) - 2019, Online]
(1) 2 s (2) 5s (3) 2.5 s (4) 3 s

28. A thin smooth rod of length L and mass M is rotating freely with angular speed w0 about an axis perpendicular
to the rod and passing through its center. Two beads of mass m and negligible size are at the center of the
rod initially. The beads are free to slide along the rod. The angular speed of the system , when the beads
reach the opposite ends of the rod, will be : [JEE (Main) - 2019, Online]
Mw0 Mw0 Mw0 Mw0
(1) (2) (3) (4)
M + 3m M+m M + 2m M + 6m

29. Consider a uniform rod of mass M = 4m and length l pivoted about its centre. A mass m moving with velocity

p
v making angle q = to the rod's long axis collides with one end of the rod and sticks to it. The angular
4
speed of the rod-mass system just after the collision is : [JEE (Main) - 2020, Online]
3 v 3 2v 4v 3v
(1) (2) (3) (4)
7 2 l 7 l 7l 7l

30. Three solid spheres each of mass m and diameter d are stuck together such that B A
the lines connecting the centres form an equilateral triangle of side of length d. d
+
The ratio I0/IA of moment of inertia I0 of the system about an axis passing the centroid
and about center of any of the spheres IA and perpendicular to the plane of the O
C
triangle is : [JEE (Main) - 2020, Online]
13 15 23 13
251\D\Allen(IIT-JEE Wing)\2020-21\Nurture\Phy\Unit-03\02-Rigid Body Dynamics

(1) (2) (3) (4)


23 13 13 15

l
31. The radius of gyration of a uniform rod of length l, about an axis passing through a point away from the
4
centre of the rod, and perpendicular to it, is : [JEE (Main) - 2020, Online]

1 7 3 1
(1) l (2) l (3) l (4) l
8 48 8 4

164
Rigid Body Dynamics
SECTION-2 : NUMERICAL ANSWER BASED QUESTIONS
32. Consider a uniform cubical box of side a on a rough floor that is to be moved by
applying minimum possible force F at a point b above its centre of mass (see figure). F
b
If the coefficient of friction is m = 0.4, the maximum possible value of 100 ×
a
for a box not to topple before moving is ______ .
[JEE (Main) - 2020, Online]
251\D\Allen(IIT-JEE Wing)\2020-21\Nurture\Phy\Unit-03\02-Rigid Body Dynamics

165
JEE-Physics

Comprehension-1*. (1 to 3)
A uniform thin cylindrical disk of mass M and radius R is attached to
two identical massless springs of spring constant k which are fixed to
the wall as shown in the figure. The springs are attached to the axle
of the disk symmetrically on either side at a distance d from its centre.
The axle is massless and both the springs and the axle are in a
horizontal plane. The unstretched length of each spring is L. The disk
is initially at its equilibrium position with its centre of mass (CM) at
a distance L from the wall. The disk rolls without slipping with velocity
r r
v 0 = v 0ˆi . The coefficient of friction is µ. [IIT-JEE 2008]

1. The net external force acting on the disk when its centre of mass is at displacement x with respect to its
equilibrium position is
2kx 4kx
(A) – kx (B) – 2kx (C) - (D) -
3 3

2. The centre of mass of the disk undergoes simple harmonic motion with angular frequency w equal to

k 2k 2k 4k
(A) (B) (C) (D)
M M 3M 3M

3. The maximum value of v0 for which the disk will roll without slipping is

M M 3M 5M
(A) µg (B) µg (C) µg (D) µg
k 2k k 2k

*4. A block of base 10 cm × 10 cm and height 15 cm is kept on an inclined plane. The coefficient of friction
between them is 3 . The inclination q of this inclined plane from the horizontal plane is gradually increased
from 0°. Then :- [IIT-JEE 2009]
(A) at q =30°, the block will start sliding down the plane
(B) the block will remain at rest on the plane up to certain q and then it will topple
(C) at q=60°, the block will start sliding down the plane and continue to do so at higher angles
(D) at q=60°, the block will start sliding down the plane and on further increasing q, it will topple at certain q

5. If the resultant of the external forces acting on a system of particles is zero, then from an inertial frame, one can
surely say that [IIT-JEE 2009]
(A) linear momentum of the system does not change in time
(B) kinetic energy of the system does not change in time
251\D\Allen(IIT-JEE Wing)\2020-21\Nurture\Phy\Unit-03\02-Rigid Body Dynamics

(C) angular momentum of the system does not change in time


(D) potential energy of the system does not change in time

C
6. A sphere is rolling without slipping on a fixed horizontal plane surface. In the
figure, A is the point of contact, B is the centre of the sphere and C is its topmost
point. Then [IIT-JEE 2009] B
r r r r r r r r
(A) v C - v A = 2(v B - v C ) (B) v C - v B = v B - v A
A
r r r r r r r
(C) v C - v A = 2 v B - v C (D) v C - v A = 4 v B

166
Rigid Body Dynamics
*7. A thin ring of mass 2 kg and radius 0.5 m is rolling without slipping on a
horizontal plane with velocity 1 m/s. A small ball of mass 0.1 kg, moving
with velocity 20 m/s in the opposite direction, hits the ring at a height of
0.75 m and goes vertically up with velocity 10 m/s. Immediately after
the collision : [IIT-JEE 2011]
(A) the ring has pure rotation about its stationary CM.
(B) the ring comes to a complete stop.
(C) friction between the ring and the ground is to the left.
(D) there is no friction the ring and the ground.

*8. The figure shows a system consisting of (i) a ring of outer radius 3R rolling clockwise without slipping on a
horizontal surface with angular speed w and (ii) an inner disc of radius 2R rotating anti-clockwise with angular
speed w/2. The ring and disc are separated by frictionless ball bearings. The system is in the x-z plane. The
point P on the inner disc is at a distance R from the origin, where OP makes an angle of 30º with the horizontal.
Then with respect to the horizontal surface. [IIT-JEE 2012]

)
(A) The point O has a linear velocity 3Rw i .

11 ) 3 )
(B) The point P has a linear velocity Rw i + Rw k
4 4
13 ) 3 )
(C) The point P has a linear velocity Rw i – Rwk
4 4
æ 3ö ) 1 )
(D) The point P has a linear velocity ç 3 – ÷ Rw i + 4 Rwk
è 4 ø

9. A thin uniform rod, pivoted at O, is rotating in the horizontal plane with constant
angular speed w, as shown in figure. At time t = 0, a small insect starts from O and
Z
moves with constant speed n with respect to the rod towards the other end. It w
reaches the end of the rod at t = T and stops. The angular speed of the system
®
remains w throughout. The magnitude of the torque (| t |) on the system about O,,
v
as a function of time is best represented by which plot ? [IIT-JEE 2012]
251\D\Allen(IIT-JEE Wing)\2020-21\Nurture\Phy\Unit-03\02-Rigid Body Dynamics

(A) (B)

(C) (D)

167
JEE-Physics
*10. A small mass m is attached to a massless string whose other end is fixed at P as shown in the
figure. The mass is undergoing circular motion If the x-y plane with centre at O and constant
angular speed w. If the angular momentum of the system , calculated about O and P are
® ®
denoted by L O and L P respectively, then [IIT-JEE 2012]
® ®
(A) L O and L P do not vary with time
® ®
(B) L O varies with time while L P remains constant
® ®
(C) L O remains constant while L P vaires with time
® ®
(D) L O and L P both vary with time

11. Two solid cylinders P and Q of same mass and same radius start rolling down a fixed inclined plane from the
same height at the same time. Cylinder P has most of its mass concentrated near its surface, while Q has most
of its mass concentrated near the axis. Which statement(s) is(are) correct? [IIT-JEE 2012]
(A) Both cylinders P and Q reach the ground at the same time.
(B) Cylinder P has larger linear acceleration than cylinder Q.
(C) Both cylinders reach the ground with same translational kinetic energy.
(D) Cylinder Q reaches the ground with larger angular speed.

Comprehension-2*. (12 to 13)


The general motion of a rigid body can be considered to be a combination of (i) a motion of its center of mass
about an axis, and (ii) its motion about an instantaneous axis passing through the centre of mass. These axes
need not be stationary. Consider for example, a thin uniform disc welded (rigidly fixed) horizontally at its rim
to a massless stick, as shown in the figure. When the disc-stick system is rotated about the origin on a horizon-
tal firctionless plane with angular speed w, the motion at any instant can be taken as a combination of (i) a
rotation of the center of mass of the disc about the z-axis and (ii) a rotation of the disc through an instanta-
neous vertical axis passing through its centre of mass (as is seen from the changed orientaion of points P and
Q). Both these motions have the same angular speed w in this case.

Now consider two similar systems as shown in the figure: Case (a) the disc with face vertical and parallel to x-
z plane; Case (b) the disc with its face making an angle of 45° with x-y plane and its horizontal diameter
parallel. to x-axis, in both the cases, the disc is welded at point P, and the systems are rotated with constant
angular speed w about the z-axis.
251\D\Allen(IIT-JEE Wing)\2020-21\Nurture\Phy\Unit-03\02-Rigid Body Dynamics

[IIT-JEE 2012]
168
Rigid Body Dynamics
12. Which of the following statements regarding the angular speed about the instantaneous axis (passing through
the centre of mass) is correct ?
(A) It is 2w for both the cases.
w
(B) It is w for case (a) ; and for case (b).
2
(C) It is w for case (a) ; and 2w for case (b).
(D) It is w for both the cases.

13. Which of the following statements about the instantaneous axis (passing through the centre of mass) is correct ?
(A) It is vertical for both the cases (a) and (b).
(B) It is vertical for case (a) ; and is at 45º to the x-z plane and lies in the plane of the disc for case (b).
(C) It is horizontal for case (a) ; and is at 45º to the x-z plane and is normal to the plane of the disc for case (b).
(D) It is vertical for case (a) ; and is at 45º to the x-z plane and is normal to the plane of the disc for case (b).

14. A lamina is made by removing a small disc of diameter 2R from a bigger disc
of uniform mass density and radius 2R, as shown in the figure. The moment 2R
of inertia of this lamina about axes passing through O and P is I0 and Ip,
respectively. Both these axes are perpendicular to the plane of the lamina. P
O 2R
Ip
The ratio to the nearest integer is : [IIT-JEE 2012]
Io

15. A uniform circular disc of mass 50 kg and radius 0.4 m is rotating with an angular velocity of
10 rad s–1 about its own axis, which is vertical. Two uniform circular rings, each of mass 6.25 kg and radius
0.2 m, are gently placed symmetrically on the disc in such a manner that they are touching each other
along the axis of the disc and are horizontal. Assume that the friction is large enough such that the rings
are at rest relative to the disc and the system rotates about the original axis. The new angular velocity (in
rad s–1) of the system is [IIT-JEE 2013]

*16. In the figure, a ladder of mass m is shown leaning against a wall. It is in static equilibrium kaing an angle q with
the horizontal floor. The coefficient of friction between the wall and the ladder is m1 and that between the floor
and the ladder is m2. The normal reaction of the wall on the ladder is N 1 and that of the floor is N 2. If the ladder
is about to slip, then [JEE(Adv.) 2014]
mg
(A) m1 = 0 m2 ¹ 0 and N2 tan q = N1 m
2 1
mg
(B) m1 ¹ 0 m2 = 0 and N1 tan q =
2
mg
(C) m1 ¹ 0 m2 ¹ 0 and N2 = 1 + m m N2
1 2 q
mg m2
(D) m1 = 0 m2 ¹ 0 and N1 tan q =
2
251\D\Allen(IIT-JEE Wing)\2020-21\Nurture\Phy\Unit-03\02-Rigid Body Dynamics

17. A horizontal circular platform of radius 0.5m and mass 0.45 kg is free to
rotate about its axis. Two massless spring toy-guns, each carrying a steel
ball of mass 0.05 kg are attached to the platform at a distance 0.25 m from
the centre on its either sides along its diameter (see figure). Each gun
simultaneously fires the balls horizontally and perpendicular to the diameter
in opposite directions. After leaving the platform, the balls have horizontal
speed of 9 ms–1 with respect to the ground. The rotational speed of the
platform in rad s–1 after the balls leave the platform is– [JEE(Adv.) 2014]

169
JEE-Physics

18. A ring of mass M and radius R is rotating with angular speed w about a fixed vertical
M w
axis passing through its centre O with two point masses each of mass at rest at O..
8
These masses can move radially outwards along two massless rods fixed on the ring
8
as shown in the figure. At some instant the angular speed of the system is w and
9
O
3
one of the masses is at a distance of R from O. At this instant the distance of the
5
other mass from O is [JEE(Adv.) 2015]
2 1 3 4
(A) R (B) R (C) R (D) R
3 3 5 5

19. Two identical uniform discs roll without slipping on two different surfaces AB and CD (see figure) starting at A
and C with linear speeds v1 and v2, respectively, and always remain in contact with the surfaces. If they reach
B and D with the same linear speed and v 1 = 3 m/s, then v2 in m/s is (g = 10 m/s2) [JEE(Adv.) 2015]

v1=3 m/s
A

30m

C v2

27m

æ rö
20. The densities of two solid spheres A and B of the same radii R vary with radial distance r as rA (r) = k çè ÷ and

5
æ rö
rB (r) = k ç ÷ , respectively, where k is a constant. The moments of inertia of the individual spheres about
è Rø

IB n
axes passing through their centres are I A and IB, respectively. If I = 10 , the value of n is
A

[JEE(Adv.) 2015]

A uniform wooden stick of mass 1.6 kg and length l rests in an inclined manner on a smooth, vertical wall of
251\D\Allen(IIT-JEE Wing)\2020-21\Nurture\Phy\Unit-03\02-Rigid Body Dynamics

21.
height h (<l) such that a small portion of the stick extends beyond the wall. The reaction force of the wall on
the stick is perpendicular to the stick. The stick makes an angle of 30° with the wall and the bottom of the stick
is on a rough floor. The reaction of the wall on the stick is equal in magnitude to the reaction of the floor on the
stick. The ratio h/l and the frictional force f at the bottom of the stick are [JEE(Adv.) 2016]

h 3 16 3 h 3 16 3
(A) = ,f = N (B) = ,f = N
l 16 3 l 16 3

h 3 3 8 3 h 3 3 16 3
(C) = ,f = N (D) = ,f = N
l 16 3 l 16 3

170
Rigid Body Dynamics

® ®
22. The position vector r of a particle of mass m is given by the following equation r (t) = at 3ˆi + bt 2ˆj ,
where a = 10/3 ms–3, b = 5 ms–2 and m = 0.1 kg. At t = 1 s, which of the following statement (s) is (are) true
about the particle ? [JEE(Adv.) 2016]
® ®
(A) The velocity v is given by v = (10iˆ + 10ˆj)ms -1
® ®
(B) The angular momentum L with respect to the origin is given by L = -5(5 / 3)kˆ N ms
® ®
(C) The force F is given by F = (iˆ + 2j)N
ˆ
® ®
(D) The torque t with respect to the origin is given by t = -(20 / 3)kˆ N m

23. Two thin circular discs of mass m and 4m, having


radii of a and 2a, respectively, are rigidly fixed by a
massless, rigid rod of length l = 24a through their
centers. This assembly is laid on a firm and flat
surface, and set rolling without slipping on the
surface so that the angular speed about the axis of
the rod is w. The angular momentum of the entire
®
assembly about the point 'O' is L (see the figure).
Which of the following statement(s) is (are) true ?
[JEE(Adv.) 2016]

(A) The magnitude of angular momentum of the assembly about its center of mass is ma 2w/2
®
(B) The magnitude of the z-component of L is 55 ma2w
(C) The magnitude of angular momentum of center of mass of the assembly about the point O is 81 ma 2w
(D) the center f mass of the assembly rotates about the z-axis with an angular speed of w/5

Comprehension-3. (24 to 25)


A frame of reference that is accelerated with respect to an inertial frame of reference is called a non-inertial
frame of reference. A coordinate system fixed on a circular disc rotating about a fixed axis with a constant
®
angular velocity w is an example of a non-inertial frame of reference. The relationship between the force F rot
®
experienced by a particle of mass m moving on the rotating disc and the force F in experienced by the particle
in an inertial frame of reference is
® ® ® ® ® ® ®
F rot = F in + 2m(v rot ´ w) + m(w ´ r ) ´ w
® ®
where v rot is the velocity of the particle in the rotating frame of reference and r is the position vector of the
particle with respect to the centre of the disc.
Now consider a smooth slot along a diameter of a disc of radius R rotating counter-clockwise with a constant
angular speed w about its vertical axis through its center. We assign a coordinate system with the origin at the
center of the disc, thex-axis along the slot, the y-axis perpendicular to the slot and the z-axis along the rotation
251\D\Allen(IIT-JEE Wing)\2020-21\Nurture\Phy\Unit-03\02-Rigid Body Dynamics

® ®
ˆ . A small block of mass m is gently placed in the slot at r = (R / 2)iˆ at t = 0 and is constrained to
axis (w = w k)
move only along the slot. [JEE(Adv.) 2016]

171
JEE-Physics

24. The distance r of the block at time t is


R 2 wt R R R wt
(A) (e + e -2wt ) (B) cos 2 wt (C) cos wt (D) (e + e -wt )
4 2 2 4

25. The net reaction of the disc on the block is


(A) - mw 2R cos wt ˆj - mg kˆ (B) mw 2R sin wt ˆj - mg kˆ

1 1
(C) mw 2R(e wt - e -wt )jˆ + mg kˆ (D) mw 2R(e 2wt - e -2wt )jˆ + mg kˆ
2 2
S
26. A wheel of radius R and mass M is placed at the bottom of a fixed
step of height R as shown in the figure. A constant force is continuously Q
P
applied on the surface of the wheel so that it just climbs the step
R
without slipping. Consider the torque t about an axis normal to the X
plane of the paper passing through the point Q. Which of the following
options is/are correct? [JEE (Adv.) 2017]

(A) If the force is applied normal to the circumference at point X then t is constant
(B) If the force is applied tangentially at point S then t ¹ 0 but the wheel never climbs the step
(C) If the force is applied normal to the circumference at point P then t is zero
(D) If the force is applied at point P tangentially then t decreases continuously as the wheel climbs

Comprehension-4. (27 to 28)


One twirls a circular ring (of mass M and radius R)
near the tip of one's finger as shown in Figure 1. In
the process the finger never loses contact with the
inner rim of the ring. The finger traces out the surface
of a cone, shown by the dotted line. The radius of the
path traced out by the point where the ring and the
finger is in contact is r. The finger rotates with an
angular velocity w0. The rotating ring rolls without
slipping on the outside of a smaller circle described
by the point where the ring and the finger is in
contact (Figure 2). The coefficient of friction between
251\D\Allen(IIT-JEE Wing)\2020-21\Nurture\Phy\Unit-03\02-Rigid Body Dynamics

the ring and the finger is m and the acceleration due


to gravity is g. [JEE (Adv.) 2017]

27. The total kinetic energy of the ring is


1 3
(A) Mw02R 2 (B) Mw02 (R - r)2 (C) Mw02 (R - r)2 (D) Mw02 (R - r)2
2 2

28. The minimum value of w0 below which the ring will drop down is
3g g 2g g
(A) (B) (C) (D)
2m(R - r) m(R - r) m(R - r) 2m(R - r)

172
Rigid Body Dynamics
29. A rigid uniform bar AB of length L is slipping from its vertical position on a frictionless floor (as shown in the
figure). At some instant of time, the angle made by the bar with the vertical is q. Which of the following
statements about its motion is/are correct ? [JEE(Adv.) 2017]

(A) When the bar makes an angle q with the vertical, the displacement of its midpoint from the initial position
is proportional to (1 – cos q)
(B) The midpoint of the bar will fall vertically downward
(C) Instantaneous torque about the point in contact with the floor is proportional to sin q
(D) The trajectory of the point A is a parabola

30. The potential energy of a particle of mass m at a distance r from a fixed point O is given by V(r)= kr 2/2, where
k is a positive constant of appropriate dimensions. This particle is moving in a circular orbit of radius R about
the point O. If v is the speed of the particle and L is the magnitude of its angular momentum about O, which of
the following statements is (are) true? [JEE (Adv.) - 2018]

k k mk 2
(A) v = R (B) v = R (C) L = mk R 2 (D) L = R
2m m 2
®
31. Consider a body of mass 1.0 kg at rest at the origin at time t = 0. A force F = (atiˆ + bˆj) is applied on the body,,
®
where a = 1.0 Ns–1 and b = 1.0 N. The torque acting on the body about the origin at time t = 1.0s is t .
Which of the following statements is (are) true ? [JEE (Adv.) - 2018]
® 1
(A) | t |= Nm
3
®
(B) The torque t is in the direction of the unit vector + k̂

® 1 ˆ ˆ -1
(C) The velocity of the body at t = 1s is v = (i + 2 j)ms
2
1
(D) The magnitude of displacement of the body at t = 1s is m
6

32. A ring and a disc are initially at rest, side by side, at the top of an inclined plane which makes an angle 60º with
the horizontal. They start to roll without slipping at the same instant of time along the shortest path. If the time
difference between their reaching the ground is ( 2 - 3 ) / 10 s , then the height of the top of the inclined
251\D\Allen(IIT-JEE Wing)\2020-21\Nurture\Phy\Unit-03\02-Rigid Body Dynamics

plane, in metres, is _____. Take g = 10 m/s–2. [JEE (Adv.) - 2018]

33. In the List-I below, four different paths of a particle are given as functions of time. In these functions, a and b
are positive constants of appropriate dimensions and a ¹ b. In each case, the force acting on the particle is
®
either zero or conservative. In List-II, five physical quantities of the particle are mentioned: p is the linear
®
momentum, L is the angular momentum about the origin, K is the kinetic energy, U is the potential energy
and E is the total energy. Match each path in List-I with those quantities in List-II, which are conserved for
that path. [JEE (Adv.) - 2018]

173
JEE-Physics
LIST–I LIST-II
® ®
P. r (t) = at ˆi + bt ˆj 1. p

® ®
Q. r (t) = a cos wt ˆi + b sin wt ˆj 2. L
®
R. r (t) = a (cos wt ˆi + sin wt ˆj) 3. K

® b
S. r (t) = at ˆi + t 2ˆj 4. U
2
5. E

(A) P ® 1, 2, 3, 4, 5; Q ® 2, 5; R ® 2, 3, 4, 5; S®5
(B) P ® 1, 2, 3, 4, 5; Q ® 3, 5; R ® 2, 3, 4, 5; S ® 2, 5
(C) P ® 2, 3, 4; Q ® 5; R ® 1, 2, 4; S ® 2, 5
(D) P ® 1, 2, 3, 5; Q ® 2, 5; R ® 2, 3, 4, 5; S ® 2, 5

34. A thin and uniform rod of mass M and length L is held vertical on a floor with large friction. The rod is
released from rest so that it falls by rotating about its contact-point with the floor without slipping. Which
of the following statement(s) is/are correct, when the rod makes an angle 60º with vertical ?
[g is the acceleration due to gravity] [JEE (Adv.) - 2019]

3g
(A) The radial acceleration of the rod's center of mass will be
4
2g
(B) The angular acceleration of the rod will be
L

3g
(C) The angular speed of the rod will be
2L

Mg
(D) The normal reaction force from the floor on the rod will be
16

251\D\Allen(IIT-JEE Wing)\2020-21\Nurture\Phy\Unit-03\02-Rigid Body Dynamics

174
Rigid Body Dynamics

ANSWERS
EXERCISE-1
Que. 1 2 3 4 5 6 7 8 9 10 11 12 13 14 15
Ans. C A B B D B C A A B A C C B A
Que. 16 17 18 19 20 21 22 23 24 25 26 27 28 29
Ans. C D B D A C A A D C A C D D

EXERCISE-2
l MULTIPLE CHOICE CORRECT QUESTIONS
1. (BC) 2. (AB) 3. (AC) 4. (CD) 5. (A) 6. (ABC) 7. (AD) 8. (AC)
9. (ABC) 10. (AC) 11. (ABCD)

l COMPREHENSION BASED QUESTIONS


12. (C) 13. (C) 14. (D) 15. (A) 16. (C) 17. (B) 18. (C) 19. (D)
20. (B) 21. (B) 22. (C) 23. (B) 24. (A)

EXERCISE-3
l NUMERICAL ANSWER BASED QUESTIONS
1. (3) 2. (2) 3. (6) 4. (17) 5. (3) 6. (1.2) 7. (2) 8. (2)
9. (7) 10. (2)

l MATRIX MATCH QUESTIONS


11. (A) Q, (B) P, (C) S (D) r 12. (A) R; (B) S; (C) Q; (D) P
13. (A) Q; (B) R; (C) P; (D) S 14. (A) S,P; (B) P; (C) S,R; (D) S,Q

EXERCISE-4(A)
Que. 1 2 3 4 5 6 7 8 9 10 11 12 13 14 15
Ans. 4 2 1 1 1 2 2 1 2 3 3 4 4 4 2
Que. 16 17 18 19 20 21 22 23 24 25 26 27 28 29 30
Ans. 4 1 3 2 4 4 2 1 3 3 4 4 3 4 2
Que. 31 32 33 34 35 36
Ans. 4 2 1 3 1 2

EXERCISE-4(B)
l SINGLE CHOICE CORRECT QUESTIONS
Que. 1 2 3 4 5 6 7 8 9 10 11 12 13 14 15
Ans. 4 4 4 3 1 2 2 3 2 4 3 2 4 3 2
Que. 16 17 18 19 20 21 22 23 24 25 26 27 28 29 30
Ans. 4 2 2 1 3 2 1 1 4 3 2 1 4 2 1
Que. 31
251\D\Allen(IIT-JEE Wing)\2020-21\Nurture\Phy\Unit-03\02-Rigid Body Dynamics

Ans. 2
l NUMERICAL ANSWER BASED QUESTIONS
32. (75)

EXERCISE-5
1. (D) 2. (D) 3. (C) 4. (B) 5. (A) 6. (BC) 7. (C) 8. (AB)
9. (B) 10. (C) 11. (D) 12. (D) 13. (A) 14. (3) 15. (8) 16. (CD)
17. (4) 18. (D) 19. (7) 20. (6) 21. (D) 22. (ABD) 23. (AD) 24. (D)
25. (C) 26. (C) 27. (A) 28. (B) 29. (ABC) 30. (BC) 31. (AC) 32. (0.75)
33. (A) 34. (ACD)

175
176
JEE-Physics

IMPORTANT NOTES

251\D\Allen(IIT-JEE Wing)\2020-21\Nurture\Phy\Unit-03\02-Rigid Body Dynamics


Centre of Mass

Not To Be Discussed in Class

SECTION - 1 : SINGLE CHOICE CORRECT QUESTIONS


r r
1. Two particles A and B start moving due to their mutual interaction only. If at any time 't', a A and a B are their
r r
respective accelerations, v A and v B are their respective velocities, and upto that time W A and WB are the
work done on A and B respectively by the mutual force, m A and mB are their masses respectively, then which
of the following is always correct.
r r r r r r
(A) v A + v B = 0 (B) m A v A + mB v B = 0 (C) WA + WB = 0 (D) a A + aB = 0

2. A small ball falling vertically downward with constant velocity 4m/s strikes
4m/s
elastically a massive inclined cart moving with velocity 4m/s horizontally as
shown. The velocity of the rebound of the ball is
4m/s
45°
(A) 4 2 m/s (B) 4 3 m/s
(C) 4 m/s (D) 4 5 m/s

3. On a smooth carom board, a coin moving in negative y–direction with a speed y


of 3 m/s is being hit at the point (4, 6) by a striker moving along negative x–axis.
The line joining centres of the coin and the striker just before the collision is
parallel to x–axis. After collision the coin goes into the hole located at the origin. (4,6) u
coin
Masses of the striker and the coin are equal. Considering the collision to be 3m/s
elastic, the initial and final speeds of the striker in m/s will be–
(0,0) x
(A) (1.2, 0) (B) (2, 0)
(C) (3, 0) (D) None of these

4. A semicircular portion of radius ‘r’ is cut from a uniform rectangular plate as shown in figure. The distance
of centre of mass 'C' of remaining plate, from point ‘O’ is :
2r 3r
(A) (B)
(3 - p) 2(4 - p)

2r 2r
(C) (D)
(4 + p) 3(4 - p)

5 ABC is a part of ring having radius R2 and ADC is a part of disc having inner radius R1 and outer R 2. Part
ABC and ADC have same mass. Then center of mass will be located, from the centre O.
(R 2 – R1 )(2R1 + R 2 )
(A) (above)
3p(R1 + R 2 )

(R 2 – R1 )(2R1 + R 2 )
(B) (below)
3p(R1 + R 2 )

2R1 + R 2
251\D\Allen(IIT-JEE Wing)\2020-21\Nurture\Phy\Unit-03\YLRE\01-Centre of Mass

(C) (above)
3p
2R1 + R 2
(D) (below)
3p

6. A thin uniform wire is bent to form the two equal sides AB and AC of triangle ABC, where AB = AC = 5 cm.
The third side BC, of length 6cm, is made from uniform wire of twice the density of the first. The distance
of centre of mass from A is :
34 11 34 11
(A) cm (B) cm (C) cm (D) cm
11 34 9 45

177
JE E-Physics
7. A railway flat car has an artillery gun installed on it. The combined system has a mass M and moves with
a velocity v 0. The barrel of the gun makes an angle a with the horizontal. A shell of mass m leaves the
barrel at a speed 'u' relative to barrel in the forward direction. The speed of the flat car so that it may stop
after the firing is

mu æ Mu ö æ mu ö
(A) (B) çè ÷ cos a (C) çè ÷ cos a (D) (M + m) u cos a
M+m M + mø M ø

8. A particle of mass 2 m is projected at an angle of 45° with horizontal with a velocity of 20 2 m/s. After 1 s
explosion takes place and the particle is broken into two equal pieces. As a result of explosion one part comes to
rest. Find the maximum height attained by the other part. Take g = 10 m/s 2.
(A) 25 m (B) 50 m (C) 15 m (D) 35 m

9. A hemisphere of mass 3m and radius R is free to slide with its base on a


smooth horizontal table. A particle of mass m is placed on the top of the
hemisphere. If particle is displaced with a negligible velocity, then find the
angular velocity of the particle relative to the centre of the hemisphere at
an angular displacement q, when velocity of hemisphere is v :
4v 3v 5v 2v
(A) (B) (C) (D)
R cos q R cos q R cos q R cos q

10. Three balls A, B and C (mA = mC = 4mB) are placed on a smooth horizontal surface. Ball B collides with ball
C with an initial velocity v as shown in the figure. Total number of collisions between the balls will be (All
collisions are elastic) v
(A) One (B) Two A B C
(C) Three (D) Four

11. A small ball moves towards right with a velocity V. It collides with the wall and returns back and continues
æ2ö
to and fro motion. If the average speed for first to and fro motion of the ball is ç 3 ÷ V, then the coefficient
è ø
of restitution of impact is :

(A) 0.5 (B) 0.8 (C) 0.25 (D) 0.75

12. Three blocks are placed on smooth horizontal surface and lie on same horizontal straight line. Block 1 and block
3 have mass m each and block 2 has mass M (M > m). Block 2 and block 3 are initially stationary, while
block 1 is initially moving towards block 2 with speed v as shown. Assume that all collisions are headon and
perfectly elastic. What value of M/m ensures that block 1 and block 3 have the same final speed?
(A) 5 + 2 (B) 5 – 2

(C) 2 + 5 (D) 3 + 5
251\D\Allen(IIT-JEE Wing)\2020-21\Nurture\Phy\Unit-03\YLRE\01-Centre of Mass

13. Two identical balls A and B lie on a smooth horizontal surface, which gradually
merges into a curve to a height 3.2 m. Ball A is given a velocity 10 m/sec to
collide head on with ball B, which then takes up the curved path. The
minimum coefficient of restitution 'e' for the collision between A and B, in
order that B reaches the highest point C of curve. (g = 10 m/sec 2)
1 3 1 3
(A) (B) (C) (D)
2 5 4 4

178
Centre of Mass
14. A striker is shot from a square carrom board from a point A exactly at midpoint of
one of the walls with a speed 2 m/sec at an angle of 45° with the x-axis as shown.
The collisions of the striker with the walls of the fixed carrom are perfectly elastic.
The coefficient of kinetic friction between the striker and board is 0.2. The coordinate
of the striker when it stops (taking point O to be the origin) is :
1 1 1
(A) , (B) 0,
2 2 2 2 2

1 1 1
(C) ,0 (D) ,
2 2 2 2 2

15. Two identical particles move towards each other with velocity 2v and v respectively. The velocity of centre of
mass is-
(A) v (B) v/3 (C) v/2 (D) zero

16. Consider the following two statements :-


A : Linear momentum of a system of particles is zero.
B : Kinetic energy of a system of particles is zero.
Then-
(A) A does not imply B and B does not imply A (B) A implies B but B does not imply A
(C) A does not imply B but B implies A (D) A implies B and B implies A

17. Two spherical bodies of mass M and 5M and radii R and 2R respectively are released in free space with initial
separation between their centres equal to 12 R. It they attract each other due to gravitational force only, then the
distance covered by the smaller body just before collision is-
(A) 2.5 R (B) 4.5 R (C) 7.5 R (D) 1.5 R

18. A body A of mass M while falling vertically downwards under gravity breaks into two parts; a body B of mass
1 2
M and, a body C of mass M. The centre of mass of bodies B and C taken together shifts compared to that
3 3
of body A towards-
(A) depends on height of breaking (B) does not shift
(C) body C (D) body B

19. The block of mass M moving on the frictionless horizontal surface collides with the spring of spring constant k
and compresses it by length L. The maximum momentum of the block after collision is-

kL2
(A) Mk L (B) M
2M

ML2
(C) zero (D)
k
l
20. A T shaped object with dimensions shown in the figure, is lying on a smooth floor. A B
251\D\Allen(IIT-JEE Wing)\2020-21\Nurture\Phy\Unit-03\YLRE\01-Centre of Mass

®
A force F is applied at the point P parallel to AB, such that the object has only
P 2l
the translational motion without rotation. Find the location of P with respect to C.
®
F
2 3 4
(A) l (B) l (C) l (D) l C
3 2 3

21. A bomb of mass 16 kg at rest explodes into two pieces of masses 4 kg and 12 kg. The velocity of the 12 kg mass
is 4 ms–1. The kinetic energy of the other mass is-
(A) 144 J (B) 288 J (C) 192 J (D) 96 J

179
JE E-Physics
22. Consider a two particle system with particles having masses m 1 and m2. If the first particle is pushed towards
the centre of mass through a distance d, by what distance should the second particle be moved, so as to keep
the centre of mass at the same position ?
m2 m1 m1
(A) d (B) d (C) d (D) d
m1 m1 + m2 m2

23. A circular disc of radius R is removed from a bigger circular disc of radius 2R such that the circumferences of
the discs coincide. The centre of mass of the new disc is aR from the centre of the bigger disc. The value of a is:
(A) 1/3 (B) 1/2 (C) 1/6 (D) 1/4

24. A particle moves in the X–Y plane under the influence of a force such that its linear momentum is
r
p(t) = A[iˆ cos(kt) - ˆj sin(kt)] , where A and k are constants. The angle between the force and the momentum is:
(A) 0° (B) 30° (C) 45° (D) 90°

25. If a ball is thrown upwards from the surface of earth:


(A) The earth remains stationary while the ball moves upwards
(B) The ball remains stationary while the earth moves downwards
(C) The ball and earth both moves towards each other
(D) The ball and earth both move away from each other

26. A ball is bouncing down a set of stairs. The coefficient of restitution is e. The height of each step is d and the
ball bounces one step at each bounce. After each bounce the ball rebounds to a height h above the next lower
step. Neglect width of each step in comparison to h and assume the impacts to be effectively head on. Which
of the following relation is correct ?
h h h 1 h 1
(A) = 1 – e2 (B) =1–e (C) = (D) =
d d d 1 - e2 d 1- e

27. A stationary body explodes into four identical fragments such that three of them fly off mutually perpendicular
to each other, each with same K.E. E 0, energy of explosion will be:
4E 0
(A) 6E0 (B) (C) 4E0 (D) 8E0
3
28. Two particles A and B each of mass m are attached by a light inextensible string of length 2l. The whole system
lies on a smooth horizontal table with B initially at a distance l from A. The particle at end B is projected across
the table with speed u perpendicular to AB. Velocity of ball A just after the jerk is
u 3
(A) (B) u 3
4
u 3 u
(C) (D)
2 2

29. Two particles A and B initially at rest, move towards each other under the mutual force of attraction. At the
instant when the speed of A is v and the speed of B is 2v, the speed of the centre of mass of the system is:–
(A) 3v (B) v (C) 1.5v (D) zero
30. A sphere of mass m moving with a constant velocity collides with another stationary sphere of same mass. The
251\D\Allen(IIT-JEE Wing)\2020-21\Nurture\Phy\Unit-03\YLRE\01-Centre of Mass

ratio of velocities of two spheres after collision will be, if the co-efficient of restitution is e-
1- e e -1 1+ e e +1
(A) (B) (C) (D)
1+ e e +1 1- e e -1

31. A cannon of mass 5m (including a shell of mass m) is at rest on a smooth horizontal ground, fires the shell with
its barrel at an angle q with the horizontal at a velocity u relative to cannon. Find the horizontal distance of the
point where shell strikes the ground from the initial position of the cannon:
4u2 sin 2q u 2 sin 2q 3u2 sin 2q 8u 2 sin 2q
(A) (B) (C) (D)
5g 5g 5g 5g

180
Centre of Mass
32. A body of mass 1 kg strikes elastically with another body at rest and continues to move in the same direction
with one fourth of the initial velocity. The mass of the other body is –
(A) 0.6 kg (B) 2.4 kg (C) 3 kg (D) 4 kg

33. A small bucket of mass M kg is attached to a long inextensible cord of length L m . The bucket is released from
rest when the cord is in a horizontal position. At its lowest position, the bucket scoops up m kg of water and
swings up to a height h. The height h in meters is
2 2
æ M ö æ M ö æ M + mö æ M + mö
(A) ç L (B) çè L (C) ç L (D) ç L
è M + m ÷ø M + m ÷ø è M ÷ø è M ÷ø

34. A body of mass 2kg is projected upward from the surface of the ground at t = 0 with a velocity of
20m/s. One second later a body B, also of mass 2kg, is dropped from a height of 20m. If they collide elastically,
then velocities just after collision are
(A) vA = 5 m/s downward, vB = 5 m/s upward (B) vA = 10 m/s downward, vB= 5 m/s upward
(C) vA = 10 m/s upward, vB = 10 m/s downward (D) both move downward with velocity 5 m/s

35. Two objects move in the same direction in a straight line. One moves with a constant velocity v 1. The other starts
at rest and has constant acceleration a. They collide when the second object has velocity 2v1. The distance
between the two objects when the second one starts moving is
v12 v 12 2 v 12
(A) zero (B) (C) (D)
2a a a
36. A piece of paper (shown in figure1) is in form of a square. Two corners of this square are folded to make it
appear like figure.2 . Both corners are put together at centre of square 'O'. If O is taken to be (0,0), the centre of
mass of new system will be at

2a O 2a O

Fig.1 Fig.2

æ a ö æ a ö æ a ö æ a ö
(A) ç - ,0÷ (B) ç - , 0÷ (C) ç , 0÷ (D) ç - , 0÷
è 8 ø è 6 ø è 12 ø è 12 ø

37. A body of mass M moves in outer space with velocity v. It is desired to break the body into two parts so that the
mass of one part is one–tenth of the total mass. After the explosion, the heavier part comes to rest while the
lighter part continues to move in the original direction of motion. The velocity of the small part will be

æ vö æ vö
(A) v (B) çè ÷ø (C) çè ÷ø (D) 10 v
2 10

38. Two persons A and B of weight 80 kg and 50 kg respectively are


standing at opposite ends of a boat of mass 70 kg and length
2m at rest. When they interchange their positions then
251\D\Allen(IIT-JEE Wing)\2020-21\Nurture\Phy\Unit-03\YLRE\01-Centre of Mass

displacement of the centre of mass of the boat will be:


(A) 60 cm towards left (B) 30 cm towards right
(C) 30 cm towards left (D) stationary

39. Two balls of same mass are dropped from the same height onto the floor. The first ball bounces upwards from
the floor elastically. The second ball sticks to the floor. The first applies an impulse to the floor of I 1 and the
second applies an impulse I2 . The impulses obey
I1 I1
(A) I2= 2I1 (B) I2= (C) I2= 4I1 (D) I2=
2 4

181
JE E-Physics
40. A particle of mass 4m which is at rest explodes into four equal fragments. All
v
4 fragments scattered in the same horizontal plane. Three fragments are found
to move with velocity v each as shown in the fig. The total energy released in the
process of explosion is 0
90
(A) mv2 (3– 2 ) 2
(B) mv (3– 2 )/2 v
1350
(C) 2mv2 (D mv2 (1+ 2 )/2
v

41. After scaling a wall of 3 m height a man of weight W drops himself to the ground. If his body comes to a
complete stop in 0.15 s. After his feet touch the ground, calculate the average impulsive force in the vertical
direction exerted by ground on his feet.
(A) 5W (B) 5.21W (C) 3W (D) 6W

42. Two masses A and B of mass M and 2M respectively are connected


by a compressed ideal spring. The system is placed on a horizontal M A uk̂
X
frictionless table and given a velocity ukˆ in the z–direction as shown
in the figure. The spring is then released. In the subsequent motion the u k̂ î
Z
line from B to A always points along the î unit vector. At some instant

of time mass B has a x–component of velocity as v x ˆi . The velocity 2M B u k̂
r
v A of mass A at that instant is
(A) v xˆi + ukˆ (B) – v xˆi + ukˆ (C) –2 v xˆi + ukˆ (D) 2 v xˆi + ukˆ

r r
43. Two particles of masses m1 and m2 in projectile motion have velocities v1 and v 2 respectively at time t = 0.
r r
They collide at time t0. Their velocities become v '1 and v '2 at time 2t0 while still moving in air. The value of:
r r r r
[(m1 v '1 +m2 v '2 ) – (m1 v 1 +m2 v 2 )] is :–
1
(A) zero (B) (m1 + m1)gt0 (C) 2(m1 + m2)gt0 (D) (m + m2)gt0
2 1

44. Two blocks of masses 10 kg and 4 kg are connected by a spring of negligible mass and placed on a frictionless
horizontal surface. An impulse gives a velocity of 14 m/s to the heavier block in the direction of the lighter
block. The velocity of the centre of mass is :–
(A) 30 m/s (B) 20 m/s (C) 10 m/s (D) 5 m/s

45. For a two-body system in absence of external forces, the kinetic energy as measured from ground frame is K o and
from center of mass frame is Kcm. Pick up the wrong statement
(A) The kinetic energy as measured from center of mass frame is least
(B) Only the portion of energy Kcm can be transformed from one form to another due to internal changes in the
system.
(C) The system always retains at least Ko – Kcm amount of kinetic energy as measured from ground frame
251\D\Allen(IIT-JEE Wing)\2020-21\Nurture\Phy\Unit-03\YLRE\01-Centre of Mass

irrespective of any kind of internal changes in the system.


(D) The system always retains at least Kcm amount of kinetic energy as measured from ground frame irrespective
of any kind of internal changes in the system

46. A non-zero external force acts on a system of particles. At any instant t, the velocity and the acceleration of CM
are found to be u0 and a0 It is possible that
(A) u0 = 0; a0 = 0 (B) u0 ¹ 0; a0 ¹ 0
(C) u0 ¹ 0; a0 = 0 (D) None of these

182
Centre of Mass
47. A ring of mass M can slide freely on a horizontal rail. A particle of mass m is
attached to ring through an inextensible string of length l and is released from M
the horizontal position as shown in the figure. Distance traveled by ring m
when string becomes vertical is l
ml ml Ml ml
(A) (B) (C) (D)
M+m M-m M-m M-m
48. A particle strikes a smooth horizontal surface at an angle of 45° with a velocity of 100 m/s and rebounds. If the
coefficient of restitution between the floor and the particle is 0.57 then the angle with which the velocity of the
particle after it rebounds will make with the floor is (approximately)
(A) 30° (B) 45° (C) 60° (D) 90°
49. Three particles of masses 1.0 kg, 2.0 kg and 3.0 kg are placed at the corners A, B and C respectively of an
equilateral triangle ABC of edge 1 m. Coordinates of the centre of mass of the system is (Taking AB as the x-axis
and A as the origin)
7 3 3 7 7 2 5 4
(A) m, m (B) m, m (C) m, m (D) m, m
12 4 12 4 4 3 12 3
50. Three man A, B & C of mass 40 kg, 50 kg & 60 kg are standing on a plank of mass 90 kg, which is kept on a
smooth horizontal plane. If A & C exchange their positions w.r.t. plank then mass B will shift
(B remains at rest w.r.t. plank) 50kg 60kg
40kg
(A) 1/3 m towards left
(B) 1/3m towards right A B C

(C) will not move w.r.t. ground


(D) 5/3 m towards left

51. Two blocks of masses 10 kg and 20 kg are placed on the X-axis. The first mass is moved on the axis by a
distance of 2 cm. By what distance should the second mass be moved to keep the position of the centre of mass
unchanged?
(A) 1 cm (B) 2 cm (C) 3 cm (D) 4 cm
52. In the arrangement shown in figure, mA = 2 kg and mB = 1 kg. String is light and inextensible.
Find the acceleration of centre of mass of both the blocks. Neglect friction everywhere.
g g
(A) upwards (B) downwards
9 9 A
(C) g upwards (D) g downwards B

53. Two blocks A and B of equal masses are released on two sides of a fixed wedge C as shown in figure. The
acceleration of centre of mass of blocks A and B is (Neglect friction)
y
r -g ˆ r -g ˆ g ˆ A B
(A) acm = j (B) acm = i+ j x
2 2 2 C
45° 45°
r g ˆ g ˆ ® g
(C) acm = i+ j (D) a CM = (-ˆj )
2 2 2
54. A block moving horizontally on a smooth surface with a speed of 20 m/s bursts into two equal parts continuing
251\D\Allen(IIT-JEE Wing)\2020-21\Nurture\Phy\Unit-03\YLRE\01-Centre of Mass

to move in the same direction. If one of the parts moves at 30 m/s the speed of the second part is
(A) 5 ms–1 (B) 10 ms–1 (C) 15 ms–1 (D) 20 B
55. A uniform rope of linear mass density l and length l is coiled on a smooth horizontal
surface. One end is pulled up with constant velocity v. Then the average power applied
by the external agent in pulling the entire rope just off the ground is :
1 ll 2 g
(A) l lv 2 + (B) llgv
2 2
llvg 1 3
(D) llgv +
3
(C) lv + lv
2 2

183
JEE-Physics
56. A light spring of spring constant k is kept compressed between two blocks of
masses m and M on a smooth horizontal surface shown in the figure. When
released, the blocks acquire velocities in opposite directions. The spring loses k
contact with the blocks when it acquires natural length. If the spring was initially m M
compressed through a distance x, then the final speeds of the block of mass m is

kM km kM km
(A) x (B) x (C) x (D) x
m(M + m) M(M + m) m(M - m) M(M - m)

57. Two block A and B of mass m are connected through a string. Block A is
moving with a velocity v as shown in figure and block B is having a velocity x axis
v1. Initially string is slackened after some time block A stops, then the impulse v1 v
acting on block B is m=0
B A
(A) mv( î ) (B) – mv ( î )
(C) m(v1 – v) ( î ) (D) m(v – v1) ( î )

58. Two identical balls A & B are kept on a smooth horizontal table. Ball A is moving with velocity v and ball B is
stationary. Ball A collide with ball B and after the collision ball B start moving at an angle of 30° to the initial
direction of the velocity of ball A, then velocity of ball A in perpendicular direction to the line of impact is

2v 3
(A) 2v (B) v/2 (C) (D) v
3 2

59. Three balls A, B, C are kept on a smooth surface. Balls A and C u 4 ms-1
-1
start moving with a constant velocity u & 4 ms respectively as
shown in the figure. If the collision between A and B is elastic then A B C
find out the value of u so that B can also collide with ball C after
m1 = 2 kg m2 = 4 kg m3 = 1 kg
some time.
(A) u > 6 m/s (B) u > 9 m/s (C) u < 6 m/s (D) u = 6 m/s

60. A ball of mass m approaches a wall of mass M (>> m) with speed 4 m/s along the normal to the wall.
The speed of wall is 1 m/s towards the ball. The speed of the ball after an elastic collision with the wall is.
(A) 5 m/s away from the wall (B) 9 m/s away from the wall
(C) 3 m/s away from the wall (D) 6 m/s away from the wall

61. A ball of mass 1 kg is suspended by an inextensible string 1 m long attached to a m=4kg


point O of a smooth horizontal bar resting on a fixed smooth supports A and B.
A O B
The ball is released from rest from the position when the string makes an angle of
30° with the vertical. The mass of the bar is 4 kg. The displacement in meters of the 300

bar when the string makes the maximum angle on the other side of the vertical is
(A) 0 (B) 0.2
(C) 0.25 (D) 0.5 m=1kg
251\D\Allen(IIT-JEE Wing)\2020-21\Nurture\Phy\Unit-03\YLRE\01-Centre of Mass

62. A smooth sphere is moving on a horizontal surface with velocity vector 2iˆ + 2ˆj immediately before it hits

a vertical wall. The wall is parallel to ĵ vector and the coefficient of restitution between the sphere and the

1
wall is e = × The velocity vector of the sphere after it hits the wall is–
2

(A) ˆi - ˆj (B) -ˆi + 2ˆj (C) -ˆi - ˆj (D) 2iˆ - ˆj

184
Centre of Mass
63. Two identical smooth balls are projected towards each other from points A and B on the horizontal ground with
same speed of projection. The angle of projection in each case is 30°. The distance between A and B is 100 m.
The balls collide in air and return to their respective points of projection. If coefficient of restitution is e = 0.7.
Find the speed of projection of either ball in m/s.
(A) 25 (B) 37 (C) 47 (D) 57

SECTION - 2 : MULTIPLE CHOICE CORRECT QUESTIONS


64. A body has its centre of mass at the origin. The x-coordinates of the particles
(A) may be all positive
(B) may be all negative
(C) may be all non-negative
(D) may be positive for some cases and negative in other cases

65. In which of the following cases the centre of mass of a rod is certainly not at its centre ?
(A) the density continuously increases from left to right
(B) the density continuously decreases from left to right
(C) the density decreases from left to right upto the centre and then increases
(D) the density increases from left to right upto the centre and then decreases

66. If the external forces acting on a system have zero resultant, the centre of mass
(A) must not move (B) must not accelerate (C) may move (D) may accelerate

67. Two blocks A and B each of mass m, are connected by a massless spring of natural length L and spring constant
k. The blocks are initially resting on a smooth horizontal floor with the spring at its natural length, as shown in
figure. A third identical block C, also of mass m, moves on the floor with a speed v along the line joining A and
B, and collides elastically with A. Then :

(A) the kinetic energy of the A-B system, at maximum compression of the spring, is zero
(B) the kinetic energy of the A-B system, at maximum compression of the spring, is mv 2/4
(C) the maximum compression of the spring is v m / K

m
(D) the maximum compression of the spring is v
2K

68. A ball hits the floor and rebounds after an inelastic collision. In this case :
(A) the momentum of the ball just after the collision is the same as that just before the collision
(B) the mechanical energy of the ball remains the same in the collision
(C) the total momentum of the ball and the earth is conserved
(D) the total energy of the ball and the earth is conserved

SECTION - 3 : COMPREHENSION BASED QUESTIONS


Comprehension-1.
251\D\Allen(IIT-JEE Wing)\2020-21\Nurture\Phy\Unit-03\YLRE\01-Centre of Mass

If net force on a system in a particular direction is zero (say in horizontal


direction) we can apply: m
åmRxR = åmLxL, åmRvR = åmLvL and åmRaR = åmLaL
Here R stands for the masses which are moving towards right and L for
4m
the masses towards left, x is displacement, v is velocity and a the
M 2m
acceleration (all with respect to ground). A small block of mass m = 1 kg
is placed over a wedge of mass M = 4 kg as shown in figure. Mass m is O +x-axis
4m
released from rest. All surfaces are smooth. Origin O is as shown.

185
JE E-Physics
69. Final velocity of the wedge is ..................... m/s :–

1 1
(A) 3 (B) 2 (C) (D)
2 3

70. The block will strike the x–axis at x = ................m :–


(A) 4.2 (B) 7.6 (C) 5.6 (D) 6.8

71. Normal reaction between the two blocks at an instant when absolute acceleration of m is 5 3 m/s2 at 60° with
horizontal is ........... N. Normal reaction at this instant is making 30° with horizontal :
(A) 6 (B) 10 (C) 4 (D) 5

72. At the same instant reaction on the wedge from the ground is .............. N.
(A) 42.5 (B) 40 (C) 43.46 (D) None of these

Comprehension-2
One particle of mass 1 kg is moving along positive x–axis with velocity 3 m/s. Another particle of mass 2 kg is
moving along y–axis with 6 m/s. At time t = 0, 1 kg mass is at (3m, 0) and 2 kg at (0, 9m), x–y plane is the
horizontal plane. (Surface is smooth for question 1 and rough for question 2 and 3)

73. The centre of mass of the two particles is moving in a straight line which equation is :
(A) y = x + 2 (B) y = 4x + 2 (C) y = 2x – 4 (D) y = 2x + 4

74. If both the particles have the same value of coefficient of friction m = 0.2. The centre of mass will stop at time
t = ......s :
(A) 1.5 (B) 4.5 (C) 3.0 (D) 2.0

75. Co–ordinates of centre of mass where it will stop finally are :–


(A) (2.0 m, 14.25 m) (B) (2.25 m, 10 m)
(C) (3.75 m, 9 m) (D) (1.75 m, 12 m)

Comprehension-3.
A ball of mass m = 1 kg is hung vertically by a thread of length l = 1.50
metre. Upper end of the thread is attached to the ceiling of a trolley of
mass M = 4 kg. Initially, trolley is stationary and it is free to move along
horizontal rails without friction. A shell of mass m = 1 kg, moving
horizontally with velocity v0 = 6 m/s, collides with the ball and gets stuck
with it. As a result, thread starts to deflect towards right. (g = 10 m/s 2)

76. Velocity of combined mass 2m just after collision is :


(A) 3 m/sec (B) 6 m/sec (C) 1 m/sec (D) 1.5 m/sec

77. Velocity of the trolley, at the time of maximum deflection of the ball is :
251\D\Allen(IIT-JEE Wing)\2020-21\Nurture\Phy\Unit-03\YLRE\01-Centre of Mass

(A) 3 m/sec (B) 6 m/sec (C) 1 m/sec (D) 1.5 m/sec

78. Maximum inclination of thread with the vertical is :


(A) 30° (B) 37° (C) 45° (D) 53°

186
Centre of Mass
SECTION - 4 : MATRIX - MATCH QUESTIONS
79. In each situation of column–I, a system involving two bodies is given. All strings and pulleys are light and
friction is absent everywhere. Initially each body of every system is at rest. Consider the system in all situation
of column I from rest till any collision occurs. Then match the statements in column – I with the corresponding
results in column–II
Column I Column II
m
(A) The block plus wedge system is placed over (p) Shifts towards right
smooth horizontal surface. After the system M
is released from rest, the centre of mass of
system

(B) The string connecting both the blocks m m (q) Shifts downwards
of mass m is horizontal. Left block is
placed over smooth horizontal table
as shown. After the two block system is released
from rest, the centre of mass of system

(C) The block and monkey have same mass. The (r) Shifts upwards
monkey starts climbing up the rope. After the
monkey starts climbing up, the centre of mass
of monkey + block system

(D) Both block of mass m are initially at rest. The (s) Does not shift
left block is given initial velocity u downwards.
Then, the centre of mass of two block system
afterwards
m m

80. Two balls of mass m and 2m each have momentum 2p and p in the direction m 2p 2m p

shown in figure. During collision they exert an impulse of magnitude p on


each other.
Column I Column II
(A) After collision momentum of m (p) 2p
(B) After collision momentum of 2m (q) p
(C) Coefficient of restitution between them (r) 1
(s) None

81. If net force on a system of particles is zero, then


Column I Column II
(A) Acceleration of centre of mass (p) Constant
(B) Velocity of centre of mass (q) Zero
(C) Momentum of centre of mass (r) May be zero
(D) Velocity of an individual particle (s) May be constant
of the system
251\D\Allen(IIT-JEE Wing)\2020-21\Nurture\Phy\Unit-03\YLRE\01-Centre of Mass

82. Column I Column II


(A) Elastic collision (p) KE is conserved
(B) Inelastic collision (q) KE after collision = KE before collision
(C) Perfectly inelastic collision (r) KE after collision ¹ KE before collision
(s) Particles stick after collision
(t) Linear momentum is conserved
(u) Relative velocity of separation after is zero

187
JE E-Physics
SECTION - 5 : NUMERICAL ANSWER BASED QUESTIONS
83. Two blocks of equal masses m are released from the top of a smooth fixed wedge as shown in the figure. Find
the magnitude of the acceleration of the centre of mass of the two blocks.

m m
30º 60º

84. A (trolley + child) of total mass 200 kg is moving with a uniform speed of 36 km/h on a frictionless track.
The child of mass 20 kg starts running on the trolley from one end to the other (10 m away) with a speed
of 10 m s–1 relative to the trolley in the direction of the trolley’s motion and jumps out of the trolley with
the same relative velocity. What is the final speed of the trolley? How much has the trolley moved from the
time the child begins to run? m

85. A man is standing on a cart of mass double the mass of the man. Initially cart
is at rest on the smooth ground. Now man jumps with relative velocity 'v' horizontally
towards right with respect to cart. Find the work done by man during the process
of jumping.
/////////////////////////////////////////////

86. A ball of mass 1 g is released down an inclined plane, just describes a circle of radius 10 cm in the vertical plane,
on reaching the bottom, the minimum height of the inclined plane is .................cm.

87. Two spheres of masses 3 kg and 2 kg collide directly. Their relative velocity before collision is 15 m/s and after
collision is 5 m/s. The total loss of K.E. in joules due to collision is ______.

88. A particle of mass 2 kg moving with a velocity 5 î m/s collides head - on with another particle of mass 3 kg
moving with a velocity -2 î m/s. After the collision the first particle has a speed of –1.6 î m/s. Find
(a) velocity of the centre of mass after the collision.
(b) velocity of the second particle after the collision.
(c) coefficient of restitution.

89. A body 'A' moving in a straight line with velocity v makes a collision with a body 'B' initially at rest. After
collision, B acquires a velocity of 1.6 v. Assuming the bodies to be perfectly elastic, what is the ratio of the mass
of A to that of B? What percentage of A's energy is transferred to B as a result of collision.

90. An object of mass 5 kg is projected with a velocity of 20 m/s at an angle of 60° to the horizontal. At the highest
point of its path the projectile explodes and breaks up into two fragments of masses 1 kg and 4 kg. The
fragments separate horizontally after the explosion. The explosion releases internal energy such that the kinetic
energy of the system at the highest point is doubled. Calculate the separation between the two fragments when
they reach the ground.

91. A cylindrical solid of mass 10–2 kg and cross–sectional area 10–4 m2 is moving parallel to its axis (the
x–axis) with a uniform speed of 103 m/s in the positive direction. At t = 0, its front face passes the plane
x = 0. The region to the right of this plane is filled with the dust particle of uniform density 10 –3 kg/m3. When
251\D\Allen(IIT-JEE Wing)\2020-21\Nurture\Phy\Unit-03\YLRE\01-Centre of Mass

a dust particles collides with the face of the cylinder, it sticks to its surface. Assuming that the dimensions of the
cylinder remain practically unchanged and that the dust sticks only to the front face of the cylinder find the
x–coordinate of the front of the cylinder at t = 150 s.

92. A coordinate axis system taking x-axis as horizontal smooth floor is shown in figure.
Two small balls of masses m and 3m attached with a string are released from some
heights on y-axis as shown in figure. The balls may collide head on or obliquely.
After a certain time mass m is at (9 cm, 20 cm) while mass 3m is 25 cm above the
x axis and the strings is taut. The balls always remain in x-y plane. Find the length of
string.

188
Centre of Mass
93. A wedge of mass M=2m rests on a smooth horizontal plane. A small block
B
of mass m rests over it at left end A as shown in figure. A sharp impulse is
applied on the block, due to which it starts moving to the right with velocity m 20cm
v0
v0 = 6 ms–1. At highest point of its trajectory, the block collides with a particle
of same mass m moving vertically downwards with velocity v = 2 ms –1 and A M
gets stuck with it. If the combined body lands at the end point A of body of
l
mass M, calculate length l. Neglect friction (g = 10 ms–2)

94. A particle moving on a smooth horizontal surface strikes a stationary wall.

////////////////////////////////////
The angle of strike is equal to the angle of rebound & is equal to 37° and the
1 37º
coefficient of restitution with wall is e = . Find the friction coefficient
5 37º

X
between wall and the particle in the form and fill value of X.:
10

95. A thin sheet of metal of uniform thickness is cut into the shape bounded by
the line x = a and y = ± k x2, as shown. Find the coordinates of the centre
of mass.

96. In the figure shown, when the persons A and B exchange their positions, then A B
(i) the distance moved by the centre of mass of the system is _____.
(ii) the plank moves toward _______.
m1
(iii) the distance moved by the plank is ____. M m2
(iv) the distance moved by A with respect to ground is _____.
(v) the distance moved by B with respect to ground is __. 2m
m1= 50kg; m2= 70kg;M = 80kg

97. A ball of mass 100 g is projected vertically upwards from the ground with a velocity of 49m/s. At the same time
another identical ball is dropped from a height of 98 m to fall freely along the same path as that followed by
the first ball. After some time the two balls collide and stick together and finally fall to the ground. Find the time
of flight of the masses.
0.1kg
98. A massless platform is kept on a light elastic spring, shown in Fig.
When a sand particle of 0.1 kg mass is dropped on the pan from
a height 0.24 m, the particle strikes the pan, and the spring compresses
by 0.01 m. From what height should the particle be dropped to cause
a compression of 0.04 m ?
251\D\Allen(IIT-JEE Wing)\2020-21\Nurture\Phy\Unit-03\YLRE\01-Centre of Mass

99. A bullet of mass M is fired with a velocity 50 m/s at an angle q with the horizontal. At the highest point of its
10
trajectory, it collides head–on with a bob of mass 3M suspended by a massless string of length metres and
3
gets embedded in the bob, after the collision the string moves through an angle of 120°. Find :
(i) the angle q,
(ii) the vertical and horizontal coordinates of the initial position of the bob with respect to the point of firing of
the bullet. (Take g = 10 m/s2).

189
JE E-Physics

100. The 4 kg sphere from rest when q =60° strikes a block mass of 5 kg placed on
q
a rough horizontal plane and comes to rest after collision. The 5 kg block 1m

comes to rest after moving a distance of 0.8m. Find µ of ground & block & 4kg

coefficient of restitution e. 5kg

101. After a completely inelastic collision two objects of the same mass and same initial speed are found to move
away together at half their initial speed. Find the angle between the initial velocities of the objects.

102. A ball of mass m=1 kg is hung vertically by a thread l=1.50 m. Upper end of
1.50m
the thread is attached to the ceiling of a trolley of mass M = 4 kg. Initially, trolley M
is stationary and it free to move along horizontal rails without friction. A shell of m m
mass m = 1 kg moving horizontally with velocity v0=6 ms–1 collides with the ball v0
and gets stuck with it. As a result, thread starts to deflected towards right. Calculate
its maximum deflection with the vertical. (g = 10 ms –2)

103. A 70 g ball B dropped from a height h0 = 9 m reaches a height h2 = 0.25 m after bouncing twice from identical
210 g plates. Plate A rests directly on hard ground, while plate C rests on a foam –rubber mat. Determine.

h0
h2
A

(i) the coefficient of restitution between the ball and the plates,
(ii) the height h1 of the ball's first bounce.

104. In the figure shown a small block B of mass m is released from the top of a
smooth movable wedge A of the same mass m. The height of wedge A
shown in figure is h = 100 cm. B ascends another movable smooth wedge C
of the same mass. Neglecting friction any where the maximum height (in cm)
attained by block B on wedge C is 20 + h . Find h :

105. Two particles A and B of masses 1 kg and 2 kg respectively are projected in the same
vertical line as shown in figure with speeds uA = 200 m/s and uB = 85 m/s respectively.
Initially they were 90 m apart. Find the maximum height attained by the centre of
mass of the system of particles A and B, from the initial position of centre of mass
of the system. Assume that none of these particles collides with the ground in that
duration Take g = 10 m/s2.
251\D\Allen(IIT-JEE Wing)\2020-21\Nurture\Phy\Unit-03\YLRE\01-Centre of Mass

106. Two blocks of mass 2 kg and M are at rest on an inclined plane and are separated
by a distance of 6.0 m as shown. The coefficient of friction between each block M
and the inclined plane is 0.25. The 2 kg block is given a velocity of 10.0 m/s up the 6.0m
inclined plane. It collides with M, comes back and has a velocity of 1.0 m/s when
2 kg
it reaches its initial position. The other block M after the collision moves 0.5 m up
and comes to rest. Calculate the coefficient of restitution between the blocks and
the mass of the block M. (Take sinq » tanq = 0.05 and g = 10 m/s2) q

190
Centre of Mass

107. A car P is moving with a uniform speed of 5 3 m/s towards a carriage of mass 9 kg at rest kept on the rails
at a point B as shown in figure. The height AC is 120 m. Cannon balls of 1 kg are fired from the car with an
initial velocity 100 m/s at an angle 30° with the horizontal. The first cannon balls hits the stationary carriage
after a time t0 and sticks to it. Determine t 0.

C
P

A B

At t0, the second cannon ball is fired. Assume that the resistive force between the rails and the carriage is
constant and ignore the vertical motion of the carriage throughout. If the second ball also hits and sticks of the
carriage, what will be the horizontal velocity of the carriage just after the second impact? [IIT-JEE 2001]

SECTION - 6 : SUBJECTIVE QUESTIONS


108. A small cube of mass ‘m’ slides down a circular path of radius ‘R’ cut
into a large block of mass ‘M’. ‘M’ rests on a table and both blocks
move without friction. The blocks initially are at rest and ‘m’ starts
from the top of the path. Find the velocity ‘v’ of the cube as it leaves
the block. Initially the line joining m and the centre is horizontal.

109. A particle of mass 4m which is at rest explodes into three fragments. Two of the fragments each of mass m are
found to move with a speed v each in mutually perpendicular directions. The total energy released in the
process of explosion is ................

110. The magnitude of the force (in newtons) acting on a body varies with
time t (in microseconds) as shown in the figure AB, BC and CD are
straight line segments. The magnitude of the total impulse of the
force on the body from t = 4 µs to t = 16 µs is ................. N–s.

111. Three rods of the same mass are placed as shown in the figure.
Calculate the coordinates of the centre of mass of the system.

112. A body of mass 1 kg initially at rest, explodes and breaks into three fragments of masses in the ratio
1 : 1 : 3. The two pieces of equal mass fly–off perpendicular to each other with a speed of 30 m/s each. What
is the velocity of the heavier fragment ?

113. A man whose mass is m kg jumps vertically into air from a sitting position in which his centre of mass is at a
251\D\Allen(IIT-JEE Wing)\2020-21\Nurture\Phy\Unit-03\YLRE\01-Centre of Mass

height h1 from the ground. When his feet are just about to leave the ground his centre of mass is h 2 from the
ground and finally rises to h3 when he is at the top of the jump. What is the average upward force exerted by the
ground on him?

114. A uniform thin rod of mass M and length L is standing vertically along the y–axis on a smooth horizontal
surface, with its lower end at the origin (0, 0). A slight disturbance at t = 0 causes the lower end to slip on the
smooth surface the positive x–axis, and the rod starts falling.
(i) What is the path followed by the centre of mass of the rod during its fall ?
(ii) Find the equation of the trajectory of a point on the rod located at a distance r from the lower end. What
is the shape of the path of this point ?

191
JE E-Physics
115. A small sphere of radius R is held against the inner surface of a larger sphere of Y
radius 6R. The masses of large and small spheres are 4M and M respectively. This
arrangement is placed on a horizontal table. There is no friction between any surfaces M,R
of contact. The small sphere is now released. Find the co–ordinates of the centre of 6R
O X
the larger sphere when the smaller sphere reaches the other extreme position. 4M(L,0)

116. Block A of mass m/2 is connected to one end of light rope which passes over a
pulley as shown in the Fig. Man of mass m climbs the other end of rope with a
relative acceleration of g/6 with respect to rope find acceleration of block A and
tension in the rope.
g/6
m/2 A m

117. A plate of mass M is moved with constant velocity v against dust particles moving
with velocity u in opposite direction as shown. The density of the dust is r and plate
area is A. Find the force F required to keep the plate moving uniformly. (r in kg /m3)

A
118. Two blocks of masses m1 and m2 are connected by a massless pulley A, m2
m1
slides along the smooth sides of a rectangular wedge of mass m, which rests
on a smooth horizontal plane. Find the distance covered by the wedge on the h m
horizontal plane till the mass m1 is lowered by the vertical distance h.

119. Two particles A and B of mass 2m and m respectively are attached to the ends of a light inextensible string of
length 4a which passes over a small smooth peg at a height 3a from an inelastic table. The system is released
from rest with each particle at a height a from the table. Find–
(i) The speed of B when A strikes the table.
(ii) The time that elapses before A first hits the table.
(iii) The time for which A is resting on the table after the first collision & before it is first jerked off.

120. Two particles, each of mass m, are connected by a light inextensible string of length 2l. Initially they lie on a
smooth horizontal table at points A and B distant l apart. The particle at A is projected across the table with
velocity u. Find the speed with which the second particle begins to move if the direction of u is :-
(i) along BA,
(ii) at an angle of 120° with AB
(iii) perpendicular to AB. In each case calculate (in terms of m and u) the impulsive tension in the string.

121. The Atwood machine in figure has a third mass attached to it by a limp string.
After being released, the 2m mass falls a distance x before the limp string becomes
taut. Thereafter both the mass on the left rise at the same speed.
251\D\Allen(IIT-JEE Wing)\2020-21\Nurture\Phy\Unit-03\YLRE\01-Centre of Mass

m 2m
What is the final speed? Assume that pulley is ideal.

122. A ball of mass m = 1 kg falling vertically with a velocity v0 = 2 m/s strikes a


wedge of mass M = 2 kg kept on a smooth, horizontal surface as shown in om
figure. The coefficient of resitution between the ball and the wedge is
M
e = 1/2. Find the velocity of the wedge and the ball immediately after collision.
300

192
Centre of Mass
123. A ball is dropped from height h to the ground. If the coefficient of restitution is 0.8, the height to which ball goes
up after it rebounds third time is ...............

124. The rate of change of total momentum of a many–particle system is proportional to the .............. on the
system.

125. In an inelastic collision of two bodies, the quantities which do not change after the collision are the ...............
of the system of two bodies.

126. Figure shows a uniform square plate from which four identical squares at the
corners will be removed.
(i) Where is the centre of mass of the plate originally.
(ii) Where is C.M. after square 1 is removed.
(iii) Where is C.M. after squares 1 and 2 removed.
(iv) Where is C.M. after squares 1 and 3 are removed.
(v) Where is C.M. after squares 1, 2 and 3 are removed.
(vi) Where is C.M. after all the four squares are removed.
Give Answers in terms of quadrants and axes.

127. A chain of length L and mass per unit length r is pulled on a horizontal surface. One end of the chain is lifted
vertically with constant velocity v by a force P.
(i) P as a function of height x of the end above the surface will be
(ii) work done by force will be
(iii) loss in energy

SECTION - 7 : ASSERTION–REASON QUESTIONS


128. Statement–1 : In an elastic collision between two bodies, the relative speed of the bodies after collision is
equal to the relative speed before the collision. [IIT-JEE 2007]
and
Statement–2 : In an elastic collision, the linear momentum of the system is conserved.
(A) Statement–1 is True, Statement–2 is True ; Statement–2 is a correct explanation for Statement–1
(B) Statement–1 is True, Statement–2 is True ; Statement–2 is not a correct explanation for Statement–1
(C) Statement–1 is True, Statement–2 is False.
(D) Statement–1 is False, Statement–2 is True.

129. Statement–1 : If there is no external torque on a body about its centre of mass, then the velocity of the centre
of mass remains constant. [IIT-JEE 2007]
and
Statement–2 : The linear momentum of an isolated system remains constant.
(A) Statement–1 is True, Statement–2 is True ; Statement–2 is a correct explanation for Statement–1
(B) Statement–1 is True, Statement–2 is True ; Statement–2 is not a correct explanation for Statement–1
(C) Statement–1 is True, Statement–2 is False.
(D) Statement–1 is False, Statement–2 is True.
251\D\Allen(IIT-JEE Wing)\2020-21\Nurture\Phy\Unit-03\YLRE\01-Centre of Mass

130. Statement–1 : In case of bullet fired from gun, the ratio of kinetic energy of gun and bullet is equal to ratio of
mass of bullet and gun.
and
Statement–2 : In firing, momentum is conserved.
(A) Statement–1 is True, Statement–2 is True ; Statement–2 is a correct explanation for Statement–1
(B) Statement–1 is True, Statement–2 is True ; Statement–2 is not a correct explanation for Statement–1
(C) Statement–1 is True, Statement–2 is False.
(D) Statement–1 is False, Statement–2 is True.

193
JE E-Physics
131. Statement–1 : When a girl jumps from a boat, the boat slightly moves away from the shore.
and
Statement–2 : The total linear momentum of an isolated system remain conserved.
(A) Statement–1 is True, Statement–2 is True ; Statement–2 is a correct explanation for Statement–1
(B) Statement–1 is True, Statement–2 is True ; Statement–2 is not a correct explanation for Statement–1
(C) Statement–1 is True, Statement–2 is False.
(D) Statement–1 is False, Statement–2 is True.

132. Statement–1 : In a two body collision, the momenta of the particles are equal and opposite to one another,
before as well as after the collision when measured in the center of mass frame.
and
Statement–2 : The momentum of the system is zero from the centre of mass frame.
(A) Statement–1 is True, Statement–2 is True ; Statement–2 is a correct explanation for Statement–1
(B) Statement–1 is True, Statement–2 is True ; Statement–2 is not a correct explanation for Statement–1
(C) Statement–1 is True, Statement–2 is False.
(D) Statement–1 is False, Statement–2 is True.

133. Statement–1 : The centre of mass and centre of gravity of a body are two different positions in general.
and
Statement–2 : The centre of mass and centre of gravity of a body coincide if gravitational field is uniform.
(A) Statement–1 is True, Statement–2 is True ; Statement–2 is a correct explanation for Statement–1
(B) Statement–1 is True, Statement–2 is True ; Statement–2 is not a correct explanation for Statement–1
(C) Statement–1 is True, Statement–2 is False.
(D) Statement–1 is False, Statement–2 is True.

134. Statement–1 : A particle of mass m strikes a smooth wedge of mass M as shown


in the figure. Linear momentum of particle along the inclined surface of wedge is m v0

conserved during collision.


and M

Statement–2 : Wedge exerts a force on particle perpendicular to inclined face of


Smooth
wedge during collision.
(A) Statement–1 is True, Statement–2 is True ; Statement–2 is a correct explanation for Statement–1
(B) Statement–1 is True, Statement–2 is True ; Statement–2 is not a correct explanation for Statement–1
(C) Statement–1 is True, Statement–2 is False.
(D) Statement–1 is False, Statement–2 is True.

135. Statement–1 : The coefficient of restitution is less than one for all collisions studied under Newton’s laws of
restitution.
and
Statement–2 : For a perfectly elastic collision, coefficient of restitution is equal to one.
(A) Statement–1 is True, Statement–2 is True ; Statement–2 is a correct explanation for Statement–1
(B) Statement–1 is True, Statement–2 is True ; Statement–2 is not a correct explanation for Statement–1
(C) Statement–1 is True, Statement–2 is False.
(D) Statement–1 is False, Statement–2 is True.
251\D\Allen(IIT-JEE Wing)\2020-21\Nurture\Phy\Unit-03\YLRE\01-Centre of Mass

136. Statement–1 : No external force acts on system of two spheres which undergo a perfectly elastic head on
collision. The minimum kinetic energy of this system is zero if the net momentum of this system is zero.
and
Statement–2 : In any two body system undergoing perfectly elastic head on collision, at the instant of
maximum deformation, the complete kinetic energy of the system is converted to deformation potential energy
of the system.
(A) Statement–1 is True, Statement–2 is True ; Statement–2 is a correct explanation for Statement–1
(B) Statement–1 is True, Statement–2 is True ; Statement–2 is not a correct explanation for Statement–1
(C) Statement–1 is True, Statement–2 is False.
(D) Statement–1 is False, Statement–2 is True.

194
Centre of Mass
137. Statement–1 : A sphere of mass m moving with speed u undergoes a perfectly elastic head on collision with
another sphere of heavier mass M at rest (M>>>m), then direction of velocity of sphere of mass m is reversed
due to collision [no external force acts on system of two spheres]
and
Statement–2 : During a collision of spheres of unequal masses, the heavier exerts more force on lighter mass
in comparison to the force which lighter mass exerts on heavier mass.
(A) Statement–1 is True, Statement–2 is True ; Statement–2 is a correct explanation for Statement–1
(B) Statement–1 is True, Statement–2 is True ; Statement–2 is not a correct explanation for Statement–1
(C) Statement–1 is True, Statement–2 is False.
(D) Statement–1 is False, Statement–2 is True.

138. Statement–1 : If a ball projected up obliquely from the ground breaks up into several fragments in its path,
the centre of mass of the system of all fragments move in same parabolic path compared to initial one till all
fragments are in air.
and
Statement–2 : In the situation of statement–1, at the instant of breaking, the fragments may be thrown in
different directions with different speeds.
(A) Statement–1 is True, Statement–2 is True ; Statement–2 is a correct explanation for Statement–1
(B) Statement–1 is True, Statement–2 is True ; Statement–2 is not a correct explanation for Statement–1
(C) Statement–1 is True, Statement–2 is False.
(D) Statement–1 is False, Statement–2 is True.

139. Statement –1 : A rigid disc rolls without slipping on a fixed rough horizontal surface with uniform angular
velocity. Then the acceleration of lowest point on the disc is zero.
and
Statement –2 : For a rigid disc rolling without slipping on a fixed rough horizontal surface, the velocity of the
lowest point on the disc is always zero.
(A) Statement–1 is True, Statement–2 is True ; Statement–2 is a correct explanation for Statement–1
(B) Statement–1 is True, Statement–2 is True ; Statement–2 is not a correct explanation for Statement–1
(C) Statement–1 is True, Statement–2 is False.
(D) Statement–1 is False, Statement–2 is True.

140. Statement –1 : A sphere rolling on a rough horizontal surface with constant velocity then it start going up on
a smooth inclined plane. Rotational KE of sphere decreases continuously on horizontal and inclined surface.
and
Statement –2 : Rotational KE decreases if torque due to friction opposes angular velocity of sphere.
(A) Statement–1 is True, Statement–2 is True ; Statement–2 is a correct explanation for Statement–1
(B) Statement–1 is True, Statement–2 is True ; Statement–2 is not a correct explanation for Statement–1
(C) Statement–1 is True, Statement–2 is False.
(D) Statement–1 is False, Statement–2 is True.

141. Statement –1 : A disc is rolling on an inclined plane without slipping. The velocity of centre of mass is v. These
others points on the disc lies on a circular arc having same speed as centre of mass.
and
Statement –2 : When a disc is rolling on an inclined plane. The magnitude of velocities of all the point from the
251\D\Allen(IIT-JEE Wing)\2020-21\Nurture\Phy\Unit-03\YLRE\01-Centre of Mass

contact point is same, having distance equal to radius r.


(A) Statement–1 is True, Statement–2 is True ; Statement–2 is a correct explanation for Statement–1
(B) Statement–1 is True, Statement–2 is True ; Statement–2 is not a correct explanation for Statement–1
(C) Statement–1 is True, Statement–2 is False.
(D) Statement–1 is False, Statement–2 is True.

195
JE E-Physics

ANSWERS
l SINGLE CHOICE CORRECT QUESTIONS
Que. 1 2 3 4 5 6 7 8 9 10 11 12 13 14 15
Ans. B D B D A A C D A B A C B A C
Que. 16 17 18 19 20 21 22 23 24 25 26 27 28 29 30
Ans. C C B A C B C A D D C A A D A

Que. 31 32 33 34 35 36 37 38 39 40 41 42 43 44 45

Ans. A A A A A D D C B A B C C C D

Que. 46 47 48 49 50 51 52 53 54 55 56 57 58 59 60

Ans. B A A A B A B D B C A A B A D

Que. 61 62 63

Ans. B B B

l MULTIPLE CHOICE CORRECT QUESTIONS


64. (CD) 65. (AB) 66. (BC) 67. (BD) 68. (CD)

l COMPREHENSION BASED QUESTIONS


69. (B) 70. (D) 71. (D) 72. (A) 73. (B) 74. (C) 75. (D)
76. (A) 77. (C) 78. (B)

l MATRIX-MATCH QUESTIONS
79. A–q, B–p, q, C–r, D–s 80. A–q, B–p, C–s
81. A–q, B–p,r, C–p,r, D–r,s 82. A–q,t, B–r,t, C–r,s,t,u

l NUMERICAL ANSWER BASED QUESTIONS

1
83. g/2 84. 9m/s, 9m 85. mv2
3

86. 25 87. 120 88. (a) 0.8 î m/s, (b) 2.4 î m/s (c) 4/7

89. 64% 90. 44.25 m 91. 105 m


92. 13 cm 93. 40 cm 94. 5
95. (3/4a) 96. (i) zero (ii) right (iii) 0.2 m (iv) 2.2 m (v) 1.8 m
97. 4.53 s 98. 3.96 m 99. (i) 370 (ii) (120, 45)
100. e=0.8, m =0.4 101. 1200 102. 370
251\D\Allen(IIT-JEE Wing)\2020-21\Nurture\Phy\Unit-03\YLRE\01-Centre of Mass

103. (i) 0.66 (ii) 4 m 104. 5 105. 5 m


106. ( e = 0.84, M = 15.21 kg ) 107. 12 s, 15.75 ms–1

l SUBJECTIVE QUESTIONS
2 gR 3m v 2
108. v = 1+ m 109. 110. 5 × 10–3
M 2

æ a aö mg ( h 3 - h1 )
111. çè , ÷ø 112. 10 2 ms–1 at 450 113.
3 3 (h2 - h1 )
196
Centre of Mass

4g 13mg
114. (i) straight line 115. 116. , 117. rA(u + v)2
9 18
x2 y2
(ii) 2
+ =1
æL ö r2
çè - r ÷ø
2

(m2 + m1 cot a ) h 2ag 3v 2v


118. 119. (i) (ii) (iii)
m + m1 + m2 3 g g

u mu u 3 mu 3 u 3 mu 3
120. (i) , (ii) , (iii) ,
2 2 8 8 4 4

3gx 1 2
121. 122. v1 = ms–1, v2 = ms–1 123. (0.8)6h
8 3 3
124. External force 125. Linear momentum
126. (i) at O, (ii) diagonally from O to 3 (iii) along OY', (iv) at O (v) diagonally from O to 4 (vi) at O

1 rv 2l
127. (i) r (gx + v2) (ii) rgL2 + rv 2L (iii)
2 2

l ASSERTION-REASON QUESTIONS
128. (D) 129. (D) 130. (A) 131. (A)
132. (A) 133. (B) 134. (A) 135. (B)
136. (C) 137. (C) 138. (B) 139. (D)
140. (D) 141. (A)
251\D\Allen(IIT-JEE Wing)\2020-21\Nurture\Phy\Unit-03\YLRE\01-Centre of Mass

197
198
JE E-Physics

IMPORTANT NOTES

251\D\Allen(IIT-JEE Wing)\2020-21\Nurture\Phy\Unit-03\YLRE\01-Centre of Mass


Rigid Body Dynamics

Not To Be Discussed in Class

SECTION - 1 : SINGLE CHOICE CORRECT QUESTIONS


1. A square plate of edge a/2 is cut out from a uniform square plate of edge 'a'
as shown in figure. The mass of the remaining portion is M. The moment
of inertia of the shaded portion about an axis passing through 'O' (centre
of the square of side a) and perpendicular to plane of the plate is :
9 3
(A) Ma 2 (B) Ma 2
64 16

5 Ma 2
(C) Ma 2 (D)
12 6

2. A uniform ladder of length 5m is placed against the wall as shown in the


figure. If coefficient of friction m is the same for both the walls, what is the
minimum value of m for it not to slip?
(A) m = 1/2 (B) m = 1/4
(C) m = 1/3 (D) m = 1/5

3. A uniform solid sphere is lying at rest between a vertical wall and a


fixed inclined plane as shown. There is no friction between sphere and
the vertical wall but coefficient of friction between sphere and the
1
fixed inclined plane is m = . Then the magnitude of frictional force
2
exerted by fixed inclined plane on sphere is

mg 3
(A) (B) mg (C) mg (D) 0
2 4

4. A uniform solid sphere rolls up (without slipping) the rough fixed inclined plane,
and then back down. Which is the correct graph of acceleration 'a' of centre of
mass of solid sphere as function of time t (for the duration sphere is on the
incline) ? Assume that the sphere rolling up has a positive velocity.

(A) (B) (C) (D)

5. A body of mass m and radius r is rotated with angular velocity w as shown in the
251\D\Allen(IIT-JEE Wing)\2020-21\Nurture\Phy\Unit-03\YLRE\02-Rigid Body Dynamics

figure & kept on a surface that has sufficient friction then the body will move:
(A) backward first and then move forward
(B) forward first and then move backward
(C) will always move forward
(D) none of these

6. Determine the acceleration a of the supporting surface required to keep the


centre G of the circular pipe in a fixed position during the motion. No slipping
takes place between pipe and its support.
(A) g sin q (B) 2g sin q
(C) g/2sin q (D) 2 g sin q

199
JEE-Physics
7. Inner and outer radii of a spool are r and R respectively. A thread is
wound over its inner surface and placed over a rough horizontal
surface. Thread is pulled by a force F as shown in fig. then in case
of pure rolling:
(A) Thread unwinds, spool rotates anticlockwise and friction act leftwards
(B) Thread winds, spool rotates clockwise and friction acts leftwards
(C) Thread winds, spool moves to the right and friction act rightwards
(D) Thread winds, spool moves to the right and friction does not come into existence.

8. Two points A & B on a disc have velocities v1 & v2 at some moment. Their directions make angles 60° and
30° respectively with the line of separation as shown in figure. The angular velocity of disc is :

3v1 v2
(A) (B)
d 3d

v 2 - v1 v2
(C) (D)
d d

9. A box of dimensions l and b is kept on a truck moving with an


acceleration a. If box does not slide, maximum acceleration for
it to remain in equilibrium (w.r.t.truck) is :
gl gb
(A) (B)
b l
(C) g (D) none of these

10. A pulley is hinged at the centre and a massless thread is wrapped around it. The thread is pulled with a
constant force F starting from rest. As the time increases,
(A) its angular velocity increases, but force on hinge remains constant
(B) its angular velocity remains same, but force on hinge increases
(C) its angular velocity increases and force on hinge increases
(D) its angular velocity remains same and force on hinge is constant

11. A solid cylinder of radius R is free to rotate about its horizontal axis. A string is wound
around it and a mass m is attached to its free end. When m falls through a distance h,
its speed at that instant is
(A) proportional to R
(B) proportional to 1 / R
(C) proportional to 1 / R2
(D) independent of R

12. A man pulls a solid cylinder (initially at rest) horizontally by a massless string as shown. The string is wrapped
on the cylinder and the cylinder performs pure rolling( that is, rolling without slipping) . Mass of the cylinder
is 100 kg, radius is p metre & tension in string is 100 N. Then the angular speed of the cylinder after one
251\D\Allen(IIT-JEE Wing)\2020-21\Nurture\Phy\Unit-03\YLRE\02-Rigid Body Dynamics

revolution will be :
4
(A) 4 rad /sec (B) rad/ sec
3
4
(C) rad/ sec (D) none of these
3

13. A mass M is moving with a constant velocity parallel to the x-axis. Its angular momentum with respect to the
origin
(A) is zero (B) remains constant
(C) goes on increasing (D) goeson decreasing

200
Rigid Body Dynamics
r r r r
14. If t ´ L = 0 for a rigid body, wheree t = resultant torque & L = angular momentum about a point and
both are non - zero. Then:
r r
(A) L = constant (B) |L| = constant
r r
(C) |L| will increase (D) |L| may increase.
15. A hollow smooth uniform sphere A of mass ‘m’ rolls without sliding
on a smooth horizontal surface. It collides elastically and headon
with another stationary smooth solid sphere B of the same mass m
and same radius. The ratio of kinetic energy of ‘B’ to that of ‘A’ just
after the collision is :
(A) 5 : 2 (B) 1 : 1 (C) 2 : 3 (D) 3 : 2

16. A rod of mass m and length l is kept on a surface that has no friction.
If a ball comes & hits the lower end with a velocity v, then the velocity
of centre of mass of system will be :
(A) zero
(B) along negative x - axis
(C) along positive x - axis
(D) cannot determine with the given data

17. Initial angular velocity of a circular disc of mass M is w1. Then two small spheres of mass m are attached gently
to two diametrically opposite points on the edge of the disc. What is the final angular velocity of the disc?
æ M+m ö æ M+m ö æ M ö æ M ö
(A) ç ÷ w1 (B) ç ÷ w1 (C) ç ÷ w1 (D) ç ÷ w1
è M ø è m ø è M + 4m ø è M + 2m ø

18. Moment of inertia of a circular wire of mass M and radius R about its diameter is-
(A) MR2/2 (B) MR2 (C) 2MR2 (D) MR2/4
C

19. A particle of mass m moves along line PC with velocity v as shown.What is


L
the angular momentum of the particle about O ?
P r
(A) mvL (B) mvl
(C) mvr (D) zero
O

20. A circular disc X of radius R is made from an iron plate of thickness t and another disc Y of radius 4R is
made from an iron plate of thickness t/4. Then the relation between the moment of inertia I X and IY is-
(A) IY = 32 IX (B) IY = 16IX (C) IY = IX (D) IY = 64 IX

21. A particle performing uniform circular motion has angular momentum L. If its angular frequency is doubled
251\D\Allen(IIT-JEE Wing)\2020-21\Nurture\Phy\Unit-03\YLRE\02-Rigid Body Dynamics

and its kinetic energy halved, then the new angular momentum is-
L L
(A) (B) 2L (C) 4L (D)
4 2
® ® ®
22. Let F be the force acting on particle having position vector r and t be the torque of this force about the
origin. Then-

® ® ® ® ® ® ® ®
(A) r . t = 0 and F . t ¹ 0 (B) r . t ¹ 0 and F . t = 0

® ® ® ® ® ® ® ®
(C) r . t ¹ 0 and F . t ¹ 0 (D) r . t = 0 and F . t = 0

201
JEE-Physics
23. Which of the following statements is false for a particle moving in a circle with a constant angular speed ?
(A) The velocity vector is tangent to the circle
(B) The acceleration vector is tangent to the circle
(C) the acceleration vector points to the centre of the circle
(D) The velocity and acceleration vectors are perpendicular to each other

24. A solid sphere is rotating in free space. If the radius of the sphere is increased keeping mass same which one
of the following will not be affected ?
(A) moment of inertia (B) Angular momentum
(C) Angular velocity (D) Rotational kinetic anergy

25. One solid sphere A and another hollow sphere B are of same mass and same outer radii. Their moment of
inertia about their diameters are respectively I A and IB such that-
IA d
(A) IA = IB (B) IA > IB (C) IA < IB (D) = A
IB dB
where dA and dB are their densities.

26. An annular ring with inner and outer radii R1 and R2 is rolling without slipping with a uniform angular speed.
F1
The ratio of the forces experienced by the two particles situated on the inner and outer parts of the ring is-
F2

2
R2 æ R1 ö R1
(A) (B) ç ÷ (C) 1 (D)
R1 è R2 ø R2

27. The moment of inertia of uniform semicircular disc of mass M and radius r about a line perpendicular to the
plane of the disc through the centre is-
1 2 2 2 1 2
(A) Mr (B) Mr (C) Mr2 (D) Mr
4 5 2

28. Four point masses, each of value m, are placed at the corners of a square ABCD of side l. The moment of
inertia of this system about an axis passing through A and parallel to BD is
(A) 2ml2 (B) 3 ml2 (C) 3ml2 (D) ml2

29. A force of –F k̂ acts on O, the origin of the co-ordinate system. The torque about the point (1, –1) is :
(A) F (ˆi - ˆj ) z

(B) – F (ˆi + ˆj )
O y
(C) F (ˆi + ˆj )
x
(D) – F (ˆi – ˆj )
251\D\Allen(IIT-JEE Wing)\2020-21\Nurture\Phy\Unit-03\YLRE\02-Rigid Body Dynamics

30. A thin circular ring of mass m and radius R is rotating about its axis with a constant angular velocity w.
Two objects each of mass M are attached gently to the opposite ends of a diameter of the ring. The ring
now rotates with an angular velocity w' = :
w(m + 2M) w(m - 2M) wm wm
(A) (B) (C) (D)
m (m + 2M) (m + M) (m + 2M)

31. For the given uniform square lamina ABCD, whose centre is O :
(A) 2 IAC = IEF (B) IAD = 3IEF

(C) IAC = IEF (D) IAC = 2 IEF

202
Rigid Body Dynamics
32. A round uniform body of radius R, mass M and moment of inertia I, rolls down (without slipping) an inclined
plane making an angle q with the horizontal. Then its acceleration is
g sin q g sin q g sin q g sin q
(A) 2 (B) 2
(C) 2 (D)
1 + I / MR 1 + MR / I 1 - I / MR 1 - MR2 / I

33. Angular momentum of the particle rotating with a central force is constant due to-
(A) constant force (B) constant linear momentum
(C) zero torque (D) constant torque

34. A particle moves in a circular path with decreasing speed. Choose the correct Statement :
(A) Angular momentum remains constant
r
(B) Acceleration ( a ) is towards the centree
(C) Particle moves in a spiral path with decreasing radius
(D) The direction of angular momentum remains constant

R
35. From a circular disc of radius R and mass 9M, a small disc of radius is
3 R
removed from the disc. The moment of inertia of the remaining disc about an 3
axis perpendicular to the plane of the disc and passing through O is : 2R
3
40 O
(A) 4MR2 (B) MR2 R
9
37
(C) 10MR2 (D) MR2
9

36. A solid sphere of radius R has moment of inertia I about its geometrical axis. If it
is melted into a disc of radius r and thickness t. If it's moment of inertia about the
tangential axis (which is perpendicular to plane of the disc), is also equal to I,
then the value of r is equal to : r
2 2
(A) R (B) R
15 5

3 3
(C) R (D) R
15 15

37. A small object of uniform density rolls up a curved surface with an initial
3v 2
velocity v. It reaches up to a maximum height of with respect to the
4g
initial position. The object is :- v
(A) ring (B) solid sphere
(C) hollow sphere (D) disc
251\D\Allen(IIT-JEE Wing)\2020-21\Nurture\Phy\Unit-03\YLRE\02-Rigid Body Dynamics

38. A boy stands over the centre of a horizontal platform which is rotating freely with a speed of
2 revolutions/s about a vertical axis through the centre of the platform and straight up through the boy. He
holds 2 kg masses in each of his hands close to his body. The combined moment of inertia of the system is
1 kg-m.2. The boy now stretches his arms so as to hold the masses far from his body. In this situation the
moment of inertia of the system increases to 2 kg-m. 2. The kinetic energy of the system in the latter case as
compared with that in the previous case will-
(A) Remain unchanged (B) Decrease (C) Increase (D) Remain uncertain

39. A rigid body can be hinged about any point on the x-axis. when it is hinged such that the hinge is at x, the moment
of inertia is given by I = x2–2x + 99. The x-coordinate of centre of mass is :–
(A) x=2 (B) x=0 (C) x=1 (D) x=3

203
JEE-Physics
40. A thin rod of length 4l, mass 4m is bent at the points as shown in the fig. What is the moment of inertia of the
rod about the axis passing point O & perpendicular to the plane of the paper ?

ml 2 10ml 2 O
(A) (B)
3 3
90° 90°
ml 2
ml 2
(C) (D)
12 24

41. A smooth tube of certain mass is rotated in gravity free space and w
released. The two balls shown in the figure move towards ends of the
tube. For the whole system which of the following quantity is not
conserved :-
(A) Angular momentum (B) Linear momentum
(C) Kinetic energy (D) Angular speed

42. A thin uniform straight rod of mass 2 kg and length 1 m is free to rotate about its
upper end when at rest It receives an impulsive blow of 10 Ns at its lowest point,
normal to its length as shown in figure. The kinetic energy of rod just after impact is
(A) 75J (B) 100 J
(C) 200 J (D) none 10 NS

43. In the adjoining figure along which axis the moment of inertia of the triangu- A

lar lamina will be maximum- [Given that AB < BC < AC]


(A) AB
(B) BC
C
(C) CA B
(D) For all axis

44. In the figure (A) half of the meter scale is made of wood while the other half of steel. The wooden part is
pivoted at O. A force F is applied at the end of steel part. In figure (B) the steel part is pivoted at O' and the
same force is applied at the wooden end:– wood steel steel wood
(A) More angular acceleration will be produced in (A)
(B) More angular acceleration will be produced in (B) O P O' P'
(C) Same angular acceleration will be produced in both conditions F F
(A) (B)
(D) Information is incomplete

45. A uniform rod of mass M and length L lies radially on a disc rotating with
angular speed w in a horizontal plane about its axis. The rod does not slip on w
the disc and the centre of the rod is at a distance R from the centre of the disc.
Then the kinetic energy of the rod is- L
æ 2 L2 ö R
1 2 1
m w çR + ÷ 2 2
251\D\Allen(IIT-JEE Wing)\2020-21\Nurture\Phy\Unit-03\YLRE\02-Rigid Body Dynamics

(A) 2 ç 12 ÷ø (B) mw R
è 2

1
(C) mw2L2 (D) None of these
24

46. Find the torque of a force F = -3 ˆi + ˆj + 5 kˆ acting at the point r = 7 ˆi + 3 ˆj + kˆ about origin:

(A) 14 ˆi - 38 ˆj + 16 kˆ (B) 4 ˆi + 4 ˆj + 6 kˆ

(C) - 21 ˆi + 4 ˆj + 4 kˆ (D) -14 iˆ + 38 ˆj - 16 kˆ

204
Rigid Body Dynamics

47. We have a rectangular slab of same thickness. E, F, G, H are the middle point of
AB, BC, CD and AD respectively then which of the following axis the moment of
inertia will be minimum :–
(A) AD (B) EG
(C) BD (D) HF

48. Off two eggs which have identical sizes, shapes and weights, one is raw and the other is half-boiled. The ratio
between the moment of inertia of the raw egg and that of the half-boiled egg about a central axis is :–
(A) one (B) greater than one (C) less than one (D) incomparable

1
49. The moment of inertia of a rod about an axis through its centre and perpendicular to it is ML2 (where M
12
is the mass and L is the length of the rod). The rod is bent in the middle so that the two half make an angle
of 60°. The moment of inertia of the bent rod about the same axis would be :–

1 1 1 ML3
(A) ML2 (B) ML2 (C) ML2 (D)
48 12 24 8 3

50. Four similar point masses (each of mass m) are placed on the circumference of a disc of mass M and radius
R. The M.I. of the system about the normal axis through the centre O will be:-
1
(A) MR2 + 4mR2 (B) MR2 + 4mR2 O
2

8
(C) MR2 + mR2 (D) None of these
5

51. For the same total mass which of the following will have the largest moment of inertia about an axis passing
through its centre of mass and perpendicular to the plane of the body
(A) a disc of radius a (B) a ring of radius a
(C) a square lamina of side 2a (D) four rods forming a square of side 2a

52. A rigid body of mass m rotates with angular velocity w about an axis at a distance d from the centre of mass
G. The radius of gyration about a parallel axis through G is k. The kinetic energy of rotation of the body is :–
1 1 1 1
(A) mk 2 w2 (B) md2 w2 (C) m(d2 + k2 )w 2 (D) m(d + k)2 w 2
2 2 2 2

53. A weightless rod is acted on by upward parallel forces of 2N and 4N at ends A and B respectively. The total
length of the rod is AB =3 m. To keep the rod in equilibrium a force of 6N should act in the following
manner:–
(A) Downwards at any point between A and B (B) Downwards at mid point of AB
251\D\Allen(IIT-JEE Wing)\2020-21\Nurture\Phy\Unit-03\YLRE\02-Rigid Body Dynamics

(C) Downwards at a point C such that AC =1m (D) Downwards at a point D such that BD =1m

54. If a ring, a disc, a solid sphere and a cylinder of same radius rolls down on inclined plane, the first one to
reach the bottom will be :
(A) disc (B) ring (C) solid sphere (D) cylinder

55. A body is rolling without slipping on a horizontal surface and its rotational kinetic energy is equal to the
translational kinetic energy. The body is :–
(A) disc (B) sphere (C) cylinder (D) ring

205
JEE-Physics
56. A disc of mass M and radius R rolls on a horizontal surface and then rolls up an inclined plane
as shown in the figure. If the velocity of the disc is v, the height to which the disc will rise will
be:

3v 2 3v 2
(A) (B)
2g 4g h
v
v2 v2
(C) (D) .
4g 2g
C A
57. A rod hinged at one end is released from the horizontal position as shown
in the figure. When it becomes vertical its lower half separates without
q
exerting any reaction at the breaking point. Then the maximum angle
'q' made by the hinged upper half with the vertical is B
(A) 300 (B) 450
(C) 600 (D) 900
C

58. A circular turn table has a block of ice placed at its centre. The system rotates with an angular speed w about
an axis passing through the centre of the table. If the ice melts on its own without any evaporation, the speed
of rotation of the system :–
(A) becomes zero (B) remains constant at the same value of w
(C) increases to value greater than w (D) decreases to a value less than w

59. An ant is sitting at the edge of a rotating disc. If the ant reaches the other end, after moving along the diameter,
the angular velocity of the disc will :-
(A) remain constant (B) first decreases and then increases
(C) first increases, then decrease (D) Increase continuously

60. Two rotating bodies have same angular momentum but their moments of inertia are I1 and I2 respectively
(I1>I2). Which body will have higher kinetic energy of rotation:–
(A) First (B) Second
(C) Both will have same kinetic energy (D) Not possible to predict

61. A disc of radius R is rolling purely on a flat horizontal surface, with a constant
angular velocity. The angle between the velocity and acceleration vectors of point
P is C
P
(A) Zero (B) 450
(C) 1350 (D) tan-1(1/2)

62. Let I be the moment of inertia of a uniform square plate about an axis AB A'
that passes through its centre and is parallel to two of its sides. CD is a line in C'
D
the plane of the plate that passes through the centre of the plate and makes
an angle q with AB. The moment of inertia of the plate about the axis CD q
A B
is then equal to:-
251\D\Allen(IIT-JEE Wing)\2020-21\Nurture\Phy\Unit-03\YLRE\02-Rigid Body Dynamics

(A) I (B) I sin2q


C
2
æqö
2
D'
(C) I cos q (D) I cos ç ÷ B'
è2ø

63. A cubical block of side 'a' moving with velocity v on a horizontal smooth plane as shown. It hits a ridge at
point O. The angular speed of the block after it hits O is :-
3v 3v a
(A) (B)
4a 2a
M v
3 O
(C) (D) zero
2a

206
Rigid Body Dynamics
64. A smooth sphere A is moving on a frictionless horizontal plane with angular velocity w and centre of mass
velocity v. It collides elastically and head on with an identical sphere B at rest. Neglect friction everywhere.
After the collision their angular speed are wA and wB respectively. Then :-
(A) wA < wB (B) wA = wB (C) wA = w (D) wB = w

65. A disc of mass M and radius R is rolling with angular speed w on a horizontal plane as shown. The magnitude
of angular momentum of the disc about the origin O is :- y
æ1ö 2
(A) ç ÷ MR w (B) MR2w w
è2ø
M
æ3ö
(C) ç 2 ÷ MR w
2
(D) 2MR2w O x
è ø

66. An equilateral triangle ABC formed from a uniform wire has two small identical
beads initially located at A. The triangle is set rotating about the vertical axis A
AO. Then the beads are released from rest simultaneously and allowed to slide g
down, one along AB and other along AC as shown. Neglecting frictional effects,
the quantities that are conserved as beads slides down are:
(A) angular velocity and total energy (kinetic and potential)
(B) total angular momentum and total energy B C
O
(C) angular velocity and moment of inertia about the axis of rotation
(D) total angular momentum and moment of inertia about the axis of rotation

67. A cubical block of side L rests on a rough horizontal surface with coefficient of F
friction µ. A horizontal force F is applied on the block as shown. If the coefficient
of friction is sufficiently high, so that the block does not slide before toppling, L
the minimum force required to topple the block is :-
mg mg
(A) infinitesimal (B) (C) (D) mg (1 – µ)
4 2

68. A thin wire of length L and uniform linear mass density r is bent into a circular loop with centre at O as
shown. The moment of inertia of the loop about the axis XX' is :-
X X'
rL3 rL3 90°
(A) (B)
8p2 16p2
O
5rL3 3rL3
(C) (D)
16p2 8 p2

69. One quarter section is cut from a uniform circular disc of radius R. This section
has a mass M. It is made to rotate about a line perpendicular to its plane and
passing through the centre of the original disc. Its moment of inertia about the
axis of rotation is :-
251\D\Allen(IIT-JEE Wing)\2020-21\Nurture\Phy\Unit-03\YLRE\02-Rigid Body Dynamics

1 1
(A) MR2 (B) MR 2
2 4
1
(C) MR 2 (D) 2 MR2
8

70. A cylinder rolls up an inclined plane and reaches some height and then rolls down (without slipping throughout
these motions). The directions of the frictional force acting on the cylinder are :
(A) up the incline while ascending and down the incline while descending
(B) up the incline while ascending as well as descending
(C) down the incline while ascending and up the incline while descending
(D) down the incline while ascending as well as descending.

207
JEE-Physics
71. A circular platform is free to rotate in a horizontal plane about a vertical axis passing through its centre. A
tortoise is sitting at the edge of the platform. Now the platform is given an angular velocity w0. When the
tortoise move along a chord of the platform with a constant velocity (with respect to the platform). The
angular velocity of the platform w(t) will vary with time t as :-

w(t) w(t) w(t) w(t)


w0 w0 w0
(A) (B) (C) w 0 (D)

t t t t

72. Consider a body, shown in figure, consisting of two identical balls, each of mass
L
M connected by a light rigid rod. If an impulse J = Mv is imparted to the body
M M
at one of its end, what would be its angular velocity:
v 2v v v J=Mv
(A) (B) (C) (D)
L L 3L 4L

73. A particle undergoes uniform circular motion. About which point on the plane of the circle, will the angular
momentum of the particle remain conserved ?
(A) Centre of circle (B) On the circumference of the circle
(C) Inside the circle (D) Outside the circle

74. A disc is rolling (with slipping) on a horizontal surface. C is its centre and Q
and P are two points equidistant from C. Let v P, vQ and vC be the magnitude Q
of velocities of points P, Q and C respectively, then :
C
(A) vQ > vC > vP (B) vQ < vC < vP P

1
(C) vQ = vP, vC = v (D) vQ < vC > vP
2 P

75. A child is standing with folded hands at the centre of a platform rotating about its central axis. The kinetic
energy of the system is K and moment of inertia is I. The child now stretches his arms so that the moment of
inertia of the system doubles. The kinetic energy of the system now is :-
K K
(A) 2K (B) (C) (D) 4K
2 4

76. A uniform disc of mass ‘m’ and radius R is placed on a smooth horizontal floor such that the plane surface of
the disc is in contact with the floor. A man of mass m/2 stands on the disc at its periphery. The man starts
walking along the periphery of the disc. The size of the man is negligible as compared to the size of the disc.
Then the centre of disc.
251\D\Allen(IIT-JEE Wing)\2020-21\Nurture\Phy\Unit-03\YLRE\02-Rigid Body Dynamics

R 2R
(A) moves along a circle of radius (B) moves along a circle of radius
3 3
R
(C) moves along a circle of radius (D) does not move along a circle
2

77. A tube of length L is filled completely with an incompressible liquid of mass M and closed at both the ends.
The tube is then rotated in a horizontal plane about one of its ends with a uniform angular velocity w. The
force exerted by the liquid at the other end is :-

Mw2L Mw2L Mw2 L2


(A) (B) Mw2L (C) (D)
2 4 2
208
Rigid Body Dynamics
x=0
78. A man can move on a horizontal plank supported symmetrically as shown.
The variation of normal reaction on support A with distance x of the man A B
from the end of the plank is best represented by
1m 1m

N N N N

(A) (B) (C) (D)

x x x x

79. Three point masses, each of mass m, are placed at the corners of an equilateral triangle of side l. Then the
moment of inertia of this system about an axis along one side of the triangle is:
(A) 3 ml2 (B) ml2 (C) 3/4 ml2 (D) 3/2 ml2

80. Three thin rods each of length L and mass M are placed along X, Y and Z-axes Z
in such a way that one end of each of the rods is at the origin. The moment of
inertia of this system about Z-axis is
L
2 2
2ML 4ML O
L
(A) (B) Y
3 3
L
5ML2 ML2
(C) (D) X
3 3

81. A small solid sphere is rolling on a frictionless surface, shown in figure with a translational velocity v m/s. If it
is to climb the inclined surface then v should be:–

(A) ³ 10 7 gh (B) > 2 gh (C) 2gh (D) 10/7gh

82. Consider the situation shown in the figure. Uniform rod of length L can
A L
rotate freely about the hinge A in vertical plane. Pulleys and strings are
light and frictionless. If the rod remains horizontal at rest when the system
is released then the mass of the rod is
4 8 16 32 M
(A) M (B) M (C) M (D) M
3 3 3 3 2M

83. The condition for beam balance as shown in figure to be in rotational l1 l2


equilibrium is
(A) F1l1 = F2l2 (B) F1l2 = F2l1
251\D\Allen(IIT-JEE Wing)\2020-21\Nurture\Phy\Unit-03\YLRE\02-Rigid Body Dynamics

F1 F2
(C) F1l12 = F2l22 (D) F1l22 = F2l12

84. A particle P of mass m is attached to a vertical axis by two strings AP and


BP of length l each. The separation AB = l . P rotates around the axis
with an angular velocity w. The tensions in the two strings are T 1 and T2. A
Find the incorrect option. L
(A) T1 = T2 L P
(B) T1 + T2 = m w2 l
(C) T1 – T2 = 2 mg L
B
2g
(D) BP will remain taut only if w ³
l

209
JEE-Physics
85. A slender rod of mass M and length L is hinged at O kept horizontal and then
released. The other end of rod strikes a solid sphere of mass M, radius R at point L
O A
P kept on a smooth horizontal surface. The point O and P are on the same M
vertical line. After the collision the rod comes to rest. The angular speed of the
L
sphere after the collision is
3g R 3g MR
(A) (B) P
L L L
m=0
L 3g
(C) (D) zero
R L

86. A thin rod of mass m and length l is hinged to a ceiling and it is free to rotate in a
vertical plane. A particle of mass m, moving with speed v strikes it as shown in the
figure and gets stick with the rod. The value of v, for which the rod becomes l/2
horizontal after collision is
56 56 60°
(A) gl (B) gl
3 3 m v

56 m, l
(C) gl (D) 2gl
3

87. A uniform rod AB of mass m and length 2a is falling freely without rotation under gravity with AB horizontal.
Suddenly the end A is fixed when the speed of the rod is v. The angular speed with which the rod begins to
rotate is :
v 4v v 3v
(A) (B) (C) (D)
2a 3a 3a 4a
Y
88. A particle of mass m moves with a constant velocity. Which of the following
D E
statements is not correct about its angular momentum about origin :
(A) it is zero when it is at A and moving along OA A C
(B) the same at all points along the line DE
(C) of the same magnitude but oppositely directed at B and D B
O X
(D) increases as it moves along the line BC

89. Two rings of the same radius and mass are placed such that their centres are at a common point and their
planes are perpendicular to each other. The moment of inertia of the system about an axis passing through
the centre and perpendicular to the plane of one of the rings is (mass of the ring = m, radius = r) :–
1 2 3
(A) mr (B) mr2 (C) mr2 (D) 2mr2
2 2

90. In an experiment with a beam balance an unknown mass m is balanced by two known masses of 16kg and
4 kg as shown in figure. The value of the unknown mass m is :–
251\D\Allen(IIT-JEE Wing)\2020-21\Nurture\Phy\Unit-03\YLRE\02-Rigid Body Dynamics

l1 l2 l1 l2

16kg m m 4kg

(A) 10 kg (B) 6 kg (C) 8 kg (D) 12 kg

91. A solid cylinder of mass M and radius R rolls without slipping down an inclined plane of length L and height
h. What is the linear speed of its centre of mass when the cylinder reaches its bottom?

3 4
(A) gh (B) gh (C) 4gh (D) 2 4gh
4 3

210
Rigid Body Dynamics
92. The moment of inertia of a uniform disc about an axis passing through its centre and perpendicular to its plane
is 1 kg-m2. It is rotating with an angular velocity 100 radians/sec. Another identical disc is gently placed on it so
that their centres coincide. Now these two discs together continue to rotate about the same axis. Then the loss
in kinetic energy in kilojoules is:
(A) 2.5 (B) 3.0 (C) 3.5 (D) 4.0

93. A solid sphere of mass M and radius R is placed vertically on a rough horizontal surface. If a horizontal force
F is applied at a distance R/6 below the centre line parallel to the diameter of the
sphere, the direction of frictional force will be
(A) forward (B) backward
(C) depends upon value of F (D) insufficient information

94. A uniform disk of mass 300kg is rotating freely about a vertical axis through its centre with constant angular
velocity w. A boy of mass 30kg starts from the centre and moves along a radius to the edge of the disk. The
angular velocity of the disk now is
w0 w0 4 w0 5w 0
(A) (B) (C) (D)
6 5 5 6

SECTION - 2 : MULTIPLE CHOICE CORRECT QUESTIONS


r r r r
95. The torque t on a body about a given point is found to be equal to A × L , wheree A is a constant
r
vector and L is the angular momentum of the body about that point. From this it follows that :-
uur
dL r
(A) is perpendicular to L at all instants of time
dt
r r
(B) the component of L in the direction of A does not change with time
r
(C) the magnitude of L does not change with time
r
(D) L does not change with time

96. A body is in equilibrium under the influence of a number of forces. Each force has a different line of action.
The minimum number of forces required is
(A) 2, if their lines of action pass through the centre of mass of the body
(B) 3, if their lines of action are not parallel
(C) 3, if their lines of action are parallel
(D) 4, if their lines of action are parallel and all the forces have the same magnitude

97. A particle falls freely near the surface of the earth. Consider a fixed point O (not vertically below the particle)
on the ground
(A) Angular momentum of the particle about O is increasing
(B) Torque of the gravitational force on the particle about O is decreasing
(C) The moment of inertia of the particle about O is decreasing
(D) The angular velocity of the particle about O is increasing
251\D\Allen(IIT-JEE Wing)\2020-21\Nurture\Phy\Unit-03\YLRE\02-Rigid Body Dynamics

98. If a cylinder is rolling down the incline with sliding


(A) after some time it may start pure rolling
(B) after some time it will start pure rolling
(C) it may be possible that it will never start pure rolling
(D) None of these

99. A ball moves over a fixed track as shown in the figure. From A to B the ball A C
rolls without slipping. If surface BC is frictionless and KA, KB and KC are kinetic
energy of the ball at A, B and C respectively, then: hA hC
(A) hA > hC; KB > KC (B) hA > hC; KC > KA B

(C) hA = hC; KB = KC (D) hA < hC; KB > KC

211
JEE-Physics
SECTION - 3 : COMPREHENSION BASED QUESTIONS
Comprehension-1.
Two discs A and B are mounted coaxially on a vertical axle. The discs have moments of inertia I and
2I respectively about the common axis. Disc A is imparted an initial angular velocity 2w using the entire
potential energy of a spring compressed by a distance x1. Disc B is imparted an angular velocity w by
a spring having the same spring constant and compressed by a distance x 2. Both the discs rotate in the
clockwise direction.
x1
100. The ratio x is :-
2

1 1
(A) 2 (B) (C) 2 (D)
2 2

101. When disc B is brought in contact with disc A, they acquire a common angular velocity in time t. The
average frictional torque on one disc by the other during this period is :-

2Iw 9Iw 9Iw 3Iw


(A) (B) (C) (D)
3t 2t 4t 2t

102. The loss of kinetic energy during the above process is :-

Iw 2 Iw 2 Iw 2 Iw 2
(A) (B) (C) (D)
2 3 4 6

Comprehension-2.
A solid sphere is kept over a smooth surface as shown is figure. It is hit by a cue at height h above the
centre C.

h
C

R R
103. In case 1, h = and in case 2, h = . Suppose in case 1 the sphere acquires a total kinetic energy K1 and
4 2
in case 2 total kinetic energy is K2. Then :-
(Note: That in both the cases, sphere is hit by the same impulse)
(A) K1 = K2 (B) K1 > K2 (C) K1 < K2 (D) data is insufficient

104. If the surface is rough, then after hitting the sphere, in which case the force of friction is in forward direction:-
(A) in case 1 (B) in case 2 (C) in both the cases (D) in none of the case
251\D\Allen(IIT-JEE Wing)\2020-21\Nurture\Phy\Unit-03\YLRE\02-Rigid Body Dynamics

Comprehension-3.
A small sphere of mass 1 kg is rolling without slipping on a stationary base

200
with linear speed v = m/s. It leaves the inclined plane at point C at
7
angle 30° with the horizontal. (Incline is fixed and sufficiently rough)

105. Find its linear speed at point C :-

100 50 100 200


(A) m/s (B) m/s (C) m/s (D) m/s
7 7 35 35

212
Rigid Body Dynamics
106. Find ratio of rotational and translational kinetic energy of the sphere when it strikes the ground after leaving
from point C :
2 2 1 1
(A) (B) (C) (D)
5 3 6 2

Comprehension-4.
A uniform disc rolls without slipping on a rough horizontal surface with uniform
angular velocity. Point O is the centre of disc and P is a point on disc as shown in
figure.

107. The velocity of point P on disc


(A) changes in magnitude with time (B) change in direction with time
(C) is always zero (D) is non-zero and remains constant in magnitude

108. The acceleration of point P on disc


(A) changes in magnitude with time (B) change in direction with time
(C) is always zero (D) is non-zero and remains constant in magnitude

109. The tangential acceleration of point P on disc


(A) changes in magnitude with time (B) change in direction with time
(C) is always zero (D) is non-zero and remains constant in magnitude

Comprehension-5.
A ring of mass M and radius R sliding with a velocity v 0 suddenly enters into
rough surface where the coefficient of friction is m , as shown in figure. v0

110. Choose the correct statement(s) Rough(m )


(A) As the ring enters on the rough surface, the limiting friction force acts on it
(B) The direction of friction is opposite to the direction of motion
(C) The friction force accelerates the ring in the clockwise sense about its centre of mass
(D) As the ring enters on the rough surface it starts rolling

111. Choose the correct statement(s)


(A) The momentum of the ring is conserved
(B) The angular momentum of the ring is conserved about its centre of mass
(C) The angular momentum of the ring conserved about any point on the horizontal surface
(D) The mechanical energy of the ring is conserved

112. Choose the correct statement(s) :


(A) The ring starts its rolling motion when the centre of mass stationary
(B) The ring starts rolling motion when the point of contact becomes stationary
v0
(C) The time after which the ring starts rolling is
2mg
251\D\Allen(IIT-JEE Wing)\2020-21\Nurture\Phy\Unit-03\YLRE\02-Rigid Body Dynamics

v0
(D) The rolling velocity is
2

113. Choose the correct alternative(s)


3v 02
(A) The linear distance moved by the centre of mass before the ring starts rolling is
8 mg
3
(B) The net work done by friction force is – mv02
8
mv 20
(C) The loss in kinetic energy of the ring is
4
mv 20
(D) The gain in rotational kinetic energy is +
8
213
JEE-Physics
Comprehension-6.
In figure, the winch is mounted on an axle, and the 6-sided
nut is welded to the winch. By turning the nut with a wrench,
a person can rotate the winch. For instance, turning the nut
clockwise lifts the block off the ground, because more and
more rope gets wrapped around the winch.
Three students agree that using a longer wrench makes it
easier to turn the winch. But they disagree about why. All
three students are talking about the case where the winch is
used, over a 10 s time interval, to lift the block one metre off
the ground.
Student 1 By using a longer wrench, the person decreases the average force he must exert on the wrench,
in order to lift the block one metre in 10 s.
Student 2 : Using a longer wrench reduces the work done by the person as he uses the winch to lift the block
1m in 10s.
Student 3 : Using a longer wrench reduces the power that the person must exert to lift the block 1m in 10s.

114. Student 1 is :
(A) correct, because the torque that the wrench must exert to lift the block doesn't depend on the wrench's
length
(B) correct, because using a longer wrench decreases the torque it must exert on the winch
(C) incorrect, because the torque that the wrench must exert to lift the block doesn't depend on the
wrench's length
(D) Incorrect, because using a longer wrench decreases the torque it must exert on the winch.

115. Which of the following is true about student 2 and 3 :-


(A) Student 2 and 3 are both correct (B) Student 2 is correct, but student 3 is incorrect
(C) Student 3 is correct, but student 2 is incorrect (D) Student 2 and 3 are both incorrect

116. If several wrenches all apply the same torque to a nut, which graph best expresses the relationship between
the force the person must apply to the wrench, and the length of the wrench :-
force
force

force
force

(1) (2) (3) (4)

length length length length

(A) 1 (B) 2 (C) 3 (D) 4

Comprehension-7.
A cylinder and a ring of same mass M and radius R are placed on the top of a sufficient rough inclined plane
of inclination q . Both are released simultaneously from the same height h

117. Choose the correct statement(s) related to the motion of each body
251\D\Allen(IIT-JEE Wing)\2020-21\Nurture\Phy\Unit-03\YLRE\02-Rigid Body Dynamics

(A) The friction force acting on each body opposes the motion of its centre of mass
(B) The friction force provides the necessary torque to rotate the body about its centre of mass
(C) Without friction none of the two bodies can roll
(D) The friction force ensures that the point of contact must remain stationary

118. Identify the correct statement(s)


(A) The friction force acting on the cylinder is more than that acting on the ring
(B) The friction force acting on the ring is more than that acting on the cylinder
(C) The velocity of centre of mass of the ring is gh when it reaches at bottom

(D) The velocity of centre of mass of each body is 2gh when it reaches at bottom

214
Rigid Body Dynamics
SECTION - 4 : MATRIX - MATCH QUESTIONS
119. Column I Column II
(A) In pure rolling work done by friction (p) is always zero
(q) may be zero
(B) In forward slipping work done by friction (r) is negative
on one of the object (s) is positive
(C) In backward slipping work done by (t) may be negative
friction on one of the object (u) may be positive

120. A disc with linear velocity v and angular velocity w is placed on rough ground. Suppose a and a be the
magnitudes of linear and angular acceleration due to friction. Then :-
Column I Column II
v
(A) When v = Rw (p) a = Ra (a ¹ 0)
w

Rw
(B) When v = (q) a > Ra
2

(C) When v = 2Rw (r) a < Ra


(s) None

121. Four rods of equal length l and mass m each form a square as shown in figure. Moment of inertia about four
axes 1, 2,3 and 4 are say I1, I2, I3 and I4.
Column I Column II

4
(A) I1 (p) ml2
3

2
(B) I2 (q) ml2
3

1
(C) I3 (r) ml2
2
(D) I4 (s) None

122. A solid sphere is rotating about an axis as shown in figure. An insect follows the dotted path on the circumference
of sphere as shown.
Column I Column II Insect
(A) Moment of inertia of system about axis of (p) will remain constant
rotation
(B) Angular velocity (q) will first increase
251\D\Allen(IIT-JEE Wing)\2020-21\Nurture\Phy\Unit-03\YLRE\02-Rigid Body Dynamics

(C) Angular momentum of system about axis of then decrease


rotation (r) will first decrease then increase
(D) Rotational kinetic energy (s) will continuously decrease
(t) will continuously increase
(u) data is insufficient

215
Rigid Body Dynamics
131. A small ring of mass m is threaded on a horizontal smooth rod which is rotating about its end with constant
angular velocity w. The ring is initially located at near the axis of rotation. When the distance of the ring from
the axis becomes r, then find the power required to rotate the system with same angular velocity.

132. A 500 g block P rests on a frictionless horizontal table at a distance of 400 mm


from a fixed pin O. The block is attached to pin O by an elastic cord of force
constant k = 100 N/m and of underformed length 900 mm. If the block is set in
motion perpendicularly, determine:
(a) the speed in the begining for which the distance from O to the block P will
reach the maximum value of 1.2 m.
(b) the speed when OP = 1.2 m
(c) the radius of curvature of the path of the block when OP = 1.2 m

133. There is a rectangular plate of mass M kg of dimensions (a × b). The plate is b


held in horizontal position by striking n small balls each of mass m per unit
area per unit time. These are striking in the shaded half region of the plate. a
The balls are colliding elastically with velocity v. What is v ? It is given n=100,
M = 3 kg, m=0.01 kg, b=2m, a=1m, g=10 m/s2.

SECTION - 6 : SUBJECTIVE QUESTIONS


y

134. A solid hemisphere of mass M and radius R is rotated about an axis y – y’ with
R
angular velocity w as shown in figure. Find the radius of gyration of the hemi
X
sphere about this axis. (0,0)
w y’

135. A wheel rotates with an angular acceleration given by a = 4at3 - 3bt2 , where t is the time and a and b are
constants. If the wheel has initial angular speed w0, write the equations for the (i) angular speed (ii) angular
displacement in terms of time t.

136. Find out the moment of inertia of an uniform semi circular disc about an axis passing through its centre
of mass and perpendicular to the plane? (Mass of semi-circular disc is M and radius R)

137. Why a force is applied at right angles to the heavy door at its outer edges while closing or opening it ?

138. A solid homogeneous cylinder of height h and base radius r is kept vertically on a conveyer belt moving
horizontally with an increasing velocity v = a + bt 2. If the cylinder is not allowed to slip find the time
when the cylinder is about to topple.
Q
251\D\Allen(IIT-JEE Wing)\2020-21\Nurture\Phy\Unit-03\YLRE\02-Rigid Body Dynamics

139. A rod of length L and mass M is hinged at point O. A small bullet of mass m
hits the rod as shown in the figure. The bullet gets embedded in the rod. Find
angular velocity of the system just after impact.
mv

140. A solid cylinder rolls without slipping on an inclined plane inclined at an angle q. Find the linear acceleration
of the cylinder. Mass of the cylinder is M.

217
JEE-Physics
141. A rectangular rigid fixed block has a long horizontal edge. A solid
homogeneous cylinder of radius R is placed horizontally at rest with its
length parallel to the edge such that the axis of the cylinder and the edge R

of the block are in the same vertical plane as shown in figure. There is
sufficient friction present at the edge, so that a very small displacement
causes the cylinder to roll of the edge without slipping. Determine :
(i) The angle qC through which the cylinder rotates before it leaves contact with the edge.
(ii) The speed of the centre of mass of the cylinder before leaving contact with the edge and
(iii) The ratio of the translational to rotational kinetic energies of the cylinder when its centre of mass is in
horizontal line with the edge.

142. A uniform rod of length 4l and mass m is free to rotate about a horizontal axis
passing through a point distance l from its one end. When the rod is horizontal, its
angular velocity is w as shown in figure. Calculate l
(i) reaction of axis at this instant, w
(ii) acceleration of centre of mass of the rod at this instant,
(iii) reaction of axis and acceleration of centre of mass of the rod when rod becomes vertical for the first time.
(iv) minimum value of w so that centre of rod can complete circular motion.

143. A semi circular track of radius R = 62.5 cm is cut in a block. Mass of block,
having track, is M = 1 kg and rests over a smooth horizontal floor. A cylinder
of radius r = 10 cm and mass m = 0.5 kg is hanging by thread such that axis
of cylinder and track are in same level and surface of cylinder is in contact with m
the track as shown in figure. When the thread is burnt, cylinder starts to move R
down the track. Sufficient friction exists between surface of cylinder and track,
M
so that cylinder does not slip. Calculate velocity of axis of cylinder and velocity
of the block when it reaches bottom of the track. Also find force applied by
block on the floor at the moment. ( g = 10 m/s2)

144. A man and a woman skate towards each other on smooth ice, but in parallel
lines. The distance between the lines is l. The mass of the man is M and
M
that of the woman is m. The velocity of the man is given by V and that of the
V
woman by v. The woman holds a stick of length l and negligible mass. The
stick is directed normal to the direction of motion as shown in the figure. l

When the couple passes each other, the man grasps the stick and the couple v
move together, each of them holding different ends of the stick. W
(i) What is the angular velocity of the rod after the couple begin moving together ?
(ii) The couple start moving towards each other by pulling the stick until the distance between them is l0(l0 < l).
What is the velocity of the centre of mass now ?
(iii) What is the angular velocity of the couple now ?
(iv) What is the work done by the couple as they move from l to l0 ?

145. A massless string passes over a massless fixed ideal pulley P and carries a block
of mass m = 1 kg at one end. The other end of the string is wound up on the
251\D\Allen(IIT-JEE Wing)\2020-21\Nurture\Phy\Unit-03\YLRE\02-Rigid Body Dynamics

circumference of a solid cylinder of mass M = 2 kg which hangs as shown.


(i) Find the acceleration of the centre of mass of M. Slipping is absent.
[Take g = 10 m/s2]
(ii) Find out the acceleration of m.

146. Find minimum height of obstacle so that the sphere can stay in equilibrium
m

R
h
q

218
Rigid Body Dynamics
147. Determine the minimum co-efficient of friction between a thin uniform rod and a floor at which a person can
slowly lift the rod from the floor without slipping, to the vertical position applying to its end a force always
perpendicular to its length.

148. Two thin circular disc of mass 2 kg and radius 10 cm each are joined by a rigid
massless rod of length 20 cm. The axis of the rod is along the perpendicular
to the planes of the disc through their centres. This object is kept on a truck in O
such a way that the axis of the object is horizontal and perpendicular to the
direction of motion of the truck. Its friction with the floor of the truck is large
20cm
enough, so that the object can roll on the truck without slipping.
Take x-axis as the direction of motion of the truck and z-axis as the vertically upwards direction. If the truck
has an acceleration 9 m/s2, calculate :
(i) the force of friction on each disc and
(ii) the magnitude and direction of the frictional torque acting on each disc about the centre of mass O of the

object. Express the torque in the vector form in terms of unit vector î , ĵ and k̂ in x, y and z-directions.

149. A moving particle in X - Y plane has its angular momentum in Z-direction only. Prove it.

150. As shown in the figure, a rod moves with v=2 m/sec and rotates
with w = 2p rad/sec. 2m/s

Find the point on the rod whose velocity is zero in this frame.
w

151. A ring of radius 3a is fixed rigidly on a table. A small ring whose mass is m and
radius a, rolls without slipping inside it as shown in the figure. The small ring is m
A a
released from position A. When it reaches at the lowest point, the speed of the 3a
centre of the ring at that time would be-

152. A sphere is placed rotating with its centre initially at rest in a corner (A) (B)
as shown in figure (A) & (B). Coefficient of friction between all
1
surfaces and the sphere is . Find the ratio of the frictional force
3

fa
fb by ground in situations (A) & (B)

153. A cylinder of mass M and radius R is resting on a horizontal platform (which is parallel to the x-y plane) with
it axis fixed along the y-axis and free to rotate about its axis. The platform is given a motion in the x-direction
given by x = Acos(wt). There is no slipping between the cylinder and platform. The maximum torque acting
on the cylinder during its motion is ........
251\D\Allen(IIT-JEE Wing)\2020-21\Nurture\Phy\Unit-03\YLRE\02-Rigid Body Dynamics

154. The translational kinetic energy is the .......... percent of total energy for a rolling hollow sphere.

155. A smooth uniform rod of length L and mass M has two identical beads of
negligible size, each of mass m, which can slide freely along the rod. Initially
the two beads are at the centre of the rod and the system is rotating with
an angular velocity w0 about an axis perpendicular to the rod and passing
through the mid-point of the rod (see fig). There are no external forces.
When the beads reach the ends of the rod, the angular velocity of the
system is .........

219
JEE-Physics
156. A carpet of mass M made of inextensible material is rolled along its length in the form of a cylinder of radius
R and is kept on a rough floor. The carpet starts unrolling without sliding on the floor when a negligibly small
push is given to it. Calculate the horizontal velocity of the axis of the cylindrical part of the carpet when its
R
radius reduces to . (Neglect vertical component of velocity of centre of mass)
2

157. A homogeneous rod AB of length L = 1.8 m and mass M is pivoted at


the centre O in such a way that it can rotate freely in the vertical plane S
(fig.). The rod is initially in the horizontal position. An insect S of the v
same mass M falls vertically with speed v on the point C, midway
between the points O and B. Immediately after falling, the insect moves
towards the end B such that the rod rotates with a constant angular A O C B
velocity w. L/2 L/4 L/4

(i) Determine the angular velocity w in terms of v and L.


(ii) If the insect reaches the end B when the rod has turned through an angle of 90°, determine v.

1
158. Four 2kg masses are connected by m long spokes to an axle as in shown
4

1
figure. A force F of 24N acts on a lever m long to produce an angular
2
acceleration a. Determine the magnitude of a.

159. A thin rod is passing through the centre of a sphere. The rod is fixed to a
w
vertical axis and the sphere is made to roll on a surface with friction. The
radius of the sphere is r, the mass is m and the length of the rod is l. The rod
l
is rotating with an angular velocity w0 . Find the energy of the sphere in terms

of w0 , m, l and r. Assume the rod to be of negligible mass.

160. A square frame is formed by four rods, each of length l = 60 cm. Mass of two rods
AB and BC is m = 25/18 kg each while that of rods AD and CD is 2kg each. The
frame is free to rotate about a fixed horizontal axis passing through its geometric
centre O shown in figure. A spring is placed on the rod AB at a distance a = 15 cm
from B. The spring is held vertical and a block is placed on upper end of the spring
so that rod AB is horizontal. Calculate mass M of the block.

161. A uniform circular disc has radius R and mass m. A particle, also of mass m, is fixed at a point A on
the edge of the disc as shown in the figure. The disc can rotate freely about a horizontal chord PQ that
R
is at a distance from the centre C of the disc. The line AC is perpendicular to PQ. Initially the disc
4
251\D\Allen(IIT-JEE Wing)\2020-21\Nurture\Phy\Unit-03\YLRE\02-Rigid Body Dynamics

is held vertical with the point A at its highest position. It is then allowed to fall, so that it starts rotation
about PQ. Find the linear speed of the particle as it reaches its lower position.

C
R
4
P Q

220
Rigid Body Dynamics
162. A man pushes a cylinder of mass m1 with the help of a plank of mass m2 as shown. There is no slipping
at any contact. The horizontal component of the force applied by the man is F, Find :

F1 m2

m1

(i) the acceleration of the plank and the centre of mass of the cylinder and
(ii) the magnitude and directions of frictional forces at contact points.

163. A rod AB of mass M and length L is lying on a horizontal frictionless surface. A particle of mass m travelling
along the surface hits the end A of the rod with a velocity v0 in a direction perpendicular to AB. The
collision is elastic. After the collision the particle comes to rest.

m
(i) Find the ratio
M
(ii) A point P on the rod is at rest immediately after collision. Find the distance AP.

pL
(iii) Find the linear speed of the point P after a time after the collision.
3V0

164. Two heavy metallic plates are joined together at 90° to each other. A laminar
sheet of mass 30 kg is hinged at the line AB joining the two heavy metallic
plates. The hinges are frictionless. The moment of inertia of the laminar
sheet about an axis parallel to AB and passing through its centre of mass
is 1.2 kg-m2. Two rubber obstacles P and Q are fixed, one on each metallic A
plate at a distance 0.5 m from the line AB. This distance is chosen, so
that the reaction due to the hinges on the laminar sheet is zero during the Q
impact. Initially the laminar sheet hits one of the obstacles with an angular
velocity 1 rad/s and turns back. If the impulse on the sheet due to each B
obstacle is 6 N-s.
(i) Find the location of the centre of mass of the laminar sheet from AB.
(ii) At what angular velocity does the laminar sheet come back after the first impact.
(iii) After how many impact, does the laminar sheet come to rest.

y
165. Three particles A, B and C each of mass m, are connected to each other
A x
by three massless rigid rods to form a rigid, equilateral triangular body w
of side l. This body is placed on a horizontal frictionless table (x-y plane)
and is hinged to it at the point A, so that it can move without friction
251\D\Allen(IIT-JEE Wing)\2020-21\Nurture\Phy\Unit-03\YLRE\02-Rigid Body Dynamics

about the vertical axis through A (see figure). The body is set into rotational
motion on the table about A with a constant angular velocity w. F B C
(i) Find the magnitude of the horizontal force exerted by the hinge on the body.
(ii) At time T, when the side BC is parallel to the x-axis, a force F is applied on B along BC (as shown).
Obtain the x-component and the y-component of the force exerted by the hinge on the body, immediately
after time T.

221
JEE-Physics
166. The figure shows a uniform rod lying along the x-axis. The locus of all the y
points lying on the xy-plane, about which the moment of inertia of the
rod is same as that about O is

x
o

167. Find the moment of inertia of uniform solid cone of mass M


and radius R about an axis AA'

168. A uniform cylinder of mass M lies on a fixed plane inclined at an angle q with
horizontal. A light string is tied to the cylinder’s right most point, and a mass m
hangs from the string, as shown. Assume that the coefficient of friction between
the cylinder and the plane is sufficiently large to prevent slipping. For the cylinder
to remain static, the value of mass m is-

SECTION - 7 : ASSERTION–REASON QUESTIONS


169. Statement –I : Two cylinders, one hollow (metal) and the other solid (wood) with the same mass and
identical dimensions are simultaneously allowed to roll without slipping down an inclined plane from the
same height. The hollow cylinder will reach the bottom of the inclined plane first.
and
Statement–2 : By the principle of conservation of energy, the total kinetic energies of both the cylinders are
identical when they reach the bottom of the incline.
(A) Statement-1 is True , Statement-2 is True ; Statement-2 is a correct explanat ion for
Statement-1
(B) Statement-1 is True, Statement-2 is True; Statement-2 is NOT a correct explanation for
Statement-1
(C) Statement-1 is True, Statement-2 is False
(D) Statement-1 is False, Statement-2 is True
251\D\Allen(IIT-JEE Wing)\2020-21\Nurture\Phy\Unit-03\YLRE\02-Rigid Body Dynamics

170. Statement –1 : A rigid disc rolls without slipping on a fixed rough horizontal surface with uniform angular
velocity. Then the acceleration of lowest point on the disc is zero.
and
Statement –2 : For a rigid disc rolling without slipping on a fixed rough horizontal surface, the velocity of
the lowest point on the disc is always zero.
(A) Statement–1 is True, Statement–2 is True ; Statement–2 is a correct explanation for Statement–1
(B) Statement–1 is True, Statement–2 is True ; Statement–2 is not a correct explanation for Statement–1
(C) Statement–1 is True, Statement–2 is False.
(D) Statement–1 is False, Statement–2 is True.

222
Rigid Body Dynamics
171. Statement –1 : A sphere rolling on a rough horizontal surface with constant velocity then it start going up on
a smooth inclined plane. Rotational KE of sphere decreases continuously on horizontal and inclined surface.
and
Statement –2 : Rotational KE decreases if torque due to friction opposes angular velocity of sphere.
(A) Statement–1 is True, Statement–2 is True ; Statement–2 is a correct explanation for Statement–1
(B) Statement–1 is True, Statement–2 is True ; Statement–2 is not a correct explanation for Statement–1
(C) Statement–1 is True, Statement–2 is False.
(D) Statement–1 is False, Statement–2 is True.

172. Statement –1 : A disc is rolling on an inclined plane without slipping. The velocity of centre of mass is v.
These others points on the disc lies on a circular arc having same speed as centre of mass.
and
Statement –2 : When a disc is rolling on an inclined plane. The magnitude of velocities of all the point from
the contact point is same, having distance equal to radius r.
(A) Statement–1 is True, Statement–2 is True ; Statement–2 is a correct explanation for Statement–1
(B) Statement–1 is True, Statement–2 is True ; Statement–2 is not a correct explanation for Statement–1
(C) Statement–1 is True, Statement–2 is False.
(D) Statement–1 is False, Statement–2 is True.

173. Statement–1 : A sphere is performing pure rolling on a rough horizontal surface with constant angular
velocity. Frictional force acting on the sphere is zero. [No other force acting on the sphere]
and
Statement–2 : Velocity of contact point is zero.
(A) Statement–1 is True, Statement–2 is True ; Statement–2 is a correct explanation for Statement–1
(B) Statement–1 is True, Statement–2 is True ; Statement–2 is not a correct explanation for Statement–1
(C) Statement–1 is True, Statement–2 is False.
(D) Statement–1 is False, Statement–2 is True.

174. Statement –1 : In case of rolling friction force can in forward and backward direction both.
and
Statement –2 : The angular momentum of a system will be conserved only about that point about which
external angular impulse is zero.
(A) Statement–1 is True, Statement–2 is True ; Statement–2 is a correct explanation for Statement–1
(B) Statement–1 is True, Statement–2 is True ; Statement–2 is not a correct explanation for Statement–1
(C) Statement–1 is True, Statement–2 is False.
(D) Statement–1 is False, Statement–2 is True.

175. Statement –1 : For the purpose of calculation of moment of inertia, a body’s mass can be thought to be
concentrated at its centre of mass.
and
Statement –2 : Moment of inertia is a measure of how the mass is distributed about a certain axis.
(A) Statement–1 is True, Statement–2 is True ; Statement–2 is a correct explanation for Statement–1
(B) Statement–1 is True, Statement–2 is True ; Statement–2 is not a correct explanation for Statement–1
251\D\Allen(IIT-JEE Wing)\2020-21\Nurture\Phy\Unit-03\YLRE\02-Rigid Body Dynamics

(C) Statement–1 is True, Statement–2 is False.


(D) Statement–1 is False, Statement–2 is True.

176. Statement –1 : If a body (ball) is rolling on a surface without slipping, no frictional force acts on it.
and
Statement –2 : In the case of rolling without slipping point of contacts are relatively at rest.
(A) Statement–1 is True, Statement–2 is True ; Statement–2 is a correct explanation for Statement–1
(B) Statement–1 is True, Statement–2 is True ; Statement–2 is not a correct explanation for Statement–1
(C) Statement–1 is True, Statement–2 is False.
(D) Statement–1 is False, Statement–2 is True.

223
JEE-Physics
r r uur ur r ur
177. Statement –1 : Torque ( t ) acting on a rigid body is defined as t = A ´ L, A is a constant vector and L is the
angular momentum of the body. The magnitude of the angular momentum of the body remains same.
and
r r r
Statement–2 : t is perpendicular to L and also perpendicular to w , hence torque does not deliver any
power to the body.
(A) Statement–1 is True, Statement–2 is True ; Statement–2 is a correct explanation for Statement–1
(B) Statement–1 is True, Statement–2 is True ; Statement–2 is not a correct explanation for Statement–1
(C) Statement–1 is True, Statement–2 is False.
(D) Statement–1 is False, Statement–2 is True.

178. Statement –1 : The moment of inertia of a rigid body is not unique, about a given axis.
and
Statement –2 : The moment of inertia of a rigid body depends on axis about which it has to be calculated.
(A) Statement–1 is True, Statement–2 is True ; Statement–2 is a correct explanation for Statement–1
(B) Statement–1 is True, Statement–2 is True ; Statement–2 is not a correct explanation for Statement–1
(C) Statement–1 is True, Statement–2 is False.
(D) Statement–1 is False, Statement–2 is True.

179. Statement –1 : A non-uniform sphere is placed such that its centre is origin of coordinate system. If Ix and
Iy be moment of inertia about x axis and y axis respectively then moment of inertia about z axis is Ix + Iy.
and
Statement –2 : According to perpendicular axis theory Iz = Ix + Iy when object is lying in x-y plane.
(A) Statement–1 is True, Statement–2 is True ; Statement–2 is a correct explanation for Statement–1
(B) Statement–1 is True, Statement–2 is True ; Statement–2 is not a correct explanation for Statement–1
(C) Statement–1 is True, Statement–2 is False.
(D) Statement–1 is False, Statement–2 is True.

251\D\Allen(IIT-JEE Wing)\2020-21\Nurture\Phy\Unit-03\YLRE\02-Rigid Body Dynamics

224
Rigid Body Dynamics

ANSWERS
l SINGLE CHOICE CORRECT QUESTIONS

Que. 1 2 3 4 5 6 7 8 9 10 11 12 13 14 15
Ans. B C D D C A B D B A D B B D D
Que. 16 17 18 19 20 21 22 23 24 25 26 27 28 29 30
Ans. B C A B D A D B B C D D C C D

Que. 31 32 33 34 35 36 37 38 39 40 41 42 43 44 45

Ans. C A C D A A D B) C B D A A B A

Que. 46 47 48 49 50 51 52 53 54 55 56 57 58 59 60

Ans. D B B B B D C D C D B C D C B

Que. 61 62 63 64 65 66 67 68 69 70 71 72 73 74 75

Ans. B A A C C B C D A B C A A A B

Que. 76 77 78 79 80 81 82 83 84 85 86 87 88 89 90

Ans. A A B C A B C A A D C D D C C

Que. 91 92 93 94

Ans. B A A D

l MULTIPLE CHOICE CORRECT QUESTIONS


95. (ABC) 96. (AB) 97. (ACD) 98. (AC) 99. (AB)

l COMPREHENSION BASED QUESTIONS


100. (C) 101. (A) 102. (B) 103. (C) 104. (B) 105. (A) 106. (C) 107. (AB)
108. (BD) 109. (AB) 110. (ABC) 111. (C) 112. ( BCD ) 113. (ACD) 114. (A) 115. (D)
116. (D) 117. (ABCD) 118. (BC)

l MATRIX-MATCH QUESTIONS
119. (A) q,t,u (B) q,t,u (C) q,t,u 120. (A) s, (B) r, (C) r
121. (A) q, (B) s, (C) q, (D) q 122. (A) q, (B) r, (C) p (D) r

l NUMERICAL ANSWER BASED QUESTIONS


123. 8.4 rad/sec2 124. 13ll3 125. 32 Ml2 126. 6.3 m/s
127. (i) 1.63 N (ii) 1.22 m 128. 1.43 m/s2 129. 106.7 rad/s 130. 12 rad/s
131. 2mw3r2 132. (a) 4.5 m/s (b) 1.5 m/s (c) 3.75 cm 133. 10 ms–1
251\D\Allen(IIT-JEE Wing)\2020-21\Nurture\Phy\Unit-03\YLRE\02-Rigid Body Dynamics

l SUBJECTIVE QUESTIONS

2 at 5 bt 4
134. K = R 135. (i) w = w0 + at 4 - bt 3 (ii) q = w0 t + -
5 5 4

é MR 2 æ 4R ö ù
2

136. ê - M çè ÷ ú 137. To maximize the torque


ëê 2 3 p ø ûú

3mv
138. gr/bh 139. 3m + M L
( )
225
JEE-Physics

2g sin q æ 4ö 4gR
140. 141. (i) q = cos -1 çè ÷ø (ii) (iii) 6
3 7 7

2 2
4 æ 7lw2 ö æ 3g ö æ 13 2ö æ 6g ö 6g
( )
2
çè 7 ÷ø + l w (iii) ç mg + mlw ÷ , ç ÷ + lw (iv)
2
142. (i) mg 1+ç (ii)
2

7 è 4g ÷ø è 7 ø è 7ø 7l
143. 2 m/s, 1 m/s, 13.57 N
2
V+v MV - mv æ V + vö æ l ö 1 Mm é l ù
2 ê1 - ú
144. (i) (ii) (iii) ç ÷ ç ÷ (iv) (V + v) l0 û
l M+ m è l ø è l0 ø 2 M+m ë

145. (i) 6 m/s2 ¯, (ii) 2 m/s2 ¯ 146. R (1 - cos q)

1
147.
2 2
( )
148. (i) 6 ˆi N (ii) 0.6 - ĵ ± kˆ , 0.85 Nm

149.

æ 1ö
150. ç ÷ m down to the centre of mass 151. 2ga
è pø
1
152. 9/10 153. 2
2 MRAw
Mw 0
154. (60) 155.
M + 6m
14Rg 12v
156. 157. (i) (ii) 3.5 m/s
3 7l
158. (12 rad/s2)

1 æ2 2 2ö 1 æ 2 2ö 2 v lw 0
çè 5 mr + ml ÷ø w 0 + çè 5 mr ÷ø w wheree w = =
2
159.
2 2 r r

11
160. kg 161. 5gR
9

4F 8F 3m1 F m1 F
162. (i) acm = 3m + 8m , a plank = 3m + 8m (ii) ( 3m + 8m ) , ( 3m + 8m )
1 2 1 2 1 2 1 2

1 2 v0
163. (i) (ii) L (iii)
4 3 2 2
164. (i) 0.1 m (ii) 1 rad/s (iii) Laminar sheet will never come to rest
F
165. (i) 3mw 2 l (ii) Fx = - 4 , F y = 3 mw2 l 166. a circle
251\D\Allen(IIT-JEE Wing)\2020-21\Nurture\Phy\Unit-03\YLRE\02-Rigid Body Dynamics

3 sin q
167. MR 2 168. M
10 1 – sin q

l ASSERTION-REASON QUESTIONS
169. (D) 170. (D) 171. (D) 172. (A)
173. (A) 174. (B) 175. (D) 176. (D)
177. (A) 178. (D) 179. (D)

226

You might also like